Sunteți pe pagina 1din 266

www.cliqueapostilas.com.

br

Apostila de
Física
www.cliqueapostilas.com.br
www.cliqueapostilas.com.br

Pr of . Romer o T avar es da Silva

15. Fluidos

Fluidos compreendem líquidos e gases. Os líquidos escoam sob a ação da gravi-


dade até preencherem as regiões mais baixas possíveis dos vasos que os contém. Os
gases se expandem até ocuparem todo o volume do vaso, qualquer que seja a sua forma.

As moléculas em um gás não têm restrição de movimento dentro do recipiente que


o contém, e podem se deslocar através de toda essa região do espaço.

Já o líquido está restrito a se mover abaixo da sua superfície. Grande parte de suas
moléculas não têm energia suficiente para vencer essa barreira imposta pela superfície,
daí a contenção entre a sua superfície e as parede do recipiente.

Na Mecânica dos Fluidos estudamos o movimento do conjunto de partículas e não


o de cada partícula, como na Mecânica Newtoniana.

Densidade

Define-se densidade ρ de um material como a relação entre a sua massa e o seu


volume. De maneira formal, analisamos apenas uma pequena porção do material de
massa ∆m e volume ∆V e definimos a sua densidade como:

∆m
ρ=
∆V

e se este material tiver uma distribuição uniforme de massa, a sua densidade será a
mesma em todas as suas partes. Nesse caso teremos ρ = m/V .

Pressão

A pressão mede a relação entre a força aplicada a uma superfície e o tamanho da


superfície considerada.

Seja ∆F a força que está sendo aplicada em um êm-


bolo de superfície ∆A . A pressão p que esta força está ∆F ∆A
exercendo no êmbolo é definida como:

∆F
p=
∆A

À rigor, a pressão é definida para o limite desta razão,


no limite quando a área tender à zero. Ou seja:

dF
p= ⇒ dF = p dA
dA

Cap 15 www. f isica. uf pb. br / ~r omer o 2


www.cliqueapostilas.com.br

Pr of . Romer o T avar es da Silva

Fluido em repouso

Para deduzir a relação entre pressão, densidade e profundidade, analisemos um


fluido de densidade ρ em repouso num dado recipiente, como mostrado na figura à se-
guir. Vamos considerar um cilindro imaginário desenhado nesse fluido. Esse cilindro tem
superfícies A paralelas à superfície do fluido e uma altura dy ao longo da profundidade
do fluido. A força líquida dFR que o fluido exerce neste cilindro é dada por:

p A - (p + dp) A = dFR

onde pA é a força que atua na super-


fície inferior e (p + dp) A é a força que
atua na superfície superior do cilindro y+dy (p+dp)A
imaginário. Como o cilindro está em
repouso, essa força deve ser igual ao
peso do cilindro. Desse modo: y pA

- dp A = dFR = g dm

Mas
dm = ρ dV = ρ A dy

ou seja:
dp = - ρ g dy (p+dp)A
logo
p2 y2

∫ dp = − ∫ ρ g dy
p1 y1
pA
Quando a densidade puder ser
considerada uniforme, ou seja quando
a densidade não variar com a altura, a
integração terá a forma:

p2 y2

∫ dp = − ρ g ∫ dy
p1 y1

ou seja:
p 2 − p1 = − ρ g (y 2 − y 1 )

Considerando que a pressão aumenta com a profundidade, vamos definir a profundidade


como h , a pressão nesta profundidade como p e a pressão superficial como p0 , e des-
se modo:
p = p0 + ρ g h

Assim encontramos que a pressão varia linearmente com a profundidade h .

Cap 15 www. f isica. uf pb. br / ~r omer o 3


www.cliqueapostilas.com.br

Pr of . Romer o T avar es da Silva

O Princípio de Pascal

A pressão aplicada a um fluido contido em um recipiente é transmitida integral-


mente a todos os pontos do fluido e às paredes do recipiente que o contém.

Se a pressão atmosférica for chamada de p0 , a pressão em uma profundidade h


deste fluido será dada por:
p = p0 + ρ g h

Caso a pressão atmosférica varie, e num certo dia ela passe para o valor p1 onde
p1 < p0 , a pressão no interior do lago também irá variar como consequência desta mu-
dança, e teremos:
p = p1 + ρ g h

O Princípio de Arquimedes

Todo corpo total ou parcialmente imerso em um fluido, recebe deste um empuxo


vertical dirigido para cima, de módulo igual ao peso do fluido deslocado pelo corpo.

Esse Princípio resume uma infinidade aspectos da influência de um líquido sobre


um corpo sólido que nele está imerso (ou parcialmente imerso).

Porque um pedaço de madeira flutua e uma pedra afunda? Porque um navio flutua,
mesmo sendo feito de ferro? Porque um submarino consegue ter controle sobre a escolha
da profundidade em que se encontra? Questões deste tipo são respondidas com a aplica-
ção do princípio de Arquimedes.

Fluidos ideais em movimento

O movimento de fluidos reais é complexo e ainda não é inteiramente compreendi-


do. Por exemplo, não existe uma compreensão clara sobre o fenômeno das turbulências.

Vamos restringir a nossa análise aos fluidos ideais. São aqueles que apresentam
um comportamento bem mais simples, e principalmente, sabemos analisar os seu movi-
mento. Um fluido ideal tem pelo menos as seguintes características:

Escoamento estacionário

A velocidade do fluido em qualquer ponto fixo não muda com o tempo. Neste tipo
de escoamento a velocidade de um elemento de volume do fluido pode variar enquanto
ele muda de posição, mas a velocidade do fluido em cada ponto do espaço permanece
constante ao longo do tempo.

Escoamento incompressível

A sua densidade é constante, independente das circunstâncias, como o aumento


de pressão ou temperatura.

Cap 15 www. f isica. uf pb. br / ~r omer o 4


www.cliqueapostilas.com.br

Pr of . Romer o T avar es da Silva

Escoamento não viscoso

Grosseiramente, a viscosidade de um fluido é uma medida da sua resistência ao


escoamento.

Escoamento irrotacional

Em um escoamento não - rotacional, um corpo não girará em torno d um eixo que


passe por seu centro de massa.

Vamos estudar o escoamento estacionário, incompressível, irrotacional e não - vis-


coso.

Linhas de corrente e a Equação da Continuidade

Uma linha de corrente é a trajetória de um elemento de volume do fluido. Enquanto


esse elemento de volume se move, ele pode variar a sua velocidade em módulo direção e
sentido. O vetor velocidade será sempre tangente á linha de corrente. Uma consequência
desta definição é que as linhas de corrente nunca se cruzam, pois caso o fizessem o ele-
mento de volume poderia ter uma das duas velocidades com diferentes direções, simulta-
neamente.

Em um escoamento podemos isolar tu-


bos de corrente, cujos limites são definidos por
linhas de corrente. Tal tubo funciona como um A2 , v2
cano, porque nenhuma partícula escapa atra- A1 , v1
vés de suas paredes - pois justamente essas
paredes definem as linhas de corrente.
B
Consideremos o tubo de corrente na figura C
ao lado, onde o fluido se move da esquerda
para a direita. O tubo tem seção transversal A1
e A2 nas posições indicadas e velocidades respectivas v1 e v2 .

Observemos durante um intervalo de tempo ∆t o fluido que cruza a área A1 . A


massa de fluido que atravessa essa superfície neste intervalo é dado por

∆m1 = ρ1 ∆V1 = ρ1 A1 ( v1 ∆t )

Como não existe fonte ou sorvedouro de massa entre A1 e A2 , essa mesma


massa de fluido atravessará a superfície A2 e será dado, nesse caso, por:

∆m2 = ρ2 ∆V2 = ρ2 A2 ( v2 ∆t )
onde concluímos que:
ρ1 A1 v1 = ρ2 A2 v2
ou seja:
ρ A v = constante

ao longo de um tubo de corrente. Algumas vezes a equação anterior é chamada de equa-


ção de continuidade para escoamento de fluidos.
Cap 15 www. f isica. uf pb. br / ~r omer o 5
www.cliqueapostilas.com.br

Pr of . Romer o T avar es da Silva

Como as linhas de corrente não se cruzam, elas se aproximam uma das outras à medida
que o tubo de corrente diminui a sua seção transversal. Desse modo o adensamento de
linhas de corrente significa o aumento da velocidade de escoamento.

A equação de Bernoulli

A equação de Bernoulli relaciona variação de pressão, variação de altura e variação


de velocidade em um fluido incompressível num escoamento estacionário. Ela é obtida
como uma consequência da conservação da energia.

Considere um tubo de largura variável por onde entra um fluido à esquerda e sai à
direita, como mostra a figura à seguir. À esquerda, o tubo tem seção transversal de área
A1 e à direita ele tem uma seção transversal de área A2 . À esquerda, parte inferior do
tubo está a uma certa altura y1 de um certo referencial e a parte superior do tubo à di-
reita está a uma altura y2 desse mesmo referencial.

Vamos considerar o movimento deste fluido que num dado instante ocupa o volume
entre os planos 1 e 2 na figura à seguir, e depois de um intervalo de tempo ∆t ele pas-
sa a ocupar o volume entre os planos 1´ e 2´ .

2 2´

p2A2

1 1´
p1A1 v2∆t
y2 y
v1∆t y1

O volume entre os planos 1 e 1´ é ∆V1 e o volume entre os planos 2 e 2´ é ∆V2


, onde temos que:
∆V1 = (v1 ∆t) . A1

∆V2 = (v2 ∆t) . A2

Considere um intervalo de tempo ∆t pequeno, tal que através da superfície A1


passe uma massa ∆m1 e através da superfície A2 passa uma massa ∆m2 . Essas
massas podem ser escritas como:

∆m1 = ρ1 ∆V1 = ρ1 [ (v1 ∆t ) A1 ]


e de modo semelhante:
∆m2 = ρ2 ∆V2 = ρ2 [ (v2 ∆t ) A2 ]

Cap 15 www. f isica. uf pb. br / ~r omer o 6


www.cliqueapostilas.com.br

Pr of . Romer o T avar es da Silva

Como a massa que entra pela esquerda deve ser igual à massa que sai à direita, temos
que
∆m1 = ∆m2

e como o fluido é considerado incompressível, a densidade à esquerda ρ1 é igual à


densidade ρ2 à direita, logo
ρ1 = ρ2
Desse modo:
∆m = ∆m1 = ∆m2

ρ = ρ1 = ρ2
ou seja:
v1 A1 = v2 A2

O trabalho W realizado pelas forças externas sobre o elemento de massa ∆m é


igual à variação da energia cinética dessa massa quando vai da esquerda para a direita.
Uma das forças externas a esse elemento de massa é a gravidade e a outra força é
uma consequência da diferença de pressão externa aplicada nas superfícies A1 e A2 .

W = WG + WP = ∆K

WG = trabalho realizado pela força da gravidade.

WP = trabalho ralizado como uma consequência da diferença de pressão externa.

2 ! !
WG = ∫ FG ⋅ dl
1

2 ! !
W P = ∫ FP ⋅ dl
1

( )( )
! !
FG ⋅ dl = − ˆj ∆m g ⋅ jˆ dy = − ∆m g dy

2
WG = ∫ (− ∆m g dy ) = −∆m g y = −∆m g (y 2 − y 1 )
y2

y1
1

Num intervalo de tempo ∆t , uma elemento de massa ∆m deixou a parte inferior


do tubo e passou para a parte superior. Logo, o sistema armazenou energia potencial
gravitacional
WG = - ∆m g ( y2 - y1 )
Por outro lado:

{( ) ( )} ( )
! !
FP ⋅ dl = kˆ p1 A1 + − kˆ p 2 A2 ⋅ kˆ dz = (p1 A1 )dz − (p 2 A2 )dz

2 2
W P = p1 A1 ∫ dz − p 2 A2 ∫ dz = p1 A1 ∆z1 − p 2 A2 ∆z 2 = p1 A1 (v 1 ∆t ) − p 2 A2 (v 2 ∆t )
1 1

Cap 15 www. f isica. uf pb. br / ~r omer o 7


www.cliqueapostilas.com.br

Pr of . Romer o T avar es da Silva

Mas
∆m
A(v ∆t ) = ∆V =
ρ
logo
∆m
WP = (p1 − p 2 )
ρ

A variação da energia cinética é dada por:

1 1
∆K = ∆m v 22 − ∆m v 12
2 2

Podemos então dizer que:

∆m
(p1 − p 2 ) − ∆m g (y 2 − y 1 ) = 1 ∆m v 22 − 1 ∆m v 12
ρ 2 2
ou ainda:
p1 − p 2 1
− g (y 2 − y 1 ) = (v 22 − v 12 )
ρ 2
ou seja:
1 1
p1 + ρ g y 1 + ρ v 12 = p 2 + ρ g y 2 + ρ v 22
2 2

de onde podemos concluir que:

1
p+ρg y + ρ v 2 = cons tan te
2

que é a equação de Bernoulli.

Cap 15 www. f isica. uf pb. br / ~r omer o 8


www.cliqueapostilas.com.br

Pr of . Romer o T avar es da Silva

O medidor de Venturi

O medidor de Venturi é um aparelho usado para medir a velocidade de escoa-


mento de um fluido de densidade ρF em um cano. O medidor é conectado entre duas
seções do cano como mostrado na figura à seguir.

A área A da seção transversal da entrada e da saída são iguais a área da seção


transversal do cano. Entre a entrada e a saída, o fluido passa por uma região estreita de
área a . Um manômetro que contém um líquido de densidade ρL conecta a parte mais
larga à parte mais estreita, onde a velocidade do fluido tem um valor V , que é maior que
a velocidade v na entrada do medidor.

! !
v 1 ; A1 v 1 ; A1
Cano ! Cano
v 2 ; A2
2
ρF
1 y2

4
y1 h
3

ρL

Vamos usar a equação de Bernoulli para analisar a variação das grandezas


envolvidas.
1
p + ρ g y + ρ v 2 = cons tan te
2

Aplicando essa equação para esse cano, nas regiões 1 e 2 , encontramos que:

1 1
p1 + ρ F v 12 + ρ F g (y 1 − h ) = p 2 + ρ F v 22 + ρ F g (y 2 − h )
2 2

onde estamos tomando como referencial da energia potencial gravitacional o ponto mais
alto do líquido dentro do manômetro, e desse modo podemos usar a Equação de bernoulli
apenas para o fluido do cano. Esta equação pode tomar a forma:

1 1
p1 + ρ F v 12 + ρ F g y 1 = p 2 + ρ F v 22 + ρ F g y 2
2 2
(p1 + ρ F g y 1 ) − (p 2 + ρ F g y 2 ) = 1 ρ F v 22 − 1 ρ F v 12
2 2

Cap 15 www. f isica. uf pb. br / ~r omer o 9


www.cliqueapostilas.com.br

Pr of . Romer o T avar es da Silva

No interior do manômetro, as pressões se equacionam do seguinte modo:


 p3 = p1 + ρ F g y 1

 p 4 = p 2 + ρ F g (y 2 − h )
 p = p + ρ gh
 3 4 L

Usando as duas primeiras equações na última, encontramos que:

(p 1 + ρ F g y 1 ) = [p 2 + ρ F g (y 2 − h )] + ρ L gh
ou seja:
(p1 + ρ F g y 1 ) − (p 2 + ρ F g y 2 ) = ρ L g h − ρ F g h = (ρ L − ρ F )g h

Identificando esta equação com a aplicação da equação de Bernoulli, encontramos


que:
1 1
ρ F v 22 − ρ F v 12 = (ρ L − ρ F )g h
2 2
ou seja:
2 (ρ L − ρ F ) g h
v 22 − v 12 =
ρF

À partir da equação da continuidade, encontramos que:

ρL v1 A1 = ρL v2 A2
ou seja:
A1
v 2 = v1
A2
e desse modo
 A12 − A22  2 (ρ L − ρ F ) g h
v − v = v 
2
2
2
1
2
1
 =
 A2
2
 ρF
e finalmente:
2 A22 (ρ L − ρ F ) g h
v1 =
(A12 − A22 )ρ F
e portanto podemos medir a velocidade v1 do fluido ao entrar no cano.

Cap 15 www. f isica. uf pb. br / ~r omer o 10


www.cliqueapostilas.com.br

Pr of . Romer o T avar es da Silva

Solução de alguns problemas

Capítulo 15 - Halliday, Resnick e Walker - 6a. edição

01 Encontre o aumento de pressão de um fluido em uma seringa quando uma enfermei-


ra aplica uma força de 42N ao êmbolo da seringa, de raio 1,1cm .

F = 42N p0
r = 1,1cm = 0,011m

F F
∆p = = = 110.487,7N/m2
A πr2 p0 +∆p

1N/m2 = 1 Pascal

1atm = 1,013x105 Pa
logo
∆p = 1,08atm

Capítulo 15 - Halliday, Resnick e Walker - 6a. edição

05 Um peixe controla a sua profundidade na água através do ajuste do conteúdo de ar


de um osso poroso ou em um saco de ar para que a sua densidade fique igual à da
água. Suponha que, com as bolsas de ar vazias, um peixe tenha a densidade de
1,08g/cm3 . Se ele quiser reduzir a sua densidade à da água, que fração do volume
do seu corpo deverá ser ocupada por ar dentro dos sacos? (Estes sacos são chama-
dos bexigas natatórias.

ρI = 1,08g/cm3
ρF = 1g/cm3
A densidade do peixe varia de ρI até ρF :

 M
 ρI = P
VI


 M P + M AR M P
ρ F = ≅
 V F VF
Na definição de ρF levamos em consideração que a massa de ar é muito
menor que a massa do peixe.

A razão entre os volumes tem a forma:


MP
VF ρ ρ
= F = I
VI MP ρF
ρI

Cap 15 www. f isica. uf pb. br / ~r omer o 11


www.cliqueapostilas.com.br

Pr of . Romer o T avar es da Silva

Mas
VF = VI + VAR
logo:
VI + V AR VF ρ V V AR ρ
= = I = 1 + AR ⇒ = I −1
VI VI ρF VI VI ρF

V AR
= 0,08
VI

Capítulo 15 - Halliday, Resnick e Walker - 6a. edição

07 Em 1654, Otto von Guericke, burgomestre de Magdeburg e inventor da bomba de ar,


deu uma demonstração diante da Dieta Imperial em que dois grupos de oito cavalos
não foram capazes de separar dois hemisférios de latão unidos, dentro dos quais se
fez vácuo.

a) Pressupondo que os dois hemisférios tenham paredes finas, de forma que R , na


figura à seguir, possa ser considerado o raio interno e externo, mostre que a for-
ça F necessária para separar os hemisférios é F = πR2 ∆p onde ∆p é a dife-
rença entre as pressões interna e externa na esfera.

A atmosfera exerce uma pressão (e


consequentemente um força) em todos ! !
os pontos dos dois hemisférios, mas F0 F0
apenas a componente z dessa força
"empurra" um hemisfério contra o outro.
As componentes x e y dessa força
são nulas.
!
Isso pode ser percebido se observar- dF
mos
! que para cada elemento de força θ
dF existe atuando um outro elemento dFz
!
dF ′ simétrico em relação ao eixo z .
! z
As componentes x e y de dF ′ anu-
larão
! as componentes equivalentes de
dF . No entanto, somar-se-ão as com-
ponentes z dessas forças elementares
simétricas. !
! dF
dF é um vetor radial, ou seja:
! θ
dF = −rˆ dF
z
As suas componentes cartesianas são:

dFX = - dF senθ cosϕ

Cap 15 www. f isica. uf pb. br / ~r omer o 12


www.cliqueapostilas.com.br

Pr of . Romer o T avar es da Silva

dFY = - dF senθ senϕ

dFZ = - dF cosθ

Considerando que:

dF = p0 dA = p0 (R2 senθ dθ dϕ)


teremos que:
dFX = - p0 R2 (sen2θ dθ) (cosϕ dϕ)

dFY = - p0 R2 (sen2θ dθ) (senϕ dϕ)

dFZ = - p0 R2 (senθ cosθ dθ) (dϕ)


Integrando, teremos:
π
2 2π
FX = ∫ dF X = − p 0 R 2 ∫ sen 2 θ dθ ∫ cos ϕ dϕ
0 0

π
2 2π
FY = ∫ dFY = − p 0 R ∫ sen 2 θ dθ ∫ sen ϕ dϕ
2

0 0

π
2 2π
FZ = ∫ dFZ = − p 0 R 2 ∫ sen θ cos θ dθ ∫ dϕ
0 0

Mas por outro lado:


 2π

 ∫ cos ϕ dϕ = sen ϕ 0 = 0
0
2π 2π
 ∫ sen ϕ dϕ = −cos ϕ 0 = 1 − 1 = 0
0 2π
 ∫ dϕ = 2π
 0

logo:
FX = FY = 0
e
π
2
FZ = 2πR p 0 ∫ sen θ cos θ dθ
2

Fazendo a substituição u = senθ , encontramos que

1
1
FZ = 2πR 2 p 0 ∫ u du = 2πR 2 p 0
0 2

Como FZ é a força resultante externa, vamos chamá-la de F0 , ou seja:

F0 = π R2 p0

Cap 15 www. f isica. uf pb. br / ~r omer o 13


www.cliqueapostilas.com.br

Pr of . Romer o T avar es da Silva

A força líquida F é a diferença entre as forças internas e externas:

F= F0 - F1 = πR2(p0 - p1) = πR2∆p

b) Fazendo R = 30cm e a pressão interna igual a 0,10atm , encontre a força que


os cavalos teriam de exercer para separar os hemisférios.

R = 30cm = 0,30m
p0 = 1atm = 1,013x105Pascal
p1 = 0,1atm = 1,013x104Pascal
∆p= p0 - p1 = 0,9atm = 91.170Pa

F = 25.777,7 Newtons

Capítulo 15 - Halliday, Resnick e Walker - 6a. edição

11 Uma piscina tem as dimensões 24m x 9m x 2,5m .

a) Quando ela está cheia de água, qual é força (devido somente à água) sobre o
fundo, nas extremidades e nos lados?

H = 2,5m
L = 9m
C = 24m

A pressão no fundo da piscina é dada


por: H C
P=ρgH

Logo, a força total no fundo será:


L
F = P A = (ρ g H) (L C)
h=0
F=ρgV h
dh
F = (103 kg/m3)(10m/s2)(2,5 . 9 . 24 m3)
h=H
6
F = 5,4 x 10 N
L
A pressão a uma profundidade genérica h é dada por:

P=ρgh

A força lateral em uma superfície dA ao longo desta profundidade e associada a


essa pressão tem a forma:

dFL = P dA = P (L dh) = ρ g L h dh

Cap 15 www. f isica. uf pb. br / ~r omer o 14


www.cliqueapostilas.com.br

Pr of . Romer o T avar es da Silva

e portanto, a força lateral é dada por:

H
ρ g LH 2
FL = ∫ ρ g L h dh =
0 2

FL = 2,8 x 105 N

Como temos duas superfícies laterais iguais:

2 FL = 5,6 x 105 N

A força ao longo do comprimento é dada por:

H
ρ g CH2
FC = ∫ ρ g C h dh =
0 2

FC = 7,4 x 105 N

Como temos duas superfícies laterais iguais:

2 FC = 1,4 x 106 N

b) Se você estiver preocupado com o fato das paredes e pisos de concreto se que-
brarem, seria apropriado levar em conta a pressão atmosférica? Porque?

Sim, por causa do princípio de Pascal. A pressão que a atmosfera exerce na su-
perfície se transmite para todos os pontos da água, inclusive os lados e o fundo.

Capítulo 15 - Halliday, Resnick e Walker - 6a. edição

12 a) Encontre o peso total da água em cima de um submarino nuclear, a uma profun-


didade de 200m , supondo que o seu casco (corte da seção transversal) tenha a
área de 3000m2 .
Submarino
2
A = 3000m
h = 200m
ρS = 1,03g/cm3 = densidade da água do
mar
A = Seção transversal do submarino
p = ρS g h

F = p A = ρS g h A

F = (1,03x103kg/m3)(10m/s2)(200m)(3000m2)

F = 6,16 x 109N

Cap 15 www. f isica. uf pb. br / ~r omer o 15


www.cliqueapostilas.com.br

Pr of . Romer o T avar es da Silva

b) A que pressão da água um mergulhador estaria submetido a essa profundidade?


Você acha que os ocupantes de um submarino danificado, a essa profundidade
poderiam escapar sem equipamento especial? Considere a densidade da água
do mar 1,03g/cm3 .

p = p 0 + ρS g h

p = (1,01x105Pa) + (1,03x103kg/m3)(10m/s2)(200m)

p = (1,01x105Pa) + (2,06x106Pa)

p = 2,1 x 106 N = 2,08 atm

Capítulo 15 - Halliday, Resnick e Walker - 6a. edição

15 Dois vasos cilíndricos idênticos, com suas bases ao mesmo nível contém um líquido
de densidade ρ . A área da base é A para ambos, mas em um dos vasos a altura
do líquido é h1 e no outro é h2 . Encontre o trabalho realizado pela força gravitacio-
nal ao igualar os níveis, quando os dois vasos são conectados.

Seja U(H) a energia potencial gravitacional


armazenada num recipiente de área transver-
sal A e altura H . h=H

A faixa de líquido a uma altura h , com uma


espessura dh , tem uma energia potencial h
gravitacional dada por:

dU = dm g h = (ρ dV) g h = (ρ A dh) g h h=0

ou seja:
dU = ρ A g h dh
e portanto:
H
H2
U (H ) = ρ A g ∫ h dh = ρ g A
0 2

Considerando a situação inicial, quando temos dois vasos que se comunicam, a


energia potencial gravitacional inicial do conjunto será:

h12 h22
U I = U (h1 ) + U (h2 ) = ρAg + ρAg
2 2
Ou seja:
ρAg 2
UI = (h1 + h22 )
2

Depois que os vasos são conectados, os seus níveis alcançam uma altura h de
equilíbrio. Como não existem perdas, a soma dos volumes dos líquidos dos dois tan-
ques permanece constante, logo:

Cap 15 www. f isica. uf pb. br / ~r omer o 16


www.cliqueapostilas.com.br

Pr of . Romer o T avar es da Silva

h1 A + h2 A = 2 h A
ou seja:
h1 + h2
h=
2

A energia potencial gravitacional final do conjunto será:

UF = U(h) + U(h) = 2 U(h)


ou seja:
 ρAgh 2   ρAg  h1 + h2  2   h1 + h2 
2

UF = 2   = 2  2  2   = ρAg  2 
 2       

ρAg 2
∆U = U F − U I = [(h1 + h22 + 2h1h2 ) − 2(h12 + h22 )]
4

ρAg
∆U = {− h12 − h22 + 2h1h2 }
4

ρAg
∆U = − (h2 − h1 )2
4
Mas
ρAg
W = −∆U = (h2 − h1 )2
4

Capítulo 15 - Halliday, Resnick e Walker - 6a. edição

19 A água se encontra a uma profundidade D abaixo da face vertical de um dique, com


ilustra a figura à seguir.

a) Encontre a força horizontal resultante exercida no dique pela pressão manométri-


ca da água.

Vamos considerar a força elementar


dA exerci sobre o dique por uma lâ- W
mina de líquido represado. Essa lâ-
mina está a uma profundidade h e
nessa profundidade existe uma pres-
são p exercida pelo líquido . Desse D
modo:

dF = p dA = p W dh

onde W é a largura do dique e dh é O


a espessura da lâmina.

Cap 15 www. f isica. uf pb. br / ~r omer o 17


www.cliqueapostilas.com.br

Pr of . Romer o T avar es da Silva

Mas
p=ρgh
logo W
dF = ρ g W h dh

e a força resultante terá a forma: dh

D
ρ g W D2 dA
F = ρgW ∫ h dh =
0 2

b) Encontre o torque resultante devido à pressão manométrica da água, em relação


ao ponto O .

O torque que a lâmina


exerce no dique, em
ralação ao ponto O é !
dado por: h dF
! ! !
dτ = r × dF D
!
r
ou seja:
O
dτ = (D - h) dF

ou ainda:
dτ = (D - h) {ρ g W h dh} = ρ g W (D - h) dh

e integrando, temos

D D D
τ = ρgW ∫ (D − h )h dh = ρgW D ∫ h dh − ∫ h 2 dh 
0  0 0 

 D 2 D 3  ρgWD 3
τ = ρgW D − =
 2 3  6

c) Encontre o braço de alavanca, em relação ao ponto O , da força horizontal re-


sultante sobre o dique.

ρgWD 3  D2 
τ = FL ⇒ 
=  ρgW  L
6  2 
ou seja:
D
L =
3

onde L é medido à partir do fundo do dique.

Cap 15 www. f isica. uf pb. br / ~r omer o 18


www.cliqueapostilas.com.br

Pr of . Romer o T avar es da Silva

Capítulo 15 - Halliday, Resnick e Walker - 6a. edição

22 Um pistom de área menor a é usado em uma prensa hidráulica para exercer uma
pequena força f num líquido confinado. Um tubo o conecta com um outro pistom
maior de área A .

a) Que força F o pistom maior sustentará?


!
Usando o princípio de Pascal, a força F
aplicada f produz no líquido uma varia- !
f
ção de pressão dada por:
a A
f F  A
∆p = = ⇒ F =  f
a A a

Se o pistom da menor se mover de d , o


pistom maior mover-se-á de D , mas os
volumes associados a esses movimen-
tos serão os mesmos. Ou seja:
a
V = ad = AD ⇒ D= d
 A
O trabalho Wf executado pela força f será:

 a  A 
W f = fd =  F  D  = FD = W F
 A  a 

e portanto as duas forças fazem o mesmo trabalho.

b) Se o pistom pequeno tem um diâmetro de l = 3,8cm e o grande de L = 53cm ,


que peso no pistom pequeno sustentará 2 toneladas no pistom maior?
2
L
π  2
A  2 L
F =f =f 2
=f 
a l l
π 
2
Como F = M g e f = m g , temos que:
2
l
m = M   = 10,28kg
L

Capítulo 15 - Halliday, Resnick e Walker - 6a. edição

26 Um objeto cúbico de dimensão L = 0,6m de lado e massa M = 450kg é suspenso


por um fio em um tanque aberto com líquido de densidade ρ = 1030kg/m3 .

a) Encontre a força total para baixo, exercida pelo líquido e pela atmosfera sobre o
objeto.

Cap 15 www. f isica. uf pb. br / ~r omer o 19


www.cliqueapostilas.com.br

Pr of . Romer o T avar es da Silva

L = 0,6m
M = 450kg
ρ = 1030kg/m3
p0 = 1atm = 1,013x105Pascal
L/2
A força total FS exercida pelo líquido
na parte superior do objeto é: L

 L
FS = p S A =  p 0 + ρg  L2
 2

FS = 37.580,4N

b) Encontre a força total para cima, na base do objeto.


! !
 3L  2 FS T
FI = p I A =  p 0 + ρg L !
 2  E

FI = 39.805,2N
!
c) Encontre a tensão no fio. ! P
FI
T = P + FS - FI

 L  3L  2
T = Mg +  p 0 + ρg  L2 −  p 0 + ρg  L = Mg − ρ L g
3

 2  2 

T = 450.10 - 39.805,2 + 37.580,4 = 4500 - 2.224,8

T = 2.275,2N

d) Calcule o empuxo sobre o objeto, usando o Princípio de Arquimedes.

E = (ρ V) g = ρ L3 g = (1030kg/m3) (0,6m)3 (10m/s2)

E = 2.224,8N

e) Qual a relação existente entre todas essas quantidades?

E = FI - FS
! ! !
P +T + E = 0

Capítulo 15 - Halliday, Resnick e Walker - 6a. edição

27 Um bloco de madeira flutua em água com dois terços do seu volume submerso. Em
óleo, flutua com 0,90 do seu volume submerso.

Cap 15 www. f isica. uf pb. br / ~r omer o 20


www.cliqueapostilas.com.br

Pr of . Romer o T avar es da Silva

a) Encontre a densidade da madeira.


!
O empuxo é proporcional ao volume do corpo E
que está submerso, porque é esse volume que
desloca o líquido.
Como o corpo está flutuando, esse empuxo é
igual ao seu peso. Considerando inicialmente o !
P
corpo de madeira flutuando na água:

EA = PM

 2V  2
 ρA  g = (ρ MV )g ⇒ ρM = ρA
 3  3

Como a densidade da água ρA = 1g/cm3 , encontramos que:

2
ρM = g / cm 3 = 666,7kg/m3
3

b) Encontre a densidade do óleo.

EO = PM

ρ M 10 20
[ρ (0,9V )]g = (ρ V )g
O M ⇒ ρO = = ρM = g / cm 3
0,9 9 27

ρO = 740,7kg/m3

Capítulo 15 - Halliday, Resnick e Walker - 6a. edição

29 Uma esfera oca, de raio interno igual a 8cm e raio externo igual a 9cm , flutua sub-
mersa pela metade em um líquido de densidade 800kg/m3 .

a) Qual a massa da esfera?

RI = 8cm = 0,08m
RE = 9cm = 0,09m
ρL = 800kg/m3

Quando a esfera flutua, temos que:

P=E
ou seja
 V 
ME g =  ρL E  g
 2 
logo:
1 4
ME = ρ L πR E3 ⇒ M E = 1,22kg
2 3
Cap 15 www. f isica. uf pb. br / ~r omer o 21
www.cliqueapostilas.com.br

Pr of . Romer o T avar es da Silva

b) Calcule a densidade do material de que ele é feita.

ME ME ME
ρE = = =
V V E − VI 4
π (R E3 − R I3 )
3

ρE = 1342,18kg/m3

Capítulo 15 - Halliday, Resnick e Walker - 4a. edição

31 Uma lata tem volume de 1200cm3 e massa de 130g . Quantos gramas de balas de
chumbo ela poderia carregar sem que afundasse na água? A densidade do chumbo
é 11,4g/cm3 .

V = 1200cm3
ML = 130g
ρPb = 11,4g/cm3
ρA = 1g/cm3 (densidade da água)

A lata tem um volume interno V e está flutuan-


do. Que massa MPb de chumbo pode ser colo-
cada em seu interior? O peso total da lata mais
balas de chumbo tem de ser igual ao empuxo
exercido pela água na lata. Ou seja:

(MPb + ML) g = E

Usando o Princípio de Arquimedes, o empuxo será igual ao volume do fluido deslo-


cado, logo:
E = (ρA V) g ⇒ (MPb + ML) g = (ρA V) g
ou seja:
MPb = ρA V - ML = 1200g - 130g

MPb = 1070g

Capítulo 15 - Halliday, Resnick e Walker - 6a. edição

36 Três crianças, cada uma pesando 356N , constroem uma jangada amarrando tron-
cos de diâmetro 0,30m e comprimento 1,80m . Quantos troncos serão necessários
para que a jangada as sustente? Considere a densidade da madeira como sendo
800kg/m3 .

P = 356N ρM = 800kg/m3
d = 0,30m ρA = 1000kg/m3
L = 1,80m

Seja VT o volume de cada tronco. Desse modo:

Cap 15 www. f isica. uf pb. br / ~r omer o 22


www.cliqueapostilas.com.br

Pr of . Romer o T avar es da Silva


2
d 
VT = π   L = 0,12m 3
2

Como a jangada será construída com N troncos, o volume V da jangada será:

V = N VT

Para que a jangada flutue com carga máxima, vamos considerar que ela ficará com-
pletamente submersa. Neste caso, o empuxo será:

E = (ρA V) g

E a jangada suportará o seu próprio peso mais o peso das crianças:

(ρA V) g = (ρM V) g + 3P
ou seja:
ρM 3P 3P
V = V+ ⇒ V =
ρA ρ Ag g (ρ A − ρ M )
Mas
3P
V = NVT ⇒ N=
VI g (ρ A − ρ M )

N = 4,45

Será necessário um número de toras maior que quatro. Supondo que a jangada será
construída com um número inteiro de toras, serão necessários cinco troncos para a
construção da jangada.

Capítulo 15 - Halliday, Resnick e Walker - 6a. edição

47 Um tanque de grande área é cheio de água a uma profundidade de 0,30m . Um bu-


raco de área A = 6,5cm2 no fundo do tanque permite que a água escoe.

a) A que taxa a água flui pelo buraco?

D = 0,30m
A = 6,5cm2 = 6,5x10-4m2 1

Vamos usar a Equação de Bernoulli: D

1
p+ρg y + ρ v 2 = cons tan te 2
2
nos pontos 1 na superfície da água h
dentro do tanque e o ponto 2 no bura- 3
co no fundo do tanque:

Cap 15 www. f isica. uf pb. br / ~r omer o 23


www.cliqueapostilas.com.br

Pr of . Romer o T avar es da Silva

1 1
p1 + ρ g y 1 + ρ v 12 = p 2 + ρ g y 2 + ρ v 22
2 2

Considerando que o buraco é pequeno em comparação à superfície da água


dentro do tanque, podemos dizer, com boa aproximação, que a velocidade que o
nível da água baixa v1 é desprezível. Ainda considerando que o buraco é pe-
queno, podemos considerar que o nível D da água varia muito pouco, e desse
modo:
D = y1 - y2
portanto:
1
p1 + ρ g D = p 2 + ρ v 22
2

Os pontos 1 e 2 estão em contato com a atmosfera, logo:

p1 = p2 = p0
logo:
1
ρg D = ρ v 22 ⇒ v 2 = 2gD = 2,4m/s
2

O fluxo de água é definido como


φ=vA
ou seja:
φ = A 2gD = 1,58m3/s

b) A que distância abaixo do fundo do tanque, a área da seção transversal do jato


será a metade da área do buraco?

A água vai fluir através do buraco e formar um tubo de corrente. Podemos usar a
equação da continuidade para calcular a velocidade quando a seção transversal
do tubo de corrente tiver a metade do valor original.

v2 A = v3 (A/2) ⇒ v3 = 2 v2 = 4,8m/s

Vamos usar a equação de Torricelli para calcular a altura h , abaixo do fundo do


tanque, em que acontece essa relação de áreas; já que a água está em queda
livre.
v 32 − v 22
h= = 0,86m
2g

Capítulo 15 - Halliday, Resnick e Walker - 6a. edição

48 Sobre a asa de um avião de área A , o ar escoa com velocidade vC e sob a asa


deste mesmo avião (também de área A) , a velocidade do ar é vB . Mostre que nesta
situação simplificada, a equação de Bernoulli prediz que a magnitude L da força de
sustentação na asa será:
1
L = ρA (v C2 − v B2 )
2
onde ρ é a densidade do ar.
Cap 15 www. f isica. uf pb. br / ~r omer o 24
www.cliqueapostilas.com.br

Pr of . Romer o T avar es da Silva

O fluxo de ar em torno da asa de um


avião tem qualitativamente a forma
desenhada ao lado. Devido ao seu
formato, existe um adensamento das
linhas de corrente acima da asa, e
portanto a velocidade nesta região é
maior que a velocidade abaixo da asa.

Usando a equação de Bernoulli, iremos calcular quais as consequências deste dese-


nho peculiar de uma asa no que diz respeito à força de sustentação de um avião:

1
p+ρg y + ρ v 2 = cons tan te
2

Aplicando essa equação para em ponto na parte superior da asa e para um outro
ponto na sua parte inferior:

1 1
pC + ρ g y C + ρ v C2 = p B + ρ g y B + ρ v B2
2 2
ou seja:

1
∆p = p B − pC = ρ (v C2 − v B2 ) + ρg (y C − y B )
2

Como a diferença de energia potencial gravitacional é desprezível frente a outras di-


ferenças de energia presentes na equação, podemos escrever que:

L 1 1
∆p = = ρ (v C2 − v B2 ) ⇒ L= ρA(v C2 − v B2 )
A 2 2

Capítulo 15 - Halliday, Resnick e Walker - 4a. edição

“49” Coloca-se um béquer de vidro, parcialmente cheio de água, em uma pia, conforme
a figura à seguir. Ele tem massa de 390g e um volume interno de 500cm3 . Come-
ça-se, então, a encher a pia com água e verifica-se por experiência que, se o bé-
quer estiver com água até menos da metade, flutuará; mas se a água nele estiver
acima da metade, permanecerá no fundo da pia até a água alcançar as suas bor-
das. Qual a densidade do material de que é feito o béquer?

MB = 390g = 0,39kg
VI = 500cm3 = 0,0005m3

Vamos considerar o caso limite, onde o


nível da água da pia atingiu a borda do
béquer, que tem metade do volume in-
terno ocupado com água.

O peso do conjunto água + béquer


será:
Cap 15 www. f isica. uf pb. br / ~r omer o 25
www.cliqueapostilas.com.br

Pr of . Romer o T avar es da Silva

P = (MA + MB) g = (ρA VI /2 + MB) g

O empuxo será
E = (ρA VE) g

onde VE é o volume externo do béquer. Além disso, a densidade do béquer será


dada por:
MB
ρB =
VE − VI

No caso limite, o empuxo E será igual ao peso P , e portanto teremos:

(ρA VE) g = (ρA VI /2 + MB) g

VI
MB + ρ A
VE = 2 = M B + VI
ρA ρA 2
Mas
MB MB
ρB = ⇒ VE = + VI
V E − VI ρB
ou seja:
M B VI M B
VE = + = + VI
ρA 2 ρB
ou ainda:
M B M B VI MB
= + ⇒ ρB = = 2,79g/cm3
ρA ρB 2 M B VI

ρA 2

Capítulo 15 - Halliday, Resnick e Walker - 6a. edição

49 Se a velocidade de escoamento, passando debaixo de uma asa, é 110m/s , que ve-


locidade de escoamento na parte de cima criará uma diferença de pressão de 900Pa
entre as superfícies de cima e de baixo? Considere a densidade do ar
ρ = 1,3x10-3g/cm3

vB = 110m/s = 396km/h
∆p = 900Pa = 0,00888atm
ρ = 1,3x10-3g/cm3 = 1,3kg/m3
1atm = 1,013x105Pa

1 2 ∆p
∆p = ρ (v C2 − v B2 ) ⇒ v C2 = v B2 +
2 ρ

vC = 116,1m/s = 417,9km/h

Cap 15 www. f isica. uf pb. br / ~r omer o 26


www.cliqueapostilas.com.br

Pr of . Romer o T avar es da Silva

Se cada asa tiver dimensões aproximadas de A = 0,5m x 3m = 1,5m2 , as duas asas


corresponderão a uma área de 3m3 . A força de sustentação, neste caso, será:

L = A ∆p = 2.700N

Capítulo 15 - Halliday, Resnick e Walker - 6a. edição

50 Suponha que dois tanques , 1 e 2 , cada um com uma grande abertura na parte de
cima, contenham dois líquidos diferentes. Um pequeno furo é feito nos dois tanques,
a uma mesma profundidade h abaixo da superfície do líquido, mas o furo no tanque
1 tem a metade da área de seção transversal do furo no tanque 2 .

a) Qual a razão ρ1 /ρ2 das densidades dos fluidos, se for observado que a vazão
de massa é a mesma nos dois furos?

m=ρV 1 2
∆m = ρ ∆V
∆m1 = ρ A ∆y1 a1 h a2
∆m2 = ρ A ∆y2

∆m é a variação de massa no
tanque quando o seu volume
varia de ∆V e o nível do líquido
varia de ∆y .
Para um intervalo de tempo ∆t temos que

∆m ∆y
= ρA
∆t ∆t
e no limite em que ∆t → 0
dm dy dm
= ρA ⇒ = ρAv S
dt dt dt

onde vS é a velocidade com que o nível da água diminui. Se considerarmos os


dois tanques, teremos que:

 dm1
 dt = ρ 1 A1v S1


 dm 2
 = ρ 2 A2 v S 2
 dt

Mas, neste problema, se observa que a vazão de massa é a mesma nos dois
furos, logo:
dm1 dm 2
= ⇒ ρ 1 A1v S1 = ρ 2 A2 v S 2
dt dt

Cap 15 www. f isica. uf pb. br / ~r omer o 27


www.cliqueapostilas.com.br

Pr of . Romer o T avar es da Silva

Quando consideramos que v é a velocidade com que o líquido flui através do


orifício de área a , podemos usar a equação da continuidade para concluir que:

ρAv S = ρav

Se usarmos esse resultado para cada um dos tanques, encontramos que:

 ρ 1 A1v S1 = ρ 1a1v 1


ρ A v = ρ a v
 2 2 S2 2 2 2

usando a igualdade da vazão das massas, temos:

ρ1 a1 v1 = ρ2 a2 v2 (1)

Aplicando a equação de Bernoulli para o tanque 1 , considerando a superfície e


um ponto do orifício, temos que:

1 1
pS1 + ρ 1v S21 + ρ 1gh = p1 + ρ 1v 12
2 2

e levando em conta que a pressão pS1 na superfície é a mesma pressão p1 em


um ponto do orifício, temos que:

v 12 = v S21 + 2gh

Como a lâmina do líquido é muito grande, ou seja A >> a , a velocidade vS1 que
o nível do líquido diminui é muito menos que a velocidade v1 desse líquido es-
capando pelo orifício, logo:
v 1 = 2gh (2)

Toda essa argumentação anterior é válida para o tanque 2 , e portanto:

v 2 = 2gh (3)

Usando as equações (2) e (3) na equação (1) , encontramos que:

ρ 1 a 2v 2 ρ1 a 2gh ρ1
= ∴ = 2 ⇒ =2
ρ2 a1v 1 ρ 2 a2 2gh ρ2
2

b) Qual é a razão entre as vazões dos dois tanques?

R = A v = vazão
Logo:
R1 av 1
= 1 1 =
R 2 a 2v 2 2

Cap 15 www. f isica. uf pb. br / ~r omer o 28


www.cliqueapostilas.com.br

Pr of . Romer o T avar es da Silva

c) Até que altura acima do furo se deve adicionar ou retirar líquido do tanque 2 ,
para igualar as vazões?

Vamos considerar que os furos agora estão em profundidades diferentes, logo

 v 1 = 2gH 1


v = 2gH
 2 2

R1 av a 2 2gH 1 H1
= 1 1 = 2 =
R 2 a 2v 2 a2 2gH 2 4H 2

Quando as vazões forem iguais, teremos:

R1 = R2 ⇒ H1 = 4 H2

Capítulo 15 - Halliday, Resnick e Walker - 6a. edição

53 A profundidade da água doce em repouso atrás de um dique é de 15m . Um tubo


horizontal de 4cm de diâmetro passa através do dique 6m abaixo da superfície da
água, como mostra a figura à seguir. Uma rolha fecha a abertura do tubo.

a) Encontre a força de atrito entre a rolha e as paredes do tubo.


1
H = 15m 3
h = 6m h
d = 4cm = 0,04m
H
Seja 1 um ponto no interior do dique e 2
próximo à rolha; e seja 2 um ponto no
exterior do dique e próximo à rolha.

Como os pontos não fazem parte de uma mesmo fluido, usando a hidrostática
nós temos então que:
 p1 = p 0 + ρ g h 
 
  ⇒ ∆p = p1 − p 2 = ρ g h
 p2 = p0 
 

Essa é a diferença de pressão que o atrito entre a rolha e as paredes do tubo têm
de suportar. Logo a força de atrito será:

F = A ∆p = ρ g h A = 73,89N

b) A rolha é removida. Que volume de água flui através do tubo em 3h ?

Cap 15 www. f isica. uf pb. br / ~r omer o 29


www.cliqueapostilas.com.br

Pr of . Romer o T avar es da Silva

Seja dV o elemento de volume que flui através do orifício, em um intervalo de


tempo dt . temos então que:
dV = (v dt) A

Considerando que a velocidade com que o a água fluirá será constante, tendo
em vista o volume do dique em comparação com o tamanho do orifício, temos
que:
V=vtA

Vamos relacionar um ponto da superfície da água do dique (3) com um ponto na


saída do tubo horizontal (2) .
1 1
p 3 + ρ g y 3 + ρ v 32 = p 2 + ρ g y 2 + ρ v 22
2 2

Considerando que a área transversal do tubo é muito menor que a lâmina d’água
do dique, usando a equação da continuidade, podemos aproximar que a veloci-
dade que o nível da água do dique vai baixar com uma velocidade muito menor
que a velocidade do fluxo d’água no tubo. Desse modo, temos que v3 ≈ 0

1
p3 + ρ g y 3 = p2 + ρ g y 2 + ρ v 22
2

Considerando que p3 = p2 = p0

1 2
ρv 2 = ρg (y 3 − y 2 ) = ρgh ⇒ v 2 = 2gh
2

O volume que fluirá será dado por:

V = tA 2gh = 147,17m3

Capítulo 15 - Halliday, Resnick e Walker - 6a. edição


Um tubo de Pitot, como esquematizado na figura à seguir, é usado para determinar a
57 velocidade de um avião em relação ao ar. Consiste em um tubo externo com um nú-
mero de pequenos furos B (são mostrados quatro na figura); o tubo é conectado a
um dos braços de um outro tubo em U , cujo segundo braço está conectado a um
buraco, A , na parte frontal do aparelho, que se alinha com a direção de vôo do avião.
Em A , o ar fica parado, logo vA = 0 . Em B , entretanto, a velocidade do ar presu-
midamente se iguala à velocidade do avião relativa ao ar. Use a equação de Bernoulli
para mostrar que
2ρ g h
v=
ρ AR

onde v é a velocidade do avião em relação ao ar e ρ é a densidade do líquido den-


tro do tubo em U .

Cap 15 www. f isica. uf pb. br / ~r omer o 30


www.cliqueapostilas.com.br

Pr of . Romer o T avar es da Silva

Considerando a diferença de
pressão entre os dois níveis do
líquido dentro do tubo em U ,
temos que:

p2 = p1 + ρ g h p2
p1
Mas, usando a equação de
Bernoulli, encontramos que:
1
pA + ρ AR v 2 = p B
2
Se
 p A ≈ p1 
 
  ⇒ ∆p = p B − p A = p 2 − p1
p ≈ p 
 B 2

ou seja:
1 2 ρgh
∆p = ρ AR v 2 = ρgh ∴ v =
2 ρ AR

Capítulo 15 - Halliday, Resnick e Walker - 6a. edição - Suplemento

68 Um sifão é um aparelho usado para remover líquido de um recipiente. Seu funciona-


mento é mostrado na figura à seguir. O tubo ABC necessita estar inicialmente cheio,
mas uma vez que isso tenha sido feito, o líquido fluirá através do tubo até que o nível
do líquido no recipiente esteja abaixo da abertura A . O líquido tem densidade ρ e
viscosidade desprezível.

a) Com que velocidade o líquido sai do tubo em C ?


B
A equação de Bernoulli tem a forma:
1 h1
p + ρ g y + ρ v 2 = cons tan te
2
D
Usando essa equação entre um ponto na saída d
do sifão C e um ponto na superfície do líquido
D , temos que: A

1 1
p D + ρg (d + h2 ) + ρ v D2 = pC + ρ v C2 h2
2 2

Supondo que a superfície do líquido tem uma


área muito maior que a seção transversal do si-
C
fão, podemos considerar que a velocidade com

Cap 15 www. f isica. uf pb. br / ~r omer o 31


www.cliqueapostilas.com.br

Pr of . Romer o T avar es da Silva

que a superfície do líquido varia(baixa) é desprezível frente a velocidade com que


o líquido entra no sifão. desse modo vD ≅ 0 , e portanto

1
ρ v C2 = p D − pC + ρ g (d + h2 )
2

Mas, tanto o ponto C quanto o ponto D estão em contato com a atmosfera em


repouso, e portanto estão a uma mesma pressão p0 , e desse modo pD = pC =
p0, logo:
v C = 2g (d + h2 )

b) Qual a pressão do líquido no ponto mais alto B ?

Usando a equação de Bernoulli para equacionar as grandezas dos pontos B e


D , encontramos:

1 1
pD + ρ v D2 + ρ g (d + h2 ) = p B + ρ v B2 + ρ g (d + h1 + h2 )
2 2
ou seja:
1
pD = pB + ρ v B2 + ρgh1
2

Usando a equação da continuidade entre os pontos B e C , encontramos que:

ρ v A = constante ⇒ vB = v C = 2g (d + h2 )
e portanto:
1 2 1
p B = p D − ρgh1 − ρv B = p D − ρgh1 − ρ [2g (d + h2 )]
2 2
Como pD = p0 , temos que:

pB = p0 - ρ g (h1 + h2 + d)

c) Teoricamente, qual a maior altura possível h , que um sifão pode elevar água?

A menor pressão que pode acontecer no ponto B será a pressão nula, logo:

pMIN = 0 ⇒ p0 - ρ g [ (h1)MAX + h2 + d ] = 0
ou seja:
p0
(h )
1 MAX = − (h2 + d )
ρg

Capítulo 15 - Halliday, Resnick e Walker - 4a. edição

“73” As janelas de um prédio de escritórios tem dimensões de 4m x 5m . Em um dia


tempestuoso, o ar passa pela janela do 530 andar , paralelo à janela, a uma veloci-
dade de 30m/s . Calcule a força resultante aplicada na janela. A densidade do ar é
1,23kg/m3 .

Cap 15 www. f isica. uf pb. br / ~r omer o 32


www.cliqueapostilas.com.br

Pr of . Romer o T avar es da Silva

v2 = 30m/s = 108km/h

Iremos usar a equação de Bernoulli, equacionando um ponto dentro e outro fora do


escritório:
1
p + ρ g y + ρ v 2 = cons tan te
2
ou seja: Dentro
1 1
p1 + ρ g y 1 + ρ v 12 = p 2 + ρ g y 2 + ρ v 22 2 1
2 2

Como os pontos estão no mesmo nível y1 = y2 , e como o


ar dentro do escritório está parado v1 = 0 , temos que:

1
∆p = p1 − p 2 = ρ v 22
2
Mas
1
F = A ∆p = ρAv 22 = 11.070Newtons
2

Cap 15 www. f isica. uf pb. br / ~r omer o 33


www.cliqueapostilas.com.br

Pr of . Romer o T avar es da Silva

16. Oscilações

Quando o movimento de um corpo descreve uma trajetória, e a partir de um certo


instante começa a repetir esta trajetória, dizemos que esse movimento é periódico. O
tempo que o corpo gasta para voltar a percorrer os mesmos pontos da trajetória é chama-
do de período.

No nosso cotidiano existem inúmeros exemplos de movimento periódico, tais como


o pêndulo de um relógio ou um sistema massa - mola, quando um desses conjuntos des-
crevem um vai e vem em torno das suas posições de equilíbrio.

O movimento harmônico simples - MHS

O movimento harmônico simples - MHS é movimento periódico, e portanto o objeto


passa novamente por uma dada posição depois de um período T . O período é o inverso
da a frequência f de oscilação:
1
T =
f

Um exemplo típico de aparato que se


movimenta segundo um MHS é sistema
massa-mola. Uma mola tem uma de suas
extremidades presa em uma parede rígida e
a outra extremidade está presa em um cor-
po que está sobre um superfície sem atrito.
Quando deslocado de sua posição de equi-
líbrio o corpo começa a oscilar.

Um objeto que se desloca em MHS tem a sua posição descrita pela equação

x(t) = xM cos(wt + ϕ)
onde

xM = amplitude de oscilação (wt + ϕ) = fase


w = frequência angular de oscilação ϕ = constante de fase

Quando a constante de fase assume o valor ϕ = - π/2 a equação anterior, que


descreve o movimento do corpo, tem a forma:

x(t) = xM sen wt

À medida que o tempo evolui, o corpo ocupa as diversas posições mostradas na fi-
gura à seguir.

Em cada posição ocupada, o corpo terá uma velocidade correspondente, como ve-
remos mais adiante.

Cap 16 www. f isica. uf pb. br / ~r omer o 2


www.cliqueapostilas.com.br

Pr of . Romer o T avar es da Silva

Também em cada posição, ele terá


uma aceleração correspondente. Tanto a
aceleração quanto a velocidade variam à
medida que a posição se altera.

O gráfico da posição em função do tempo toma diversas formas quando modifica-


mos a amplitude, frequência ou constante de fase.

Quando alteramos a amplitude de


oscilação, o movimento se consuma para
deslocamentos máximos diferentes, mas
com mesma frequência e mesma constante
de fase. Desse modo os dois movimentos
alcançam os extremos no mesmo instante.

Quando aumentamos a frequência (e con-


sequentemente diminuímos o período), os
movimentos terão a forma descrita a seguir
onde a função de maior período é a verme-
lha e a de menor período é azul.

Quando variamos a constante de fase, a


função mantém a forma, mas sofre um
deslocamento, como é mostrado a seguir.

Cap 16 www. f isica. uf pb. br / ~r omer o 3


www.cliqueapostilas.com.br

Pr of . Romer o T avar es da Silva

Como o movimento é periódico, teremos que as posições se repetem depois de um


tempo igual ao período T , ou seja:

x(t) = x(t + T)
e portanto:
x(t + T) = xM cos[w(t + T) + ϕ] = x(t) = xM cos[(wt + ϕ) + wT]

logo:
 2π
w =
wT = 2π ⇒  T
w = 2π f

MHS - A velocidade

dx
v (t ) = = − wx M sen(wt + ϕ )
dt

Definindo a amplitude da velocidade vM = w xM , encontramos que:

v (t ) = − v M sen(wt + ϕ )

MHS - A aceleração

dv
a(t ) = = − w v M cos(wt + ϕ )
dt

Definindo a amplitude da aceleração aM = w vM = w2 xM , encontramos que:

a(t ) = − aM cos(wt + ϕ )

ou ainda

Cap 16 www. f isica. uf pb. br / ~r omer o 4


www.cliqueapostilas.com.br

Pr of . Romer o T avar es da Silva

a(t ) = − w 2 x(t )

MHS - A Lei da força

Considerando um sistema massa - mola que obedeça à Lei de Hooke e supondo


que a resultante das forças que atuam na massa é a força restauradora da mola, encon-
tramos que:
F = ma = −m w 2 x
Mas
F = -k x
logo
 k
 w=
 m
k = mw 2 ⇒ 
 m
T = 2π
 k

MHS - Considerações sobre energia

A energia potencial elástica de um sistema massa - mola é definido como:

1 1
U (t ) = k x 2 = k x M2 cos 2 (wt + ϕ )
2 2

e a energia potencial desse sistema é definida como:

1 1
m v 2 = m[− w x M sen(wt + ϕ )]
2
K (t ) =
2 2

Se considerarmos que m w2 = k , encontramos que:

1
K (t ) = k x M2 sen 2 (wt + ϕ )
2

A energia mecânica E , definida como a soma das energias cinética K e potencial


U , terá a forma:
1
E = U + K = k x M2 = constante
2

Cap 16 www. f isica. uf pb. br / ~r omer o 5


www.cliqueapostilas.com.br

Pr of . Romer o T avar es da Silva

A equação para o MHS

d 2x
F =m = −k x
dt2
ou seja:
d 2x  k 
+  x = 0
d t2 m
ou ainda:
d 2x k
+ w 2x = 0 onde w=
dt 2
m

A solução mais geral da equação anterior tem a forma:

x(t ) = Ae αt

onde A e α são constantes a determinar. Usando a solução, encontramos:

 dx
 = A α e αt
 dt

d 2 x
 2 = Aα e
2 αt

d t

Aplicando estes resultados na equação do MHS, temos que:

Aα 2 e αt + w 2 Ae αt = 0
ou ainda:

Cap 16 www. f isica. uf pb. br / ~r omer o 6


www.cliqueapostilas.com.br

Pr of . Romer o T avar es da Silva

Ae αt (α 2 + w 2 ) = 0

Como A e α são diferentes de zero, em princípio, a única forma da equação aci-


ma se anular será quando:

α 2 + w 2 = 0 ∴ α 2 = −w 2 ⇒ α = ±iw

A solução da equação do MHS toma, então, a forma:

x(t ) = A1e + iαt + A2 e − iαt

A solução da equação do MHS poderá tomar outra forma se redefinirmos as cons-


tantes A1 e A2 , da seguinte forma:

 1
 A1 = 2 x M e
+ iϕ



 1
 A2 = 2 x M e
− iϕ

1 1
x (t ) = x M e + i (wt +ϕ ) + x M e − i (wt +ϕ )
2 2

Considerando a fórmula de De Moivre:

1 + iθ
e iθ = cos θ + i sen θ ⇒ cos θ = (e + e −iθ )
2
temos que:
x (t ) = x M cos(wt + ϕ )

Um oscilador harmônico simples angular - O pêndulo de torção

Vamos considerar um disco preso a um fio


que passa pelo seu centro e perpendicular à sua su-
perfície, como mostra a figura ao lado.

Se giramos o disco à partir de sua posição de


equilíbrio (θ = 0 ) e depois soltarmos, ele irá oscilar
em torno daquela posição em Movimento Harmônico θM
Simples - MHS entre os ângulos (θ = - θM ) e 0
(θ = + θM ) θM

Rodando o disco de um ângulo θ em qual-


quer direção, faremos surgir um torque restaurador
dado por
τ=-κθ

Cap 16 www. f isica. uf pb. br / ~r omer o 7


www.cliqueapostilas.com.br

Pr of . Romer o T avar es da Silva

onde kapa ( κ ) é a constante de torção.

Como a força restauradora é a única que atua no plano do disco, ela provocará o
torque resultante:
τ=Iα

onde I é o momento de inércia do disco e α é a sua aceleração angular. Desse modo,


temos que:

d 2θ
τ =I = −κθ
d t2
ou seja:
d 2θ  κ 
+  θ = 0
dt2  I 

A equação anterior define a frequência angular de oscilação do pêndulo de torção:

κ I
w= ⇒ T = 2π
I κ

e tem como solução:


θ(t) = θM cos(wt + δ)

Pêndulos

Os pêndulos fazem parte de uma classe de osciladores harmônicos simples nos


quais a força restauradora está associada à gravidade, ao invés das propriedades elásti-
cas de um fio torcido ou de uma mola comprimida.

O pêndulo simples

O pêndulo simples é composto de um corpo


suspenso através de um fio de massa desprezível, e
ele é posto a oscilar em torno de sua posição de equi-
líbrio. No seu movimento a corpo descreve um arco de
circunferência.
θ
A componente do peso, tangencial ao desloca-
mento é a força de restauração desse movimento, L
porque age no corpo de modo a trazê-lo de volta à sua
!
posição central de equilíbrio. T
A componente do peso, perpendicular ao deslo-
camento é equilibrada pela tração exercida pelo fio, de s
modo que a resultante das forças tem a forma: θ
!
P

Cap 16 www. f isica. uf pb. br / ~r omer o 8


www.cliqueapostilas.com.br

Pr of . Romer o T avar es da Silva

d 2s
F = −mg sen θ = m
d t2

onde s é o deslocamento medido ao longo do arco que descreve a oscilação, e o sinal


negativo indica que a força age na direção da posição de equilíbrio - como no caso do
sistema massa - mola. O arco s é definido como

d 2s d 2θ
s = Lθ ⇒ =L 2
dt2 dt
temos que:
d 2θ  g 
+   sen θ = 0
dt2 L 

Para pequenas oscilações do pêndulo, podemos aproximar senθ ≈ θ , e teremos


então:
d 2θ  g 
+  θ = 0
dt2 L

A equação anterior define a frequência angular de oscilação do pêndulo simples:

g L
w= ⇒ T = 2π
L g

e tem como solução:


θ(t) = θM cos(wt + δ)

O pêndulo físico

A maior parte dos pêndulos do mundo real não é nem ao menos aproximadamente
simples.

Vamos considerar um objeto de forma arbitrá-


ria, que pode oscilar em torno de um eixo que passa
pelo ponto O , perpendicular à folha de papel. O eixo h O
está a uma distância h do centro de massa, onde
atua a força peso.

Quando o pêndulo da figura ao lado é deslo- CM θ


cado de sua posição de equilíbrio de um ângulo θ ,
surge um torque restaurador
θ
! ! ! !
τ = r ×F P
com módulo:
τ = - (mg senθ) h

e esse é o torque resultante, portanto:

Cap 16 www. f isica. uf pb. br / ~r omer o 9


www.cliqueapostilas.com.br

Pr of . Romer o T avar es da Silva

d 2θ
τ = Iα = I
dt2
ou seja:
d 2θ
τ = −mgh sen θ = I
dt2
ou ainda:
d 2θ  mgh 
+  sen θ = 0
dt2  I 

Para pequenas oscilações do pêndulo, podemos aproximar senθ ≈ θ , e teremos


então:
d 2θ  mgh 
+ θ = 0
dt2  I 

A equação anterior define a frequência angular de oscilação do pêndulo físico:

mgh I
w= ⇒ T = 2π
I mgh

e tem como solução:


θ(t) = θM cos(wt + δ)

MHS e o movimento circular e uniforme

Vamos considerar um corpo que descreve um movimento circular ! e uniforme, com


velocidade constante v em um círculo de raio R . O vetor posição r (t ) que descreve a
trajetória do corpo tem módulo constante, e suas projeções nos eixos cartesianos são da-
das por:
!
r (t ) = iˆx (t ) + ˆjy (t ) y
onde
x(t) = R cos(wt + ϕ) !
e r (t )
y(t) = R sen(wt + ϕ) wt + ϕ

Observando a forma funcional de x


x(t) podemos concluir que o Movimento
Harmônico Simples é a projeção do movi-
mento circular e uniforme num diâmetro do
círculo onde este último acontece.

Cap 16 www. f isica. uf pb. br / ~r omer o 10


www.cliqueapostilas.com.br

Pr of . Romer o T avar es da Silva

y y
!
v (t ) !
a(t )
wt + ϕ wt + ϕ
x x

A velocidade tem a forma: A aceleração tem a forma:


! !
! dr ! dv
v (t ) = a( t ) =
dt dt
! !
v (t ) = iˆv X + ˆjv Y a(t ) = iˆa X + ˆjaY

vX = - w R sen(wt + ϕ) aX = - w2 R cos(wt + ϕ)

vy = + w R cos(wt + ϕ) ay = - w2 R sen(wt + ϕ)

MHS amortecido

Em diversas situações do nosso cotidiano, os movimentos oscilatórios têm uma du-


ração finita, eles têm um começo e um fim. Não ficam se movendo no ir e vir de modo
indefinido. Isso acontece, basicamente, devido a atuação de forças dissipativas tais como
as forças de atrito.

Em uma situação simples as forças dissipativas podem ser representadas por uma
função que depende linearmente da velocidade.

Vamos considerar um sistema composto de uma mola de constante elástica k


com uma das extremidades presa ao teto e a outra suspendendo um corpo de massa m .
Nesse corpo está presa uma haste vertical que tem a sua outra extremidade presa a um
anteparo que está mergulhado em um líquido. Quando o anteparo se move no líquido

esse movimento é amortecido por uma força que surge devido à viscosidade do líquido.

Essa força dissipativa pode ser descrita por uma equação do tipo:

FA = - b v

onde b é chamado de constante de amortecimento. A resultante das forças que atuam


no corpo de massa m é dada por:

Cap 16 www. f isica. uf pb. br / ~r omer o 11


www.cliqueapostilas.com.br

Pr of . Romer o T avar es da Silva

F=-kx-bv
ou seja:
ma=-kx-bv

A forma diferencial da equação anterior é:

d 2x dx
m = −kx − b
dt 2
dt
ou
d 2x  b  d x
+  + w 02 x = 0
dt 2
m dt
onde
k
w0 =
m

A solução da equação diferencial anterior tem a forma:

x(t) = A eαt

onde A e α são constantes a serem determinadas. Aplicando essa forma na equação


diferencial encontramos que:

b
Aα 2 e αt +   Aαe αt + w 02 Ae αt = 0
m
ou seja:
 b 
Ae αt α 2 +  α + w 02  = 0
 m 

Como Ae αt ≠ 0 , teremos então que:

b
α 2 +  α + w 02 = 0
m
cujas soluções são:
2
b b
− ±   − 4w 02
m m
α=
2

ou ainda:
2
b  b 
α=− ±   − w0
2

2m  2m 

Cap 16 www. f isica. uf pb. br / ~r omer o 12


www.cliqueapostilas.com.br

Pr of . Romer o T avar es da Silva

Vamos considerar inicialmente que o movimento é sub-amortecido :


2
 b 
w >
2
0 
 2m 
e definir:
2
 b 
w A = w 02 −  
 2m 
logo:
b
α=− ± iwA
2m

A função x(t) terá, então, a forma:


bt bt
− + iw A t − − iw A t
x(t ) = A1e 2m
+ A2 e 2m

ou seja:
bt

x(t ) = (A1e + iw t + A2 e − iw )e −
A At 2m

e usando uma transformação equivalente àquela do MHS, temos que:

cos(w A t + ϕ )
− bt m
x (t ) = x M e 2

A equação da posição em função do


tempo tem a forma da curva da figura ao
lado. Ela é um cosseno multiplicado por
uma exponencial, e o resultado é um cos-
seno cuja amplitude de oscilação vai dimi-
nuindo à medida que as oscilações se pro-
cessam.

Um exemplo típico dessa situação é a


porta dos saloons dos filmes de bang-bang.
Quando alguém passa pela porta ela inicia
a oscilação com uma grande amplitude, que
vai diminuindo com o tempo.

Quando supomos que o movimento é super-amortecido , temos que:


2
 b 
w <
2
0 
 2m 

temos
2
 b 
wB =   −w0
2

 2m 

Cap 16 www. f isica. uf pb. br / ~r omer o 13


www.cliqueapostilas.com.br

Pr of . Romer o T avar es da Silva

e o parâmetro α agora tem a forma:

b
α =− ± wB
2m

e à partir dele encontramos a equação da posição em função do tempo:


bt

x (t ) = (A1e +w t + A2 e −w )e −
B Bt 2m

ou, se redefinirmos as constantes:

cosh(w B t + ϕ )
− bt m
x (t ) = x M e 2

A equação da posição em função do


tempo tem a forma da curva da figura ao
lado. Ela é um cosseno hiperbólico multipli-
cado por uma exponencial, e o resultado é
um decréscimo monotônico da amplitude.

Na realidade não chega a acontecer ne-


nhuma oscilação, e à medida que o tempo
evolui , a amplitude de oscilação vai ficando
sempre menor.

Um exemplo típico dessa situação é a


porta dos escritórios. Quando alguém passa
pela porta ela inicia a um movimento em
direção ao repouso na posição de equilíbrio.

Cap 16 www. f isica. uf pb. br / ~r omer o 14


www.cliqueapostilas.com.br

Pr of . Romer o T avar es da Silva

Solução de alguns problemas

Capítulo 16 - Halliday, Resnick e Walker - 6a. edição

01 Um objeto sujeito a um movimento harmônico simples leva 0,25s para ir de um


ponto de velocidade zero até o próximo ponto onde isso ocorre. A distância entre es-
ses pontos é de 36cm .

a) Calcule o período do movimento. A

A = 36cm = 0,36m = 2xM


T/2 = 0,25s -xM x=0 +xM

Considerando o movimento harmônico simples, a velocidade é nula nos dois


pontos de elongação máxima x = ± xM . Por outro lado, o tempo para ir de um
extremo ao outro é igual a metade do período. Desse modo:

T = 0,5s

b) Calcule a frequência do movimento.

f = 1/T = 1/0,5 ∴ f = 2Hz

c) Calcule a amplitude do movimento.

xM = 0,18m

Capítulo 16 - Halliday, Resnick e Walker - 4a. edição

03 Um bloco de 4,0Kg está suspenso de uma certa mola, estendendo-a a 16,0cm


além de sua posição de repouso.

a) Qual a constante da mola?

m1 = 4Kg
L = 16cm = 0,16m

Como o bloco está em repouso, existe


o equilíbrio entre as forças que estão
atuando nele. O peso e a força restau-
radora elástica são iguais, logo:
! ! !
F + P1 = 0 L T
ou seja:
k L - m1 g = 0
m1
!
P1
Cap 16 www. f isica. uf pb. br / ~r omer o 15
www.cliqueapostilas.com.br

Pr of . Romer o T avar es da Silva

m1g 4 x 9,8
k= =
L 0,16

k = 245N/m

k 245 2π
w1 = = = 7,8rad / s ⇒ T1 = = 0,8s
m1 4 w1

b) O bloco é removido e um corpo de 0,5Kg é suspenso da mesma mola. Se esta


mola for então puxada e solta, qual o período de oscilação?

m2 = 0,5Kg

k 245 2π
w2 = = = 22,1rad / s ⇒ T2 = = 0,28s
m2 0,5 w2

Capítulo 16 - Halliday, Resnick e Walker - 6a. edição

10 O diafragma de um alto-falante está vibrando num movimento harmônico simples


com a frequência de 440Hz e um deslocamento máximo de 0,75mm .

a) Qual é a frequência angular deste diafragma?

w = 2π f = 2764,60Hz f = 440Hz
xM = 0,75mm = 7,5x10-4m

b) Qual é a velocidade máxima deste diafragma?

vM = w xM = 2,07m/s

c) Qual é a aceleração máxima deste diafragma?

aM = w2 xM = 5732,25m/s2

Capítulo 16 - Halliday, Resnick e Walker - 6a. edição

11 Podemos considerar que um automóvel esteja montado sobre quatro molas idênti-
cas, no que concerne às suas oscilações verticais. As molas de um certo carro estão
ajustadas de forma que as vibrações tenham uma frequência de 3,0Hz .

a) Qual a constante de elasticidade de cada mola, se a massa do carro é de 1450kg


e o peso está homogeneamente distribuído entre elas?

f = 3Hz
M = 1450Kg

Como o peso está distribuído uniformemente entre as quatro molas, cada mola
suportará a quarta parte do peso total. Logo podemos definir m = M/4 e então:
Cap 16 www. f isica. uf pb. br / ~r omer o 16
www.cliqueapostilas.com.br

Pr of . Romer o T avar es da Silva

k k M
= (2πf ) (2πf )2
2
w= = 2πf ∴ ⇒ k=
m m 4

k = 128.798,33N/m = 1,29x105N/m

b) Qual será a frequência de vibração se cinco passageiros, com média de 73kg


cada um, estiverem no carro? (Novamente, considere uma distribuição homogê-
nea de peso.)
mP = 73Kg

O peso dos cinco passageiros será distribuída uniformemente entre as quatro


molas, portanto:
w 1 k 1 4k
f = = = ∴ f = 2,68Hz
2π 2π M 5m P 2π M + 5m P
+
4 4

Capítulo 16 - Halliday, Resnick e Walker - 6a. edição

15 Um corpo oscila com movimento harmônico simples de acordo com a equação:

x(t) = (6,0m) cos[(3π rad/s) t + π/3rad]


a) Em t = 2,0s , qual é o deslocamento nesse movimento?

x(2) = xM cos(2w + ϕ) x(t) = xM cos(wt + ϕ)


Mas
cos(2w + ϕ) = cos(2.3π + π/3) = cos(19π/3) = 0,5 xM = 6m
w = 3π rad/s
x(2) = 6 cos(19π/3) = 3m ϕ = π/3 rad
b) Em t = 2,0s , qual é a velocidade nesse movimento?

dx
v (t ) = = −w x M sen(wt + ϕ )
dt

v(2) = -w xM sen(2w + ϕ)
Mas
sen(2w + ϕ) = sen(2.3π + π/3) = sen(19π/3) = 0,866

v(2) = - 3π 6 sen(19π/3) = -48,97m/s

c) Em t = 2,0s , qual é a aceleração nesse movimento?

dv
a(t ) = = −w 2 x M cos(wt + ϕ )
dt

a(2) = -w2 xM cos(2w + ϕ)

cos(2w + ϕ) = cos(2.3π + π/3) = cos(19π/3) = 0,5

Cap 16 www. f isica. uf pb. br / ~r omer o 17


www.cliqueapostilas.com.br

Pr of . Romer o T avar es da Silva

a(2) = - ( 3π)2 6 cos(19π/3) = -266,47m/s2

d) Em t = 2,0s , qual é a fase nesse movimento?

Fase = Φ(t) = wt + ϕ

Φ(2) = 2w + ϕ = 19π/3 = 39,79rad

e) Qual é a frequência deste movimento?

f = w/2π = 3π/2π = 1,5Hz

f) Qual é o período deste movimento?

T = 1/f = 2/3 s

Capítulo 16 - Halliday, Resnick e Walker - 6a. edição

16 Dois blocos ( m = 1,0kg e M = 10,0kg ) e uma única mola ( k = 200N/m ) estão co-
locados em uma superfície horizontal sem atrito, como ilustra a figura abaixo. O coe-
ficiente de atrito estático entre os dois blocos é µE = 0,40 . Qual a máxima amplitude
possível do movimento harmônico simples, se não houver deslizamento entre os blo-
cos?

Vamos considerar que na figura ao lado o m


!
conjunto está em movimento e passou da FA
posição x = 0 ( primeira figura) e se en-
caminha para a posição x = +xM . A força M
máxima que os blocos exercerão entre si
acontecerá quando x = ±xM pois nessa Indo
situação a = ±aM .

Se F(x) for a força que a mola exerce no x


conjunto dos dois blocos, teremos essa !
força, numa posição genérica, com a for- N
ma: !
FA
F(x) = (m + M) a = k x
k
Como o conjunto está sendo retar- !
dado, a tendência do bloco menor é es- P
corregar para frente, daí a força de atrito
ser dirigida para trás.
Na posição de elongação máxima
da mola, teremos:
x
FM = (m + M) aM = k xM
ou seja

Cap 16 www. f isica. uf pb. br / ~r omer o 18


www.cliqueapostilas.com.br

Pr of . Romer o T avar es da Silva

 k 
aM =   xM
m +M 

Se considerarmos isoladamente o bloco menor, teremos que:

 FA = ma M


N = p = mg

Mas como FA = µE N , concluímos que:

m a M = µE m g ∴ a M = µE g
Mas
 k  µ E (m + M )g
aM = µ E g =   xM ⇒ xM =
m+M  k

xM = 0,22m = 22cm

Capítulo 16 - Halliday, Resnick e Walker - 6a. edição

18 Um bloco está num pistom que se move verticalmente em um movimento harmônico


simples.

a) Se o MHS tem um período de 1,0s , em que amplitude do movimento o bloco e o


pistom irão se separar?
!
O bloco está sobre o pistom que oscila entre N
os limites x = ± xM . Usando a Segunda Lei +xM
de Newton, temos que:
! ! !
N + P = ma x=0
!
Acima da posição x = 0 , ou seja para x ≥ 0 , P1
!
nós temos que a = −iˆ a -xM

Nessa região (x ≥ 0 ) a Segunda Lei de Newton toma a forma:

N - P = - ma ∴ N = m(g - a)

Quando o pistom está subindo desacelerado, depois de passar por x = 0 , o va-


lor da normal N começa a diminuir, até chegar ao seu valor mínimo em x = + xM.
Se a frequência aumentar, a desaceleração também aumentará. Existe um valor
limite da desaceleração para a qual o bloco ainda manterá contato com o pistom.

Nesse limite teremos a = g e consequentemente N = 0 , segundo a equação


anterior. Com a maior desaceleração para uma dada frequência acontece nos
extremos do movimento, o pistom e o bloco ainda manterão o contato se em

Cap 16 www. f isica. uf pb. br / ~r omer o 19


www.cliqueapostilas.com.br

Pr of . Romer o T avar es da Silva

x = + x0 , a = g
Mas
x(t) = xM cos(wt + δ)

a(t0) = - w2 xM cos(wt0 + δ) = - w2 x0

x(t0) = x0 ⇒ |a(t0)| = w2 x0
Logo
2
g T 
w x0 = g ∴ x0 = 2 = g 
2

w  2π 

xM = 0,248m = 24,8cm

b) Se o pistom tem uma amplitude de 5,0cm , qual a frequência máxima em que o


bloco e o pistom estarão continuamente em contato?

xM = 5cm = 0,05m

Do item anterior temos que:


g g 1 g
xM = 2 = ⇒ f = = 2,22Hz
w (2π f )2 2π xM

Capítulo 16 - Halliday, Resnick e Walker - 6a. edição

22 Duas partículas executam um movimento harmônico simples com as mesmas ampli-


tudes e frequências ao longo da mesma linha reta. Elas passam uma pela outra, mo-
vendo-se em sentidos opostos, cada vez que o seu deslocamento é a metade da
amplitude. Qual a diferença de fase entre elas?

As partículas se passam uma pela


outra em dois instantes: t = t1 e
t=t2.
Quando t=t1 temos que:
 xM
 x A (t 1 ) = x B (t 1 ) = 2 -xM -xM/2 0 +xM/2 xM


 v (t ) = −v (t )
 A 1 B 1

Da primeira equação temos que:

xM cos(wt1 + ϕA) = xM cos(wt1 + ϕB)


= xM/2
ou seja:

wt1 + ϕA = 2nπ ± π/3 (1) -xM -xM/2 0 +xM/2 xM


e

Cap 16 www. f isica. uf pb. br / ~r omer o 20


www.cliqueapostilas.com.br

Pr of . Romer o T avar es da Silva

wt1 + ϕB = 2nπ ± π/3 (2)

Por outro lado: + π/3

dx
v=
dt
ou seja:

vA(t1) = -w xM sen(wt1 + ϕA) - π/3


e
vB(t1) = -w xM sen(wt1 + ϕB)

Considerando que nesse problema as velocidades devem ter sentidos contrários:

sen(wt1 + ϕA) = - sen(wt1 + ϕB)

Para que a equação anterior juntamente com as equações (1) e (2) sejam válidas
simultaneamente, deveremos ter:

ΦA(t1) = wt1 + ϕA = 2nπ + π/3


e
ΦB(t1) = wt1 + ϕB = 2nπ - π/3

onde Φ(t) é a fase do movimento de oscilação considerado no instante t e ϕ é a


constante de fase.
∆Φ = ΦA(t1) - ΦB(t1) = 2π/3

∆Φ = 2π/3 = 1200

Capítulo 16 - Halliday, Resnick e Walker - 6a. edição

23 Duas partículas oscilam em um movimento harmônico simples ao longo de um seg-


mento de reta comum de comprimento A . Cada partícula tem um período de 1,5s ,
mas diferem em fase de π/6rad .

a) Qual a distância entre elas, em termos de A , 0,5s após a partícula mais atra-
sada deixar uma das extremidades do percurso?

T = 1,5s ⇒ w = 2π/T = 4π/3 A


∆ϕ = ϕB - ϕA = π/6
∆t = t2 - t1 = 0,5s
- xM + xM

xA(t) = xM cos(wt + ϕA)

xB(t) = xM cos(wt + ϕB)

Em t = t1 a partícula A estará na extremidade, então:

Cap 16 www. f isica. uf pb. br / ~r omer o 21


www.cliqueapostilas.com.br

Pr of . Romer o T avar es da Silva

xA(t1) = xM cos(wt1 + ϕA) = ± xM


e isso implica que:
(wt1 + ϕA) = nπ
Considerando que t2 = t1 + ∆t , temos:
xA(t2) = xM cos(wt2 + ϕA) =
onde
wt2 = w ( t1 + ∆t) = w t1 + w ∆t
ou seja
xA(t2) = xM cos[ ( wt1 + ϕA ) + w ∆t ] = xM cos[ nπ + w ∆t ]
e como
w ∆t = (4π/3) 0,5 = 2π/3
temos que
xA(t2) = xM cos[ nπ + 2π/3 ]
Mas
cos[ nπ + 2π/3 ] = cos(nπ)cos(2π/3)- sen(nπ)sen(2π/3) = (-1)n+1(0,5)
logo
xA(t2) = xM cos[ nπ + 2π/3 ] = (-1)n+1(0,5)
Por outro lado
xB(t2) = xM cos( wt2 + ϕB )
Como
ϕB = ϕA + ∆ϕ
temos que
wt2 + ϕB = w ( t1 + ∆t ) + ( ϕA + ∆ϕ ) = ( wt1 + ϕA ) + ( w∆t + ∆ϕ )
ou seja:
wt2 + ϕB = nπ + ( w∆t + ∆ϕ )
onde
w∆t = ( 4π/3) 0,5 = 2π/3
∆ϕ = π/6
Logo
wt2 + ϕB = nπ + 5π/6

xB(t2) = xM cos[ nπ + 5π/6 ]


Mas
3
cos[ nπ + 5π/6 ] = cos(nπ)cos(5π/6)- sen(nπ)sen(5π/6) = (-1)n+1
2
ou seja:
3
xB(t2) = xM cos[ nπ + 5π/6 ] = (-1)n+1
2

A distância ∆x que separa as duas partículas será dada por:

3
∆x = | xA(t2) - xB(t2) | = xM | (-1)n+1(0,5) - (-1)n+1
2

∆x = xM | 0,5 - 0,866 | = 0,366 xM


Mas como
A = 2 xM
∆x = 0,183 A

Cap 16 www. f isica. uf pb. br / ~r omer o 22


www.cliqueapostilas.com.br

Pr of . Romer o T avar es da Silva

b) Elas estão se movendo no mesmo sentido, em direção uma da outra ou estão se


afastando?

vA(t2) = w xM sen(wt2 + ϕA) = - w xM sen(2π/3 + nπ)

vB(t2) = w xM sen(wt2 + ϕB) = - w xM sen(5π/6 + nπ)


Mas
sen(α + β) = senα cosβ + cosα senβ
logo
sen(2π/3 + nπ) = sen(2π/3)cos(nπ) + sen(nπ)cos(2π/3) = (-1)n sen(2π/3)

ou seja
3
sen(2π/3 + nπ) = (− 1)
n +1

2
Por outro lado:

sen(5π/6 + nπ)= sen(5π/6)cos(nπ) + sen(nπ)cos(5π/6) = (-1)n sen(5π/6)

ou seja
1
sen(5π/6 + nπ) = (− 1)
n +1

2
e finalmente:
 3
v A (t 2 ) = (− 1)
n+2

 2

 1
 v B (t 2 ) = (− 1)
n +2

 2

Como as duas partículas têm velocidades com mesmo sinal, elas estão se mo-
vendo no mesmo sentido.

Capítulo 16 - Halliday, Resnick e Walker - 6a. edição

24 Duas molas idênticas estão ligadas a um bloco de massa m e aos dois suportes
mostrados na figura ao lado. Mostre que a frequência de oscilação na superfície sem
atrito é:
1 2k
f =
2π m
k1 k2
Vamos distinguir as molas com os
rótulos k1 e k2 . Considerando
que o corpo deslocou-se de uma x=0 x

Cap 16 www. f isica. uf pb. br / ~r omer o 23


www.cliqueapostilas.com.br

Pr of . Romer o T avar es da Silva

distância x para a direita, à partir ! !


de sua posição de equilíbrio em F1 F2
x=0 , temos que:
! k1 k2
F1 = −k 1 x iˆ
!
F2 = −k 2 x iˆ
x x
Se considerarmos que o corpo vai
sentir a ação das duas molas k1 = k2 = k
como se fosse apenas uma mola,
teremos: !
F = −κ x iˆ

Mas de acordo com a suposição, a força equivalente é igual à soma das duas forças,
e portanto:
κ k1 + k 2
κ = k1 + k 2 ∴ w = =
m m
Mas k1 = k2 = k , ou seja κ = 2k , e desse modo:

κ 2k w 1 2k
w= = ⇒ f = =
m m 2π 2π m

Capítulo 16 - Halliday, Resnick e Walker - 6a. edição

25 Suponha que as duas molas da figura do problema 33 têm constantes diferentes k1


e k2 . Mostre que a frequência f das oscilações do bloco é então dada por:

f = f12 + f 22
Como já foi deduzido
κ 2k w 1 2k
w= = ⇒ f = =
m m 2π 2π m
logo:
k1 k 2
w2 = + = w 12 + w 22
m m
ou seja:
(2π ) f = (2π ) (f12 + f 22 ) ⇒
2 2
2
f = f12 + f 22

Capítulo 16 - Halliday, Resnick e Walker - 6a. edição

27 Duas molas estão ligadas entre si e conectadas a determinada massa m , como


mostra figura ao lado. A superfície é sem atrito. Se ambas as molas tiverem uma
constante de elasticidade k , mostre que a frequência da oscilação de m é dada
por:

Cap 16 www. f isica. uf pb. br / ~r omer o 24


www.cliqueapostilas.com.br

Pr of . Romer o T avar es da Silva

1 k
f =
2π 2m
k1 k2
Vamos distinguir as molas com os ró-
tulos k1 e k2 . Vamos considerar que a
mola 1 se distende de x1 e a mola 2
se distende de x2 , e a distensão do
conjunto é x . Logo:
x=0
x = x1 + x2
! ! !
Diante destas distensões, surgem as F3 F2 F1
forças
! representadas na figura ao lado:
F3 = força que a parede faz na mola da
!′
esquerda. F3 = força que a mola da
esquerda faz na parede. De acordo !′ !′ !′
!′ ! F3 F2 F1
com a Terceira Lei de Newton F3 = - F2
x
.
A convenção anterior será utilizada
para todos os pares de forças. !
R1
Quando temos apenas uma mola subs- !
tituindo as duas molas mencionadas: R2

"
R1 = −iˆ k x
! ′ !′
Como as molas têm massa desprezí- R2 R1
vel, é nula a resultante das forças que x
nela atuam, ou seja:
!′ !
R1 + R 2 = 0
Pela Terceira Lei de Newton:
! !
R = −R ′
 1 1


 R! = R! ′
 2 2

Usando as três últimas equações, constatamos que:


! ! ′
R 1 = −R 2

ou seja: a força que a mola faz no bloco tem o mesmo módulo da força que esta mola
faz na parede. Estamos aptos a fazer a comparação entre a mola única e o conjunto
de molas no que diz respeito as interações desses sistemas com a parede e o bloco.

Por outro lado, considerando o deslocamento de cada mola, teremos que:

Cap 16 www. f isica. uf pb. br / ~r omer o 25


www.cliqueapostilas.com.br

Pr of . Romer o T avar es da Silva

!
 F ′ = + iˆ k x
 1 1 1


F! ′ = + iˆ k x
 2 2 2

Se observarmos as forças que atuam no sistema das duas molas encontramos que:
! !
 F = −F ′ ! !
F ′ + F = 0
 !1 !1 ′  1 2

F2 = −F2 e 
! !′ F! ′ + F! = 0
F
 3 = − F3  2 3

ou seja: todas as forças envolvidas têm o mesmo módulo, e portanto:

′ ′
R1 F1 F2 1 1 1
x = x1 + x 2 ⇒ = + ⇒ = +
k k1 k2 k k1 k 2
logo:
k1 k 2
κ =
k1 + k 2
e então:
w 1 κ 1 1 k1 k 2
f = = =
2π 2π m 2π m k1 + k 2
Se k1 = k2 = k
1 k
f =
2π 2m

Capítulo 16 - Halliday, Resnick e Walker - 6a. edição

29 Uma mola uniforme, cujo comprimento de repouso é L , tem uma constante de força
k . A mola é cortada em duas partes com comprimentos de repouso L1 e L2 .

a) Quais as correspondentes constantes de força k1 e k2 em termos de n e k .

L = L1 + L2

Quando a mola se distende de x , os pedaços distender-se-ão respectivamente


de x1 e x2 , tal que:
x = x1 + x2

Como a mola é uniforme, podemos supor que ao distender-se o comprimento dos


pedaços manterão a mesma relação de proporcionalidade. Se D é o compri-
mento da mola quando distendida, temos que:

D=L+x ⇒ D1 = nD2 ∴ L1 + x1 = n(L2 + x2)

Cap 16 www. f isica. uf pb. br / ~r omer o 26


www.cliqueapostilas.com.br

Pr of . Romer o T avar es da Silva

ou seja:
x1 = n x2
logo
L = L1 + L2 = nL2 + L2 = (n+1)L2
e
x = x1 + x2 = nx2 + x2 = (n+1)x2

No problema 35 temos duas molas alinhadas e formando um conjunto, e encon-


tramos que todas as forças envolvidas têm o mesmo módulo. Assim:

F = F1 = F2 ∴ k x = k1 x1 = k2 x2
Logo
k x = k1 x1 ⇒ k[(n+1)x2] = k1[nx2] ∴ k1 = k[(n+1)/n]
e
k x = k2 x2 ⇒ k[(n+1)x2] = k2x2 ∴ k2 = k(n+1)

b) Se um bloco for ligado à mola original, oscila com frequência f . Se esta última
for substituída por pedaços L1 ou L2 , a frequência correspondente é f1 ou f2 .
Ache f1 e f2 em termos de f .

 1 k
f =
 2π m
 1 k1
 f1 =
 2π m
 1 k2
f 2 = 2π m

f1 k1 n +1 n +1
= = ∴ f1 = f
f k n n

f2 k2
= = n + 1 ∴ f1 = f n + 1
f k

Capítulo 16 - Halliday, Resnick e Walker - 6a. edição

36 Um bloco de massa M , em repouso numa mesa horizontal sem atrito, é ligado a um


suporte rígido por uma mola de constante k . Uma bala de massa m e velocidade v
atinge o bloco como mostrado na figura à seguir. A bala penetra no bloco.

a) Determine a velocidade do bloco imediatamente após a colisão.


!
Usando a conservação do mo- v M
mento linear, temos que:
m v = (m + M) V m
ou seja:
 m 
V = v
m +M 

Cap 16 www. f isica. uf pb. br / ~r omer o 27


www.cliqueapostilas.com.br

Pr of . Romer o T avar es da Silva

b) Determine a amplitude do movimento harmônico simples resultante.

A energia cinética do conjunto bala + massa logo após a colisão transformar-se-á


em energia potencial elástica quando a mola for comprimida e o bloco para à di-
reita. Logo:
2
1 1  m + M  2  m + M   m  
(m + M )V = k x M ⇒ x M = 
2 2 2
V =    v
2 2  k   k   m + M  

ou seja:
m 2v 2
xM =
k (m + M )

Capítulo 16 - Halliday, Resnick e Walker - 6a. edição

37 Quando o deslocamento no movimento harmônico simples é metade da amplitude


xM

a) Que fração da energia total é cinética? Que fração da energia total é potencial?

x(t) = xM cos(wt + ϕ)

 1 1
U (t ) = k x(t ) 2 = k x M2 cos 2 (wt + ϕ )
 2 2

 1 1
K (t ) = m v (t ) 2 = k x M2 sen 2 (wt + ϕ )
 2 2

Para um dado instante t = t0 o deslocamento é metade da amplitude, logo:

x(t0) = xM/2 ⇒ cos(wt0 + ϕ) = 1/2

A fase Φ(t0) tem a forma:

Φ(t0) = wt0 + ϕ = π/3

A energia total, ou energia mecânica E é a soma das energias cinética e poten-


cial:
1
E = K + U = k x M2
2

 2

 U (t 0 ) = 1 k x M2 cos 2  π  = 1 k x M2  1  = 1 E
 2 3 2 2 4


  3
2
1 2π  1
K (t 0 ) = k x M sen   = k x M 
2 2
 = 3E
2 3 2  
  2  4

Cap 16 www. f isica. uf pb. br / ~r omer o 28


www.cliqueapostilas.com.br

Pr of . Romer o T avar es da Silva

c) Com que deslocamento, em termos da amplitude, a energia do sistema é metade


cinética e metade potencial?

Para um dado instante t = t1 a energia cinética é igual à energia potencial e cada


uma delas é a metade da metade da energia mecânica:

1 1
2E = k x M2 cos 2 (wt 1 + ϕ ) = k x M2 sen 2 (wt 1 + ϕ )
2 2
Desse modo
cos(wt1 + ϕ) = ± sen(wt1 + ϕ)

Φ(t1) = wt1 + ϕ = nπ ± π/4


Logo:
x(t1) = xM cos(wt1 + ϕ) = xM cos(π/4)

x(t 1 ) π  2
= cos  =
xM 4 2

Capítulo 16 - Halliday, Resnick e Walker - 6a. edição

41 A roda de balanço de um relógio oscila com uma amplitude angular de π rad e um


período de 0,5s .

a) Ache a velocidade angular máxima da roda.


θ(t) = θM cos(wt + ϕ)
θM = π rad
T = 0,5s

d θ (t )
θ#(t ) ≡ = −wθ M sen(wt + ϕ )
dt
 
2π 2π 
[θ#(t )]
M = wθ M =
T
θ M = 
1  [ ]
π ∴ θ#(t ) M = 4π 2
 
 2

b) Ache a velocidade angular da roda quando o seu deslocamento for de π/2 rad .
Vamos considerar que o deslocamento tem o valor estipulado quando t = t1 .
Desse modo:
π
θ (t 1 ) = θ M cos(wt 1 + ϕ ) =
2
π
1 π
cos(wt 1 + ϕ ) = 2 = ∴ wt 1 + ϕ = ±
θM 2 3
Logo:
 2π   π  2π   3 
θ#(t 1 ) = −wθ M sen(wt 1 + ϕ ) = −  θ M sen ±  = − π  ±
T   3  0,5   2 

θ#(t 1 ) = $ 2π 2 3 rad / s

Cap 16 www. f isica. uf pb. br / ~r omer o 29


www.cliqueapostilas.com.br

Pr of . Romer o T avar es da Silva

c) Ache a aceleração angular da roda quando o seu deslocamento for de π/4 rad .

Vamos considerar que o deslocamento tem o valor estipulado quando t = t2 .


Desse modo:
π
θ (t 2 ) = θ M cos(wt 2 + ϕ ) =
4
π
1
cos(wt 2 + ϕ ) = 4 = ∴ wt 2 + ϕ = ±1,318 rad
θM 4
 2π   2π 
θ#(t 2 ) = −wθ M sen(wt 2 + ϕ ) = −  θ M sen(± 1,318 ) = −  π (± 0,968 )
T   0,5 

θ#(t 1 ) = $ 3,872 π 2 rad / s

Capítulo 16 - Halliday, Resnick e Walker - 6a. edição

46 Um pêndulo físico consiste em um disco sólido uniforme (de massa M e raio R) ,


suportado num plano vertical por um eixo localizado a uma distância d do centro do
disco - ver figura à seguir. O disco é deslocado um pequeno ângulo e liberado. Ache
uma expressão para o movimento harmônico simples resultante.
! ! !
Seja P o peso do disco e T a força que o eixo T
exerce sobre esse disco. Quando esse sistema
está em repouso a resultante das forças e o tor-
que resultante são nulos. Quando ele começa a d
oscilar, o torque resultante é diferente de zero, e
tem a forma:
!
τ = - P d senθ = Iα P

onde I é o momento de inércia do disco em


relação ao eixo de giro. Por outro lado: !
T
I = ICM + Md2
1 R2 
I = MR + Md = M 
2 2
+ d 2 
2  2 
Da primeira equação temos que:

Pd !
α+ sen θ = 0 P
I
Para pequenas oscilações podemos aproximar o seno pelo seu argumento, logo:

d 2θ  Mgd  Mgd
+ θ = 0 ∴ w =
2

dt 2
 I  I
Mgd 2gd
w= =
I R + 2d 2
2

Cap 16 www. f isica. uf pb. br / ~r omer o 30


www.cliqueapostilas.com.br

Pr of . Romer o T avar es da Silva

Capítulo 16 - Halliday, Resnick e Walker - 4a. edição

50 Um cilindro sólido está ligado a uma mola horizontal sem massa de forma que ele
possa rolar, sem deslizamento, sobre uma superfície horizontal. A constante da mola
é k = 3,0N/m . Se o sistema for liberado de uma posição de repouso em que a mola
esteja distendida de 0,25m ,

Mostre que nessas condições o centro de massa do cilindro executa um movi-


mento harmônico simples com período
3M
T = 2π
2k
onde M é a massa do cilindro. ( Sugestão: Ache a derivada da energia mecânica
total em relação ao tempo) .

ICM = MR2/2 M
k
K = KRot + KTrans

1 1
K = I CM w 2 + Mv CM
2

2 2
Mas
vCM = wR

2  v CM
2
11 1 1 1 3
K=  MR  2 + Mv CM = Mv CM + Mv CM = Mv CM
2 2 2 2

22 R 2 4 2 4

 1
K Trans = 2 Mv CM
2



 1
 K Rot = 4 Mv CM
2

3 1
E = K +U = Mv CM
2
+ k x2
4 2

Como o sistema é conservativo a energia mecânica não varia, e portanto:

dE 3  dv CM  1  dx 
=0 ⇒ M  2 v CM  + k2 x =0
dt 4  dt  2  dt 
ou seja:
 3 d 2x 
 M + k x  v CM = 0
2 dt
2

Mas como vCM ≠ 0 , temos que:

d 2 x  2k 
+ x = 0
d t 2  3M 
Cap 16 www. f isica. uf pb. br / ~r omer o 31
www.cliqueapostilas.com.br

Pr of . Romer o T avar es da Silva

O sistema considerado obedece a equação diferencial acima, e portanto ele tem


frequência angular natural de:

2k 1 3M
w= ⇒ T =
3M 2π 2k

a) Ache a energia cinética translacional do cilindro quando ele passa pela posição
de equilíbrio.

No ponto de elongação máxima a posição é dada por xM e nessa ocasião a ve-


locidade é nula. No ponto de equilíbrio a elongação é nula e a velocidade é má-
xima com o valor vM . Desse modo, considerando a conservação da energia
mecânica:
1 3 2k 2
E = k x M2 = M v CM
2
∴ v CM
2
= xM
2 4 3M
e finalmente:
1 1  2k 2  1
K Trans = Mv CM
2
= M x M  ∴ K Trans = k x M2
2 2  3M  3

b) Ache a energia rotacional do cilindro quando ele passa pela posição de equilíbrio.

1 1
K Rot = Mv CM
2
∴ K Rot = k x M2
4 6

Capítulo 16 - Halliday, Resnick e Walker - 6a. edição

52 Uma haste de comprimento L oscila como um pêndulo físico, com eixo no ponto O ,
como mostra a figura à seguir.

a) Deduza uma expressão para o período do pêndulo em termos de L e x a dis-


tância do ponto de suspensão ao centro de massa do pêndulo.
! !
Seja P o peso da haste e T a força que o eixo exerce sobre essa haste.
Quando esse sistema está em repouso a resultante das forças e o torque resul-
tante são nulos. Quando ela começa a oscilar, o torque resultante é diferente de
zero, e tem a forma:

τ = - P x senθ = Iα !
T
onde I é o momento de inércia da haste em L/2
relação ao eixo de giro. Por outro lado:
x
I = ICM + Mx2

1  L2  L/2
I= ML + Mx = M 
2 2
+ x 2  !
12  12  P

Cap 16 www. f isica. uf pb. br / ~r omer o 32


www.cliqueapostilas.com.br

Pr of . Romer o T avar es da Silva

Da primeira equação temos que:

Px
α+ sen θ = 0
I

Para pequenas oscilações podemos aproxi-


mar o seno pelo seu argumento, logo: !
T
d 2θ  Mgx  Mgx
+ θ = 0 ∴ w =
2

dt 2
 I  I

Mgx 12gx
w= =
I L + 12 x 2
2

!
P
L2 + 12 x 2
T = 2π
12gx

b) Para qual valor de x/L o período é mínimo?

 x 
2 2

L2 1 + 12   x
1 + 12 
  L   L L
T = 2π = 2π
  x  g x
L 12g   12 
  L  L
Vamos definir:
L x
T0 ≡ 2π e u=
g L
logo:
1
1 + 12u 2  1  2

T (u ) = T0 = T0  u + 
12u  12u 

− 12
dT  1  1   1 
= T0  −  u +  1 − 
du  2  12u   12u 
2

1
1−
dT T 12u 2 1 1
=− 0 1
=0 ⇒ 1− = 0 ∴ uM =
du 2  1  2 12u 2
12
u + 
 12u 

xM 1 L
uM = = ⇒ xM =
L 12 12

c) Mostre que se L = 1,0m , e g = 9,8m/s2 , esse mínimo é 1,53s .

Cap 16 www. f isica. uf pb. br / ~r omer o 33


www.cliqueapostilas.com.br

Pr of . Romer o T avar es da Silva

1
 1  2

TM = T (u M ) = T0  u M + 
 12u M 

TM = 1,519s

Capítulo 16 - Halliday, Resnick e Walker - 6a. edição

53 Uma haste longa e uniforme de comprimento L e massa m roda livremente no pla-


no horizontal em torno de um eixo vertical, através de seu centro. Uma determinada
mola com constante de força k é ligada horizontalmente entre um extremidade da
haste e uma parede fixa, conforme figura à seguir. Quando a haste está em equilíbrio
fica paralela à parede.
Qual o período das pequenas oscilações que resultam, quando a haste é ligeira-
mente girada e liberada?

Quando a haste se desloca de um


ângulo θ um ponto de sua extremi-
dade traça um arco de comprimento k
s , e este ponto está distante x da
posição de equilíbrio.
L/2
A mola exerce uma força F na haste
e essa força produz um torque τ

τ = - F (L/2) cosθ

Para pequenas oscilações podemos


aproximar cosθ ≈ 1 , logo x s

τ = -F (L/2) !
F
Mas F = k x , e como θ é pequeno
podemos aproximar a corda ( x ) pelo θ
arco ( s = θ . L/2 ) , ou seja:

x ≈ s = θ (L/2)

Desse modo:
L L L  kL2 
τ = −k . x . ≈ −k  θ  ∴ τ = − θ
2 2 2  4 
Mas, por outro lado:
 mL2  ##
τ = Iα =  θ
 12 
ou seja:
 mL2  ##  kL2   3k 
τ =  θ = − θ ∴ θ## +  θ = 0
 12   4  m

Cap 16 www. f isica. uf pb. br / ~r omer o 34


www.cliqueapostilas.com.br

Pr of . Romer o T avar es da Silva


e portanto:
m
T = 2π
3k

Capítulo 16 - Halliday, Resnick e Walker - 6a. edição

58 Uma roda gira livremente em torno de seu eixo fixo. Uma mola está ligada a um de
seus raios, a uma distância r do eixo, como mostra a figura à seguir.

a) Considerando que a roda é um aro de


raio R e massa m , obtenha a fre-
quência angular de pequenas oscila-
ções deste sistema em termos de m , k
R , r e a constante da mola k . R
r
Como no problema 75, temos que:

τ = - F r cosθ ≈ - F r
Mas
F=kx≈krθ
Logo
τ = - (k r θ) r = - k r2 θ
Mas por outro lado:
τ = Iα = (mR 2 )θ##
ou seja:
 kr 2 
τ = (mR )θ## = −kr 2θ
2
∴ θ + 
## θ = 0
2 
 mR 
2 2
k r  2 r  r
w =   = w0   ∴ w = w0
2

m R  R  R

b) Como mudaria o resultado se r = R ?

Quando r = R , teremos:
k
w = w0 =
m
c) Como mudaria o resultado se r = 0 ?

θ
Se r = 0 , a mola estará fixa no eixo,
e consequentemente não exercerá
influência na possível oscilação. Da
equação que deduzimos para a fre-
quência em função dos parâmetros
chegamos ao resultado que

w=0

nessa situação.

Cap 16 www. f isica. uf pb. br / ~r omer o 35


www.cliqueapostilas.com.br

Pr of . Romer o T avar es da Silva

17. Ondas I - Ondas em meios elásticos

Quando você joga uma pedra no meio de um lago, ao se chocar com a água ela
criará uma onda que se propagará em forma de um círculo de raio crescente, que se
afasta do ponto de choque da pedra. As ondas também podem se propagar em um corda
esticada, presa por suas extremidades; se introduzirmos uma perturbação num ponto
qualquer dessa ela se propagará ao longo da corda. Esses são dois exemplos de ondas
que necessitam de um meio para se propagar.

O som necessita de um meio para se propagar. A luz também é uma onda, e em


particular uma onda eletromagnética. Ondas eletromagnéticas podem se propagar em um
meio ou no vácuo.

Ondas e partículas

Escrever uma carta ou usar o telefone são duas maneiras de se entrar em contato
com uma amiga numa cidade distante.

A primeira opção (a carta) envolve o conceito de partícula. Um objeto material se


desloca de um ponto para outro, carregando consigo a informação e energia.

A segunda opção (o telefone) envolve o conceito de onda. Numa onda, informação


e energia se deslocam de um ponto para outro, mas nenhum objeto material está reali-
zando esta viagem. Em uma onda não existe o transporte de matéria

Ondas

As ondas no mar movem-se com


velocidade perceptível. Mas cada partícula
de água meramente oscila em torno de
seu ponto de equilíbrio.

As partículas descrevem um movi-


mento circular e temos uma combinação
de um movimento na direção de movi-
mento da onda com um movimento per-
pendicular à direção de movimento da
onda.

Ondas transversais e longitudinais

Inicialmente a corda está esticada horizontalmente e em repouso. Introduz-se um


perturbação de modo a se criar uma corcova na corda, e a onda dessa forma se propaga.
Depois da passagem da perturbação por um dado pedaço da corda ela retornará a sua
situação original de repouso.

Cap 17 www. f isica. uf pb. br / ~r omer o 2


www.cliqueapostilas.com.br

Pr of . Romer o T avar es da Silva

Numa corda esticada temos a propagação de ondas


transversais. Nas ondas transversais, o meio no qual a onda
se propaga oscila na direção perpendicular à direção de pro-
pagação da onda. Se isolarmos para observação um elemento
de corda, ele oscilará para cima e para baixo enquanto a onda
se propagará horizontalmente.

Por outro lado, se considerarmos uma mola, teremos a


propagação de ondas longitudinais. Nas ondas longitudinais, o
meio no qual a onda se propaga oscila na direção de propaga-
ção da onda.

Um exemplo
típico de onda lon-
gitudinal é mostrado
ao lado, onde pul-
sos periódicos estão
sendo comunicados
à uma mola

Ondas progressivas

Vamos considerar um pul-


so em forma de corcova se
propagando em uma corda. No
instante t = 0 , o pulso tem o x(t)
formato da esquerda e num x(0)
x'(t)
instante t posterior o pulso
manteve o mesmo formato,
mas se moveu para a direita.

A função que descreve o formato da corda em t = 0 é dada por:

y(x,0) = f(x)

Num instante posterior t , a função que descreverá a forma da corda é dada por:

y(x,t) = f(x')

Se o pulso na corda move-se com velocidade com velocidade v , depois de um tem-


po t , todos os pontos da corcova mover-se-ão de uma distância v t .

Se estivermos observando um dado ponto específico da corcova, por exemplo onde


ela tem metade do valor máximo. Em t = 0 esse ponto está distante de x da coordena-
da do ponto de máxima altura, mas num tempo t posterior ele estará distante x' do má-
ximo, que se moveu de v t com toda a corcova. A relação entre essas grandezas é tal
que:
x = x' + v t ⇒ x' = x - v t

Cap 17 www. f isica. uf pb. br / ~r omer o 3


www.cliqueapostilas.com.br

Pr of . Romer o T avar es da Silva

Desse modo teremos que para uma onda progressiva que se move no sentido positi-
vo do eixo x ,
y( x , y ) = f( x - v t )

Uma onda progressiva, independente da sua forma, depende de x e t como mos-


trado na equação anterior.

Por outro lado, se tivéssemos uma onda progressiva viajando para a esquerda (quer
dizer na direção negativa do eixo x ), ela teria uma dependência funcional em x e t da
forma:
y( x , y ) = g( x + v t )

Se tivéssemos ondas progressivas viajando nos dois sentidos, elas seriam represen-
tadas funcionalmente por:

y( x , y ) = f( x - v t ) + g( x + v t )

Comprimento de onda e frequência

Se estivermos observando a λ
propagação de uma onda harmô-
nica em uma corda, denomina-
mos comprimento de onda λ
distância entre dois pontos equi-
valentes consecutivos. Na figura
ao lado consideramos o compri-
mento de onda como a distância
entre dois máximos consecutivos.

Se estivermos observando
um pequeno pedaço da corda
enquanto uma onda harmônica
se propaga, notaremos que esse
elemento de corda irá se mover T
para cima e para baixo. 1,0

Se medirmos cada posição


desse pedaço de corda à medida 0,5

que o tempo evolui, ao desenhar


o gráfico das posições desse pe- 0,0
Y

daço versus o tempo encontra- 0,0 0,5 1,0 1,5 2,0

remos uma curva do tipo mostra-


do à esquerda. -0,5

Denominamos período T o
tempo entre dois pontos equiva-
-1,0
t
lentes consecutivos. Na figura ao
lado consideramos o período como a distância entre dois máximos consecutivos.

Cap 17 www. f isica. uf pb. br / ~r omer o 4


www.cliqueapostilas.com.br

Pr of . Romer o T avar es da Silva

Velocidade de propagação de uma onda

Um caso particular muito im- 1,0

portante de onda progressiva tem


a forma de uma senóide: 0,5

y(x,t) = yM sen(kx - wt)

Y
0,0
0,00 0,25 0,50 0,75 1,00 1,25 1,50 1,75 2,00
No instante t = 0 a função
tem a forma da curva de traço
contínuo e para um tempo poste-
-0,5

rior ∆t a função tem a forma da


curva tracejada. -1,0
X

Chamamos a grandeza k de
número de onda (ou vetor de onda) e o definimos como:


k=
λ

Chamamos w de frequência angular e a definimos como:


w=
T

Chamamos de fase ϕ(x,t) o argumento da senóide, ou seja:

ϕ(x,t) = kx - wt

Um ponto de fase constante ocupa uma certa posição relativa na onda. Se marcar-
mos um certo ponto de máximo e passarmos a acompanhá-lo, iremos verificar que mes-
mo com a onda se movimentado á medida que o tempo evolui, a fase daquele máximo se
mantém constante.

Assim, se quisermos calcular a velocidade com que uma onda se propaga devemos
acompanhar um dado ponto dela, ou seja um ponto de fase constante:

ϕ(x,t) = kx - wt = constante

dx dx w λ
k −w = 0 ⇒ v= = = ∴ λ = vT
dt dt k T

Cap 17 www. f isica. uf pb. br / ~r omer o 5


www.cliqueapostilas.com.br

Pr of . Romer o T avar es da Silva

Velocidade de uma onda numa corda esticada

Para calcular a velocidade de uma onda em uma corda vamos considerar um pe-
queno pulso se propagando da esquerda para a direita em uma corda de densidade linear
de massa µ e que é esticada através de uma tensão T aplicada nas suas extremidades.
No sentido de facilitar a visualização apresentamos à seguir uma ampliação do pequeno
pulso que se propaga.

Vamos analisar um pequeno pedaço de comprimento ∆L na parte superior do pulso.


esse pedaço ∆L pode se considerado aproximadamente com o formato de um arco de
círculo de raio R e definindo um pequeno ângulo θ .

A análise ficará adequada aos nossos propósitos se observarmos o movimento do


pulso em um referencial que o acompanha com mesma velocidade. Neste referencial que
!
se move com velocidade v em relação aos suportes que prendem a corda, observamos
a corda se mover e tomar a forma de pulso. Se observarmos apenas o pedaço de com-
primento ∆L veremos que momentaneamente ele tem uma trajetória circular. Teremos a
percepção de um pulso congelado e a corda escorregando através dele, como se existis-
se um tubo na forma de pulso e a corda escorregasse por dentro desse pulso.

Como as forças que atuam na corda não se alteram devido a essa mudança de refe-
rencial, temos que é nula a resultante horizontal das forças que atuam no pedaço de cor-
da
! e é não nula a resultante vertical. E como no referencial que se move com velocidade
v o pedaço de corda descreve movimento circular, esta resultante vertical é a força cen-
trípeta que atua neste pedaço de corda.
!
v

∆L

θ !/2 θ!/2
TE TD

Logo:
 θ  θ  
 TD cos 2  − TE cos 2  = 0 
     
 
 θ  θ  
 TD sen  + TE sen  = FR 
 2 2 
Como a tensão da esquerda TE é igual à tensão da direita TD , ou seja: TE = TD = T te-
mos que:
θ 
2T sen  = FR
2

Cap 17 www. f isica. uf pb. br / ~r omer o 6


www.cliqueapostilas.com.br

Pr of . Romer o T avar es da Silva

Considerando que o ângulo é muito pequeno, temos que:

θ   ∆L   ∆L 
2T sen  ≅ 2T   = T 
2  2R  R 
e por outro lado:
v2
FR = m
R
logo:
 ∆L  v T T T
2

T =m ⇒ v2 = = ∴ v=
R  R m ∆L µ µ

Energia e potência numa onda progressiva

Quando consideramos a propagação de uma onda progressiva em uma corda o


movimento oscilatório de um elemento de corda será no sentido perpendicular à sua pro-
pagação. Levando em conta que o deslocamento de um elemento de corda que se en-
contra na posição x no instante t é dado por y(x,y)

y(x,t) = yM sen(kx - wt)

esse elemento de corda deslocar-se-á transversalmente com uma velocidade dada por
u(x,t) :
∂y ( x, t )
u ( x, t ) = = −w y M cos( kx − wt )
∂t

Num dado instante a porção da corda


à esquerda deste elemento de corda, exer-
ce nele uma força transversal à direção de y
propagação dada por
!
FY = −T sen θ T

Considerando que os ângulos envolvidos


serão muito pequenos, podemos aproximar θ

∂y x
FY = −T tan θ = −T
∂x

Portanto, a potência transmitida a um elemento de corda específico por seu vizinho


da esquerda é dada pelo produto da força exercida pela velocidade desse elemento:

 ∂y   ∂y 
P (x, t ) = FY (x, t )u (x, t ) = −T     = −T [ky M cos( kx − wt )][− wy M cos( kx − wt ]
 ∂x   ∂t 

P (x, t ) = w k T y M2 [cos( kx − wt )]
2

Cap 17 www. f isica. uf pb. br / ~r omer o 7


www.cliqueapostilas.com.br

Pr of . Romer o T avar es da Silva

Para uma análise global da propagação da onda na corda é interessante que sai-
bamos qual o valor médio da potência comunicada por um elemento ao seu vizinho, e
esse resultado é o fluxo de energia na corda por unidade de tempo.

Considerando que:

τ
1 1
[cos(kx − wt ]2 = ∫ dt [cos( kx − wt )] =
2

τ 0 2

onde usamos que τ é o período da função, e desse modo:

1
P = P (x, t ) = µ v w 2 y M2
2

onde usamos que T = µ v2 e w = k v .

O Princípio da Superposição

Quando estamos ouvindo uma orquestra chegam si-


multaneamente aos nossos ouvidos os sons de todos os
instrumentos que estão sendo tocados num dado instante.
Isto significa que uma o mais ondas sonoras podem se
propagar ao mesmo tempo numa dada região do espaço.
O efeito global que percebemos será a soma dos efeitos
que cada uma das ondas produziria se estivesse se pro-
pagando isoladamente.

Chamamos de princípio da superposição ao efeito


global ser a soma dos efeitos isolados, como se depreen-
de da figura ao lado que represente a interação entre duas
ondas progressivas em uma corda.

Num dado instante as ondas viajam uma na direção


da outra, produzem um efeito cumulativo ao se encontrar,
e depois disso se afastam com o formato original.

Interferência - ondas no mesmo sentido

Vamos considerar o efeito da interação entre duas ondas que viajam no mesmo
sentido. Para simplificar a análise, sem perder muito em generalidade, vamos considerar
que essas ondas tenham mesma frequência, mesmo comprimento de onda, mesma am-
plitude, mas tenham uma defasagem. A primeira onda tem constante de fase nula e a se-
gunda onda tem constante de fase ϕ . Elas têm a forma:

y1(x,t) = yM sen(kx - wt)

y2(x,t) = yM sen(kx - wt + ϕ)

Cap 17 www. f isica. uf pb. br / ~r omer o 8


www.cliqueapostilas.com.br

Pr of . Romer o T avar es da Silva

Vamos usar a identidade trigonométrica:

α + β  α − β 
sen α + sen β = 2 sen  cos 
 2   2 

A onda resultante será a soma das duas ondas, ou seja:

y(x,t) = y1(x,t) + y2(x,t)


logo:
  ϕ   ϕ
y ( x, t ) = 2y M cos  sen kx − wt + 
  2   2

A onda resultante tem uma amplitude modificada de acordo com o valor da diferen-
ça de fase entre as ondas formadoras. Alguns casos simples podem ser analisados facil-
mente:

a. ϕ = 0

y(x,t) = 2 yM sen(kx - wt)

Esse é um exemplo de uma interferência construtiva, as ondas se somam de modo a


alcançar a maior amplitude possível.

b. ϕ = π

y(x,t) = 0

Esse é um exemplo de uma interferência destrutiva, as ondas interagem e o resul-


tado é a anulação de uma pela outra.

Interferência - ondas em sentido contrário

Vamos analisar o resultado da interação entre duas ondas que se propagam em


sentidos contrários

y1(x,t) = yM sen(kx - wt)

y2(x,t) = yM sen(kx + wt)

Para simplificar a análise, sem perder muito em generalidade, vamos considerar


que essas ondas tenham mesma frequência, mesmo comprimento de onda, mesma am-
plitude, e mesma constante de fase.

Novamente vamos usar a identidade trigonométrica:

α + β  α − β 
sen α + sen β = 2 sen  cos 
 2   2 

Cap 17 www. f isica. uf pb. br / ~r omer o 9


www.cliqueapostilas.com.br

Pr of . Romer o T avar es da Silva

A onda resultante será a soma das duas ondas, ou seja:

y(x,t) = y1(x,t) + y2(x,t)


logo:
y(x,t) = [ 2 yM sen(kx) ] cos(wt)

Esta não é uma onda progressiva, porque não depende de x e t na forma (kx -wt)
mas no entanto a corda oscila para cima e para baixo.

Existem alguns pontos na corda onde a amplitude é máxima, e eles são localizados
quando kx assumem valores múltiplos ímpares de π/2 . Ou seja:

π 3π 5π π  1
kx = ; ; ⇒ kx = (2n + 1) = n + π ; n = 0;1; 2; 3 ;"
2 2 2 2  2

A partir do resultado anterior podemos encontrar os valores de x para os quais a


amplitude é máxima. Esse pontos são chamados antinodos. Temos que k = 2π/λ , logo

 1 λ
xN = n +  ; n = 0;1; 2; 3;"
 2 2

Por outro lado existem pontos onde a amplitude de oscilação é sempre nula, ou
seja: a corda não se move. Esses pontos são localizados quando kx assume valores
múltiplos de π .

kx = 0;π ; 2π ; 3π ;" ⇒ kx = nπ ; n = 0;1; 2; 3;"

A partir do resultado anterior podemos encontrar os valore de x para os quais a


amplitude é nula. Esse pontos são chamados nós. Temos que k = 2π/λ , logo

λ
xN = n ; n = 0;1; 2; 3;"
2

Cap 17 www. f isica. uf pb. br / ~r omer o 10


www.cliqueapostilas.com.br

Pr of . Romer o T avar es da Silva

Reflexão de ondas na extremidade de uma corda

Uma corda pode ter a sua


extremidade presa a um ponto
fixo ou a uma presilha móvel.

Uma onda quando incide na


extremidade de uma corda será
refletida de um modo quando
tem-se a extremidade fixa e de
modo diverso quando a extremi-
dade é móvel.

As duas situações podem


ser vistas nas figuras vizinhas, e
uma dedução desses resultados
pode ser encontrada no Vol 2 do
Curso de Física Básica de H
Moysés Nussenzveig .

Ondas estacionárias e ressonância

Quando uma presa por am-


bas as extremidades é posta para
vibrar em certa frequência as on-
das se propagam nos dois senti-
dos formando um padrão de in-
terferência, como já foi analisado
anteriormente.

Para algumas frequências


específicas a corda entra em res-
sonância, e acontecem as ondas
estacionárias

Na primeira figura à direita


temos uma onda estacionária
com três nós intermediários. O nó
é um ponto onde a corda não se
movimenta. Obviamente, as ex-
tremidades são dois nós. Numa
onda estacionária, essa situação
define o primeiro padrão de osci-
lação, ou seja:

L = λ /2

Cap 17 www. f isica. uf pb. br / ~r omer o 11


www.cliqueapostilas.com.br

Pr of . Romer o T avar es da Silva

É um padrão de oscilação
onde a onda estacionária tem
meio comprimento de onda.

Num segundo padrão de


oscilação temos um nó interme-
diário e desse modo:

L=λ

É um padrão de oscilação
onde a onda estacionária tem um
comprimento de onda.

Num terceiro padrão de os-


cilação temos dois nós intermedi-
ário e desse modo:

L=λ

É um padrão de oscilação
onde a onda estacionária tem
três meios comprimentos de
onda.
L = 3 λ /2

Podemos generalizar dizendo que a condição para existir um padrão de oscilação


para uma onda estacionária é que:

λ 2L
L=n ⇒ λN =
2 n

Já mostramos anteriormente que:

v v
λ = vT = ⇒ f =
f λ

Mas para uma corda presa pelas extremidades, apenas algumas frequências especí-
ficas podem desenvolver uma onda estacionária, portanto:

n n T
fN = v ⇒ fN =
2L 2L µ

Essas frequências específicas são chamadas frequências de ressonância, e como


pode-se notar elas são múltiplas de uma certa frequência mais baixa (n=1) . Chama-se a
frequência mais baixa (n=1) de fundamental ou primeiro harmônico. O segundo har-
mônico corresponde a (n=2) . Chama-se série harmônica o conjunto dos possíveis mo-
dos de oscilação, enquanto n é chamado de número harmônico.

Cap 17 www. f isica. uf pb. br / ~r omer o 12


www.cliqueapostilas.com.br

Pr of . Romer o T avar es da Silva

Solução de alguns problemas

Capítulo 17 - Halliday, Resnick e Walker - 6a. edição

05 Mostre que y(x,t) = yM sen(k x - w t) pode ser reescrito nas seguintes formas alterna-
tivas:

a) y(x,t) = yM sen[k (x - v t)]

 w 
kx − wt = k  x − t  = k (x − vt )
 k 

b) y(x,t) = yM sen[2π (x / λ - f t)]

2π x 
kx − wt = x − 2πf t = 2π  − f t 
λ λ 

c) y(x,t) = yM sen[w (x / v - t) ]

w x 
kx − wt = x − wt = w  − t 
v v 

d) y(x,t) = yM sen[2π [x / λ - t / T)]

2π 2π x t 
kx − wt = x− t = 2π  − 
λ T λ T 

Capítulo 17 - Halliday, Resnick e Walker - 4a. edição

“09” Um pulso isolado, cuja forma de onda é dado pela função h(x - 5 t) é mostrado na
figura à seguir para t = 0 , onde x é dado em centímetros e t é dado em segun-
dos.

a) Qual a velocidade de propagação deste pulso?

Um ponto com fase 4

constante na onda é
definido por: 3

ϕ(x,t) = x - 5 t = cte
h(X)

A velocidade desse 1
ponto é a velocidade da
onda, logo: 0
0 1 2 3 X 4 5 6 7

dx
v = = +5cm / s t=0
dt

Cap 17 www. f isica. uf pb. br / ~r omer o 13


www.cliqueapostilas.com.br

Pr of . Romer o T avar es da Silva

b) Qual o sentido de propagação deste pulso?

O sentido positivo do eixo x .

c) Trace o gráfico h(x - 5 t) como uma função de x para t = 2s .


4

Como é uma onda pro-


gressiva em um meio 3
não dispersivo e sem

h(x-vt)
atenuação, a forma da 2
onda manter-se-á a
mesma. Assim, basta
calcular onde um ponto
1

do pulso vai estar. Va-


mos escolher o ponto 0
10 11 12 13 X 14 15 16 17
mais à esquerda da
onda que se encontra na t = 5s
posição inicial LI = 1cm.
No intervalo de tempo ∆t = 2s esse ponto move-se de ∆L, onde

∆L = v ∆t = 5 . 2 = 10cm

A posição final LF desse ponto será:

LF = LI + ∆L = 1 + 10 = 11cm

d) Trace o gráfico h(x - 5 t) como uma função de t para x = 10cm .


4
Seja tE o tempo neces-
sário para que a parte da 3
esquerda do pulso al-
h(x-vt)

cance o ponto x = 10cm


2
. O máximo do pulso já
passou por esse ponto
um tempo ∆tM anterior e
1

a parte da direita do pul-


so já passou um tempo 0
1,4 1,6 1,8 2,0 t
2,2 2,4 2,6 2,8 3,0
∆tD .
Temos três tempos ca- x = 10cm
racterísticos tE ;
tM = tE + ∆tM e tD = tE +∆tD .
d E 10 − 1
tE = = = 1,8s
v 5

∆x M 3 − 1
∆t M = = = 0,4s ⇒ t M = t E + ∆t M = 2,2s
v 5

∆x D 4 − 1
∆t D = = = 0,6s ⇒ t D = t D + ∆t M = 2,4s
v 5

Cap 17 www. f isica. uf pb. br / ~r omer o 14


www.cliqueapostilas.com.br

Pr of . Romer o T avar es da Silva

Capítulo 17 - Halliday, Resnick e Walker - 4a. edição

“11” A equação de uma onda transversal se propagando em uma corda é dada por:

y(x,t) = (2,0mm) sen[(20m-1)x - (600s-1)t]

a) Ache a amplitude, frequência, velocidade e comprimento de onda.

yM = 2,0mm

w = 600rad/s ⇒ f = w/2π = 95,5Hz

k = 20rad/m ⇒ λ = 2π/k = 0,31m

v = w/k = 30m/s

b) Ache a velocidade escalar máxima de uma partícula da corda.

u( x, t ) =
∂y ( x, t )
∂t
( ) [( ) (
= − 600s −1 (2,0mm )cos 20m −1 x − 600s −1 t )]
uM = 1200mm/s = 1,2m/s

Capítulo 17 - Halliday, Resnick e Walker - 6a. edição

12 A tensão num fio preso em ambos os extremos é duplicada sem que haja qualquer
mudança considerável em seu comprimento. Qual é a razão entre as velocidades das
ondas transversais nesse fio, antes e depois do aumento de tensão?
!
TF = 2 TI T1

A velocidade de propagação de !
uma onda numa fio é dada por: T2

T
v=
µ

vI TI µF
=
vF TF µI

Como o fio não foi alterado, não aconteceu mudança nas densidades de massa,
logo:
vI TI TI 1
= = = ⇒ v F = v I 2 = 1,414v I
vF TF 2TI 2

Cap 17 www. f isica. uf pb. br / ~r omer o 15


www.cliqueapostilas.com.br

Pr of . Romer o T avar es da Silva

Capítulo 17 - Halliday, Resnick e Walker - 6a. edição

13 A densidade linear de uma corda vibrante é 1,6x10-4kg/m . Uma onda transversal se


propaga na corda e é descrita pela seguinte equação:

y(x,t) = (0,021m) sen[(2,0m-1)x + (30s-1)t]

a) Qual é a velocidade da onda?


µ = 1,6x10-4kg/m
v = w/k = 15m/s w = 30rad/s
k = 2rad/m
b) Qual é a tensão na corda?

T
v= ⇒ T = µv2
µ

T = 0,036N

Capítulo 17 - Halliday, Resnick e Walker - 4a. edição

“15” Prove que, se uma onda transversal está se propagando ao longo de uma corda,
então a inclinação de qualquer ponto da corda é numericamente igual à razão entre
a velocidade escalar da partícula e a velocidade da onda naquele ponto

y(x,y) = yM sen(kx - wt)

v = velocidade da onda y
v = w/k

u(x,t) = velocidade de um elemento


de corda
∂y (x, t )
u (x, t ) = = −wy M cos( kx − wt )
∂t θ

tan θ = inclinação da corda

∂y ( x, t )
tan θ = = k y M cos( kx − wt )
∂x

k u ( x, t )
tan θ = u ( x, t ) =
w v

Capítulo 17 - Halliday, Resnick e Walker - 6a. edição

20 Na figura à seguir a corda 1 tem uma densidade linear µ1 = 3,0g/m e a corda 2 tem
uma densidade linear µ2 = 5,0g/m . Elas estão sob tensão devido a um bloco sus-
penso de massa M = 500g .
Cap 17 www. f isica. uf pb. br / ~r omer o 16
www.cliqueapostilas.com.br

Pr of . Romer o T avar es da Silva

a) Calcule a velocidade da onda em cada corda.

µ1 = 3,0g/m Corda 1 Corda 2


µ2 = 5,0g/m
M = 500g

As tensões T1 e T2 que
distendem as cordas são
iguais porque as cordas
estão conectadas e esti- Nó
cadas pela ação da mas-
sa M . Dito de outra for-
ma:
Mg M
T1 = T2 =
2

Estamos aptos a calcular as velocidades de propagação de uma onda em cada


uma das cordas:
T1
v1 = = 28,57m / s
µ1
T2
v2 == 22,13m / s
µ2
b) O bloco agora é dividido em dois (com massas M1 + M2 = M), de acordo com a
configuração á seguir. Determine as massas M1 e M2 para que as velocidades
de uma onda nas duas cordas sejam iguais.

T1 M 1g Corda 1 Corda 2
v1 = =
µ1 µ1

T2 M2g
v2 = =
µ2 µ2
M2
Como v1 = v2 , temos:
M1

M1 M 2 M1 µ 3 3
= ⇒ = 1 = ∴ M1 = M 2
µ1 µ2 M2 µ2 5 5

Mas M1 + M2 = M = 500g , logo

M1 = 187,5g e M2 = 312,5g

Capítulo 17 - Halliday, Resnick e Walker - 6a. edição

23 Uma corda uniforme de massa m e comprimento L está pendurada no teto.

Cap 17 www. f isica. uf pb. br / ~r omer o 17


www.cliqueapostilas.com.br

Pr of . Romer o T avar es da Silva

a) Mostre que a velocidade de uma onda transversal na corda é função de y , a


distância até a extremidade mais baixa, e é dada por v = g y .

Vamos considerar um elemento de corda Y


de comprimento ∆L .

Existem duas forças atuando nesse ele-


mento: o pedaço acima puxa o elemento θ /2 !
com uma força F1 , que é uma reação à θ F1
força peso do elemento de corda mais o
pedaço abaixo. A segunda força F2 é o !
∆L v
peso de pedaço abaixo do elemento de
corda. Seja F a resultante das forças que
atuam no elemento de corda: !
y F2
 θ  θ  θ /2
FX = F1 sen 2  + F2 sen 2 
    

 θ  θ  X
 FY = F1 cos  − F2 cos 
 2 2
onde
F1 = µ (y + ∆L )g
 m
 e µ=
 F =µyg L
 2

Por outro lado, vamos considerar que a onda tenha uma amplitude pequena
comparada com o seu comprimento, de modo que o ângulo possa ser considera-
do pequeno:
 θ  θ ∆L
sen 2  ≅ 2 = 2R
∆L   
θ = ; se θ << 1 ⇒ 
R  θ 
 cos  ≅ 1
 2

 θ  ∆L µgy µg
FX = (F1 + F2 )sen 2  = (2µ g y + µ g ∆L ) 2R = R ∆L + 2R (∆L )
2

  

 θ 
 FY = (F1 − F2 )cos  = F1 − F2
 2

Considerando que se ∆L << 1 teremos que ∆L >> (∆L)2 , então teremos que:

Cap 17 www. f isica. uf pb. br / ~r omer o 18


www.cliqueapostilas.com.br

Pr of . Romer o T avar es da Silva

 µgy
FX ≅ R ∆L
  µ g ∆L 
 ⇒ FR = FX =  y
 F ≅0  R 
 Y

No entanto, em um referencial que esteja se movimentando com a mesma velo-


cidade do pulso, o elemento de corda tem movimento circular com aceleração
centrípeta dada por:
v2
FR = (µ ∆L )
R
e desse modo encontramos que:

v 2  µ g ∆L 
FR = (µ ∆L ) = y ∴ v = gy
R  R 

b) Mostre que o tempo que uma onda transversal leva para percorrer o compri-
mento da corda é dado por t = 2 L .
g

dy dy t L
dy
v= = gy ⇒ dt = ∴ ∫ dt ' = ∫
dt gy 0 0 gy


1 L

1

1

1
L
t =g 2
∫y 2
dy = 2g 2 L 2
⇒ t =2
0 g

Capítulo 17 - Halliday, Resnick e Walker - 6a. edição

27 Duas ondas idênticas que se propagam, deslocando-se no mesmo sentido, têm uma
diferença de fase de π/2rad . Qual é a amplitude da onda resultante em termos da
amplitude comum yM das duas ondas?

y1(x,t) = yM sen(kx - wt)

y2(x,t) = yM sen(kx - wt + π/2)

y(x,t) = y1(x,t) + y2(x,t)

y(x,t) = y1(x,t) = yM [ sen(kx - wt) + sen(kx - wt + π/2) ]


Mas:
α + β  α − β 
sen α + sen β = 2 sen  cos 
 2   2 
logo:
 π  2α + π 2  π 
sen α + senα +  = 2 sen  cos 
 2  2  4

Cap 17 www. f isica. uf pb. br / ~r omer o 19


www.cliqueapostilas.com.br

Pr of . Romer o T avar es da Silva

e portanto
  π   π
y ( x, t ) = 2y M cos  sen kx − wt + 
  4   4

A amplitude A desta onda resultante é dada por:

π  A
A = 2y M cos  = y M 2 ⇒ = 2
4 yM

Capítulo 17 - Halliday, Resnick e Walker - 6a. edição

32 Uma corda sob tensão TI , oscila no terceiro harmônico com uma frequência f3 , e as
ondas na corda têm comprimento de onda λ3 . Se a tensão for aumentada para
TF = 4TI e a corda for novamente levada a oscilar no terceiro harmônico,

a) qual será a frequência de oscilação em termos de f3 ?

 TI
 vI =
 µ

 TF 4TI
v F = = ∴ v F = 2v I
 µ µ

λN 2L
L=n ⇒ λN =
2 n

v n n T
fN = = v ⇒ fN =
λ N 2L 2L µ

 I 3 3v I
 f 3 = 2L v I
 f I
v 1
 ⇒ 3
= 2L = I = ∴ f 3F = 2 f 3I
 F 3 f 3F 3v F vF 2
f 3 = 2L v F 2L

b) qual será o comprimento de onda em termos de λ3 ?

v λI3  v i  f 3F  v i f 3F 1
λ3 = ⇒ =    = = ⋅ 2 = 1 ∴ λ F3 = λI3
f3 λF3  f 3I  v F  v F f3
I
2

Capítulo 17 - Halliday, Resnick e Walker - 6a. edição

34 Duas ondas senoidais com amplitudes e comprimentos de onda idênticos se propa-


gam em sentidos contrários ao longo de uma corda, com velocidade escalar de
10cm/s . Se o intervalo de tempo entre os instantes em que a corda fica retilínea é
0,50s , quais os seus comprimentos de onda?
Cap 17 www. f isica. uf pb. br / ~r omer o 20
www.cliqueapostilas.com.br

Pr of . Romer o T avar es da Silva

y1(x,t) = yM sen(kx - wt)


y2(x,t) = yM sen(kx + wt)

y(x,t) = y1(x,t) + y2(x,t) = yM [ sen(kx - wt) + sen(kx + wt) ]

α + β  α − β 
sen α + sen β = 2 sen  cos 
 2   2 

y(x,t) = 2 yM sen(kx) cos(wt)

O intervalo de tempo entre os instantes em que a corda fica retilínea é igual à meio
período, logo:
∆t = T/2 = 0,50s ⇒ T = 1s

v = 10cm/s = 0,1m/s

λ = v T = (0,1) (1) ⇒ λ = 0,1m

Capítulo 17 - Halliday, Resnick e Walker - 6a. edição

35 Uma corda fixada em ambas as pontas tem 8,40m de comprimento, com uma mas-
sa de 0,120kg . Ela está submetida a uma tensão de 96N e é colocada em oscila-
ção.

a) Qual a velocidade escalar das ondas na corda?

M L = 8,4m
µ= M = 0,120kg
L
T = 96N
T LT
v= =
µ M
v = 81,97m/s
b) Qual o mais longo comprimento de onda possível para uma onda estacionária?

L = λMax/2 ⇒ λMax = 2 L ∴ λMax = 16,8m

c) Dê a frequência dessa onda.

f = v /λMax ⇒ f = 4,87Hz

Capítulo 17 - Halliday, Resnick e Walker - 4a. edição

“38” Uma fonte S e um detetor de ondas de rádio D estão localizados ao nível do solo
a uma distância d , confirme a figura à seguir. Ondas de rádio de comprimento λ
chegam a D , pelo caminho direto ou por reflexão numa certa camada da atmosfe-
ra. Quando a camada está numa altura H , as duas ondas chegam em D exata-
mente em fase. À medida que a camada sobe, a diferença de fase entre as duas
ondas muda, gradualmente, até estarem exatamente fora de fase para uma altura
de camada H + h . Expresse o comprimento de onda λ em termos de d , h e H .
Cap 17 www. f isica. uf pb. br / ~r omer o 21
www.cliqueapostilas.com.br

Pr of . Romer o T avar es da Silva

Vamos definir as grandezas:


h
d1 = distância entre a fonte e o receptor.

d2 = distância percorrida pelo som ao


ser refletido numa altura H . H

d3 = distância percorrida pelo som ao S D


ser refletido numa altura H + h .
d /2 d /2
Desse modo temos que:

 d d
 d1 = + = d
 2 2
 d 
2

 d 2 = 2 H +   = 4H 2 + d 2
2

 2
 d
2

d 3 = 2 (H + h )2 +   = 4(H + h )2 + d 2
 2

∆d1 = d2 - d1 = n λ ⇒ Interferência construtiva

∆d2 = d3 - d1 = ( n + 1/2 ) λ ⇒ Interferência destrutiva

∆d2 - ∆d1 = λ/2 ⇒ λ = 2 ( ∆d2 - ∆d1 )

λ = 2 4(H + h ) + d 2 − 2 4H 2 + d 2
2

Capítulo 17 - Halliday, Resnick e Walker - 6a. edição

40 Dois pulsos se propagam ao longo de uma corda em sentidos opostos, como na figu-
ra à seguir.

a) Se a velocidade da onda v = 2,0m/s e os pulsos estão a uma distância de


6,0cm em t = 0 , esboce os padrões resultantes para t = 5 ; 10 ; 15 e 20ms.

Vamos chamar de x1(t) a y d


localização do máximo do !
pulso 1 , x2(t) a localiza- 1 +v
ção do máximo do pulso x
!
2 , e D(t) a separação −v 2
entre os máximos.

Inicialmente os pulsos estão localizados nas posições x01 e x02 respectiva-


mente, e eles se movem com velocidade v , logo

Cap 17 www. f isica. uf pb. br / ~r omer o 22


www.cliqueapostilas.com.br

Pr of . Romer o T avar es da Silva

x1(t) = x01 +vt


e
x2(t) = x02 - vt
portanto

D(t) = |x2(t) - x1(t)|


e
D(0) = |x01 - x02| = d = 6,0cm
Podemos dizer que:
D(t) = |(x01 - x02)| - 2vt = d - 2vt

Os pulsos terão seus máximos no mesmo ponto quando D(tE) = 0 , ou seja:

d - 2vtE = 0 ⇒ tE = d /2v = 0,015s = 15ms

Para t < tE os dois pulsos


estão se aproximando um y D(t)
do outro. !
1 +v
Quando t = tE os máximos x
!
dos pulsos estão na mes- −v 2
ma posição e tem lugar
uma interferência destrutiva
Neste instante a corda tem
a forma de uma linha reta. y D(t)
!
Quanto t > tE os dois pul- 1 +v
sos estão se afastando um x
!
do outro −v 2

b) O que aconteceu com a energia em t = 15ms ?

Neste instante a corda tem a forma de uma linha reta e aparentemente não exis-
tem pulsos na corda. Mas é como se a energia dos pulsos estivesse armazenada
em forma de energia potencial.

Cap 17 www. f isica. uf pb. br / ~r omer o 23


www.cliqueapostilas.com.br

Pr of . Romer o T avar es da Silva

18. Ondas II - Ondas sonoras

Ondas sonoras são familiares à nossa existência e faz parte de nosso cotidiano a
convivência com corpos que produzem sons. Esses sons podem ser ruídos de choque
entre dois corpos ou melodias produzidas por instrumentos musicais.

As ondas sonoras necessitam de um meio elástico para se propagarem, e não


existe essa propagação no vácuo. Num sólido podemos ter ondas longitudinais ou ondas
transversais. Como os fluidos (líquidos e gases) não suportam tensão de cisalhamento,
apenas as ondas longitudinais se propagam neste meio.

A velocidade do som

As ondas se caracterizam por ser um transporte de energia, associado a uma os-


cilação da matéria. A energia se propaga através da interação de elementos de volume
adjacentes. Como cada material se caracteriza por um arranjo específico da matéria, a
interação entre os elementos de volume adjacentes se dá de um modo peculiar para cada
material que consideremos. Por isso a onda sonora se propaga com uma velocidade dife-
rente para cada meio. Em particular, a sua velocidade no ar a 200C é de vS = 343m/s .

Uma onda sonora se propaga numa sucessão de compressões e rarefações, e em


cada material esses movimentos têm uma característica peculiar. Existe uma grandeza
que dá conta dessas variações em um meio: é o módulo volumétrico da elasticidade B ,
que leva em conta a variação de pressão e a variação fracional de volume. Ele é definido
como:
∆p
B=−
 ∆V 
 
 V 
e no limite quando ∆V → 0 , temos que
 dp 
B = −V  
 dV 

Outro modo de apresentar B é usando-se a densidade volumétrica de massa ρ =


M/V ao invés do volume. Temos que

dp  dp   dρ   dp   M  ρ  dp 
=     =   − 2  = −  
dV  dρ   dV   dρ   V  V  dρ 
logo
 ρ  dρ    dρ 
B = −V −   ⇒ B = ρ  
 V  dp   dp 

A velocidade do som em um meio elástico é dada por:

B
v=
ρ

Cap 18 www. f isica. uf pb. br / ~r omer o 2


www.cliqueapostilas.com.br

Pr of . Romer o T avar es da Silva

Para deduzir a equação da velocidade do som, vamos considerar a propagação de


um pulso em um tubo longo.

Consideremos um fluido
de densidade volumétrica ρ e
pressão P preenchendo o t = t0
tubo desenhado ao lado. Num
dado instante comprimimos
esse fluido movimentando o v ∆t
êmbolo para á direita com ve-
locidade u durante um inter-
valo de tempo ∆t . O movi- t = t0+∆t
mento do pistão é transmitido
às moléculas do fluido pelas u ∆t
colisões que elas
efetuam com o pistão e pelas colisões entre elas.

À medida que as moléculas colidem com a superfície do pistão, elas adquirem veloci-
dades maiores que a média, transmitindo através dos choques essa propriedade para as
moléculas adjacentes. A região hauchuriada comporta-se como um pulso propagando-se
para a direita.
! !
O impulso dado pelo pistão F1 F2
ao volume representado pela
área hauchuriada será igual à
sua variação da quantidade de
movimento, ou seja:
Impulso = I = F ∆t
Mas
F = F1 - F2 = (p + ∆p)A - pA

F = ∆p A
ou seja:
I = (A ∆p) ∆t

A variação da quantidade de movimento do volume perturbado é dado por:

variação da quantidade de movimento = ∆m v

onde ∆m é a massa do fluido que entra em movimento depois de um intervalo ∆t em


que aconteceu o movimento do êmbolo, ou seja:

∆m = ρ ∆V = ρ (u ∆t A)

Considerando que o impulso é igual à variação da quantidade de movimento, temos


que:
F ∆t = ∆m v ⇒ ∆p = ρ v u

Mas o módulo da elasticidade é:

Cap 18 www. f isica. uf pb. br / ~r omer o 3


www.cliqueapostilas.com.br

Pr of . Romer o T avar es da Silva

 dp 
B = −V  
 dV 
onde, usando as nossas convenções:

∆V = VF - VI < 0

∆V = - (u ∆t) A

V= (v ∆t) A
logo:
∆V  − u ∆t A  u u B
∆p = −B = −B  = B ⇒ ∆p = ρ v u = B ∴ v=
V  v ∆t A  v v ρ

Quando consideramos a propagação de uma onda como um processo adiabáti-


co, ou seja: a propagação é um evento tão rápido que não possibilita a troca de calor no
meio, devemos considerar a equação de estado:

p Vγ = constante
onde:
 ∂U 
 
c P  ∂T  P
γ = =
cV  ∂U 
 
 ∂T  V

Diferenciando ambos os lados da equação de estado, temos que:

 γp  dp
V γ dp + γV γ −1 pdV = 0 ⇒ V γ dp + dV  = 0 ∴ − V = γp
 V  dV
logo:
dp B γp
B = −V = γp ⇒ v = =
dV ρ ρ

Propagação de ondas sonoras

À medida que uma onda sonora avança num tubo, cada volume elementar do fluido
oscila em torno de sua posição de equilíbrio.

Os deslocamentos se realizam para direita e para esquerda sobre a direção x , na


qual a onda se propaga.

De modo geral, uma onda progressiva s(x,t) que se propaga no sentido positivo
do eixo x , tem a forma:

s(x,t) = f(x - vt)

Cap 18 www. f isica. uf pb. br / ~r omer o 4


www.cliqueapostilas.com.br

Pr of . Romer o T avar es da Silva

Considerando uma onda harmônica progressiva, temos que:

s(x,t) = sM cos(kx -wt)

Vamos considerar uma


situação simplificada, mas sem x1 x2
perda de generalidade. Num ins-
tante t1 = t0 dois elementos de
volume estão nas suas respecti-
vas posições de equilíbrio, e num
instante posterior t2 = t0 + ∆t s1 s2
eles sofreram os deslocamentos
de acordo com a equação anteri-
or.

onde
s1 = s(x1 , t2)
e
s2 = s(x2 , t2)

∆x = x2 - x1 ∆V = A ( s2 - s1) = A[s(x2 , t2) - s(x1 , t2)]

V = A ∆x ∆V = A ∆s

∆p ∆V
B=− ⇒ ∆p = −B
 ∆V  V
 
 V 
Mas
B
v= ⇒ B = ρv 2
ρ
logo
∆V A ∆s
∆p = − ρ v 2 = −ρ v 2
V A ∆x

e no limite quando ∆x → 0 , teremos:

 ∂s   ∂s 
∆p = −B   = − ρ v 2  
 ∂x   ∂x 

que nos fornece uma relação entre a posição s(x0 ,t) de um elemento de volume que tem
a sua posição de equilíbrio em um ponto genérico x0 e a variação de pressão ∆p(x0 ,t)
que está acontecendo nesse ponto x0 .

∆p = + ρ v2 k sM sen(kx - wt)

onde podemos considerar a variação máxima de pressão ∆pM = ρ v2 k sM , teremos:

∆p = ∆pM sen(kx - wt)


Cap 18 www. f isica. uf pb. br / ~r omer o 5
www.cliqueapostilas.com.br

Pr of . Romer o T avar es da Silva

Intensidade e nível do som

A intensidade de uma onda é definida como a potência média transmitida por uni-
dade de área. Quando no nosso cotidiano dizemos que o som está alto, estamos na reali-
dade dizendo que é alta a intensidade som emitido pelo aparelho. Os músicos dizem que
um som é alto quando a sua frequência é alta.

P ( x, t )
I=
A

Mas a potência instantânea que atua em um elemento de volume pode ser definida
com o produto da força por sua velocidade, ou seja:

∂s( x, t ) ∂s( x, t ) P ( x, t ) ∂s( x, t )


P ( x, t ) = F ( x, t ) = A ∆p ∴ = ∆p
∂t ∂t A ∂t

P ( x, t )
= [∆p M sen(kx − wt )][w s M sen(kx − wt )] = ρ v 2 k w s M2 sen 2 (kx − wt )
A

I = ρ v 2 k w s M2 sen 2 (kx − w )

Pode-se mostrar que

1T 1
sen 2 (kx − wt ) = ∫ dt sen 2 (kx − wt ) =
T 0 2
logo
1
I= ρ v 2 k w s M2
2

Fontes sonoras musicais

Nós percebemos claramente a diferença de som quando ouvimos uma flauta e logo
depois um trombone. Mesmo que os dois instrumentos estejam tocando a mesma nota
musical. Isso acontece porque eles têm timbres diferentes.

Uma nota musical específica está associada com uma certa frequência, e a essa
frequência corresponde um período determinado. A frequência da nota musical é caracte-
rizada pela variação de pressão causada no ar durante um intervalo de tempo periódico.
Pode ser um seno, um dente de serra, ou a variação específica de um instrumento.

Para a variação específica de um dado instrumento nós denominamos timbre.


Cada instrumento tem uma forma específica de produzir uma mesma nota musical, daí
nós percebermos quando está sendo tocado uma flauta ou um trombone.

Cap 18 www. f isica. uf pb. br / ~r omer o 6


www.cliqueapostilas.com.br

Pr of . Romer o T avar es da Silva

Batimentos

Um tipo peculiar de interferência entre duas ondas acontece quando elas se propa-
gam no mesmo sentido, têm mesma amplitude, mas as suas frequências w diferem ligei-
ramente. Como elas estão se propagando no mesmo meio elástico elas têm a mesma
velocidade v de propagação e portanto k = w/v . Desse modo, se as frequências são
próximas, isso também acontece com o número de onda k .

Vamos considerar as duas ondas do tipo:

y1(x,t) = yM cos(k1 x - w1 t)
e
y2(x,t) = yM cos(k2 x - w2 t)
logo:
y(x,t) = y1(x,t) + y2(,x,t)

y(x,t) = yM [ cos(k1 x - w1 t) + cos(k2 x - w2 t) ]

Vamos definir algumas grandezas:


  w1 + w 2 
∆w = w 1 − w 2 w =  
  2 

 e 
 ∆k = k − k   k1 + k 2 
 1 2
k = 
  2 

onde supomos que w1 > w2 e k1 > k2 . Por outro, como as frequências diferem ligeira-
mente, estamos assumindo que w >> ∆w e k >> ∆k . Podemos colocar as equações
anteriores na forma:

 ∆w  ∆k
w 1 = w + 2 k1 = k + 2
 
 e 
 ∆w  ∆k
w 2 = w − 2 k 2 = k − 2
 
ou seja:
  ∆k   ∆w    ∆k   ∆w   
y ( x, t ) = y M cos  k +  x − w +  t  + cos  k −  x − w − t 
  2   2    2   2   

Considerando a identidade trigonométrica:

α + β  α − β 
cos α + cos β = 2 cos  cos 
 2   2 
encontramos que
 ∆k ∆w 
y ( x, t ) = 2y M cos x− (
t  cos k x − w t )
 2 2 

Cap 18 www. f isica. uf pb. br / ~r omer o 7


www.cliqueapostilas.com.br

Pr of . Romer o T avar es da Silva

e se definirmos a amplitude de oscilação como A(x,t) , teremos

 ∆k ∆w 
A( x, t ) = 2y M cos x− t
 2 2 
ou seja:
(
y ( x, t ) = A( x, t ) cos k x − w t )
Como exemplo, estamos 2

mostrando ao lado o gráfico em


x = 0 , resultante da soma de
duas ondas com amplitudes uni-
tárias e frequência w1 = 1
20,94rad/s e w2 = 17,80rad/s .

Temos então que a diferen-


ça ∆w = 3,14rad/s e o valor mé- 0
dio w = 19,37rad / s . 0 1 2 3 4

 2π
 ∆w = 3,14 ⇒ ∆T = ∆w = 2 -1


 2π
w = 19,37 ⇒ T = w = 0,32

-2

Um batimento, ou seja um
máximo de amplitude, ocorrerá 2

sempre que a amplitude global


apresentar um extremo: máximo
ou mínimo.
1

Neste exemplo, o período de


batimento será ∆T = 2s como se
pode observar na figura, a fre-
quência angular de batimento 0

vale ∆w = 3,14rad/s e a fre- 0 1 2 3 4

quência, ∆f = 0,5Hz .

-1

-2

Cap 18 www. f isica. uf pb. br / ~r omer o 8


www.cliqueapostilas.com.br

Pr of . Romer o T avar es da Silva

O Efeito Doppler

O som é um tipo de onda que necessita de um meio para se propagar. Quando


estamos analisando a produção e a captação de uma onda sonora, estamos diante de
três participantes: a fonte sonora, o meio onde ela se propaga e o observador que está
captando as ondas. Temos então três referenciais bem definidos.

O tipo de onda captada dependerá de como a fonte e o observador se movem em


relação ao meio de propagação da onda. Vamos considerar o meio parado em relação ao
solo. Neste caso temos ainda três situações diferentes: a fonte se movimenta e o obser-
vador está parado; a fonte está parada e o observador está em movimento; a fonte e o
observador estão em movimento. Nos três casos podemos ter uma aproximação ou um
afastamento entre a fonte e o observador.

Fonte e observador em repouso

A fonte emite uma onda


!
harmônica de frequência f e v
comprimento de onda λ . Vamos
desenhar apenas as frentes de
onda. As frentes de onda esféricas
concêntricas viajam com velocida- Observador
de v . Como todos os participan-
tes (fonte, observador e meio) es-
tão em repouso, o observador vai
perceber uma onda exatamente do
mesmo tipo que foi emitida pela
fonte.

v=λf λ

Fonte em movimento - observador em repouso

Como a fonte está em mo-


!
vimento, as frentes de onda não v
são mais esferas concêntricas.
Quando a fonte emitir a segunda
frente ela já não estará mais na
mesma posição de quando emitiu Observador
uma primeira onda.

Seja T é o período da onda


que a fonte está emitindo. Como a
fonte está se aproximando do ob-
servador ele irá perceber uma
distância λ' entre as frentes de !
onda menor que um comprimento vF
de onda λ original, como pode-se

Cap 18 www. f isica. uf pb. br / ~r omer o 9


www.cliqueapostilas.com.br

Pr of . Romer o T avar es da Silva

depreender pela figura ao lado. Se em um tempo T (período) uma frente de onda viajou
uma distância λ = v T (comprimento de onda original), como a fonte se aproximou do
observador de vF T , o observador perceberá um comprimento de onda λ' diferente do
original:
λ' = λ - vF T
ou seja:
λ' = v T - vF T = (v - vF)/f
Mas
λ' = v / f'

onde f' é a frequência que o observador vai perceber nas circunstâncias atuais. Portanto:

v v −vF  v 
= ⇒ f ' =   f
f' f v − vF 

Quando a fonte estiver se afastando do observador em repouso, teremos uma si-


tuação semelhante a essa descrita, e encontraremos que:

λ' = λ + vF T
ou seja:
λ' = v T + vF T = (v + vF)/f
logo:
 v 
f ' =   f
v + vF 

Fonte em repouso - observador em movimento

Quando a fonte está em repouso em relação ao meio a propagação se dará de


modo a formarem-se frentes de ondas esféricas concêntricas.

Como a frequência é uma medida do número de frentes de ondas por unidade de


tempo que atingem o observador, neste caso chegam a si f = v / λ frentes de onda por
unidade de tempo. Se a frequência for f = 1Hz o período T = 1s , e atingirá o observador
uma frente de onda por segundo. Se f = 0,5Hz teremos T = 2s e portanto atingirá o ob-
servador uma frente de onda a cada 2s , que é metade do número do caso anterior.

Se o observador se aproxima da fonte com velocidade vo , ele irá de encontro às


frentes de onda, encontrando vo /λ mais frentes de onda por unidade de tempo que se
estivesse em repouso. Desse modo, o número de frentes de onda por unidade de tempo
f' que ele encontra será:

v vo vo v + vo 
f'= + ⇒ f'= f + f ∴ f'=  f
λ λ v  v 

Quando o observador estiver se afastando da fonte em repouso, teremos uma situ-


ação semelhante a essa descrita, e encontraremos que:

Cap 18 www. f isica. uf pb. br / ~r omer o 10


www.cliqueapostilas.com.br

Pr of . Romer o T avar es da Silva

v vo vo v − vo 
f'= − ⇒ f'= f −f ∴ f'=  f
λ λ v  v 

Quando o observador estiver se afastando da fonte em repouso, teremos uma si-


tuação semelhante a essa descrita, e encontraremos que:

v + vo 
f'=  f
 v 

Fonte e observador em movimento

Quando fonte e observador estiverem em movimento teremos uma combinação


dos resultados anteriores.

 v ± vo  sinal sup erior : aproximando − se


f ' = f   
v " vF   sinal inf erior : afas tan do − se

Cap 18 www. f isica. uf pb. br / ~r omer o 11


www.cliqueapostilas.com.br

Pr of . Romer o T avar es da Silva

Solução de alguns problemas

Capítulo 18 - Halliday, Resnick e Walker - 6a. edição

01
a) Uma regra para encontrar a sua distância de um relâmpago é contar quantos se-
gundos se passam, desde a visão do raio até ouvir o trovão e, então, dividir o
número por cinco. O resultado é por suposição, a distância em milhas. Explique o
funcionamento dessa regra e determine a porcentagem de erro a 200C .

vL = 3x108m/s = 300.000.000m/s
vS = 343m/s = 767,291mi/h Raio Observador

Considerando a propagação do som do d


trovão, temos que:

d = vS tS

e considerando a propagação da luz do relâmpago, temos que:

d = vL tL

O observador percebe os dos fenômenos com uma diferença de tempo ∆t dada


por:
d d v − vS 
∆t = t S − t L = − ⇒ ∆t = d  L 
vS vL  v Lv S 

Mas como vL >> vS , teremos:

 v  d
∆t ≅ d  L  ⇒ ∆t ≅
 v Lv S  vS

Considerando a distância em milhas e a velocidade em milhas por hora, temos:

 767,291 ∆t ∆t
d = v S ∆t =  ∆t = ⇒ dE =
 3600  4,69 5

∆d d − d E d  ∆d 
= = 1 − E = 0,062 ∴  % = 6,2%
d d d  d 

b) Desenvolva uma regra semelhante para obter a distância em quilômetros.

Considerando a distância em metros e o tempo em segundos, temos

∆t ∆t
d = v S ∆t = (343 )∆t = (343 x10 −3 km / s )∆t = ⇒ dE =
2,91 3

Cap 18 www. f isica. uf pb. br / ~r omer o 12


www.cliqueapostilas.com.br

Pr of . Romer o T avar es da Silva

∆d d − d E d  ∆d 
= = 1 − E = 0,03 ∴  % = 3%
d d d  d 

Capítulo 18 - Halliday, Resnick e Walker - 6a. edição

04 Uma coluna de soldados, marchando a 120 passos/min , segue a música da banda à


frente do pelotão. Observa-se que os soldados atrás da coluna avançam com o pé
esquerdo, enquanto os músicos da banda avançam com o pé direito. Qual o tamanho
da coluna, aproximadamente?

f = 120passos/min = (120/60)passos/s Banda Pelotão

ou seja:

f = 2Hz ⇒ T = 0,5s
d
Os componentes da banda estão defasa-
dos de meio período em relação aos soldados que marcham no fim da coluna. A dife-
rença de tempo ∆t é dada por:

∆t = T/2 = 0,25s

O tamanho d do pelotão será, então:

d = vS ∆t = (343m/s) (0,25s)

onde vS = 343m/s é a velocidade do som no ar. Logo

d = 85,75m

Capítulo 18 - Halliday, Resnick e Walker - 6a. edição

05 Terremotos geram ondas sonoras na Terra. Ao contrário do que ocorre em um gás,


podem ser geradas ondas longitudinais (P) e ondas transversais (S) em um sólido . A
velocidade das ondas S é aproximadamente vS≅ 4,5km/s e as ondas P aproxima-
damente vP ≅ 8,0km/s . Um sismógrafo registra as ondas S e as ondas P de um
terremoto. As primeiras ondas P aparecem ∆t = 3min antes das primeiras ondas S.
Supondo que as ondas viajam em linha reta, a que distância ocorreu o terremoto?

Vamos chamar de L a distância entre o


ponto onde aconteceu o terremoto e a vS = 4,5km/s
posição do observador; tS o tempo para vP = 8km/s
uma onda S percorrer esta distância e ∆t = 3min = 180s
tP o tempo para uma onda P percorrer
esta distância.

Cap 18 www. f isica. uf pb. br / ~r omer o 13


www.cliqueapostilas.com.br

Pr of . Romer o T avar es da Silva

 L 
t S = v 
 S
  1 1  v − vS 
  ⇒ ∆t = t S − t P = L −  = L P 
 L vS vP   v Pv S 
t P = v 
 P 

 v v 
L = ∆t  P S  = 1.851,4km
 vP − vS 

Capítulo 18 - Halliday, Resnick e Walker - 6a. edição

06 A velocidade do som em um certo metal é vM . Em uma extremidade de um longo


tubo deste metal de comprimento L , se produz um som. Um ouvinte do outro lado
do tubo ouve dois sons, um da onda que se propaga pelo tubo e outro que se propa-
ga pelo ar.

a) Se vS é a velocidade do som no ar, que intervalo de tempo ∆t ocorre entre os


dois sons?

L = vM tM = vS tS

L L v − vS 
= L M
∆t = t S − t M =
− 
vS vM  v Mv S 
b) Suponha que ∆t = 1s e que o metal é ferro, encontre o comprimento L .

∆t = 1s
vS = 343m/s
vM = 5.941m/s
 v v 
L = ∆t  M S  ∴ L = 364m
 vM − vS 

Capítulo 18 - Halliday, Resnick e Walker - 6a. edição

07 Uma pedra é jogada num poço. O som da pedra se chocando com a água é ouvido
∆t = 3s depois. Qual a profundidade do poço?

Vamos considerar que h é a profundidade do poço, tP é o tempo gasto para a pe-


dra chocar com a água no fundo do poço e tS é o tempo necessário para o som da
colisão subir até a boca do poço. Logo temos que ∆t = tP + tS . Por outro lado:

1 2
h= g tP = v StS
2
logo
2h 2 2v  2v ∆t   2v 
t P2 = = (v S t S ) = S (∆t − t P ) =  S  −  S t P
g g g  g   g 

Cap 18 www. f isica. uf pb. br / ~r omer o 14


www.cliqueapostilas.com.br

Pr of . Romer o T avar es da Silva

ou seja:
 2v   2v ∆t 
t P2 +  S t P −  S  = 0
 g   g 
Resolvendo, temos que:
− v S ± v S2 + 2 v S g ∆t  + 2,88s
tP = =
g − 72,88s

Como o temo é positivo, escolhemos a primeira solução tP = 2,88s . Desse modo,


temos que:
tS = ∆t - tP = 3,00 - 2,88 = 0,12s =

e portanto
1 2
h= gt P = 40,64m
2

Capítulo 18 - Halliday, Resnick e Walker - 6a. edição

10
a) Uma onda senoidal longitudinal contínua é envidada através de determinada
mola, por meio de uma fonte oscilante conectada a ela. A frequência da fonte é
de 25Hz e a distância entre pontos sucessivos de máxima expansão da mola é
de 24cm . Encontre a velocidade com que a onda se propaga na mola.

v = w /k = λ/T = λ f = (25Hz) (0,24m) f = 25Hz


λ = 24cm = 0,24m
v = 6m/s

b) Se o deslocamento longitudinal máximo de uma partícula na mola é de 0,30cm


e a onda se move no sentido - x , escreva a equação da onda. Considere a fonte
em x = 0 e o deslocamento nulo em x = 0 quanto t = 0 também é zero.

s(x,t) = sM cos(kx + wt + ϕ) sM = 0,30cm = 0,0030m


w = 2π f = 50 π rad/s
s(0,0) = 0 = sM cosϕ k = 2π/λ = 5π/6 rad/m= 8,33πrad/m
logo
ϕ = π/2
ou seja
s(x,t) = sM sen(kx + wt)
e finalmente:
s(x,t) = (0,0030m) sen( 5πx/6 + 50πt)

Capítulo 18 - Halliday, Resnick e Walker - 6a. edição

11 A pressão em uma onda sonora progressiva é dada pela equação:

∆p = (1,5Pa) sen π [(1m-1)x - (330s-1)t]

Cap 18 www. f isica. uf pb. br / ~r omer o 15


www.cliqueapostilas.com.br

Pr of . Romer o T avar es da Silva

a) Encontre a amplitude de pressão

∆pM = 1,5Pa

b) Encontre a frequência

w 330π
f = = = 165Hz
2π 2π

c) Encontre o comprimento de onda

2π 2π
λ= = = 2m
k π
d) Encontre a velocidade da onda

w 330π
v= = = 330m / s
k π

Capítulo 18 - Halliday, Resnick e Walker - 6a. edição

12 Duas fontes pontuais de ondas sonoras, de comprimentos de onda λ e amplitudes


idênticas, estão separadas por uma distância D = 2 λ . As fontes estão em fase.

a) Quantos pontos de sinal máximo (interferência construtiva) existem em um gran-


de círculo em torno da fonte?
P
Vamos considerar um grande círculo, P
ou seja: a distância entre as fontes é L2 L1
bem menor que o raio deste círculo. r

Seja P um ponto desse círculo, e L1 e


L2 as distâncias de cada uma das fon- D
tes a esse ponto.

Vamos definir a origem das coordena-


das coincidindo com o centro do círculo.

Podemos então definir:


!
! ! D
 L1 = r −
 2 ! #
 L2 L1
!
! ! D
L2 = r + !
 2 r
Logo:
2
!
D ! D θ
L1 = r +   − 2r ⋅
2 2
! !
2 2 D D
ou seja −
2 2

Cap 18 www. f isica. uf pb. br / ~r omer o 16


www.cliqueapostilas.com.br

Pr of . Romer o T avar es da Silva

 2 D
2

L1 = r +   − rD cos θ
2

 2

 2 D
2

L2 = r +   + rD cos θ
2

 2
portanto
L22 − L21 = 2rD cos θ
Mas por outro lado:

 L2 + L1 ≅ 2r 
 
  ⇒ L22 − L21 = (L2 − L1 )(L2 + L1 ) ≅ 2r∆L
L − L = ∆L 
 2 1 
logo
∆L ∆L
L22 − L21 = 2rD cos θ ≅ 2r∆L ∴ cos θ = =
D 2λ

Para que tenhamos uma interferência construtiva é necessário que ∆L = ± n λ ,


ou seja:
n
cos θ = ±
2

n=0 ⇒ cosθ = 0 ⇒ θ = 900 ou θ = 2700


n = +1 ⇒ cosθ = + 1/2 ⇒ θ = 600 ou θ = 3000
n = -1 ⇒ cosθ = - 1/2 ⇒ θ = 1200 ou θ = 2400
n = +2 ⇒ cosθ = + 1 ⇒ θ = 00
n = -2 ⇒ cosθ = - 1 ⇒ θ = 1800

Existem, portanto oito pontos de máximo.

b) Quantos pontos de sinal mínimo (interferência destrutiva) existem em um grande


círculo em torno da fonte?

Para o cálculo de pontos com interferência destrutiva, o procedimento é equiva-


lente:

∆L ∆L
L22 − L21 = 2rD cos θ ≅ 2r∆L ∴ cos θ = =
D 2λ

Para que tenhamos uma interferência destrutiva é necessário que

 λ λ
∆L = ± nλ +  = ± (2n + 1)
 2 2
ou seja:
 2n + 1
cos θ = ± 
 4 

Cap 18 www. f isica. uf pb. br / ~r omer o 17


www.cliqueapostilas.com.br

Pr of . Romer o T avar es da Silva

n=0 ⇒ cosθ = + 1/4 ⇒ θ = 75,520 ou θ = 284,440


n=0 ⇒ cosθ = - 1/4 ⇒ θ = 104,470 ou θ = 255,520
n = +1 ⇒ cosθ = + 3/4 ⇒ θ = 41,400 ou θ = 318,590
n = -1 ⇒ cosθ = - 3/4 ⇒ θ = 138,59 ou θ = 221,400

Existem, portanto oito pontos de mínimo.

Capítulo 18 - Halliday, Resnick e Walker - 6a. edição

13 Na figura á seguir, dois alto-falantes, separados por uma distância de 2m , estão em


fase. Supondo que a amplitude dos sons dos dois seja, de modo aproximado, a
mesma na posição do ouvinte, que está a 3,75m diretamente à frente de um dos
alto-falantes,

a) Para quais frequências audíveis (20Hz - 20kHz) existe um mínimo?

D = 3,75m Ouvinte
d = 2m

Por construção, temos que triângulo retângu-


lo, logo:
L D
L = d + D = 4,25m
2 2

Para que tenhamos um mínimo, a


interferência entre as ondas deve ser destruti-
va, e isso acontece quando a diferença de
percurso for igual a meio comprimento de d Alto-falante
onda.
Ou de modo geral, for igual a um número inteiro de comprimentos de
onda mais meio comprimento de onda
λ
L − D = nλ +
2
ou ainda:
λ 2(L − D )
L − D = (2n + 1) ⇒ λN =
2 2n + 1
Mas
v v
fN = = (2n + 1)
λN 2(L − D )
Como:
v
= 343Hz
2(L − D )
teremos:
f0 = 343Hz
f1 = 3 f0 = 1029Hz
f2 = 5 f0 = 1715Hz

Cap 18 www. f isica. uf pb. br / ~r omer o 18


www.cliqueapostilas.com.br

Pr of . Romer o T avar es da Silva

b) Para quais frequências o som fica ao máximo?

Para que tenhamos um máximo, a interferência entre as ondas deve ser


construtiva, e isso acontece quando não existir diferença de percurso.

Ou de modo geral, for igual a um número inteiro de comprimentos de onda:

L − D = nλ

λN =
(L − D )
n
Mas
v v
fN = =n
λN (L − D )
Como:
v
= 686Hz
2(L − D )

f1 = 686Hz
f2 = 2 f1 = 1372Hz
f3 = 2058Hz

Capítulo 18 - Halliday, Resnick e Walker - 6a. edição

16 Uma onda sonora de comprimento de onda 40cm entra no tubo mostrado na figura à
seguir. Qual deve ser o menor raio r , de modo que um mínimo seja registrado no
detetor?

A diferença entre os percursos é dada


por:
∆L = πr - 2r = (π - 2) r

Para que aconteça uma interferência


destrutiva é necessário que a diferença
de percurso tenha a forma:

λ
∆L = (2n + 1) ⇒ (π − 2)r = (2n + 1) λ
2 2

Para se calcular o menor raio possível, basta fazer n = 0 na equação anterior, ou


seja:
λ
r = = 17,51cm
2(π − 2)

Capítulo 18 - Halliday, Resnick e Walker - 4a. edição


Duas ondas sonoras, originárias de duas fontes diferentes e com a mesma frequên-
“19” cia f = 540Hz , viajam à velocidade de 330m/s . As fontes estão em fase. Qual a
diferença das fases das ondas em um ponto que dista 4,4m de uma fonte e 4m
de outra?. As ondas se propagam na mesma direção.

Cap 18 www. f isica. uf pb. br / ~r omer o 19


www.cliqueapostilas.com.br

Pr of . Romer o T avar es da Silva

Vamos considerar as on- 1 2 P


das com as formas:
x
s1(x,t) = sM cos(kx - wt) D d2

s2(x,t) = sM cos(kx - wt + d1
ϕ)

Vamos considerar que as fontes estão respectivamente nos pontos x = 0 e x = D .


Desse modo, no instante t = 0 as fontes estão emitindo ondas tais que, no local de
emissão temos:
s1(0,0) = s

s2(D,0) = sM cos(kD + ϕ)
Mas como as fontes estão emitindo em fase, devemos ter que:

s2(D,0) = sM ⇒ cos(kD + ϕ) = 1 ∴ ϕ = - kD

ou seja:
s2(x,t) = sM cos[k(x-D) - wt]

Assim temos o formato das duas ondas para quaisquer valores de x, e t . Para um
ponto específico x = d1 , temos que:

s1(d1,t) = sM cos(kd1 - wt)


e
s2(d1,t) = sM cos[k(d1-D) - wt]

com as respectivas fases:

Φ1(d1,t) = kd1 - wt
Φ2(d1,t) = k(d1-D) - wt
∆Φ = Φ1 - Φ2 = kD = 2 π D / λ = 2 π f D / v

∆Φ = 4,11rad

Capítulo 18 - Halliday, Resnick e Walker - 4a. edição

“20” Em um certo ponto no espaço, duas ondas produzem variações de pressão dadas
por:
∆p1 = ∆pM sen(wt)
e
∆p2 = ∆pM sen(wt - ϕ)

Qual é a amplitude de pressão da onda resultante nesse ponto quando ϕ = 0 ;


ϕ = π/2 ; ϕ = π/3 e ϕ = π/4 ?

Cap 18 www. f isica. uf pb. br / ~r omer o 20


www.cliqueapostilas.com.br

Pr of . Romer o T avar es da Silva

∆p = ∆p1 + ∆p2 = ∆pM [sen(wt) + sen(wt - ϕ)]


Mas
α + β  α − β 
sen α + sen β = 2 sen  cos 
 2   2 
logo
  ϕ   ϕ
∆p = 2∆p M cos  sen wt − 
  2   2

onde a amplitude de pressão resultante é dada por:

ϕ 
∆PM = 2∆p M cos 
2

Para cada uma das situações mencionadas teremos os valores á seguir:

i. ϕ=0
∆PM = 2∆p M
ii. ϕ = π/2
π 
∆PM = 2∆p M cos  = 2 ∆p M
4
iii. ϕ = π/3
π 
∆PM = 2∆p M cos  = 3 ∆p M
6
iv. ϕ = π/4
π 
∆PM = 2∆p M cos 
8

Capítulo 18 - Halliday, Resnick e Walker - 6a. edição

30 Uma corda de violino de 15cm , presa em ambas as extremidades, oscila em seu


modo n = 1 . A velocidade das ondas na corda é de 250m/s e a velocidade do som
no ar é de 348m/s .

a) Qual é a frequência da onda emitida?

L = 15cm = 0,15m v = 250m/s


n=1 vS = 348m/s

Quando a corda de um violino está vibrando, devido à reflexão nas extremidades,


forma-se uma onda estacionária. A condição para uma onda estacionária neste
caso é:
λ 2L
L=n ⇒ λN =
2 n

Cap 18 www. f isica. uf pb. br / ~r omer o 21


www.cliqueapostilas.com.br

Pr of . Romer o T avar es da Silva

v n n T
fN = = v =
λ N 2L 2L µ

v
f1 = =833,3Hz
2L

b) Qual é o comprimento de onda da onda emitida?

Quando estiver no ar, essa onda vai se propagar com a velocidade do som vS e
desse modo teremos que:
v
λ = S = 0,419m
f1

Capítulo 18 - Halliday, Resnick e Walker - 6a. edição

45 Duas cordas de piano idênticas têm uma frequência fundamental de 600Hz , quando
colocadas sob a mesma tensão. Que aumento fracionário na tensão de uma corda irá
levar à ocorrência de 6batimentos , quando as cordas oscilarem juntas?

Vamos considerar a interação de duas ondas:

s1(x,t) = sM cos(k1 x - w1 t)
e
s2(x,t) = sM cos(k2 x - w2 t)
logo:
s(x,t) = s1(x,t) + s2(,x,t)

s(x,t) = sM [ cos(k1 x - w1 t) + cos(k2 x - w2 t) ]

Vamos definir algumas grandezas:


  w1 + w 2 
∆w = w 1 − w 2 w =  
  2 

 e 
 ∆k = k − k   k1 + k 2 
 1 2
k = 
  2 

Considerando a identidade trigonométrica:

α + β  α − β 
cos α + cos β = 2 cos  cos 
 2   2 
encontramos que
 ∆k ∆w 
s( x, t ) = 2s M cos x− (
t  cos k x − w t )
 2 2 

Para simplificar, e sem perda de generalidade, vamos analisar a interferência entra


as ondas para o ponto x = 0 . Neste caso:

Cap 18 www. f isica. uf pb. br / ~r omer o 22


www.cliqueapostilas.com.br

Pr of . Romer o T avar es da Silva

 ∆w 
s(0, t ) = 2s M cos t  cos(w t )
 2 

onda a frequência de batimento wB = ∆w . Por outro lado:

∆f = f B = 6batimentos / s = 6Hz




 f1 = 600Hz

f2 = f1 - ∆f = 600 - 6

f2 = 594Hz

Como as duas cordas tem a mesma densidade e o mesmo tamanho, vão vibrar com
mesmo comprimento de onda, mas com frequências diferentes.

T 1 T
v= = λf ⇒ f =
µ λ µ
ou seja:
1 T1
2
f1 λ µ T1 T1  f1 
= = ⇒ = 
f2 1 T2 T2 T2  f 2 
λ µ
logo
2
∆T T1 − T 2 T f 
= = 1 − 2 = 1 −  2  =1 - 0,9801 = 0,0199
T1 T1 T1  f1 

 ∆T 
 % = 1,99%
 T 

Capítulo 18 - Halliday, Resnick e Walker - 6a. edição

46 O vigilante rodoviário B está perseguindo o motorista A por uma estrada estreita.


Ambos se movem a velocidade de 160km/h . O vigilante rodoviário, não conseguindo
alcançar o infrator faz soar a sua sirene. Considera a velocidade do som no ar como
sendo 343m/s e a frequência da sirene como sendo 500Hz .Qual a mudança Do-
ppler na frequência ouvida pelo motorista A ?

vF = vo = 160km/h = 44,45m/s
v = 343m/s
f = 500Hz
 v  sinal sup erior : aproximando − se
f ' = f   
 v " v F   sinal inf erior : afas tan do − se
Cap 18 www. f isica. uf pb. br / ~r omer o 23
www.cliqueapostilas.com.br

Pr of . Romer o T avar es da Silva

Neste problema: a fonte se aproxima do observador e este observador se afasta da


fonte. Com o adendo que as duas velocidades são iguais, logo:

v −vo 
f ' = f   ∴ f ' = f
v −vF 

Capítulo 18 - Halliday, Resnick e Walker - 4a. edição

“48” Uma onda sonora de frequência 1000Hz, se propagando através do ar, tem uma
amplitude de pressão de 10Pa .

∆pM = 10Pa
f = 103Hz

a) Qual é o comprimento de onda?

v = 343m/s
v
λ= = 0,343m
f

b) Qual é a amplitude de deslocamento da partícula?

w = 2π f = 6,28x103rad/s
k = 2π/λ = 18,31rad/m
∆p = ∆pM sen(kx - wt)

s(x,t) = sM cos(kx - wt)

 ∂s   ∂s 
∆p = −B   = − ρ v 2  
 ∂x   ∂x 

∆p = - B [- k yM sen(kx - wt)]
ou seja:
∆p M ∆p M ∆p M
∆p M = kBs M ⇒ sM = = =
kB kρv 2
ρvw

sM = 3,83x10-7m

c) Qual é a velocidade máxima da partícula?

∂s( x, t )
u( x, t ) = = ws M sen( kx − wt )
∂t

uM = w sM = 2,4x10-3m/s = 0,24cm/s

d) Um tubo de órgão, aberto nas duas extremidades, tem essa frequência como
fundamental. Qual o comprimento do tubo?

Cap 18 www. f isica. uf pb. br / ~r omer o 24


www.cliqueapostilas.com.br

Pr of . Romer o T avar es da Silva

Quando temos um tubo aberto em ambas as extremidades:

 λ 2L
L = n 2 ⇒ λ N = n


 v v
 f = λ ⇒ f N = n 2L

λ
n =1 ⇒ L= ∴ L = 0,171m
2

Capítulo 18 - Halliday, Resnick e Walker - 6a. edição

48 Uma ambulância, tocando sua sirene a 1600Hz ultrapassa um ciclista, que estava
pedalando uma bicicleta a 2,44m/s . Depois da ambulância ultrapassá-lo, o ciclista
escuta a sirene a 1590Hz . Qual a velocidade da ambulância?

f = 1600Hz v = 343m/s
f' = 1590Hz vo = 2,44m/s

 v ± vo  sinal sup erior : aproximando − se


f ' = f   
v " vF   sinal inf erior : afas tan do − se

Depois que a ambulância ultrapassa o ciclista, ela passa a se afastar dele que cami-
nha na direção dela: a fonte se afasta do observador que se aproxima desta fonte:

 v + vo  f − f' f 
f ' = f   ⇒ vF =   v +   v o = 4,61m/s
v + vF   f'  f'

Capítulo 18 - Halliday, Resnick e Walker - 6a. edição

49 Um apito de frequência 540Hz move-se em uma trajetória circular de raio 60cm


com uma velocidade de 15rad/s .

Quais são as menores e maiores frequências ouvida por um ouvinte a uma gran-
de distância e em repouso em relação ao centro do círculo?

f = 540Hz
vF = w r = 9m/s r = 60cm = 0,6m
w = 15rad/s

Cap 18 www. f isica. uf pb. br / ~r omer o 25


www.cliqueapostilas.com.br

Pr of . Romer o T avar es da Silva

 v ± vo  sinal sup erior : aproximando − se


f ' = f    1 2
v " vF   sinal inf erior : afas tan do − se

Quando o observador está fixo, temos duas possíveis


situações:
 v 
f1\ = f   fonte aproximando − se
v − vF 
 v 
f 2\ = f   fonte afas tan do − se
 v + v F 

f'2 = 525,66Hz
Observador
f'1 = 555,14Hz

Capítulo 18 - Halliday, Resnick e Walker - 4a. edição

“50” Uma onda sonora em um meio fluido é refletida em uma barreira, de tal modo que
uma onda estacionária é formada. A distância entre os nós é de 3,8cm e a veloci-
dade de propagação é de 1500m/s .Encontre a frequência.

A barreira funciona com um nó e a fonte também será considerada como um nó.


Desse modo, o maior comprimento de onda dessa onda estacionária será tal que:

λ
d=
2
Desse modo, temos que:
v v
f = = = 19.736,8Hz
λ 2d

Capítulo 18 - Halliday, Resnick e Walker - 6a. edição

51 Um submarino francês e um submarino norte-americano movem-se um em direção


ao outro, durante manobras em águas paradas no Atlântico Norte. O submarino
francês move-se a 50,0km/h e o subma-
rino americano a 70,0km/h . O submari- VFR VAM
no francês envia um sinal de sonar (onda
sonora na água) a 1.000Hz . As ondas
de sonar se propagam a uma velocidade Francês Americano
de 5470km/h .

a) Qual a frequência do sinal quando detectado pelo submarino norte-americano?

VFR = 50km/h f = 1.000Hz


VAM = 70km/h VS = 5.470km/h

Cap 18 www. f isica. uf pb. br / ~r omer o 26


www.cliqueapostilas.com.br

Pr of . Romer o T avar es da Silva

Quando o submarino francês emite uma onda de frequência f e ela é captada


pelo submarino americano com uma frequência f' enquanto os dois se aproxi-
mam, temos uma situação onde a fonte se aproxima do observador que por sua
vez está também se aproximando desta fonte. Considerando que:

 v ± vo  sinal sup erior : aproximando − se


f ' = f   
v " vF   sinal inf erior : afas tan do − se

temos que:
 V + V AM 
f ' =  S  f = 1022,2Hz
 VS − VFR 

b) Qual a frequência detectada pelo submarino francês do sinal refletido de volta


para ele pelo submarino norte-americano?

Quando o submarino americano refletir as ondas emitidas pelo submarino fran-


cês, o americano funcionará como uma fonte que se aproxima do observador e
o francês como um observador que se aproxima da fonte. Desse modo:

 V + VFR 
f ' ' =  S  f '
 VS − V AM 
ou seja:
 V + VFR   VS + V AM 
f ' ' =  S    f = 1044,4Hz
 VS − V AM   VS − VFR 

Capítulo 18 - Halliday, Resnick e Walker - 4a. edição

54 Um morcego está voando rapidamente sem ficar em um lugar por muito tempo em
uma caverna, navegando por meio de pulsos sonoros ultra-sônicos. Suponha que a
frequência de emissão sonora do morcego seja de 39.000Hz. Durante uma rápida
arremetida em direção à uma superfície de uma parede plana, o morcego está se
movendo a 0,025 a velocidade do som. Que frequência o morcego escuta refletida
pela parede?
f = 39.000Hz
vM = 0,025 vS
 v ± v o  sinal sup erior : aproximando − se
f ' = f   
 v " v F   sinal inf erior : afas tan do − se

Um observador junto à parede observará uma onda vindo do morcego com fre-
quência
 vS 
f ' =   f
v
 S − v M 

Essa será a frequência refletida pela parede. Como o morcego está se aproximando
desta “nova fonte”, ele observará vindo da parede uma onda com frequência:

Cap 18 www. f isica. uf pb. br / ~r omer o 27


www.cliqueapostilas.com.br

Pr of . Romer o T avar es da Silva

Essa será a frequência refletida pela parede. Como o morcego está se aproximando
desta “nova fonte”, ele observará vindo da parede uma onda com frequência:

v + vM 
f " =  S f '
 vS 
Logo
v + vM  v S  v + vM 
f " =  S   f =  S  f
 vS  v S − v M   vS − vM 
ou seja:
f” = 1,051 f = 40.989Hz

Capítulo 18 - Halliday, Resnick e Walker - 4a. edição

55 Uma menina está sentada próxima à janela de um trem que está se movendo com
uma velocidade de 10m/s para o Leste. O tio da menina está de pé próximo aos
trilhos e vê o trem se afastar. O apito da locomotiva emite um som com a frequência
de 500Hz . O ar está parado.
!
vT = 10m/s Tio vT
v = 343m/s
f = 500hz
 v ± vo  sinal sup erior : aproximando − se
f ' = f   
v " vF   sinal inf erior : afas tan do − se

a) Que frequência o tio ouve?

Como o tio - observador está parado e a fonte – trem está em movimento, te-
mos que:
 v 
f ' =   f = 485,71Hz
 v + vT 

b) Que frequência a menina ouve?

A menina – observador se move na direção do apito – fonte que move-se afas-


tando-se da menina, e como ambos estão ligados à locomotiva, eles movimen-
tam-se com a mesma velocidade. Desse modo temos que:

 v + vo 
f ' =   f
v + vF 
e como vo = vF , temos que
f = f’ = 500Hz

c) Um vento começa a soprar vindo do Leste a 10m/s . Que frequência o tio ouve
agora?

O ar é o referencial privilegiado. Em relação à atmosfera, o tio viaja para o leste


Cap 18 www. f isica. uf pb. br / ~r omer o 28
www.cliqueapostilas.com.br

Pr of . Romer o T avar es da Silva

com velocidade vO = 10m/s , e o trem viaja para leste com velocidade


vF= 10m/s + 10m/s = 20m/s . Desse modo, teremos que:

 v + vO   343 + 10 
f ' =   f =  f
v + vF   343 + 20 

f’ = 486,11Hz

d) Que frequência a menina ouve agora?

Apesar da menina e o apito terem modificado as suas velocidades, elas conti-


nuam sendo iguais entre si, logo teremos o mesmo resultado anterior:

 v + vo 
f ' =   f
v + vF 
e como vo = vF , temos que
f = f’ = 500Hz

Capítulo 18 - Halliday, Resnick e Walker - 4a. edição

“69” Uma fonte F gera ondas na superfície de um lago, como mostradas na figura à
seguir. A velocidade das ondas é 5,5m/s e a distância de crista à crista é 2,3m .
Você está em um pequeno bote, se dirigindo diretamente para F com velocidade
constante de 3,3m/s em relação à costa. Qual a frequência das ondas que você
observa?

v = 5m/s
λ = 2,3m
vo = 3,3m/s vo

v
f = = 2,17Hz
λ

v + vo v + vo v + vo 
f'= = ⇒ f'=   f = 1,66 . 2,17Hz
λ v  v 
f
f' = 3,6Hz

Capítulo 18 - Halliday, Resnick e Walker - 4a. edição


Um apito usado para chamar cães tem uma frequência de 30kHz . O cão, entre-
“71” tanto o ignora. O dono do cão, que não pode escutar frequências acima de 20kHz ,
decide usar o efeito Doppler para descobrir se o apito funciona de maneira adequa-
da. Pede a uma migo que sopre o apito no interior de um carro em movimento, en-
quanto ele permanece parado ouvindo.

a) Qual precisa ser a velocidade do carro para que o dono escute o apito a 20kHz
(se ele estiver funcionando) ?

Cap 18 www. f isica. uf pb. br / ~r omer o 29


www.cliqueapostilas.com.br

Pr of . Romer o T avar es da Silva

f = 30kHz
f' = 20kHz
v = 343m/s
 v  sinal sup erior : aproximando − se
f ' = f   
v " vF   sinal inf erior : afas tan do − se

Como desejamos detectar uma frequência f' menor que aquela emitida, deve-
mos escolher a situação tal que:
 v 
f ' = f  
v + vF 

ou seja, o amigo no carro deve adotar uma direção tal que se afaste do dono do
cão. Desse modo temos que:

f − f'
v F = v  = 171,6m/s = 617km/h
 f' 

b) Refaça para uma frequência do apito igual a 22kHz, em vez de 30kHz .

Se a frequência do apito for mudada para f = 22kHz , teremos:

vF = 34,3m/s = 123, 48km/h

Cap 18 www. f isica. uf pb. br / ~r omer o 30


www.cliqueapostilas.com.br

Pr of . Romer o T avar es da Silva

19. Temperatura, Calor e Primeira Lei da Termodinâmica

Temperatura
O tato constitui uma das maneiras mais simples de fazer uma distinção entre cor-
pos quentes e frios. Mas essa maneira de avaliação é bastante imprecisa, e além do mais
poderá causar dificuldades se as temperaturas dos corpos estiverem muito próximas. Se
construirmos uma experiência com três recipientes contendo água, onde um deles está a
temperatura ambiente, o segundo a uma temperatura acima da ambiente e o terceiro a
uma temperatura abaixo da ambiente. Vamos mergulhar uma das mãos no recipiente com
água a uma temperatura acima da ambiente e a outra mão no recipiente com água a uma
temperatura abaixo da ambiente, e permanecer pouco mais de um minuto nessa situação.
Ao mergulhar as duas mãos no recipiente a temperatura ambiente iremos ter a sensação
estranha onde uma mão manda a informação que a água está numa certa temperatura
enquanto a outra mão manda uma informação de uma temperatura diferente. A mão que
estava no recipiente com água mais fria sente a água mais quente, e a mão que estava
no recipiente com água mais quente sente a água mais fria.
Felizmente existem substâncias que nos dão uma medida da temperatura de ou-
tros corpos e a relação entre elas. São chamadas de substâncias termométricas.
A temperatura é uma medida da agitação das partículas que compões um certo
material. Se considerarmos as moléculas um gás, quanto maior a sua temperatura mais
energia cinética terão essas moléculas.

Equilíbrio térmico
Dois corpos em contato físico, estão em equilíbrio térmico quando param de trocar
energia, quando o fluxo líquido de energia entre eles é nulo. Quando isso acontece, a
temperatura dos dois corpos é a mesma.

Lei Zero da Termodinâmica


Se dois corpos A e B estão em equilíbrio térmico com um terceiro corpo C (o
termômetro) , eles também estarão em equilíbrio térmico entre si.

Medindo a Temperatura
Existem várias grandezas que variam as suas características quando varia a nossa
percepção fisiológica de temperatura. Entre essas grandezas estão:
- o volume de um líquido,
- o comprimento de uma barra
- a resistência elétrica de um material
- o volume de um gás mantido a pressão constante

Qualquer dessas pode ser usada para construir um termômetro, isto é: estabelecer
uma determinada escala termométrica. Uma tal escala termométrica é estabelecida pela
escolha de uma determinada substância termométrica e também uma propriedade ter-
mométrica desta substância.

Cap 19 www. f isica. uf pb. br / ~r omer o 2


www.cliqueapostilas.com.br

Pr of . Romer o T avar es da Silva

Deve-se entender que a cada escolha de uma substância, da sua respectiva proprie-
dade termométrica, e da relação admitida entre essa propriedade e a temperatura, conduz
a uma escala termométrica específica. As medidas obtidas nesta escala não devem coin-
cidir necessariamente com as medidas realizadas em outra escala termométrica definida
de forma independente. Justamente por essa liberdade na construção de uma escala ter-
mométrica, historicamente apareceram diversas escalas com leituras completamente dife-
rentes de temperaturas.

Esse caos foi removido utilizando como padrão uma dada substância termométrica, e
a dependência funcional entre a propriedade termométrica dessa substância e a tempe-
ratura T . Como exemplo, consideremos que exista uma relação linear entre uma proprie-
dade termométrica X e a temperatura, de modo que:

T(X) = a X + b

onde X é o comprimento da uma coluna de


mercúrio em um termômetro e a e b são X
constantes a serem determindas.
Analisando essa relação para duas temperaturas diferentes T1 e T2 , encontramos que:

 T ( X 2 ) − T ( X1 )
 a =
T ( X 1 ) = aX 1 + b X 2 − X1
 
 ∴ 
T ( X ) = aX + b  X 2T ( X 1 ) − X 1T ( X 2 )
 2 2
b =
 X 2 − X1

usando os valores das constantes, temos que:

T ( X 2 ) − T ( X 1 )   X 2T ( X 1 ) − X 1T ( X 2 ) 
T(X) =  X +  
 X 2 − X1   X 2 − X1 
ou ainda:
 X − X1   X2 − X 
T(X) =  T ( X 2 ) +  T ( X 1 )
 X 2 − X1   X 2 − X1 
e finalmente
 X − X1 
T ( X ) = T ( X 1 ) +   [T ( X 2 ) − T ( X 1 )]
X
 2 − X 1 

A escala Celsius
Para calibrar este termômetro na escala Celsius vamos considerar que as tempe-
raturas T(X1)=00C e T(X2)=1000C são respectivamente o ponto de vapor e o ponto do
gelo, e que X1 e X2 são os respectivos comprimentos da coluna de mercúrio. Desse
modo, encontramos que:
 X − X0 
TC ( X ) = 
X − X
0
 100 C ( )
 100 0 

Cap 19 www. f isica. uf pb. br / ~r omer o 3


www.cliqueapostilas.com.br

Pr of . Romer o T avar es da Silva

Isso equivale a dividir a escala entre X0 e X100 em cem partes iguais, cada subdivisão
correspondendo a 10C , ou seja equivale a dizer que a dilatação da coluna de mercúrio é
linear com T(X).

A escala Fahrenheit
A escala Fahrenheit é usada nos Estados Unidos e Inglaterra. Para calibrar este
termômetro na escala Celsius vamos considerar que as temperaturas T(X1)=320C e
T(X2)=2120C são respectivamente o ponto de vapor e o ponto do gelo, e que X1 e X2
são os respectivos comprimentos da coluna de mercúrio. Desse modo, encontramos que:

 X − X 32 
TF ( X ) = 32 0 F +  ( 0
 180 F )
 X 212 − X 32 

Relação entre as escalas Celsius e Fahrenheit


Se considerarmos dois termômetros de mesmo formato, feitos do mesmo material
e calibrados nestas escalas, podemos dizer que quando estiverem medindo a mesma si-
tuação, a coluna terá um tamanho X , e portanto:

TF − 32 0 F TC X − X Gelo
0
= 0
=
180 F 100 C X Vapor − X Gelo
ou seja:
9
TF = 32 +   TC
5
ou ainda:
5
TC =  (TF − 32)
9

A escala Kelvin
Se considerarmos o comportamento de um gás de N moléculas, constata-se expe-
rimentalmente que para uma dada temperatura:

pV
= const
N

onde p é a pressão do gás e V é o volume ocupado por ele. Esta é a equação dos ga-
ses ideais é comprova-se que ela é válida sempre que a densidade N/V for pequena. A
escala de temperaturas Kelvin é definida de modo que a relação entre a constante e a
temperatura seja de proporcionalidade. Em outras palavras, a escala Kelvin é tal que:

pV
= k BT
N
onde kB é a constante de Boltzmann. Usando o raciocínio anterior, relembramos que a
substância termométrica nesse caso é um gás e a propriedade termométrica é a pressão
desse gás a volume constante. Temos então que:

Cap 19 www. f isica. uf pb. br / ~r omer o 4


www.cliqueapostilas.com.br

Pr of . Romer o T avar es da Silva

T ( X 0 ) 
T ( X ) = aX ∴ T(X) =  X
 X0 

Considerando o ponto triplo da água, escolhemos a temperatura de calibração na


escala Kelvin.
 273,16 K 
T =   p
 pTr 
Uma vez calibrada a escala obtemos o valor de kB = 1,38x10-23J/K . A correspon-
dência entre as escalas Celsius e Kelvin é tal que:

00C = 273,16K
1000C = 373,15K
ou seja:
TK = TC + 273,16

Dilatação térmica
Quando aumentamos a temperatura de um sólido ele se dilata. A dilatação térmica
desse sólido está associada ao aumento da distância entre os átomos vizinhos que o
compõe. Poderíamos dizer que a força de interação elétrica entre esses átomos já não é
suficiente para mantê-los tão próximos um dos outros devido a agitação térmica oriunda
do aumento da temperatura.
Consideremos que em uma temperatu-
ra inicial TI um sólido tenha um compri- L0 ∆L
mento L0 . Se aumentarmos a temperatura
de ∆T , esse sólido aumentará o seu com-
primento de ∆L . Para uma dada variação L
de temperatura podemos entender que a
a dilatação do sólido ∆L será proporcional ao seu comprimento inicial L0 . Para uma va-
riação de temperatura suficientemente pequena, podemos ainda inferir que a dilatação do
sólido ∆L também será proporcional ao aumento da temperatura ∆T . Desse modo, po-
demos resumir, como:
∆L = α L0 ∆T
onde a constante de proporcionalidade α é chamada de coeficiente de dilatação linear
do material considerado. Como
∆L = L – L0
L = L0 ( 1 + α ∆T )
Para muitos sólidos os coeficientes de dilata-
ção é o mesmo nas suas diversas dimensões. Dize- L01
mos que eles têm uma dilatação isotrópica. Vamos
considerar que uma chapa plana tenha dimensões L01
e L02 para uma dada temperatura inicial. Quando va-
riamos a temperatura de ∆T as dimensões se alteram L02
para L1 e L2 conforme a figura ao lado. Consideran-
do que os coeficiente de dilatação são os mesmos nas
duas dimensões, teremos que: L1

L1 = L01 ( 1 + α ∆T )
L2 = L02 ( 1 + α ∆T ) L2

Cap 19 www. f isica. uf pb. br / ~r omer o 5


www.cliqueapostilas.com.br

Pr of . Romer o T avar es da Silva

As áreas inicial e final podem ser definidas como:

A0 = L01 L02
e
A = L1 L2

A =[ L01 ( 1 + α ∆T )] [ L02 ( 1 + α ∆T )]
ou seja:
A = A0 [ 1 + 2 α ∆T + (α ∆T)2 ]

A aproximação da dilatação térmica ∆L = α L0 ∆T é válida apenas igualmente para


todos os materiais apenas em circunstâncias restritas, ou seja quando α ∆T << 1 , e des-
se modo podemos afirmar que:
α ∆T >> (α ∆T)2
ou seja:
A = A0 [ 1 + 2 α ∆T]
Quando lidamos com dilatação volumétrica de sólidos, podemos usar um raciocínio
similar e encontrar que:
V = V0 [ 1 + 3 α ∆T]

Em sólidos isotrópicos o coeficiente de dilatação superficial é definido como γ = 2α


e o coeficiente de dilatação volumétrica é definido como β = 3α .

Calor
No final do século XVIII, existiam duas hipóteses alternativas sobre o calor. A hi-
pótese mais aceita considerava o calor como uma substância fluida indestrutível que
“preencheria os poros” dos corpos e escoaria de um corpo mais quente a um mais frio.
Lavoisier chamou esta substância hipotética de “calórico”. A implicação era que o calor
pode ser transferido de um corpo a outro, mas a quantidade total de “calórico” se conser-
varia, ou seja, existiria uma lei de conservação de calor.
A hipótese rival, endossada entre outros por Francis Bacon e Robert Hooke, foi as-
sim expressa por Newton em 1704: “O calor consiste num minúsculo movimento de vibra-
ção das partículas dos corpos”.
A principal dificuldade estava na “lei de conservação do calórico”, pois a quantidade
de calórico que podia ser “espremida para fora” de um corpo por atrito era ilimitada. Com
efeito, em 1798, Rumford escreveu: “Foi por acaso que me vi levado a realizar as experi-
ências que vou relatar agora...Estando ocupado ultimamente em supervisionar a perfura-
ção de canhões nas oficinas do arsenal militar de Munique, chamou-me a atenção o ele-
vado grau de aquecimento de um canhão de bronze, atingido em tempos muito curtos,
durante o processo de perfuração...A fonte de calor gerado por atrito nestas experiências
parece ser inesgotável ... e me parece extremamente difícil de conceber qualquer coisa
capaz de ser produzida ou transmitida da forma como o calor o era nestas experiências,
exceto o MOVIMENTO.
Rumford foi levado a endossar a teoria alternativa de que “...o calor não passa de
um movimento vibratório que tem lugar entre as partículas do corpo”.
H. Moysés Nussenzveig
Curso de Física Básica – Vol2 – 4a. edição
Editora Edgard Blücher Ltda.
São Paulo - 2002

Cap 19 www. f isica. uf pb. br / ~r omer o 6


www.cliqueapostilas.com.br

Pr of . Romer o T avar es da Silva

Um olhar mais de perto no Calor e Trabalho


Calor Q é energia em trânsito de um corpo para outro devido à diferença de tem-
peratura entre eles.
Trabalho W é a energia que é transferida de um sistema para outro de tal modo
que a diferença de temperaturas não esteja envolvida.
As grandezas Q e W não são características do estado de equilíbrio do sistema,
mas sim dos processos termodinâmicos pelos quais o sistema passa quando vai de um
estado de equilíbrio para outro. Desse modo, se um sistema vai de um estado de equilí-
brio inicial para um outro estado de equilíbrio final, por dois caminhos diversos, para cada
caminho ele terá um valor de Q e W específico.
Q e W são definidos como:
Q = calor transferido para o sistema
W = trabalho realizado pelo sistema
De modo geral, nós separamos uma certa quantidade de material que desejamos
analisar. A esse material chamamos de sistema, que pode estar isolado (ou não) da sua
vizinhança. A interação com a vizinhança pode ser de vários tipos: trocando calor, trocan-
do trabalho, ou ambos os casos simultaneamente.
Um sistema sofre transformações que o levarão de um estado de equilíbrio inicial a
um estado final, através de diversos estados intermediários. O caminho entre os estados
inicial e final, através dos estados intermediários se dá por causa da interação do sistema
com a sua vizinhança.
Para exemplificar, calculemos o tra-
balho feito por um sistema formado por um
gás isolado no interior de um pistão, cujo
êmbolo pode movimentar-se livremente sem
atrito. Considere que inicialmente o êmbolo
estava preso e continha um volume Vi ,
após ser solto ele moveu-se e o volume Vf
passou a ser Vf , quando então ele tornou a Vi
ser preso. O êmbolo subiu como conse-
quência da pressão p exercida pelo gás. O
trabalho elementar feito por esse sistema é
definido como:
dW = F dx = p A dx
ou seja: quando o êmbolo moveu-se de dx , sob a ação de uma pressão interna p , o
sistema executou um trabalho dW . A área do êmbolo é A , daí a variação de volume
associada a dx é igual a dV = A dx , e portanto:

dW = p dV

O trabalho total executado pelo siste- p


ma entre os estados inicial e final, é definido
como:
f pi i
W if = ∫ p dV
i
e considerando a definição de integral, te- pf a f
mos que esse trabalho será a área abaixo
da curva que vai do estado inicial até o es-
tado final. Vi Vf V
Cap 19 www. f isica. uf pb. br / ~r omer o 7
www.cliqueapostilas.com.br

Pr of . Romer o T avar es da Silva

Como já tínhamos antecipado o valor do trabalho associado á mudança de estado


do sistema não é único. Quando o sistema for do estado inicial até o final através do esta-
do dos percursos ia e af o trabalho associado a esse percurso será diferente daquele
considerado inicialmente.

A absorção de Calor
Quando uma certa quantidade de calor é transmitida para um corpo, na maioria dos
casos a sua temperatura cresce. A quantidade de calor necessária para aumentar de um
certo valor a temperatura de uma substância, depende da quantidade dessa substância, e
varia de acordo com a substância. Se foi necessário 3min para ferver 1litro de água
numa certa chama, serão necessários 6min para ferver 2litros de água na mesma cha-
ma. Se no entanto formos aquecer 1litro azeite na mesma chama, será necessário um
tempo maior que 3min.
A propriedade física que define a quantidade de calor Q necessária para aquecer
determinado material de ∆T é chamada capacidade térmica, e é definida como:

Q = C . ∆T

Desse modo poderemos calcular a capacidade térmica de 1litro de água, de


2litros de água, de 1litro azeite e etc. A capacidade térmica é uma característica de uma
amostra de determinada substância. Outra amostra diferente dessa mesma substância
terá uma capacidade térmica diferente.
Fica claro que são limitadas as vantagens dessa propriedade física, a capacidade
térmica. Mas à partir dela, definiu-se uma outra propriedade chamada calor específico c ,
que é uma característica de cada substância.
A propriedade física que define a quantidade de calor Q necessária para aquecer
de ∆T uma massa m de determinado material é chamada calor específico, e é definida
como:
Q = m . c . ∆T

Como foi mencionado, calor é uma forma de energia e portanto a unidade de calor
é a mesma de energia. Mas por razões históricas, ainda se usa como unidade de calor a
caloria ou cal, que se define como a quantidade de calor necessária para aquecer 1g
de água de 14,50C até 15,50C. Desse modo, a unidade do calor específico será cal/g.0C.
Como foi mencionado, uma substância altera a sua temperatura quando ela troca
calor com a sua vizinhança. No entanto, existem algumas situações onde não acontece
exatamente desse modo; um corpo pode absorver certa quantidade de calor e no entanto
manter-se com a sua temperatura constante. Quando isso acontece, diz-se que o corpo
passou por uma mudança de fase. Existe um exemplo corriqueiro: uma pedra de gelo
numa temperatura de 00C é retirada do congelado e colocada dentro de um copo na
temperatura ambiente de 300C . Esse material irá absorver calor da sua vizinhança e
paulatinamente transformar-se-á em água a uma temperatura de 00C .
A propriedade física que define a quantidade de calor Q necessária para uma mu-
dança de fase de uma massa m de determinada substância é chamada calor latente, e é
definida como:
Q=mL

Quando estamos considerando a mudança do estado sólido para o estado líquido,


chamamos de calor latente de fusão LF , e quando estamos considerando a mudança do

Cap 19 www. f isica. uf pb. br / ~r omer o 8


www.cliqueapostilas.com.br

Pr of . Romer o T avar es da Silva


estado líquido para o estado gasoso, chamamos de calor latente de vaporização LV . A
unidade do calor latente é cal/g .

Primeira Lei da Termodinâmica

Quando um sistema termodinâmico


vai de um estado inicial i para um estado p
final f , ele pode fazer este “caminho” atra-
vés de vários “percursos”. Na figura ao lado
estão ilustrados dois “percursos” ; direta- pi i
mente ao longo da curva - (1) ou ao pas-
sando pelo estado a – (2) . em cada per-
curso o trabalho executado pelo sistema pf a f
tem um resultado diferente. Por outro lado,
a troca do o sistema com a sua vizinhança
também é diferente em cada um dos dois Vi Vf V
percursos.
Define-se uma grandeza, chamada energia interna E , caracterizada pelos diversos
tipos de energia possíveis de existir em uma substância quando ela está em determinado
estado.
Se tivéssemos um gás diatômico, a energia interna desse gás em determinado es-
tado teria uma parte associada ao seu movimento (energia cinética de translação), outra
parte associada a rotação de um átomo em torno do outro (energia cinética de rotação),
outra parte associada à oscilação de um átomo em relação ao outro (energia potencial
elástica), e outros tipos de energia, de acordo com o modelo usado para descrever a mo-
lécula e o gás a que ela pertence.
No caso, mais simples, de um gás ideal monoatômico, a energia interna depende
apenas do movimento dos átomos.
A diferença de energia interna entre os estados inicial e final ∆EInt = EF - EI é uma
grandeza de grande importância na termodinâmica, porque independente do percurso
usado para ir de um estado para o outro, teremos sempre que:

∆EInt = QIF – WIF = QIAF – WIAF

onde podemos definir a Primeira Lei da Termodinâmica como:

∆EInt = Q - W

A diferença entre a quantidade de calor Q e o trabalho envolvidos em um percurso


entre os estados inicial e final, depende apenas dos estados, e fornece o mesmo valor
independente do percurso escolhido.

Alguns casos específicos da Primeira Lei da Termodinâmica

Processos adiabáticos
É um processo em que não existe troca de calor entre o sistema e a sua vizinhan-
ça, ou seja: o sistema está muito bem isolado termicamente. Na Natureza existem pro-
cessos que podemos aproximar como adiabáticos. São aqueles que ocorrem tão

Cap 19 www. f isica. uf pb. br / ~r omer o 9


www.cliqueapostilas.com.br

Pr of . Romer o T avar es da Silva

rapidamente que o sistema chega ao seu estado final antes que possa trocar calos com a
vizinhança. Num processo adiabático, Q = 0 e de acordo com a Primeira Lei da Termodi-
nâmica:
∆EInt = - W

Processos a volume constante


São os chamados processos isométri-
cos. Usando a definição de trabalho execu- p
tado pelo sistema entre os estados inicial e
final, encontramos que:
f pi i
W if = ∫ p dV = 0
i
porque não aconteceu variação de volume. pf f
Através da Primeira Lei da Termodinâmica
encontramos que:
Vi = Vf V
∆EInt = Q

Processos cíclicos
Num processo cíclico o sistema passa por várias transformações, mas ao final do
processo ele retorna ao estado inicial. Desse modo, temos que EI = EF e portanto não
existe variação de energia interna, logo:
Q=W

Mecanismos de transferência de Calor


A transferência de calor de um ponto a outro de um meio se dá através de três pro-
cessos diferentes: convecção, radiação e condução.
A convecção ocorre tipicamente num fluido, e se caracteriza pelo fato de que o ca-
lor é transferido pelo movimento do próprio fluido, que constitui uma corrente de convec-
ção. Um fluido aquecido localmente em geral diminui de densidade e por conseguinte
tende a subir sob o efeito gravitacional, sendo substituído por um fluido mais frio, o que
gera naturalmente correntes de convecção. O borbulhar da água fervente em uma panela
é o resultado de correntes de convecção.
A radiação transfere calor de um ponto a outro através da radiação eletromagnéti-
ca. A radiação térmica é emitida de um corpo aquecido e ao ser absorvida por outro corpo
pode aquecê-lo, convertendo-se em calor. O aquecimento solar é uma forma de aprovei-
tamento da radiação solar para a produção de calor. Um ferro em brasa emite radiação
térmica e aquece a região que o rodeia.
A condução de calor só pode acontecer através de um meio material, sem que haja
movimento do próprio meio. Ocorre tanto em fluidos quanto em meios sólidos sob o efeito
de diferenças de temperatura.
H. Moysés Nussenzveig
Curso de Física Básica – Vol2 – 4a. edição
Editora Edgard Blücher Ltda.
São Paulo – 2002

Cap 19 www. f isica. uf pb. br / ~r omer o 10


www.cliqueapostilas.com.br

Pr of . Romer o T avar es da Silva

Quando colocamos uma panela com água no fogo, ele começa a aquecer a água.
Esse processo inicial de aquecimento se dá por condução de calor, e a parte na superfície
da água vai sendo aquecida paulatinamente. No entanto a taxa de aquecimento da água
no fundo da panela é maior do que a taxa de aquecimento da água na superfície. A água
entre o fundo e a superfície não dá conta da condução do calor que é comunicado através
do fogo. Começam a se formar no fundo bolsões de água mais quentes que a vizinhança,
e esses bolsões começam a subir para a superfície. Nesse instante a convecção passa a
ser o processo principal de condução de calor na panela. E isso acontece por causa da
incapacidade da água conduzir calor de maneira adequada nesta panela sobre o fogo.

Condução
Consideremos dois reservatórios tér-
micos que estão a temperaturas diferentes L
TQ e TF, tais que TQ > TF . Estes dois reser-
vatórios serão conectados por uma placa de
área transversal A e comprimento L ,
conforme mostra a figura ao lado. Vamos
supor que a placa está isolada das vizi-
nhanças, de modo que através dela passa
apenas o fluxo de calor entre os reservató- x
rios. Intuitivamente pode-se perceber que a
taxa de transferência de calor dQ/dt que
flui através da placa é proporcional à sua Reservatório quente Reservatório frio
área e a diferença de temperatura entre os TQ TF
reservatórios de calor, e inversamente pro- TQ > TF
porcional ao seu comprimento. Ou seja:
dQ T − TF
= kA Q
dt L

onde a constante de proporcionalidade k é conhecida como condutividade térmica da


barra. Se considerarmos uma placa de comprimento ∆x , que una dois reservatórios que
têm uma diferença de temperatura ∆T , encontraremos que:

dQ ∆T
= −kA
dt ∆x

onde o sinal negativo exprime o fato que o calor flui de temperaturas mais quentes para
temperaturas mais frias. Quando tivermos ∆x → 0 , encontraremos que:

dQ dT
= −kA
dt dx

No estado estacionário, a temperatura na barra não depende mais do tempo t , e o


fluxo de calor é o mesmo em qualquer parte da barra. Desse modo dQ/dt é uma cons-
tante, e a equação anterior toma a forma:

dT Ρ Ρ
Ρ = −kA ⇒ dT = − dx ∴ TF − TQ = − (x F − x Q )
dx kA kA

Cap 19 www. f isica. uf pb. br / ~r omer o 11


www.cliqueapostilas.com.br

Pr of . Romer o T avar es da Silva

ou seja: L
Ρ
∆T = L
kA
logo:
dQ ∆T T − TF
Ρ= = kA = kA Q
dt L L
x
e desse modo poderemos calcular o fluxo
de calor através da placa. Se quisermos
saber como varia a temperatura ao longo da TQ TF
barra, podemos usar que:
T
Ρ Ρ TQ
dT = − dx ⇒ T (x) = − x + TQ
kA kA

 T − TQ  TF
T ( x ) =  F  x + TQ
 L  L x

Condução através de uma parede composta


Consideremos dois reservatórios térmi-
cos que estão a temperaturas diferentes TQ L2 L1
e TF, tais que TQ > TF . Estes dois reservató-
rios serão conectados por duas placas de
mesma área transversal A ; comprimentos
L1 e L2 e condutividades térmicas k1 e k2
respectivamente , conforme mostra a figura
ao lado.
Encontre a temperatura na junção x
das placas o fluxo de calor através delas.

O fluxo de calor que sair da fonte Reservatório quente Reservatório frio


quente e atravessar a primeira placa, será o TQ TF
mesmo que irá atravessar a segunda placa TQ > TF
e chegar até a fonte fria. Portanto o fluxo Ρ1
que atravessa a primeira placa é igual ao T
fluxo Ρ2 que atravessa a segunda placa TQ

dQ dQ1 dQ 2 TX
= =
dt dt dt
TF
Mas
dQ dT
Ρ= = −kA L2 L1+L2 x
dt dx
 dT TQ − T X
 Ρ1 = −k 1 A ∴ Ρ1 = k 1 A
 dx L2

 dT T X − TF
Ρ2 = −k 2 A ∴ Ρ2 = k 2 A
 dx L1

Cap 19 www. f isica. uf pb. br / ~r omer o 12


www.cliqueapostilas.com.br

Pr of . Romer o T avar es da Silva

No entanto
TQ − T X T X − TF
Ρ1 = Ρ2 ⇒ k2 A = k1A
L2 L1
ou seja:
TX ( L2 k1 + L1 k2 ) = TQ L1 k2 + TF L2 k1

 k k 
L1L2 TQ 2 + TF 1 
T L1k 2 + TF L2 k 1  L2 L1 
TX = Q =
L2 k 1 + L1k 2 k k 
L1L2  2 + 1 
 L2 L1 

k2 k
TQ + TF 1
L2 L1
TX =
k 2 k1
+
L2 L1
Por outro lado:
dQ dQ1 dQ 2
= =
dt dt dt
ou seja:
 k2 k  
 TQ + TF 1  
dQ k 1 A k 1 A  L2 L1 
= (T X − TF ) = 
 − TF 
dt L1 L1  k 2 k1
 +  
 L2 L1  
e finalmente:
dQ A (TQ − TF )
=
dt L2 L1
+
k 2 k1

Radiação
A taxa Ρ com que um objeto emite radiação depende da área A da superfície
deste objeto e da temperatura T dessa área em Kelvins, e é dada por:

Ρ = σ ε A T4

Nesta equação σ = 5,67x10-8W/m2K4 é chamada a constante de Stefan-Boltzmann. E a


grandeza ε é a emissividade da superfície do objeto que vale entre 0 e 1 dependendo
da composição da superfície.

Cap 19 www. f isica. uf pb. br / ~r omer o 13


www.cliqueapostilas.com.br

Pr of . Romer o T avar es da Silva

Solução de alguns problemas

Capítulo 19 - Halliday, Resnick e Walker

03 Um certo termômetro a gás é construído com dois bulbos contendo gás, cada um dos
quais é colocado em banho-maria, como mostrado na figura à seguir. A diferença de
pressão entre os dois bulbos é medida por um manômetro de mercúrio como mostra-
do. Reservatórios apropriados, não mostrados no diagrama, mantêm o volume de
gás constante nos dois bulbos.
i. Não há nenhuma diferença de pressão
quando as duas cubas estão no ponto
tríplice da água.
ii. A diferença de pressão é de 120Torr
quando uma das cubas está no ponto
tríplice e a outra está no ponto de ebuli-
ção da água.
iii. Ela vale 90Torr quando uma das cubas
está no ponto tríplice da água e a outra
está a uma temperatura desconhecida a
ser medida.
Qual a temperatura a ser medida?

1Torr = 1mmHg

i. Esse termômetro será construído considerando-se que um dos bulbos estará na


temperatura do ponto triplo e o outro numa temperatura desconhecida, a ser medida.
A diferença de pressão ∆p é a propriedade termométrica a ser usada neste termô-
metro, logo:
T = a ∆p + b
Quando o segundo bulbo também estiver na temperatura do ponto triplo, teremos
que:
TTr = a . 0 + b ∴ b = TTr

ii. Quando o segundo bulbo estiver no ponto de ebulição, teremos que:

TEb = a . p1 + b ; p1 = 120Torr

TEb − b TEb − TTr


a= ∴ a=
p1 p1
ou seja:
 T − TTr 
T =  Eb  ∆p + TTr
 p1 

iii. Para a temperatura desconhecida teremos que:

T = a . p2 + b ; p2 = 90Torr
ou seja:

Cap 19 www. f isica. uf pb. br / ~r omer o 14


www.cliqueapostilas.com.br

Pr of . Romer o T avar es da Silva

 T − TTr 
T =  Eb  p 2 + TTr
 p1 

90Torr
T = (373,16K − 273,16K ) + 273,16
120Torr

T = 348,16K

Capítulo 19 - Halliday, Resnick e Walker

05 A que temperatura os seguintes pares de escala possuem a mesma leitura, se isto


acontecer:

a) Fahrenheit e Celsius.

A relação entre estas escalas é:


5
TC =  (TF − 32)
9

e portanto teremos mesma leitura T0 quando:

5
T0 =  (T0 − 32)
9
ou seja:
T0= - 400C = - 400F
b) Fahrenheit e Kelvin.
Temos que
5
TC =  (TF − 32)
9
e
TK = TC + 273,16
ou seja:
5
TK = 273,16 +  (TF − 32)
9
e portanto teremos mesma leitura T0 quando:
5
T0 = 273,16 +  (T0 − 32)
9
ou seja:
T0 = 574,610F = 574,61K
c) Celsius e Kelvin
A relação entre estas escalas é:
TK = TC + 273,16

e como é uma relação aditiva, não existe a possibilidade de termos as mesmas


leituras nas duas escalas.

Cap 19 www. f isica. uf pb. br / ~r omer o 15


www.cliqueapostilas.com.br

Pr of . Romer o T avar es da Silva

Capítulo 19 - Halliday, Resnick e Walker

07 Observa-se no dia-a-dia que objetos quentes e frios se esfriam o aquecem até a


temperatura do ambiente ao seu redor. Se a diferença de temperatura ∆T ente um
objeto e o seu ambiente (∆T = TObj – TAmb ) não for muito grande, a taxa de resfria-
mento ou de aquecimento de um objeto é proporcional, aproximadamente, a essa
diferença de temperatura; ou seja:
d (∆T )
= − A (∆T )
dt

onde A é constante. O sinal negativo aparece porque ∆T diminui com o tempo se


∆T for positivo e aumenta com o tempo se ∆T for negativo. Essa equação é conhe-
cida como a Lei de resfriamento de Newton.

a) De que fatores depende A ? Qual é a sua unidade?

A depende principalmente da condutividade térmica do objeto. O lado esquerdo


da equação tem unidades de temperatura sobre tempo, e desse modo, a unidade
de A é o inverso de tempo: s-1 .

b) Se em algum instante t = 0 a diferença de temperatura for ∆T0 , mostre que em


um instante posterior ela será
∆T = ∆T0 e – A t

Da equação diferencial, encontramos que:

d (∆T )
= − A dt
∆T
e quando integramos:
ln(∆T ) = − At + c 1
ou seja
∆T (t ) = e c1 e − At = c 2 e − At

Considerando as condições iniciais:

∆T (0) = c 2 = ∆T0
chegamos a:
∆T = ∆T0 e – A t

Capítulo 19 - Halliday, Resnick e Walker

09 Suponha que em uma escala linear de temperatura X , a água ferva a -53,50X e se


congele a -1700X . Qual a temperatura de 340K na escala X ?

Vamos supor que a relação entre a escala X e a escala Kelvin seja linear, ou seja:

X(K) = a . K + b
e ainda temos que:

Cap 19 www. f isica. uf pb. br / ~r omer o 16


www.cliqueapostilas.com.br

Pr of . Romer o T avar es da Silva

X K
T1 -53,50X 373,16K
T2 -170,00X 273,16K
Desse modo:
X1 = a K1 + b

X2 = a K2 + b
logo:
X1 − X 2 − 53,5 − 170,0
X 1 − X 2 = a (K 1 − K 2 ) ∴ a = =
K1 − K 2 373,16 − 273,16
ou seja:
a = 1,165 0X/K
E ainda:
b = X1 – a K1 = - 488,0450X
Portanto:
X(K) = 1,165 . K – 488,045

Quando a temperatura T0 = 340K , usando essa relação anterior, encontramos

T0 = - 91,9450X

Capítulo 19 - Halliday, Resnick e Walker

16 A área S de uma placa retangular é ab .O seu coeficiente de expansão linear é α .


Após um aumento de temperatura ∆T , o lado a aumenta de ∆a e o lado b au-
menta de ∆b . Mostre que se a pequena quantidade (∆a ∆b)/ab for desprezada,
então ∆S = 2 α S ∆T .
∆a = α a ∆T
e a ∆a
∆b = α b ∆T

S=ab b

S + ∆S = ( a + ∆a) ( b + ∆b)
∆b
S + ∆S = a b + a ∆b + b ∆a + ∆a ∆b

S + ∆S = a b + 2 ab α ∆T + a b (α ∆T)2
Considerando que:
2 α ∆T >> (α ∆T)2
teremos
∆S = 2 α S ∆T

Capítulo 19 - Halliday, Resnick e Walker

18 A 200C , uma haste mede exatamente 20,05cm de comprimento em uma régua de


aço. Tanto a haste quanto a régua são colocadas em um forno a 2700C , onde a
haste passa a medir 20,11cm na mesma régua.
Qual o coeficiente de expansão térmica para o material do qual é feita a haste?
Cap 19 www. f isica. uf pb. br / ~r omer o 17
www.cliqueapostilas.com.br

Pr of . Romer o T avar es da Silva

Ti = 200C L0 = 20,05cm L0
Tf = 2700C L’ = 20,11cm
∆T = 2500C αA = 11x10-6 0C-1

Se tivéssemos duas réguas idênticas de aço,


uma a uma temperatura de 200C e a outra a l0
uma temperatura de 2700C , graduadas em
cm , teríamos que l0 a 200C e l a 2700C
se relacionam da seguinte maneira: L’

l
l = l 0 (1 + α A ∆T ) ⇒ = 1 + α A ∆T = 1,00275 l
l0

ou seja: a gradação dilatou-se de 0,275% e consequentemente as medidas efetua-


das deverão ser alteradas desta fração. A gradação da régua sofreu uma variação
percentual igual a variação percentual da régua como um todo. Desse modo, deverí-
amos fazer uma correção na medida L’ realizada pela régua dilatada:

 l 
L =   L' = (1,00275 ).(20,11) ⇒ L = 20,165cm
 l0 
O comprimento da haste dilatada L , medido pela régua não dilatada ( a 200C )
forneceria o resultado L0 :
Como queremos saber o quanto a haste se dilatou, devemos fazer as medidas
antes e depois da dilatação com um instrumento que não se dilatou. Devemos usar L
como sendo o comprimento da haste medido por uma régua que não sofreu dilata-
ção, logo:
L − L0
L = L0 (1 + α H ∆T ) ∴ α B =
L0 ∆T
ou seja:
αH = 23x10-6 0C-1

Capítulo 19 - Halliday, Resnick e Walker

21 Mostre que quando a temperatura de um líquido em um barômetro varia de ∆T e a


pressão é constante, a altura h do líquido varia de ∆h = β h ∆T onde β é o coefici-
ente de expansão volumétrica deste líquido. Despreze a expansão do tubo de vidro.

Vamos considerar que o líquido se expande de acordo


com a equação:
V = V0 ( 1 + β ∆T ) ∆h

Mas como o tubo de vidro do barômetro tem uma dila-


tação desprezível, o líquido só poderá expandir-se ao
longo do comprimento do tubo, que está vazio. Desse
modo, temos que:

V0 = A0 h 0 e V = A0 h

Cap 19 www. f isica. uf pb. br / ~r omer o 18


www.cliqueapostilas.com.br

Pr of . Romer o T avar es da Silva

ou seja:
V – V0 = ( h – h0 ) A0 = ∆h A0
Mas por outro lado:
V – V0 = V0 β ∆T = β h0 A0 ∆T
Portanto:
∆h = h0 β ∆T

Capítulo 19 - Halliday, Resnick e Walker

22 Quando a temperatura de uma moeda de cobre é elevada de 1000C o seu diâmetro


aumenta de 0,18% . Calcule com dois algarismos significativos:

∆T = 1000C
 ∆d   ∆d 
 % = 0,18% ∴   = 0,0018
 d0   d0 
logo
d − d 0 ∆d
d = d 0 (1 + α∆T ) ∴ = = α∆T = 0,0018
d0 d0

a) O aumento percentual da área de uma face.

A − A0 ∆A
A = A0 (1 + γ∆T ) ∴ = = γ∆T = 2α∆T = 0,0036
A0 A0
logo:
 ∆A 
 % = 0,36%
 A0 

b) O aumento percentual da espessura.

∆L
= α∆T = 0,0018
L0
logo:
 ∆L 
 % = 0,18%
 L0 

c) O aumento percentual do volume.

∆V
= β∆T = 3α∆T = 0,0054
V0
logo:
 ∆V 
 % = 0,54%
 V0 

Cap 19 www. f isica. uf pb. br / ~r omer o 19


www.cliqueapostilas.com.br

Pr of . Romer o T avar es da Silva

d) O aumento percentual da massa da moeda.

A massa obviamente não se modifica quando aumenta a temperatura.

e) O coeficiente de expansão linear da moeda.

d − d 0 ∆d
d = d 0 (1 + α∆T ) ∴ = = α∆T = 0,0018
d0 d0
logo:
∆d
α= = 18 x10 − 6 0 C −1
d 0 ∆T

Capítulo 19 - Halliday, Resnick e Walker

23 Um relógio de pêndulo com um pêndulo feito de latão é projetado para medir com
precisão o tempo a 200C . Se o relógio operar a 00C , qual a intensidade de seu
erro, em segundos por hora? O relógio adianta ou atrasa?

Ti = 200C αL = 0,7x10-6 0C-1


Tf = 00C l
∆T = -200C Τ = 2π
g
O período do pêndulo Τ vai se alterar da seguinte maneira:

l'

Τ' g l' l (1 + α L ∆T )
= = = = 1 + α L ∆T = 0,999997
Τ l l l

g
ou seja:
Τ' = Τ 1 + α L ∆T

Como o tempo esfria, a haste do pêndulo se contrai diminuindo o seu tama-


nho, e portanto diminuindo o seu tempo correspondente ao seu período, ou seja : Τ’ <
Τ. Desse modo, o mesmo intervalo de tempo passa a ter mais períodos que antes.
Como o tempo é medido nesse tipo de relógio em relação ao número de períodos o
relógio irá adiantar. Se inicialmente em 10s temos 10 períodos, depois do esfria-
mento teremos mais períodos neste intervalo de tempo, e o relógio irá indicar um in-
tervalo de tempo maior que os 10s iniciais.
Imaginemos a medição de um certo a medição de um certo intervalo de tem-
po t que corresponde a um certo número n de períodos Τ . Temos então que:

t
n=
Τ

Para calcular qual intervalo de tempo t’ será medido quando a temperatura


variar, devemos multiplicar o número de períodos n pelo valor do novo período Τ’ .
Ou seja:
Cap 19 www. f isica. uf pb. br / ~r omer o 20
www.cliqueapostilas.com.br

Pr of . Romer o T avar es da Silva

 Τ'   Τ'   ∆Τ 
t ' = n Τ' = t   ∴ ∆t = t − t ' = t 1 −  = t  
Τ  Τ  Τ 

∆t = t 1 + α L ∆T = 7 x10 −6 t

t - intervalo ∆t - atraso
1 hora 0,0252s
1 dia 0,6048s
1 mês 18,144s

Capítulo 19 - Halliday, Resnick e Walker

25 Como resultado de uma elevação de temperatura de 320C , uma barra com uma fis-
sura no seu centro empena para cima. Se a distância fixa L0 for 3,77m e o coefici-
ente de expansão linear da barra for 25x10-6/0C , determine a elevação x do centro
da barra.

∆T = 320C
L0 = 3,77m
α = 25x10-6 0C-1
L0
L = L0 ( 1 + α ∆T )
e
2 2
L  L0  2
x
  =  +x
2  2
ou seja: L0
2
L    L 
2

x 2 =  0  1 −   
 2    L0  

L0
x= (1 + α∆T )2 − 1 = 0,02L0 = 0,0754m
2

Capítulo 19 - Halliday, Resnick e Walker – Edição antiga

“32” Consideremos um termômetro de mercúrio em vidro. Suponhamos que a seção


transversal do capilar seja constante, A0 , e que V0 seja o volume do tubo do ter-
mômetro a 00C . Se o mercúrio for exatamente o suficiente para encher o tubo a
00C , mostre que o comprimento L da coluna de mercúrio no capilar, depois de
uma variação de temperatura ∆T , será:

V0
L= (β − 3α ) ∆T
A0

ou seja: é proporcional à temperatura; β é o coeficiente de dilatação volumétrica do


mercúrio e α é o coeficiente de dilatação linear do vidro.

Cap 19 www. f isica. uf pb. br / ~r omer o 21


www.cliqueapostilas.com.br

Pr of . Romer o T avar es da Silva

Quando a temperatura varia, o volume do tubo es-


férico de vidro varia para VV e o mercúrio que o
preenchia inicialmente, varia para VM . desse
modo, temos que:
VM = V0 ( 1 + βM ∆T )
e L
VV = V0 ( 1 + βV ∆T )

Se existir um aumento de temperatura, o mercúrio


transbordará do tubo esférico.
Seja ∆V o volume de mercúrio que transbordará:

∆V = VM – VV = V0 ( 1 + βM ∆T ) - V0 ( 1 + βV ∆T ) = V0 (βM - βV ) ∆T
Mas
∆V = A0 L
logo
∆V = V0 (βM - βV ) ∆T = A0 L

Como temos que βM = β e βV = 3 α , teremos:

V0
L= (β − 3α )∆T
A0

Capítulo 19 - Halliday, Resnick e Walker – Edição antiga

“33” Dois tubos verticais contém um líquido e estão ligados, pelas extremidades inferio-
res, por um tubo capilar horizontal. Um dos tubos verticais encontra-se em um ba-
nho que contém gelo e água em equilíbrio (00C) e o outro está em um banho de
água quente (t0C) . A diferença entre as alturas nas colunas líquidas nos dois tubos
é ∆h ; h0 é a altura da coluna a 00C .

a) Mostrar que esse aparelho,


usado originalmente por Du-
long e Petit em 1816, pode ∆h
ser utilizado para medir o ver-
dadeiro coeficiente de dilata- 00C t0C
ção β de um líquido ( e não a
dilatação diferencial entre ele h0
e o vidro.
Como os tubos verticais 1 2
se comunicam e estão co-
nectados por um tubo capilar
horizontal, as suas pressões nos pontos mais baixos são iguais, ou seja:

p1 = p2 ∴ p0 + ρ1gh0 = p0 + ρ2g(h0 + ∆h) ⇒ h0(ρ1-ρ2) = ρ2∆h

É o mesmo líquido que preenche os dois tubos e o capilar, e portanto o peso


desse líquido na coluna direita é igual ao peso na coluna esquerda. As colunas

Cap 19 www. f isica. uf pb. br / ~r omer o 22


www.cliqueapostilas.com.br

Pr of . Romer o T avar es da Silva

têm alturas diferentes devido a diferença de densidade dos líquidos, e as densi-


dades são diferentes por as temperaturas nas colunas são diferentes. Como os
pesos das colunas são iguais, temos que as massas de líquido nas colunas são
iguais. A densidade é definida como:

M
ρ=
V
ou seja:
M M  M  V − V1  ∆h
h0  −  = ∆h ⇒ h0 M  2  = M
 V1 V2  V2  V1V2  V2
logo:
h0 (V2 – V1) = V1 ∆h

As massas das colunas são iguais, e os volumes são diferentes devido a dife-
rença de temperatura, logo eles estão relacionados como:

V2 = V1 ( 1 + β ∆T )
onde
∆T = t – 00C = t
ou seja:
V2 – V1 = V1 β t
e portanto:
V1∆h ∆h
V2 − V1 = = V1 βt ∴ β =
h0 h0 t

b) Determine β sabendo-se que quando t = 160C , tem-se h0=126cm e


∆h=1,5cm .

β = 7,4x10-4 0C-1

Capítulo 19 - Halliday, Resnick e Walker

35 Um pequeno aquecedor elétrico de imersão é usado para aquecer 100g de água


para uma xícara de café instantâneo. O aquecedor está rotulado com “200Watts” , o
que significa que ele converte energia elétrica em energia térmica com essa taxa.
Calcule o tempo necessário para levar toda essa água de 230C para 1000C , igno-
rando quaisquer perdas.
c = 1cal/g.0C
m = 100g Ti = 230C
Ρ = 200W Tf = 1000C

Q = m . c . ∆T = 100. 1 . (100 – 23) = 7.700cal


ou seja:
Q = 32.232,2 Joules
Mas
Q Q 32.233,2Joules
Ρ= ⇒ t= = = 161,1s
t Ρ 200Watts

Cap 19 www. f isica. uf pb. br / ~r omer o 23


www.cliqueapostilas.com.br

Pr of . Romer o T avar es da Silva

Capítulo 19 - Halliday, Resnick e Walker - 4a. edição

43 Que massa de vapor d’água a 1000C deve ser misturada com 150g de gelo no seu
ponto de fusão, em um recipiente isolado termicamente, para produzir água líquida a
500C ?
mG = 150g T1 = 00C
0
c = 1cal/g. C T2 = 500C
LF = 79,5cal/g T3 = 1000C
LV = 539cal/g

Como todo esse material está isolado, a quantidade de calor que esse sistema troca
com a vizinhança é nulo. Se um material que tem calor específico c , com massa M,
varia a sua temperatura de Ti até Tf ele absorveu de sua vizinhança uma quanti-
dade de calor Q , dada por
Q = M . c . (Tf – Ti)

Se Q < 0 dizemos que ele cedeu calor para a vizinhança. Por outro lado se uma
massa M de gelo se transforma em água ela absorveu calor M LF da vizinhança, e
se vapor d’água de transforma em líquido ele cedeu calor M LV para a vizinhança.
Desse modo, temos que:
∆Q = 0

+ mG LF + mG . c . (T2 – T1) – m LV + m . c . (T2 – T3) = 0


Logo
mG LF + mG c (T2 − T1 )
m=
LV + c (T3 − T2 )
ou seja:
m = 32,97g

Capítulo 19 - Halliday, Resnick e Walker - 4a. edição

46 Uma garrafa térmica isolada contém 130cm3 de café quente, a uma temperatura de
800C . Você insere um cubo de gelo de 12g no seu ponto de fusão para esfriar o
café. De quantos graus o seu café esfriou quando o gelo se derreteu? Trate o café
como se ele fosse água pura e despreze as transferências de energia para o ambi-
ente.
ρA = 1g/cm3 ,
Mas
mA = ρA . VA
logo:
VA = 130cm3 ⇒ mA = 130g

LF = 79,5cal/g mA = 130g mG = 12g


TA = 800C TG = 00C
Como o sistema está isolado, temos que

∆Q = 0
ou seja:
mA . c . (TF – TA) + mG LF + mG . c . (TF – TG) = 0
Cap 19 www. f isica. uf pb. br / ~r omer o 24
www.cliqueapostilas.com.br

Pr of . Romer o T avar es da Silva

m A cT A + mG cTG − mG LF
TF = = 66,520C
m A c + mG c
Mas
∆T = TA – TF = 800C – 66,520C
ou seja:
∆T = 13,480C

Capítulo 19 - Halliday, Resnick e Walker - 4a. edição

49 Uma amostra de gás se expande de 1m3 para 4m3 enquanto a sua pressão diminui
de 40Pa para 10Pa . Quanto trabalho é realizado pelo gás se a sua pressão varia
com o volume passando por cada uma das três trajetórias mostradas no diagrama p-
V da figura ao lado?

2
W B = W12 = ∫ p dV 1 A 3
1
Para calcular esta integral deve- B
mos saber com a pressão p varia
com o volume V ao longo da
trajetória B . Através do gráfico C
constatamos que a curva é uma 4 2
reta, do tipo:

p=aV+b
onde
p 2 − p1 10 − 40
a= = = - 10 Pa/m3
V2 − V1 4 −1
e
b = p 1 – a V1 ⇒ b = 50Pa
ou seja:
p = -10 V + 50
e desse modo:
V2 4
V2
W B = ∫ (− 10V + 50 )dV = −10
4
+ 50V
V1
2 1
1

logo:
WB = 75Joules
Por outro lado:
WC = W14 + W42 = W42 = (10Pa) . (4-1)m3
ou seja:
WC = + 30Joules
e também:
WA = W13 + W32 = W32 = (40Pa) . (4-1)m3
ou seja:
WA = + 120Joules

Cap 19 www. f isica. uf pb. br / ~r omer o 25


www.cliqueapostilas.com.br

Pr of . Romer o T avar es da Silva

Capítulo 19 - Halliday, Resnick e Walker - 4a. edição

50. Um sistema termodinâmico é levado de um estado inicial A para um outro estado B


e de volta ao estado A , passando pelo estado C, como é mostrado pela trajetória
ABCA no diagrama p-V da figura à seguir.

Q W ∆EINT
A → B + + + C
B → C + 0 +
C → A - - -
A B
a) Complete a tabela acima pre-
enchendo-a com + ou -
para o sinal de cada grandeza
termodinâmica associada com
cada etapa do ciclo.

A primeira lei da termodinâmica diz que:

∆E = Q - W
A→B
WAB = pA (VB – VA) > 0
mas como ∆EAB > 0 ,
QAB > WAB > 0
B→C
WBC = 0
mas como QBC > 0 ,
∆EBC > 0
C→A
A
WCA = ∫ p dV 〈 0
C

pois envolve uma compressão: VC > VA . Por outro lado:

∆EAB = EB – EA > 0
e
∆EBC = EC - EB > 0
ou seja:
EC – EA > 0
e portanto
∆ECA = EA – EC < 0

Como ∆ECA < 0 e WCA < 0 , podemos usar a primeira lei da termodinâmica e
concluir que QCA < 0 .
b) Calcule o valor numérico do trabalho realizado pelo sistema para o ciclo ABCA
completo.

O trabalho é a área abaixo da curva no gráfico p versus V. Em um ciclo, o

Cap 19 www. f isica. uf pb. br / ~r omer o 26


www.cliqueapostilas.com.br

Pr of . Romer o T avar es da Silva

trabalho W será a área no interior da curva. Como já foi explicado WCA < 0 , e
portanto o trabalho no ciclo será negativo.

1 1
W = (base ).(altura) = (3 − 1)( 40 − 20)m 3 Pa
2 2

W = 20Joules

Capítulo 19 - Halliday, Resnick e Walker - 4a. edição

53 Quando um sistema é levado do estado i para o estado f ao longo da trajetória iaf


na figura à seguir, Q = 50cal e W = 20cal . Ao longo da trajetória ibf , Q = 36cal .

a) Qual o valor de W ao longo da trajetória ibf ?

 Q = 50cal p
iaf : 
W = 20cal a f
e
ibf : {Q = 36cal
i b
Usando a primeira lei da termodinâmi-
ca, encontramos que:
V
∆Eif = Qiaf – Wiaf = 30cal

Mas, por outro lado


∆Eif = Qibf – Wibf
ou seja:
Wibf = Qibf - ∆Eif = 6cal

b) Se W = -13cal para a trajetória de volta fi , qual será Q para essa trajetória?

∆Eif = Ef – Ei ∴ ∆Efi = Ei – Ef = - ∆Eif = - 30cal


logo:
Qfi = ∆Efi + Wfi = - 43cal

c) Considere Ei = 10cal , qual é o valor de Ef ?

∆Eif = Ef – Ei ∴ Ef = ∆Eif + Ei = 30cal + 10cal = 40cal

d) Considere Eb = 22cal , qual o valor de Q para as trajetórias ib e bf ?

∆Eib = Eb – Ei = 22 – 10 = 12cal
∆Ebf = Ef – Eb = 40 – 22 = 18cal
e
Wibf = Wib + Wbf
Mas Wbf = 0 , logo
Wib = Wibf = 6cal

Cap 19 www. f isica. uf pb. br / ~r omer o 27


www.cliqueapostilas.com.br

Pr of . Romer o T avar es da Silva

Portanto:
Qib = ∆Eib + Wib = 12 + 6 = 18cal

Qbf = ∆Ebf + Wbf = 18 + 0 = 18cal

Capítulo 19 - Halliday, Resnick e Walker - 4a. edição

57 Considere a placa mostrada na figura à seguir. Suponha que L = 25cm , A = 90cm2


e que o material seja cobre. Se TQ = 1250C , TF = 100C e for alcançado o regime
permanente, encontre a taxa de condução através da placa.

kCu = 401W/m.K L

dQ T − TF
Ρ= = kA Q
dt L

(
Ρ = 401. 90 x10 − 4 m 2 )125 − 10 TQ TF
25 x10 − 2
TQ > TF
Ρ = 1.660,14 Watts

Capítulo 19 - Halliday, Resnick e Walker - 4a. edição

60 Quatro pedaços de isolantes feitos de dois materiais diferentes, todos com a mesma
espessura L e área A , estão disponíveis para cobrir uma abertura de área 2 A .
Isto pode ser feito de duas maneiras mostradas na figura ao lado. Que arranjo (a) ou
(b) , fornece o menor fluxo de energia se k1 ≠ k2 .

Se tivermos apenas uma placa de condutividade k2


térmica k ; área A ; e comprimento L entre du- k1 k1
as fontes de calor, o fluxo de calor Ρ será dado k1
por k2 k2
dQ T − TF
Ρ= = kA Q (a) (b)
dt L

Se tivermos duas placas entre duas fontes de calor, o fluxo de calor Ρ será dado por

dQ A (TQ − TF )
Ρ= =
dt L2 L1
+
k 2 k1

Vamos considerar que nos casos a e b , os arranjos estão entre duas fontes de
calor com temperaturas TQ e TF .

No arranjo a , dois pares de placas iguais formam o conjunto: duas placas com k1 e
duas placas com k2 . O fluxo de calor através das placas k1 tem a forma:

Cap 19 www. f isica. uf pb. br / ~r omer o 28


www.cliqueapostilas.com.br

Pr of . Romer o T avar es da Silva

 TQ − TF
 Ρ1 = k 1 A
 2L  k + k 2   TQ − TF 
 ⇒ ΡA = Ρ1 + Ρ2 =  1  A 
 TQ − TF  2   L 
Ρ2 = k 2 A
 2L
No arranjo b duas placas diferentes formam um conjunto, e dois desses conjuntos
formam o arranjo. O fluxo através de cada arranjo será dados por:
T − TF  k k  T − TF 
Ρ1 = A Q = A  1 2  Q 
L L k
 1 + k 2  L 
+
k1 k 2
Como os fluxos nos dois conjuntos são iguais:
 k k  T − TF 
ΡB = 2Ρ1 = A  2 1 2  Q 
 k 1 + k 2  L 
Para encontrar em qual arranjo teremos o maior fluxo, vamos calcular a razão:

 k1 + k 2 
 
ΡA  2  (k 1 + k 2 )2
= =
ΡB  k 1k 2  4k 1 k 2
 2 
 k1 + k 2 
(k1 + k 2 )2
〉1 ⇒ k 12 + k 22 + 2k 1k 2 〉 4k 1k 2 ⇒ k 12 + k 22 − 2k 1k 2 〉 0
4k 1k 2
ou seja:
(k 1 − k 2 )2 〉 0
E como a equação anterior é sempre verdadeira, concluímos que:

ΡA > ΡB

Capítulo 19 - Halliday, Resnick e Walker - 4a. edição

61 Duas hastes metálicas retangulares idênticas são soldadas extremidade com extre-
midade, como mostrado na figura (a) , e 10J são conduzidos (em um processo em
regime estacionário) através das hastes sob a forma de calor em 2min . Quanto
tempo levaria para que 10J fossem conduzidos através das hastes se elas fossem
soldadas uma na outra como mostrado na figura (b) ?
A
dQ A (T Q − T F )
Ρ= = 00C 1000C
dt L2 L1
+
k 2 k1
B
A (TQ − TF )  kk  TQ − TF 
ΡA = = A 1 2   00C 1000C
L L  k1 + k 2  L 
+
k 2 k1

Cap 19 www. f isica. uf pb. br / ~r omer o 29


www.cliqueapostilas.com.br

Pr of . Romer o T avar es da Silva

TQ − TF
Ρ = kA
L
logo:
TQ − TF
ΡB = A (k 1 + k 2 )
L
A razão entre os fluxos:
ΡA k 1k 2
=
ΡB (k 1 + k 2 )2
e como as placas são iguais:
ΡA 1
= ⇒ ΡB = 4ΡA
ΡB 4

 QA
ΡA = t
 A ΡA Q A t B
 ⇒ =
 QB ΡB QB t A
ΡB =
 tB
Como QA = QB
ΡA t A
tB = t A =
ΡA 4
ou seja:
tB = 0,5min

Capítulo 19 - Halliday, Resnick e Walker - 4a. edição

65 Um tanque de água ficou destampado em tempo frio, e uma placa de gelo de 5cm
de espessura se formou na sua superfície. O ar acima do gelo está a -100C . Calcule
a taxa de formação de gelo (em centímetros por hora) na placa de gelo. Adote a con-
dutividade térmica e massa específica do gelo como 0,0040cal/s.cm.0C e
0,92g/cm3. Suponha que não haja transferência de energia através das paredes ou
pelo fundo do tanque.

k = 0,0040cal/s.cm.0C Ar
ρ = 0,92g/cm3 T1
LF = 79,5cal/g Gelo 5cm
T1 = -100C
T2 = 00C T2
L = 5cm
Água
Vamos considerar que a camada de gelo
vá se aprofundando, de modo que num
intervalo de tempo dt , se forme uma ca-
mada de gelo de área A e espessura
dx , ou seja, se formaria um volume de
gelo dV = A dx , e a esse volume corres-
ponde uma massa dM , tal que:

Cap 19 www. f isica. uf pb. br / ~r omer o 30


www.cliqueapostilas.com.br

Pr of . Romer o T avar es da Silva

dM = ρ dV = ρ A dx

A quantidade de calor que deve ser retirada para a formação deste camada de gelo,
é dada por:
dQ = - LF dM = - ρ A LF dx

A taxa de calor retirado no tempo, ou fluxo de calor será dada por:

dQ dx
= − ρALF
dt dt

onde dx/dt é a velocidade com que a camada dx de gelo aumenta, ou seja é a taxa
de formação da placa de gelo. Mas por outro lado, o fluxo de calor que sai do gelo
para a atmosfera através da placa de gelo já formada é dada por:

dQ T − T1
= kA 2
dt L

Como o gelo irá sendo formado como consequência desse fluxo, temos que:

dQ dx T − T1
= − ρALF = kA 2
dt dt L
ou seja:
dx k T2 − T1
= =1,09x10-4cm/s
dt ρLF L
e ainda
dx
= 0,39cm/hora
dt

Cap 19 www. f isica. uf pb. br / ~r omer o 31


www.cliqueapostilas.com.br

Pr of . Romer o T avar es da Silva

20. Teoria Cinética dos Gases

Quando consideramos um gás contido em um recipiente podemos analisá-lo de


uma maneira global usando a Termodinâmica, e calcular as suas propriedades macros-
cópicas tais como temperatura, pressão, volume e etc.
Por outro lado, se quisermos entender os porquês do comportamento macroscópi-
co, devemos analisar os constituintes deste gás, como eles interagem entre si e como
interagem com as paredes do volume que os contém.

Uma nova maneira de ver os Gases

Os gases são constituídos de pequenas entidades, que podem ser átomos, molé-
culas ou ambos. Ele será um gás monoatômico quando composto apenas de átomos (ou
seja: moléculas monoatômicas) ou um gás poliatômico, dependendo das suas caracterís-
ticas moleculares.
As moléculas interagem entre elas, e essa interação acontece aos pares, ou seja
elas interagem duas a duas. Se neste gás existirem N moléculas cada molécula interage
com todas as outras N-1 moléculas. Cada molécula deve ter o seu movimento governa-
do pela segunda lei de Newton, e portanto temos N equações referentes a aplicação
dessa lei, uma para cada molécula. Como cada molécula interage com as restantes, o
seu movimento irá interferir no movimento de todas as outras, e dizemos então que essas
equações estão acopladas uma as outras.
O número de equações resultante deste modelo torna a sua solução numérica im-
possível, mesmo usando os melhores computadores contemporâneos.

O Número de Avogadro

Mas quantas moléculas existem em uma amostra macroscópica de uma dada


substância? Vamos definir uma grandeza adequada para lidar com moléculas, é o mol.
Um mol é o número de moléculas que existem em 12g de carbono-12. Experimental-
mente se determina quantas moléculas existem em um mol, e esse é o chamado número
de Avogadro NA ,
NA = 6,02x1023moléculas

Desse modo, já podemos relacionar número de moles µ e número de moléculas


N , ou seja:
N
N = µ NA ⇒ µ =
NA

Gases ideais

Se considerarmos uma amostra com 12g de carbono-12 , teremos neste material


NA = 6,02x1023moléculas , e se desejarmos usar a segunda lei de Newton para calcular as
trajetórias das moléculas, teremos que resolver NA equações acopladas. O que fazer
nesta situação?
A aproximação mais drástica possível será considerar que as moléculas não

Cap 20 www. f isica. uf pb. br / ~r omer o 2


www.cliqueapostilas.com.br

Pr of . Romer o T avar es da Silva

interagem, elas se ignoram, e desse modo elas interagem apenas com as paredes do re-
cipiente que contém a mostra do gás. Apesar desta aproximação ser drástica, ela se
aproxima da realidade em muitas situações práticas, quando a densidade do gás é sufici-
entemente baixa. Nesta circunstâncias, uma amostra de um gás real se aproxima do mo-
delo do gás ideal.
Trabalhos experimentais com gases ideais mostraram que a pressão, temperatura
e volume se relacionam de tal modo que:

pV=µRT

onde µ é o número de moles do gás presentes na amostra considerada e


R=8,31J/mol.K é a constante universal dos gases. A equação anterior é chamada equa-
ção dos gases ideais. Por outro lado, se ao invés de moles estivermos usando o número
de moléculas, a equação tomará a forma

p V = N kB T

onde N é o número de moléculas do gás presentes na amostra considerada e


kB=1,38x10-23J/K é a constante de Boltzmann. Pode-se notar que:

µ R
µR = Nk B ⇒ kB = R= ∴ R = kBNA
N NA

Trabalho com temperatura constante

Vamos considerar um sistema em con-


tato com um reservatório térmico. Nes- 450
400
sas condições esse sistema pode sofrer p350
T1
mudanças de pressão e volume mas 300
manterá sempre a mesma temperatura, 250
T2
200
que é a temperatura do reservatório 150
térmico. O trabalho realizado pelo sis- 100 T3
50
tema é definido como: 0
Vf
0,025 0,075 0,125 0,175
W if = ∫ p dV V
Vi T1 > T2 > T3

Mas como o gás é ideal e a temperatura é mantida constante ao logo da transformação,


temos que:
Vf
dV V 
= µRT (lnV ) Vf = µRT (lnVf − lnVi ) = µRT ln f
V
W if = µRT ∫ 
Vi
V i
 Vi 

Cálculo cinético da pressão

Vamos considerar N moléculas um gás ideal em um recipiente em forma de um


cubo de aresta L e considerar os eixos cartesianos paralelos as arestas, como na figura à
seguir.

Cap 20 www. f isica. uf pb. br / ~r omer o 3


www.cliqueapostilas.com.br

Pr of . Romer o T avar es da Silva

As moléculas desse gás estão continu-


amente colidindo com as paredes do recipi- - mvx
ente. Vamos analisar especificamente a co- x
lisão de uma molécula, que se dirige para
colidir com a parede do recipiente paralela +mvx
ao plano yz e que passa pela origem.
Quando ela colide com a parede, não acon-
tecerá mudança nas componentes y e z
do momento linear, mas a componente x y
do momento linear mudará de sinal, aconte-
cerá uma reversão neste movimento. Esta-
mos considerando que as colisões são
perfeitamente elásticas. A variação do mo-
mento dever-se-á apenas a mudança da A2
componente x . A1

∆p = pf – pi = mvx – (-mvx) = 2mvx x

Sejam A1 e A2 as paredes do cubo z


perpendiculares ao eixo x . A molécula vai
colidir com a face A1 e levar um intervalo
de tempo ∆t para colidir com a face oposta A2 e depois colidir novamente com A1 .
O tempo t necessário para essa molécula ir de uma face até outra é dado por
t=L/vx , e desse modo:
2L
∆t = 2 t =
vX

A variação do momento linear de uma molécula, num intervalo ∆t entre duas coli-
sões com a mesma face do recipiente é dada por:

∆p X 2mv X mv 2X
= =
∆t 2L / v X L

A equação anterior nos dá a força que uma molécula exerce na face considerada.
Para se encontrar a força total exercida por todas as moléculas, devemos considerar as
contribuições de todas as N moléculas:

m 2
FX =
L
(
v X 1 + v X2 2 + ! + v XN
2
)
A pressão que essas moléculas exercerão dependerá da força média e será dada
por:
FX
p=
L2
=
m
L3
(
v 2X 1 + v X2 2 + ! + v XN
2
)
onde estamos representando o valor médio de uma grandeza A por <A> . Como as
moléculas não são distinguíveis, os valores médios das componentes x de cada uma
das moléculas são iguais, ou seja:

Cap 20 www. f isica. uf pb. br / ~r omer o 4


www.cliqueapostilas.com.br

Pr of . Romer o T avar es da Silva

(v 2
X1 + v 2X 2 + ! + v 2XN )= N v 2
X

Considerando que neste cubo não existe direção privilegiada, os valores médios
das diversas componentes serão iguais, ou seja:

1 2
v 2 = v 2X + v Y2 + v Z2 = 3 v 2X ⇒ v 2X = v
3

e como temos N moléculas nesse gás ideal;

(v 2
X1 + v X2 2 + ! + v XN
2
)= N v 2
X =
N 2
3
v
Desse modo:
FX
p=
L 2
=
m
L3
(
v X2 1 + v X2 2 + ! + v 2XN ) = mN
3V
v2

onde consideramos que o volume do cubo é V = L3 . Podemos ainda dizer que:

mN 2
pV = v
3

Mas Nm é a massa total do gás pois: N é número de moléculas e m é a massa


de cada molécula. Por outro lado, a massa total também pode ser expressa como µM
pois: µ é o número de moles e M é a massa molar. Portanto, usando a equação dos ga-
ses ideais:
µM 2 3RT
pV = v = µRT ⇒ v 2 =
3 M

e se definirmos
v RMS = v2

(RMS = root mean square) encontramos que:

3RT
v RMS =
M

Entretanto a massa molar M é igual ao número de Avogadro vezes a massa m


de uma molécula M=NAm , e a constante universal dos gases pode ser escrita como
R=NAkB , e desse modo teremos que:

3k B T
v RMS =
m

Cap 20 www. f isica. uf pb. br / ~r omer o 5


www.cliqueapostilas.com.br

Pr of . Romer o T avar es da Silva

Energia cinética de translação

Como já foi mencionada, em um gás ideal as moléculas não interagem, portanto


não existem energia potencial e o único tipo de energia possível é a energia cinética de
translação. A energia cinética média de uma partícula é dada por:

1 m 2 m 3k BT
K = mv 2 = v =
2 2 2 m

3
K = k BT
2

Percurso livre médio

Entre colisões sucessivas, o movimento de uma


molécula de um gás ideal é retilíneo e uniforme . A
distância média que uma molécula percorre entre duas
colisões sucessivas é chamado percurso livre médio.
Se tivermos duas moléculas de diâmetro d, ocorrerá d
uma colisão quando os seus centros se aproximarem de
uma distância d .
Uma descrição equivalente das colisões entre mo-
léculas consiste em considerar uma delas pontual e a
outra com diâmetro 2d , pois colisão ocorrerá quando os
seus centros se aproximarem de uma distância d , como
na situação anterior.
Se estivermos observando uma molécula nas suas d
múltiplas colisões, podemos considerar que ela tem um
diâmetro 2d e as outras são pontuais.
Se ela tem diâmetro 2d e velocidade média <v> ,
num intervalo de tempo t , ela terá descrito um cilindro
de seção reta πd2 e comprimento <v>t . Se a densida-
de de partículas no gás for n = N/V , existirão no cilindro 2d
N partículas, onde:
N = n V = n (πd2 . <v>t)

Este número de partículas N será exatamente o <v>t


número de colisões num dado intervalo de tempo t . O percurso livre médio <L> será a
distância percorrida num intervalo de tempo t dividido pelo número de colisões que
acontecerá neste trajeto.

v t v t 1
L = = 2
=
N n πd v t n πd 2
ou ainda
V 1
L =
N πd 2

Cap 20 www. f isica. uf pb. br / ~r omer o 6


www.cliqueapostilas.com.br

Pr of . Romer o T avar es da Silva

Esse resultado é apenas uma primeira aproximação, por que ele se baseia na hi-
pótese que todas as moléculas estão em repouso, e apenas uma se move.

Distribuição de velocidades moleculares


Vamos considerar um número N de moléculas que estão no interior de um recipi-
ente de volume V . As moléculas têm velocidade diferentes, mas essas velocidades se
distribuem segundo uma característica própria.

Se considerarmos uma situação genérica, onde a energia interna E de cada mo-


lécula é composta da soma de sua energia cinética K mais sua energia potencial U , e
desse modo:
1
E = mv 2 + U (x, y , x )
2

A função que explicita a distribuição de velocidades, é a distribuição de Maxwell-


Boltzmann, e tem a forma:
f (E ) = Ae − E / kBT

onde A é uma constante. Essa constante pode ser determinada se considerarmos que
integral da função de distribuição deve ser igual ao número de moléculas. Quando esta-
mos analisando um gás ideal, a energia potencial é desprezada, e temos como energia
interna apenas a energia cinética:

1 1
E=
2
mv 2 = m v 2X + v Y2 + v Z2
2
( )
e portanto:
f (v ) = Ae − m (v X +v Y +v Z )/ 2k BT
2 2 2

+∞ +∞ +∞

∫ dv X
−∞
∫ dv Y
−∞ −∞
∫ dv Z f (v X , v Y v Z ) = N

ou seja:
+∞ +∞ +∞
− mv X2 / 2 kT − mv Y2 / 2 kT − mv Z2 / 2 kT
A ∫e dv X ∫e dv Y ∫e dv Z = N
−∞ −∞ −∞

e por outro lado, seja:


+∞
− aX 2
B= ∫e
−∞
dX

Podemos dizer que:

+∞ +∞ 2π ∞ ∞
2π −u π
du −u
(0 − 1) = π
2 2 2 ∞
B2 = ∫ e dX ∫ e dY = ∫ dθ ∫ r dr e = 2π ∫
− aX − aY − ar
e =− e =−
−∞ −∞ 0 0 0
2a 2a 0 a a
ou seja:
+∞
− aX 2 π
B= ∫e
−∞
dX =
a
e portanto
Cap 20 www. f isica. uf pb. br / ~r omer o 7
www.cliqueapostilas.com.br

Pr of . Romer o T avar es da Silva

+∞
− mv 2X / 2 kT 2πkT
∫e
−∞
dv X =
m
e
+∞ +∞ +∞
2 2
− mv Z2 / 2 kT
A ∫ e − mv X / 2kT dv X ∫ e − mv Y / 2kT dv Y ∫e dv Z = N
−∞ −∞ −∞
logo
3 3/2
 2πkT   m 
A  =N ⇒ A = N 
m   2πkT 
 
e portanto
3/2
 m 
e − m (v X +v Y +v Z )/ 2k BT
2 2 2
f (v ) = N  
 2πkT 

Se fizermos a mudança de variáveis para coordenadas esféricas, encontraremos


que:
∞ 3
2 ∞
 m  2

∫0 4πv dv f (v ) = 4πN  2πkT  ∫v


/ 2 k BT
2 2
e − mv =N
0

Podemos então definir uma função de distribuição de velocidades F(v) que de-
pende do módulo do vetor velocidade, ou seja:

3
 m  2 2 − mv 2 / 2k BT
F (v ) = 4π   v e
 2πkT 

Pode-se mostrar que:


0,25
F(v)
∞ T1
∫ F (v ) dv = 1
0
0,2

0,15
Tem-se que: T2
0,1

8kT
v = ∫ v F (v ) dv = 0,05
0
πm
e 0

3kT 0 5 10 15 v 20
v2 = ∫ v 2 F (v ) dv =
0
m T1 < T2

A velocidade mais provável em uma gás é aquela na qual a função de distribuição


de velocidades F(v) é máxima, e nestas circunstâncias:

dF (v ) 2kT
=0 ⇒ vP =
dv m

Cap 20 www. f isica. uf pb. br / ~r omer o 8


www.cliqueapostilas.com.br

Pr of . Romer o T avar es da Silva

Calores específicos molares de um gás ideal

Se tivermos uma certa massa m de uma substância, podemos tentar relacionar


qual a variação de temperatura ∆T que sofrerá essa massa, quando ela absorver uma
quantidade de calor ∆Q . Existe uma relação, que tem a forma:

∆Q = m c ∆T

onde chamamos a grandeza c de calor específico. Quando lidamos com gases, surge a
necessidade de definir uma relação mais específica que leve em contas as especificida-
des deste fluido. Definimos o calor específico a volume constante para relacionar variação
de temperatura ∆T que sofrerá um gás, quando ele absorver uma quantidade de calor
∆Q ; na situação em que o recipiente que contém o gás é mantido a volume constante. De
maneira equivalente, definimos o calor específico a pressão constante para relacionar
variação de temperatura ∆T que sofrerá um gás, quando ele absorver uma quantidade
de calor ∆Q ; na situação em que o recipiente que contém o gás é mantido a pressão
constante

A energia interna EINT

Vamos considerar uma gás ideal monoatômico, ou seja as suas moléculas têm
apenas um átomo. Ao nível dessa nossa descrição da Natureza, não estamos conside-
rando a estrutura interna dos átomos e portanto eles podem ter apenas um tipo de ener-
gia: a energia associada ao seu movimento. Desse modo, a energia total das N molécu-
las monoatômicas que compõe esse gás terá a forma:

3 3
E INT = Nk BT = µRT
2 2

Calor específico molar a volume constante – CV

Como mencionado anteriormente, podemos definir o calor específico molar a volu-


me constante como:
dQV = µ CV dT
ou ainda:
1  dQ 
CV =  
µ  dT V

Usando a primeira lei da Termodinâmica, temos que:

dEINT = dQ – p dV

e se considerarmos uma transformação isovolumétrica:

(dEINT )V = dQV
ou seja:

Cap 20 www. f isica. uf pb. br / ~r omer o 9


www.cliqueapostilas.com.br

Pr of . Romer o T avar es da Silva

1  dQ  1  ∂E 
CV =   =  INT 
µ  dT V µ  ∂T V

e para um gás ideal, encontramos


3
CV = R
2

Calor específico molar a pressão constante – CP

Como mencionado anteriormente, podemos definir o calor específico molar a pres-


são constante como:
dQP = µ CP dT
ou ainda:
1  dQ 
CP =  
µ  dT  P

Usando a primeira lei da Termodinâmica, temos que:

dEINT = dQ – p dV

e se considerarmos uma transformação que envolva uma variação de temperatura, mas


com o sistema mantido a pressão constante, temos que:

 ∂E INT   dQ   ∂V 
  =  − p 
 ∂T  P  dT  P  ∂T  P

onde lembramos que dQ não é uma diferencial exata, daí o aparente contra-senso ao
envolver derivadas parciais e total, na equação anterior. Usando as definições de um gás
ideal, temos que:
 3  ∂E INT  3
E INT = µRT ∴   = µR
 2  ∂T  P 2

 µRT  ∂V 
 V = ∴ p  = µR
 p  ∂T  P
ou seja:
3 5
µR = µC P − µR ⇒ C P = R
2 2

Relação entre CV e CP para um gás ideal

Vamos considerar um sistema formado por µ moles de uma gás ideal, e a sua
temperatura será aumentada T até alcançar T+ ∆T de duas formas diferentes. As cur-
vas que representam transformações isotérmicas nas duas temperaturas mencionadas

Cap 20 www. f isica. uf pb. br / ~r omer o 10


www.cliqueapostilas.com.br

Pr of . Romer o T avar es da Silva

estão representadas no gráfico ao lado. A


primeira transformação será feita a volume p
1000,000
900,000
constante, e o gás vai do estado a até o 800,000 c
estado c . A primeira lei da Termodinâmica 700,000
600,000
diz que: 500,000
dEINT = dQ – p dV 400,000 b
300,000
200,000 a T+∆T
e neste caso teremos que 100,000 T
0,000
0,010 0,030 0,050 0,070 V
0,090
∆Eac = ∆QV = µ CV ∆T

A segunda transformação será feita a pressão constante, e o gás vai do estado a até o
estado b . A primeira lei da Termodinâmica diz que:

dEINT = dQ – p dV
e neste caso teremos que

∆Eab = ∆QP – p ∆V = µ CP ∆T – p (∆V)P

Como a energia interna de uma gás ideal depende apenas da sua temperatura,
temos que:
∆Eac = ∆Eab
e portanto:
µ CV ∆T = µ CP ∆T – p (∆V)P
ou seja:
p  ∆V  p  µR 
C P − CV =  =   = R ∴ C P = CV + R
µ  ∆T  µ  p 

Transformação adiabática de um gás ideal

Uma expansão adiabática é caracterizada por ser uma transformação onde o sis-
tema não troca calor com as suas vizinhanças. Nestas circunstâncias, temos então que:

dE = dQ – p dV ⇒ dE = µ CV dT = - p dV
ou seja:
p
dT = − dV
µCV

Mas por outro, se diferenciarmos a equação do gás ideal encontramos que:

pdV + Vdp
pV = µRT ⇒ pdV + Vdp = µRdT ∴ dT =
µR

e igualando os termos em dT, temos que:

pdV + Vdp p
dT = =− dV
µR µCV

Cap 20 www. f isica. uf pb. br / ~r omer o 11


www.cliqueapostilas.com.br

Pr of . Romer o T avar es da Silva


ou seja:
(CV + R) p dV + CV V dp = 0

Mostramos anteriormente que para um gás ideal:

CP = CV + R
logo:
CP p dV + CV V dp = 0
ou seja:
C P dV dp
+ =0
CV V p
Vamos definir γ = CP/CV

dV dp
γ + =0 ⇒ γ lnV + ln p = const = ln a
V p
e portanto:
( )
ln pV γ = ln a ∴ pV γ = a = const

Cap 20 www. f isica. uf pb. br / ~r omer o 12


www.cliqueapostilas.com.br

Pr of . Romer o T avar es da Silva

Solução de alguns problemas


Capítulo 20 - Halliday, Resnick e Walker

10 Uma quantidade de oxigênio ocupando um volume de 1000cm3 a 400C e uma


pressão de 1,01x105Pa se expande até um volume de 1500cm3 e pressão
1,06x105Pa

a) Encontre o número de moles de oxigênio no sistema.

V1 = 1000cm3 = 10-3m3
T1 = 400C = 313K
p1 = 1,01x105Pa
R = 8,314J/mol . K
pV = µRT ⇒
pV
µ= =
(
1,01x10 5 10 −3 )( )
=3,8x10-2moles
RT (8,314 )(313 )
b) Encontre a temperatura final do sistema.

V2 = 1500cm3 = 1,5x10-3m3
p2 = 1,06x105Pa
pV pV  p  V 
µ= 1 1 = 2 2 ⇒ T2 = T1  2  2  = 492,74K
RT1 RT2  p1  V1 

T2 = 219,740C

Capítulo 20 – Halliday e Resnick – Edição antiga

“10” Um manômetro de mercúrio selado, tem dois ramos desiguais à mesma pressão p0,
como mostra a figura abaixo à esquerda. A área da seção reta do manômetro é
1,0cm2 . Através de uma torneira no fundo do manômetro, admite-se no recipiente
um volume adicional de mercúrio, igual a 10cm3 . O nível da esquerda sobe de
6,0cm e o nível da direita sobe de 4,0cm . Determine a pressão p0 .

he’ = 50cm hd’ = 30cm


He = 6cm Hd =4cm
he
∆H = He - Hd = 2cm
he’
he = he’ – He = 44cm hd’ hd
hd = hd’ – Hd = 26cm He
Hd
2
A = 1cm
∆V = 10cm3

Tanto na situação inicial pe pd


como na final, existe um
gás acima do nível Pe Pd

Cap 20 www. f isica. uf pb. br / ~r omer o 13


www.cliqueapostilas.com.br

Pr of . Romer o T avar es da Silva

do líquido, e a sua composição deve ser basicamente de mercúrio. Vamos conside-


rar esse gás como ideal. Desse modo, considerando a situação inicial, teremos que:

p0 Vd’ = µd R T
e
p0 Ve’ = µe R T

onde V é o volume ocupado por esse gás e µ é o número de moléculas contido


nele. Logo temos que:
µ RT µ RT Vd' µd
p0 = d ' = e ' ⇒ =
Vd Ve Ve' µe
ou ainda:
p 0Vd' p 0Ve'
µd = e µe =
RT RT

Depois de adicionado um volume ∆V de mercúrio, as colunas ficarão com níveis


diferentes. Usando a hidrostática, poderemos relacionar as pressões em diferentes
pontos do manômetro.
Pd = pd + ρ g Hd
e
Pe = pe + ρ g He

Como as pressões no mesmo nível horizontal do líquido são iguais, subtraímos a


penúltima equação da última e encontramos que:

pd – pe = ρ g ( He - Hd ) = ρ g ∆H

Por outro lado, o gás acima do nível de mercúrio terá um volume disponível dife-
rente da situação inicial, e será diverso em cada ramo do manômetro. Ou seja:

 µ d RT
 p =
 p d Vd = µ d RT d
Vd
 
 ⇒ 
 p V = µ RT  µ e RT
 e e e
 pe =
 Ve
e usando a equação anterior, encontramos que:
µ µ  ρ g ∆H
p d − p e =  d − e  RT = ρ g ∆H ⇒ RT =
 Vd V e   µd µe




 V d Ve 
e usando que
Vd' µ d
=
Ve' µ e
encontramos que
 V V V' 
µ e RT = ρ g ∆H  ' d e e '  = p 0Ve'

 Vd V e − V d V e 

Cap 20 www. f isica. uf pb. br / ~r omer o 14


www.cliqueapostilas.com.br

Pr of . Romer o T avar es da Silva

ou seja:
 VV 
p 0 = ρ g ∆H  ' d e ' 

 Vd V e − Vd V e 

Lembrando que os volumes considerados são partes dos ramos do manômetro, que
têm seção reta A , e desse modo V = A h e portanto:

 h h 
p 0 = ρ g ∆H  ' d e ' 

 hd he − hd he 

Usando que a densidade do mercúrio ρ = 1,36x104kg/m3 encontramos que:

p0 = 1,55x105N/m2 = 1,55atm

Capítulo 20 - Halliday, Resnick e Walker

11 A pressão p , o volume V e a temperatura T de um certo material estão relaciona-


dos através da equação:
AT − BT 2
p=
V

onde A e B são constantes. Encontre uma expressão para o trabalho realizado


pelo material se a temperatura variar de T1 até T2 enquanto a pressão permanece
constante.

O trabalho realizado pelo sistema quando ele passa de um estado para outro é defi-
nido como:
2
W12 = ∫ pdV
1

e como a pressão permanece constante (p1 = p2) nesse processo, temos que:

2
W12 = p1 ∫ dV = p1 (V2 − V1 ) = p 2V2 − p1V1
1

Usando a dependência funcional mencionada:

[ ] [ ] (
W12 = AT 2 − BT 22 − AT1 − BT12 = A(T2 − T1 ) − B T22 − T12 )

Capítulo 20 - Halliday, Resnick e Walker

12 Um recipiente encerra dois gases ideais. Dois moles do primeiro gás estão presen-
tes, com massa molar M1 .O segundo gás possui massa molar M2 = 3M1 , e 0,5mol
deste gás está presente. Que fração da pressão total na parede do recipiente pode
ser atribuída ao segundo gás?

Cap 20 www. f isica. uf pb. br / ~r omer o 15


www.cliqueapostilas.com.br

Pr of . Romer o T avar es da Silva

(A explicação da pressão da teoria cinética conduz à descoberta experimentalmente


de pressões parciais para uma mistura de gases que não reagem quimicamente: a
pressão total exercida pela mistura é igual à soma das pressões que os vários gases
exerceriam separadamente se cada um deles ocupasse o recipiente sozinho.)

M1 M2
µ1 = 2moles µ2 = 0,5mol

( mi )= ( µi ) ( Mi )
(Massa) = (Número de moles) ( Massa molar)

pi V = µi R T

p = p1 + p2 = ( µ1 + µ2 ) RT/V

p1 µ 1RT / V µ1
= = =0,8
p (µ1 + µ 2 )RT / V µ1 + µ 2
e de modo equivalente:
p2 µ2
= = 0,2
p µ1 + µ 2

Capítulo 20 - Halliday, Resnick e Walker

15 Uma bolha de ar com volume de 20cm3 está no fundo de um lago a 40m de pro-
fundidade, onde a temperatura é 40C . A bolha sobe até a superfície, que está na
temperatura de 200C . Considere que a temperatura da bolha de ar é a mesma que a
da água ao seu redor. Exatamente quando a bolha atinge a superfície, qual o seu
volume?

Vi = 20cm3 = 2x10-5m3 ρA = 103kg/m3 f


Ti = 40C = 277K p0 = 1,013x105Pa
h = 40m
Tf = 200C = 293K
h
Vamos chamar de situação inicial quando a
bolha está no fundo do lago e situação final
quando ela alcança a superfície. Temos que: i
 µRTi
 p i = p 0 + ρgh = V
 i

 µRTf
 pf = p0 =
 Vf

Temos duas equações e duas incógnitas, Vf e µ .

T T   p V  Ti Tf   T  V  
ρgh = µR  i − f  =  0 f  −  = p0  i  f  − 1
 Vi Vf   Tf  Vi Vf   Tf  Vi  

Cap 20 www. f isica. uf pb. br / ~r omer o 16


www.cliqueapostilas.com.br

Pr of . Romer o T avar es da Silva

Ou seja:
 Ti  Vf  ρgh T  ρgh 
   = 1 + ⇒ Vf = Vi  f 1 +  = 103cm3
 Tf  Vi  p0  Ti  p0 

Capítulo 20 - Halliday, Resnick e Walker

16 Um tubo de comprimento L = 25m que está aberto em uma extremidade, contém ar


a pressão atmosférica. Ele é empurrado na vertical para dentro de um lago de água
doce até que a água suba até a metade do tubo, como mostrado na figura ao lado.
Qual a profundidade h da extremidade inferior do tubo? Suponha que a temperatura
é a mesma em todos os pontos e que não varie com o tempo.

L = 25m p0 = 1,013x105Pa
ρ = 103kg/m3
L/2
A pressão na superfície do líquido dentro do
tubo, é a mesma do gás acima desta superfície,
e é dada por:
pf = p0 + ρ g (h - L/2) h

onde estamos explicitando que esta é a situação L/2


final do tubo. Na situação inicial, este tubo está a
pressão atmosférica. Como foi dito, a temperatu-
ra é a mesma em todos os pontos e não varia
com o tempo, temos que:
V   2V 
p 0Vi = µRT = p f Vf ⇒ p f = p 0  i  = p 0  f  ∴ p f = 2 p 0
 Vf   Vf 
ou seja:
pf = p0 + ρ g (h - L/2) = 2p0 ⇒ p0 = ρ g (h - L/2)
logo:
L p
h = + 0 = 22,83m
2 ρg

Capítulo 20 - Halliday, Resnick e Walker


O recipiente A da figura abaixo contém um gás ideal a uma pressão de 5,0x105Pa
17 e a uma temperatura de 300K . Ele está ligado por um tubo fino (e uma válvula fe-
chada) ao recipiente B , com quatro vezes o volume de A . O recipiente B con-
tém, o mesmo gás ideal a uma pressão 1,0x105Pa e a uma temperatura de 400K .
A válvula é aberta para permitir que as pressões se igualem, mas a temperatura de
cada recipiente é mantida constante em seus valores iniciais. Qual será então a
pressão nos dois recipientes?

pA = 5x105Pa pB 1x105Pa
TA = 300K TB 400K
VB = 4VA
A
B

Cap 20 www. f isica. uf pb. br / ~r omer o 17


www.cliqueapostilas.com.br

Pr of . Romer o T avar es da Silva

Temos claramente duas situações, antes da válvula ser aberta e depois que ela foi
aberta. Depois que ela foi aberta existiu um fluxo de gás de um recipiente para outro
de modo que as pressões foram equilibradas, mas a quantidades total de gás per-
maneceu a mesma. Logo:
µ = µA + µB = µ’A + µ’B

onde os µ são os números de moles em cada recipiente, antes e depois da válvula


ser aberta. Usando a equação dos gases ideais encontramos que:

 p AV A = µ A RT A  pV A = µ ' A RT A
 
 e 
 p V = µ RT  pV = µ ' RT
 B B B B  B B B

ou seja:
p AV A p BVB V A  p A 4 p B 
µ = µ A + µB = + =  + 
RT A RTB R  T A TB 
e também
pV A pVB pV A  1 4 
µ = µ' A +µ'B = + =  + 
RT A RTB R  T A TB 
ou ainda:
VA  p A 4pB  pV A  1 4 
µ=  +  =  + 
R  TA TB  R  T A TB 
e portanto:
 p A 4 pB 
 + 
 TA TB 
p= = 2,0x105Pa
 1 4 
 + 
 T A TB 

Capítulo 20 - Halliday, Resnick e Walker

19 A temperatura mais baixa possível no espaço sideral é 2,7K . Qual a velocidade mé-
dia quadrática das moléculas de hidrogênio a esta temperatura?

R = 8,31J/mol.K M = 2,02x10-3kg/mol

3RT
v QM = = 182m/s
M

Capítulo 20 - Halliday, Resnick e Walker

23 Um feixe de moléculas de hidrogênio (H2) está dirigido contra uma parede, segundo
um ângulo de 550 com a normal à parede. Cada molécula no feixe possui uma velo-
cidade escalar de 1,0km/s e uma massa de 3,3x10-24g . O feixe bate na parede so-
bre uma área de 2,0cm2 , à uma taxa média de 1023 moléculas por segundo . Qual a
pressão do o feixe sobre a parede?

Cap 20 www. f isica. uf pb. br / ~r omer o 18


www.cliqueapostilas.com.br

Pr of . Romer o T avar es da Silva

n = 1023moléculas/s θ = 550 "


A = 2cm2 = 2x10-4m2 v = 1km/s = 103m/s pf
m = 3,3x10-24g = 3,3x10-27kg

Como as moléculas só apresentam variação θ


de momento na direção do eixo x , temos que: x
θ
∆p = pfx – pix = (-m vX) - (+mvx) = - 2 m vX

vX = v cos550 ⇒ ∆p = -2 m v cos550 "


pi

A força total que as moléculas exercem na parede é resultado das contribuições


de todas as N moléculas que colidem num intervalo de tempo ∆t , ou seja:

∆p N
F =N = ∆p = n ∆p
∆t ∆t

A pressão Ρ é definida em termos da força exercida pelas moléculas na parede,


ou seja:
F n 2nmv cos 55 0
Ρ = = ∆p =
A A A

Ρ = 1,89x103Pa = 1,8x10-2atm

Capítulo 20 – Halliday e Resnick – Edição antiga

“27” Mostre que a variação de pressão na atmosfera terrestre, suposta isotérmica, é


dada por:
p(y) = p0 e - Mgy / RT

Considerando a atmosfera um fluido em repouso, temos que:

dp = - ρ g dy

onde estamos considerando a superfície da Terra como a origem do eixo y, que


mede a altura de um elemento de volume. Da equação anterior, temos que:

dp
= − ρg
dy

A equação dos gases ideais, nos diz que:

m
pV = µRT = RT
M

onde m = µ M é a massa de um elemento de volume, µ é o número de moles con-


tido nesse elemento de volume e M é a massa molecular da substância considera-
da. Desse modo:
Cap 20 www. f isica. uf pb. br / ~r omer o 19
www.cliqueapostilas.com.br

Pr of . Romer o T avar es da Silva

m RT RT pM
p= =ρ ⇒ ρ=
V M M RT

onde ρ é a densidade do material considerado. A equação da variação da pressão


terá a forma:
dp  pM  dp  Mg 
= − g ⇒ = − dy
dy  RT  p  RT 

Integrando, temos que:


 Mg 
ln p − ln p 0 = − (y − y 0 )
 RT 

Considerando que a superfície da Terra como origem do referencial, y0 = 0 , logo:

 p   Mg 
ln  = − y ⇒ p( y ) = p 0 e −Mgy / RT
 p0   RT 

Capítulo 20 - Halliday, Resnick e Walker

28 Mostre que a equação dos gases ideais p V = µ R T pode ser escrita na forma alter-
nativa p = ρ R T / M onde ρ é a massa específica do gás e M é a massa molar.

pV=µRT
onde
m massa da amostra
µ= =
M massa molar
logo:
m  m  RT
pV = RT ⇒ p= 
M V  M
e portanto:
RT
p=ρ
M

Capítulo 20 - Halliday, Resnick e Walker

33 Qual a trajetória livre média para 15 balas de goma esféricas em um saco que é sa-
cudido vigorosamente? O volume do saco é 1litro e o diâmetro de uma bala é igual
a 1,0cm . Considere colisões de balas com balas, não colisões de balas com o saco.

N = 15balas d = 1,0cm = 10-2m


V = 1l = 10-3m3
V 1
L =
N πd 2

Devemos corrigir essa equação ao considerar que todas as moléculas estão se

Cap 20 www. f isica. uf pb. br / ~r omer o 20


www.cliqueapostilas.com.br

Pr of . Romer o T avar es da Silva

movimentando. A equação corrigida tem a forma:

V 1
L = = 0,150m = 15,0cm
C
N 2 πd 2

Capítulo 20 - Halliday, Resnick e Walker

36 Vinte e duas partículas têm as seguintes velocidades ( Ni representa o número de


partículas que possuem velocidade vi )

Ni 2 4 6 8 2
vi (cm/s) 1,0 2,0 3,0 4,0 5,0

a) Calcule a sua velocidade média vM .

∑v
i =1
i
2 x1 + 4 x 2 + 6 x 3 + 8 x 4 + 2 x 5 70
v = = = = 3,18m/s
N 2+4+6+8+2 22

b) Calcule a sua velocidade média quadrática vRMS .

∑v 2
i
2 x12 + 4 x 2 2 + 6 x 3 2 + 8 x 4 2 + 2 x 5 2 250
v2 = i =1
= = =11,36m2/s2
N 2+4+6+8+2 22

v RMS = v 2 = 3,37m/s

c) Das cinco velocidades mostradas, qual a velocidade mais provável vP ?

vP = 4,0m/s

Capítulo 20 - Halliday, Resnick e Walker

43 A figura abaixo mostra uma distribuição hipotética de velocidades para uma amostra
de N partículas de um gás (observe que P(v) = 0 para v > 2 v0 ) .

a) Expresse a em termos de N e v0 . P(v)

Observando o gráfico de P(v) versus v , po- a


demos notar que:

 a 
 v para 0 ≤ v ≤ v 0
 v 0  0 v0 2v0 v
P (v ) =  a para v 0 ≤ v ≤ 2v 0
 0 para v ≥ 2v 0


Cap 20 www. f isica. uf pb. br / ~r omer o 21
www.cliqueapostilas.com.br

Pr of . Romer o T avar es da Silva

A condição de normalização no diz que:

∫ P (v )dv = 1
0

e portanto:
v0 2v 0
 a  
∫0  v 0 v dv +
 
∫ [a]dv = 1
 v0

 a  v 02 av 0 3av 0
  + a(2v 0 − v 0 ) = + av 0 = =1
v0  2 2 2
ou seja:
 2 
a =  
 3v 0 

b) Quantas das partículas possuem velocidades entre 1,5v0 e 2,0v0 ?

A fração de partículas (N1/N) , com velocidade destro deste intervalo, tem a for-
ma:
2,0 v 0 2,0 v 0
N1  2  v 0  1
= ∫ P (v )dv = ∫ [a ]dv = av 1,5v 0 = a(0,5v 0 ) = 
2,0 v
  =
N 1,5v 0 1,5 v 0
0
 3v 0  2  3
ou seja:
N
N1 =
3

c) Expresse a velocidade média das partículas em termos de v0 .


v = ∫ v P (v ) dv
0

v0 2v 0 v0 2v 0
 a   a v3 v2
3
a v0 a
v = ∫ v  v dv + ∫ v [a]dv = v 0 3 +a = (
+ 4v 02 − v 02 )
0  v 0   v0 0
2 v0
v0 3 2

a 2 3a 2 11 2 11  2  2 11
v = v0 + v 0 = av 0 =  v 0 = v 0
3 2 6 6  3v 0  9
d) Determine vRMS .


v 2
= ∫ v 2 P (v ) dv
0

v0 2v 0 v0 2v 0
 a    a v 4 v3  a  v 04  a  3
v 2
= ∫ v 
2
v dv + ∫v v [a]dv =  v 0
2
 +a =   (
+   8v 0 − v 03 )
0  v 0   0 4 0
3 v0 v0  4 3

Cap 20 www. f isica. uf pb. br / ~r omer o 22


www.cliqueapostilas.com.br

Pr of . Romer o T avar es da Silva

2 av 03 7av 03  1 7   31  2  3  31  2
v = + = av 03  +  =   v 0 =  v 0
4 3  4 3   12  3v 0   18 

31
v RMS = v 2 = v0
18

Capítulo 20 - Halliday, Resnick e Walker

45 Um mol de um gás ideal sofre uma expansão isotérmica. determine a energia adicio-
nada ao gás sob a forma de calor em termos dos volumes inicial e final e da tempe-
ratura.

Como o gás é ideal, a sua energia interna é uma função apenas da temperatura. Se
a transformação for isotérmica, a temperatura se mantém constante e portanto não
existe variação da energia interna nesse processo. Desse modo, usando a primeira
lei da termodinâmica, encontramos que:

(dE)T = (dQ)T – (dW)T = 0 ⇒ (dQ)T = (dW)T

f Vf
dV
= µRT (lnVf − lnVi )
Vf
W if = ∫ pdV = µRT ∫ = µRT lnV
i Vi
V Vi

V 
Q if = W if = µRT ln f 
 Vi 

Capítulo 20 - Halliday, Resnick e Walker

47 Um recipiente contém uma mistura de três gases que não reagem entre si: µ1 moles
do primeiro gás com calor específico molar a volume constante C1 , e assim por di-
ante. Determine o calor específico molar a volume constante da mistura, em termos
dos calores específicos molares e das quantidades dos gases em separado.
O número total de moles desta mistura de três gases é dada por:

µ = µ1 + µ2 + µ3

e a quantidade de calor total absorvido (a volume constante) pela mistura será a


soma dos calores absorvidos pelos diversos componentes:

dQV = dQV1 + dQV2 + dQV3


Calculando as derivadas:

 dQ   dQ1   dQ   dQ 
  =  + 1 + 1 ⇒ µCV = µ 1CV 1 + µ 2CV 2 + µ 3CV 3
 dT V  dT V  dT V  dT V

µ 1CV 1 + µ 2CV 2 + µ 3CV 3


CV =
µ1 + µ 2 + µ 3

Cap 20 www. f isica. uf pb. br / ~r omer o 23


www.cliqueapostilas.com.br

Pr of . Romer o T avar es da Silva

Capítulo 20 - Halliday, Resnick e Walker

57 Sejam µ moles de um gás ideal que se expande adiabaticamente de uma tempera-


tura inicial T1 até uma temperatura final T2 . Prove que o trabalho realizado pelo gás
é µ CV (T2 – T1) , onde CV é o calor específico molar a volume constante.

O calor específico molar a volume constante é definido como:


1  ∂E 
CV =  Int 
µ  ∂T  V =const
Mas a energia interna do gás ideal depende exp0licitamente apenas da temperatura,
e neste caso, a derivada parcial se transforma em derivada total, ou seja:

1 dE Int
CV = ⇒ dE Int = µCV dT
µ dT
A primeira lei da Termodinâmica diz que:

dEInt = dQ - dW
e para uma gás ideal, temos que:
µ CV dT = dQ – dW

Quando a transformação for adiabática , não existe troca de calor com o ambiente,
logo:
µ CV dT = - dW
e portanto:
T2

W12 = − µCV ∫ dT
T1

ou seja:
W12 = µ CV (T1 – T2)

Capítulo 20 - Halliday, Resnick e Walker

61 Um mol de um gás ideal monoatômico percorre o ciclo 123 da figura abaixo. O pro-
cesso 1 → 2 ocorre a volume constante, o processo 2 → 3 é adiabático e o proces-
so 3 → 1 ocorre a pressão constante.
a) Calcule o calor Q , a variação de
energia interna ∆EI e o trabalho rea- p
lizado W , para cada um dos três
processos e para o ciclo como um 2
todo.

T1 = 300K
T2 = 600K
T3 = 455K 1 3

dEInt = dQ – p dV
V
O processo 1 → 2 é realizado a

Cap 20 www. f isica. uf pb. br / ~r omer o 24


www.cliqueapostilas.com.br

Pr of . Romer o T avar es da Silva

volume constante:
dEInt = dQ ⇒ ∆EInt = Q12

Como se trata de um gás ideal monoatômico:

3
E Int = µRT
2
ou seja:
3
Q12 = µR (T2 − T1 )
2
e como temos apenas um mol:
3
Q12 = R (T2 − T1 )
2
e portanto:
∆EInt = Q12 = 3.740J
W12 = 0

O processo 2 → 3 é realizado adiabaticamente, ou seja dQ = 0 e pV γ = const .

dEInt = - dW ⇒ ∆EInt = - W12

Como se trata de um gás ideal monoatômico:

3
E Int = µRT
2
ou seja:
3
W 23 = − µR (T3 − T2 )
2
e como temos apenas um mol:
3
W 23 = − R (T3 − T2 )
2
e portanto:
∆EInt = W23 = 1.807J
Q23 = 0

O processo 3 → 1 é realizado a pressão constante. Usando a definição de tra-


balho, encontramos que:

1 V1

W 31 = ∫ pdV = p1 ∫ dV = p1 (V1 − V3 )
3 V3

e como o gás é ideal


pV=µRT
ou seja:
W31 = R (T1 – T3) = - 1288J

A energia interna de um gás ideal é dada por:

Cap 20 www. f isica. uf pb. br / ~r omer o 25


www.cliqueapostilas.com.br

Pr of . Romer o T avar es da Silva

3
E Int = µRT
2
e portanto:
3
∆E Int = R (T1 − T3 ) = - 1932J
2

Usando a primeira lei da Termodinâmica, temos que:

∆EInt = Q31 – W31 ⇒ Q31 = ∆EInt + W31


ou seja:
5
Q31 = R (T1 − T3 ) = -3220J
2

b) A pressão no ponto 1 é 1,00atm . determine a pressão e o volume nos pontos 2


e 3 . Use 1,00atm = 1,013x105Pa e R = 8,314J/mol . K

p1 = 1,00atm = 1,013x105Pa T1 = 300K


R = 8,314J/mol . K T2 = 600K
T3 = 455K
RT1
V1 = = 0,246m 3
p1

 p 3 = p1


 RT3
V3 = = 0,0373m 3
 p3

 V2 = V1


 RT2
p2 = = 2,0 x10 5 N / m 2 = 2,0atm
 V2

Cap 20 www. f isica. uf pb. br / ~r omer o 26


www.cliqueapostilas.com.br

Prof. Romero Tavares da Silva

21. Entropia e a Segunda Lei da Termodinâmica

Processos reversíveis e irreversíveis

Segundo o Dicionário Aurélio, que reflete o nosso linguajar coloquial, algo é rever-
sível quando se pode reverter; ou que pode retornar ao estado inicial. Em Física, um pro-
cesso é reversível quando pode parti do estado final e alcançar o estado inicial usando os
mesmos micro-estados que utilizou para alcançar o estado final.

Consideremos um sistema em equilíbrio, e apenas nessas circunstâncias podemos


caracterizar um estado termodinâmico e, nesse estado podemos atribuir valores para as
funções termodinâmicas de estado tais como temperatura, pressão, energia interna, e etc.
Quando um sistema sofre variações através de absorção de calor ou trabalho, ele sai
momentaneamente do estado de equilíbrio, e depois de um certo tempo de relaxação en-
contra outro estado de equilíbrio.

Quando a variação sofrida pelo sistema for infinitesimal, as suas funções termodinâ-
micas também sofrerão variações infinitesimais. E podemos caracterizar os novos valores
das funções termodinâmicas para essa nova situação de equilíbrio. Essas transformações
infinitesimais são chamadas às vezes de transformações quasi-estáticas. Quando subme-
temos um sistema a várias transformações quasi-estáticas, podemos definir uma sequên-
cia de valores pra as suas funções de estado, que irão caracterizar cada uma das peque-
nas transformações. Podemos desse modo executar a mudança de um sistema físico en-
tre dois estados termodinâmicos afastados, utilizando uma sequência de pequenas trans-
formações quasi-estáticas.

Um exemplo dessa situação seria considerar um gás em equilíbrio, contido em um


êmbolo, que está mantido nessa posição por uma certa quantidade de pequenos pesos. À
medida que retiramos um pequeno peso, a pressão exercida no êmbolo diminui infinitesi-
malmente, fazendo com que o gás encontre outra situação de equilíbrio, infinitesimalmen-
te próxima da situação de equilíbrio anterior. Quando terminarmos de retirar os pesos, o
gás encontra-se em um estado termodinâmico final distante do estado termodinâmico ini-
cial. E o gás alcançou o estado final seguindo um percurso de estados intermediário que
foram sendo conhecidos enquanto ele sofria as transformações infinitesimais.

Se quisermos fazer o gás retornar ao estado inicial pelo mesmo percurso, será ne-
cessário apenas ir recolocando paulatinamente os pesos em sues lugares originais, e o
sistema voltará usando os mesmos estados do percurso de ida.

Cap 21 www.fisica.ufpb.br/~romero 2
www.cliqueapostilas.com.br

Prof. Romero Tavares da Silva

Consideremos o mesmo sistema anterior, no mesmo estado inicial. A única diferen-


ça da configuração seria que os pequenos pesos forma substituídos por um único peso de
mesma massa. Quando esse peso é retirado, o sistema sofre uma mudança brusca até
atingir o equilíbrio. Durante esse processo, para as funções termodinâmicas de estado
tais como temperatura, pressão, energia interna; ficam indefinidas, pois são caracteriza-
das apenas em situações de equilíbrio.

Como não estamos limitando as possibilidades de interação entre o gás e o ambien-


te, as quantidades de calor e trabalho envolvidas no percurso de volta podem ser diferen-
tes das quantidades do percurso da vinda.

No percurso inicial, quando retiramos o peso, o gás enfrentou um processo de forte


desequilíbrio no qual não é possível definir as funções termodinâmicas. Essa é a essência
de um processo irreversível: a impossibilidade de definir os estados intermediários de
uma transformação termodinâmica. Como não podemos conhecer o percurso utilizado,
não podemos reverter o processo pelo mesmo caminho.

Existe uma outra faceta que caracteriza os processos irreversíveis. São que pro-
cessos que naturalmente acontecem apenas em uma direção.

Na experiência cotidiana percebemos que o calor sempre vai naturalmente do cor-


po mais quente para o mais frio, até que as temperaturas se equilibrem. Mas nunca acon-
tece o contrário: o calor naturalmente ir do corpo mais frio para corpo o mais, esquentan-
do o mais quente e esfriando o mais frio. Essa frase anterior chega a incomodar do ab-
surdo que ela reflete. Porque acontece isso se as duas transformações são equivalentes
em termos energéticos: a energia seria conservada em ambas as situações.

As mudanças que acontecem com a energia dentro de um sistema fechado não


impõem o sentido de processos irreversíveis. Essa direção é imposta pela análise da va-
riação de uma outra grandeza termodinâmica: a entropia. A entropia está associada com
o grau de organização de um sistema. E esse grau de organização não pode nunca dimi-
nuir naturalmente.

Quando um sistema esfria significa que diminuiu a sua energia interna e, portanto a
amplitude de seus movimentos, o números de graus de liberdade. Isso implica em torná-lo
mais organizado. Nessa situação, esfriar o sistema significaria diminuir a entropia, e por
isso em um sistema isolado a temperatura nunca diminui.

Cap 21 www.fisica.ufpb.br/~romero 3
www.cliqueapostilas.com.br

Prof. Romero Tavares da Silva

Máquinas térmicas

Máquina térmica ou motor é um dispositivo que extrai energia do ambiente, na for-


ma de calor, e realiza trabalho útil. No interior de toda máquina térmica está uma substân-
cia de trabalho, que sofre as transformações termodinâmicas que possibilitam as mudan-
ças de forma da energia.

Para que uma máquina funcione de maneira permanente é necessário que ela ope-
re em ciclos, ou seja: a substância de trabalho passa por diversas transformações termo-
dinâmicas até retornar ao estado inicial, completando um ciclo.

De modo geral as máquinas térmicas operam em ciclo entre duas fontes térmicas
com temperaturas diferentes. Uma máquina térmica retira calor da fonte quente e rejeita
parte desse calor para uma fonte fria e transforma essa diferença de energia em trabalho
mecânico.

Uma máquina de Carnot

p a

Em um ciclo de uma máquina de Car- Q2


not a substância de trabalho passa por qua-
tro processos diferentes, onde dois proces- b
sos são isotérmicos (ab e cd) e os outros T2
dois processos são adiabáticos (bc e da). d
c
Q1 T1
V

T2
O sistema absorve uma quantidade de calor Q2
isotermicamente a uma temperatura T2 quando vai
do estado a para o estado b . E de maneira equiva- Q2
lente, o sistema rejeita uma quantidade de calor Q1
isotermicamente a uma temperatura T1 quando vai W
do estado c para o estado d . As transformações
entre os estados b e c , bem como entre os estados Q1
d e a acontecem adiabaticamente, ou seja: sem
que ocorra troca de calor com o ambiente.
T1

O trabalho executado pelo sistema quando acontece a transformação isotérmica


entre os estados a e b é calculado como:

b
Wab = ∫ p dV
a

Cap 21 www.fisica.ufpb.br/~romero 4
www.cliqueapostilas.com.br

Prof. Romero Tavares da Silva

e considerando a substância de trabalho como um gás ideal, temos que:

µRT
p=
V
ou seja:
V 
Vb
dV
Wab = µRT2 ∫ =µRT2 ln b 
Va
V  Va 

De maneira equivalente calculamos trabalho executado pelo sistema quando acon-


tece a transformação isotérmica entre os estados c e d como sendo:

V 
Vd
dV
Wcd = µRT1 ∫ =µRT1 ln d 
Vc
V  Vc 

Considerando que a substância de trabalho é um gás ideal, a sua energia interna


depende explicitamente apenas da temperatura, e desse modo ela se mantém constante
ao longo de uma transformação isotérmica. Ou seja:

E (T2 ) = E a = E b


E (T ) = E = E
 1 c d

Tendo em conta a primeira lei da termodinâmica

dE = dQ - dW
encontramos que:

 ∆E ab = Qab − Wab ⇒ Q2 ≡ Qab =W ab




∆E = Q − W ⇒ Q3 ≡ Qcd = Wcd
 cd cd cd

ou seja:

µRT1 lnVb 
Q1 Wab
= =  Va 
Q2 Wcd µRT2 Vd 
 Vc 

Ainda considerando as propriedades de um gás ideal, quando ele é submetido a


uma transformação adiabática, temos que:

TV γ −1 = cons tan te
ou seja:
T2Vbγ −1 = T1Vcγ −1


T V γ −1 = T V γ −1
 2 a 1 d

Cap 21 www.fisica.ufpb.br/~romero 5
www.cliqueapostilas.com.br

Prof. Romero Tavares da Silva

logo:
Vbγ −1 Vcγ −1 V  V  V 
= ⇒ ln b  = ln c  = − ln d 
Vaγ −1 Vdγ −1  Va   Vd   Vc 
e finalmente
µRT1 lnVd 
Q1 Wcd
= =  Vc  = − T1
Q2 Wab µRT2 Vb  T2
 Va

Eficiência de uma máquina de Carnot

A eficiência de uma máquina térmica qualquer é definida como a sua capacidade


de transformar calor em trabalho. Ou seja:

trabalho efetivo WE
ε= =
calor absorvido Qabsorvido

onde o trabalho efetivo é entendido como a soma de todos os trabalhos envolvidos em


cada etapa do ciclo completo, e o calor absorvido é considerado como o soma de todos
os calores absorvidos (positivos), ignorando-se os calores rejeitados (negativos).

Em um ciclo de Carnot, como esse considerado anteriormente:

WE = Wab + Wcd = |Wab| - |Wcd|

onde enfatizamos que Wcd < 0 . Por outro lado, o calor absorvido foi Q2 > 0. E desse
modo temos que:

Wab − Wcd Q2 − Q1 Wab − Wcd Q1


ε= = = = 1−
Q2 Q2 Wab Q2
ou seja:
T1
ε = 1−
T2

Refrigeradores

Refrigerador é um dispositivo cuja função é transferir calor de um reservatório tér-


mico em uma temperatura mais baixa para um outro reservatório térmico em uma tempe-
ratura mais alta. Em um processo natural o calor se transfere de um reservatório com
temperatura mais alta para outro com uma temperatura mais baixa. Para conseguir reali-
zar uma transferência de calor num sentido contrário ao sentido natural, o refrigerador
necessita executar trabalho na substância de trabalho.

A região onde são armazenados os alimentos no interior de uma geladeira domés-


tica é o reservatório frio, e o reservatório quente é o ambiente que rodeia a geladeira. Pa-
ra um ar-condicionado o reservatório frio é o interior do aposento onde ele está instalado,
e o reservatório quente é o ambiente externo a esse aposento.

Cap 21 www.fisica.ufpb.br/~romero 6
www.cliqueapostilas.com.br

Prof. Romero Tavares da Silva

De maneira semelhante a uma má-


quina térmica ideal, em um refrigerador ide- p a
al todos os processos são reversíveis.
Q2
Em um refrigerador de Carnot temos
um ciclo passando pelos mesmos estados b
de uma máquina de Carnot, mas com uma T2
seqüência de transformações em um senti- d
do contrário, como mostra a figura ao lado. c
Q1 T1
V

O equivalente à eficiência de uma máquina tér- T2


mica é definido como coeficiente de desempenho de
um refrigerador K :
Q2
o que queremos Q1
K= = W
o que pagamos Q2 − Q1
Q1
Para um refrigerador de Carnot temos que:

T1 T1
KC =
T2 − T1

Teorema de Clausius

Quando estávamos analisando o ciclo de Carnot, encontramos que:

µRT1 lnVd 
Q1 Wcd
= =  Vc  = − T1
Q2 Wab µRT2 Vb  T2
 Va

Podemos então dizer que quando uma máquina térmica realiza um ciclo reversível
usando duas transformações isotérmicas de temperaturas T1 e T2 e duas transforma-
ções adiabática que partem de uma isotérmica e alcança a outra, como foi indicado ante-
riormente, nós temos que:
Q1 Q2
+ =0
T1 T2

Se tivermos um grande número de transformações reversíveis alternadamente iso-


térmicas e adiabáticas, como na situação anterior, de modo que esse sistema complete
um ciclo, poderemos generalizar a equação anterior como:

Qi
∑T
i
=0
i

Cap 21 www.fisica.ufpb.br/~romero 7
www.cliqueapostilas.com.br

Prof. Romero Tavares da Silva

A generalização da equação anterior é conhecida como o teorema de Clausius.

Seja dQ a quantidade de calor que um dado sistema troca com o ambiente que o
rodeia, e seja T a temperatura em que se dá essa troca de calor; segundo o teorema de
Clausius nós temos que:

 dQR
∫ T = 0 ; num ciclo reversível
C

 dQ
 ∫ T ≤ 0 ; num ciclo irreversível
C

Como foi dito anteriormente, um processo reversível é composto de pequenos pro-


cessos entre estados termodinâmicos muito próximos. Acontece uma pequena mudança
no estado de equilíbrio de um sistema, e ele encontra um novo estado de equilíbrio pró-
ximo ao estado inicial.

Apesar da grandeza dQR/T de modo geral não ser uma função de estado, para um
processo reversível ela comporta como uma função de estado, e podemos definir a entro-
pia S como sendo essa grandeza, de tal modo que:

dQ R
dS =
T

Como a entropia é uma função de es-


tado, a diferença entre os valores de da en- p
tropia de dois estados independe do cami-
nho usado para se ir de um estado até o
outro. Vamos considerar um processo re- f
versível cíclico, partindo do estado i até o 1
estado f pelo percurso 1 e voltando até o
estado original pelo percurso 2 . Desse 2
modo, temos que: i

dQ R

C
T
= ∫ dS = 0
C
V

ou seja:
f i f i f f

∫ dS +
i (1)
∫ dS = 0
f ( 2)
⇒ ∫ dS = − ∫ dS ∴
i (1) f (2)
∫ dS =
i (1)
∫ dS
i ( 2)

Como os percursos 1 e 2 foram escolhidos genericamente, podemos concluir


que num processo reversível a variação de entropia entre dois estados de equilíbrio não
depende do percurso usado para ir de um estado até o outro.

Cap 21 www.fisica.ufpb.br/~romero 8
www.cliqueapostilas.com.br

Prof. Romero Tavares da Silva

A segunda lei da Termodinâmica

A primeira lei da termodinâmica incorpora ao princípio geral da conservação da e-


nergia o reconhecimento de que calor é uma forma de energia. Qualquer processo cuja
energia total seja conservada é compatível com a primeira lei da termodinâmica.

No entanto, existem processos que só acontecem em um sentido, são os proces-


sos irreversíveis. A segunda lei da termodinâmica dá consta desta questão, assim como
das possíveis maneiras de transformar calor em trabalho.

Enunciado de Kelvin
É impossível realizar um processo cujo único efeito seja remover ca-
lor de um reservatório térmico e produzir uma quantidade equivalen-
te de trabalho.

Consequências do enunciado de Kelvin


- A geração de calor por atrito a partir de trabalho mecânico é irreversível.
- A expansão livre de um gás é um processo irreversível.
- A condução de calor, que se dá sempre do corpo mais quente para o mais frio, é um
processo irreversível.

Curso de Física Básica - Vol 2 - item 10.2


Moysés Nussenzveig

Enunciado de Clausius
É impossível realizar um processo cujo único efeito seja transferir
calor de um corpo mais frio para um corpo mais quente.

Variação da entropia - casos particulares

Transformação adiabática reversível

Em uma transformação adiabática reversível o sistema não troca calor com o am-
biente e, portanto:

dQR = 0 ⇒ dS = 0 ⇒ ∆S = Sf − Si = 0

Variação da entropia em uma transição de fase

Em uma transição de fase o sistema absorve (ou fornece) calor sem que exista
uma variação de temperatura:

Cap 21 www.fisica.ufpb.br/~romero 9
www.cliqueapostilas.com.br

Prof. Romero Tavares da Silva

f f
dQR
∆S = Sf − Si = ∫ dS = ∫
i i
T

e como a temperatura é constante

∆QR mL
∆S = =
T T

onde m é a massa do sistema e L é o calor latente desse sistema nessa transição de


fase.

Variação de entropia de um gás ideal

De acordo com a primeira lei da termodinâmica

dE = dQ − dW ⇒ TdS = dQ = dE + pdV
ou seja:
dE p
dS = + dV
T T

Considerando que para um gás ideal:


dE = µCv dT


 p µR
 T = V
encontramos:
f f f
dT dV
∆S = Sf − Si = ∫ dS = ∫ µCv + ∫ µR
i i
T i
V

Se considerarmos o calor específico constante na região de integração, teremos


que:

T  V 
∆S = Sf − Si = µCv ln f  + µR ln f 
 Ti   Vi 

Probabilidade e entropia

Uma amostra de um gás comum contém um número muito grande de átomos ou


moléculas. Para termos uma idéia da ordem de grandeza de quão grande é esse número
basta lembrarmos que em um mol de hidrogênio (2 gramas) existem 1023 moléculas (nú-
mero de Avogadro).

Cap 21 www.fisica.ufpb.br/~romero 10
www.cliqueapostilas.com.br

Prof. Romero Tavares da Silva

Para lidar uma grande quantidade de moléculas vamos introduzir conceitos de pro-
babilidade e estatística, e para tal vamos analisar um gás composto por pouquíssimas
partículas.

Consideremos um gás com apenas duas moléculas idênticas, que ocupam um re-
cipiente dividido em duas partes; à parte da esquerda e a parte da direita.

Quais as possíveis configurações que esse gás pode apresentar? Podemos ter as
possibilidades mostradas adiante:

1 2 A As duas moléculas ocupam o lado esquerdo do recipiente.

1 2 B Uma molécula ocupa o lado esquerdo do recipiente enquanto


a outra molécula ocupa o lado direito.

As posições são invertidas, a molécula que na configuração


2 1 C anterior ocupava o lado esquerdo passa a ocupar o lado di-
reito do recipiente, e a molécula que na configuração anterior
ocupava o lado direito passa a ocupar o lado esquerdo do
recipiente.

1 2 D As duas moléculas ocupam o lado direito do recipiente.

Mas afinal, esse gás de duas moléculas se apresentará em qual configuração? Es-
sa situação se apresenta de uma forma nova, pois o gás pode se apresentar em qualquer
uma das configurações. A pergunta deve ser feita de uma maneira diferente: qual a pro-
babilidade do gás se apresentar em cada uma das configurações? Para responde a essa
pergunta vamos construir uma tabela:

Configuração Molécula 1 Molécula 2 nE nD No estados Probabilidade


A E E 2 0 1 1/4
B E D 1 1
C D E 1 1 2 2/4 = 1/2
D D D 0 2 1 1/4
Total 4 1

Nós temos dois estados equivalentes, e são aqueles associados com as configura-
ções B e C. O total de estados acessíveis para as duas moléculas, nestas circunstân-
cias, é 2N = 22 = 4 . Considerando que cada uma das configurações são igualmente pro-
váveis, a probabilidade de encontrar o sistema em cada uma delas é ¼ . Como temos
duas configurações equivalentes (B e C), a probabilidade de encontrar sistema em uma
delas é 2.(1/4) = ½ . As duas configurações são equivalentes (B e C) por que são indis-
tinguíveis, não se pode distinguir em qual das configurações o sistema está.

Cap 21 www.fisica.ufpb.br/~romero 11
www.cliqueapostilas.com.br

Prof. Romero Tavares da Silva

Para tentar entender o comportamento de um gás real, devemos analisar um gás


com um número cada vez maior de moléculas. Nesse sentido, vamos refazer os cálculos
anteriores considerando dessa vez um gás com 4 moléculas.

N=4
Molécula No de Probabilidade
1 2 3 4 nE nD estados P(nE, nD)
4 1
= 4
1
E E E E 4 0 1 =  
0 16 2
D E E E  4 4 1  4 1
E D E E 3 1 4 =   = =   4
E E D E  1 16 4  1  2
E E E D
D D E E
D E D E  4 6 3 4 1
D E E D 2 2 6 =   = =   4
E D D E  2 16 8  2  2
E D E D
E E D D
D D D E  4 4 1  4 1
D D E D 1 3 4 =   = =   4
D E D D 3 16 4  3  2
E D D D
4 1 1
= 4
D D D D 0 4 1 =  
4 16 2
Totais 16 = 24 1

Estamos usando a notação:

N  N!  N 
  = =  
 n  n! (N − n )!  N − n 

É possível generalizar os resultados obtidos para a situação onde o sistema é


composto por um número N de moléculas. As probabilidades calculadas para cada uma
das situações têm a forma:
N  1
P (nE , nD ) =   N
 nE  2

N = 2, 4, 8, 16, 100 N = 1000


0,030

0,60
0,025

0,020
0,40
P(n,N
P(n,N

0,015

0,20 0,010

0,005
0,00
0,00 0,25 0,50 0,75 1,00 0,000
0,00 0,25 0,50 0,75 1,00
n/N
n/N

Cap 21 www.fisica.ufpb.br/~romero 12
www.cliqueapostilas.com.br

Prof. Romero Tavares da Silva

Como mostra a figura anterior na medida que aumenta o número N de moléculas


do gás o máximo em torno de nE = nD vai se tornando cada vez mais agudo. Podemos
entender que quando N for pequeno, não são muito diferentes as probabilidades do sis-
tema ocupar um de seus estados acessíveis. No entanto, quando N assume valores a-
preciáveis, existe uma grande quantidade de estados equivalentes em torno de nE = nD e
desse modo existe uma grande probabilidade do sistema ocupar um estado onde nE = nD
onde o número de moléculas na parte esquerda do recipiente é igual ao número de molé-
culas na parte direita do recipiente.

Curso de Física Básica - Vol 2 - item 12.5


Moysés Nussenzveig

Uma visão estatística da entropia

No item anterior encontramos que o número de estados acessíveis W(N, nE ) para


uma dada escolha de nE , ou seja, a multiplicidade de estados com essa mesma caracte-
rística é dado por
N  N!  N 
W (N, nE ) =   = =  
 n  n! (N − n )!  N − n 

O Físico austríaco Ludwig Boltzmann deduziu uma relação entre a entropia S de


um sistema e a multiplicidade W(N, nE ) , e essa relação tem a forma:

S = kB ln W

Calor, trabalho e energia

Calor é a energia que se transfere de um corpo para o outro corpo devido a uma di-
ferença de temperatura entre eles.
Trabalho é a energia que se transfere de um corpo para o outro devido a uma força
que age entre eles.

Cap 21 www.fisica.ufpb.br/~romero 13
www.cliqueapostilas.com.br

Prof. Romero Tavares da Silva

Solução de alguns problemas

Capítulo 21 - Halliday, Resnick e Walker

01 Uma amostra de 2,5moles de um gás ideal se expande reversível e isotermicamen-


te a 360K até que o seu volume seja dobrado. Qual o aumento de entropia do gás?

µ = 2,5moles
T = 360K
Vf = 2 Vi

Para um gás ideal a energia interna é função apenas da temperatura, e desse modo
em uma transformação isotérmica a energia interna não varia. Considerando a pri-
meira lei da termodinâmica, para uma transformação isotérmica ( dE = 0) , temos
que:
dE = dQ - dW ⇒ dQ = dW = p dV
Por outro lado:
dQ dW pdV dV
dS = = = = µR
T T T V

onde a última igualdade é uma consequência da equação de estado para um gás


ideal. Integrando a equação anterior, temos que:

V 
Vf
dV
∆S = S f − S i = µR ∫ = µR ln f  = µR ln 2
Vi
V  Vi 

∆S = 14,41 J/K

Capítulo 21 - Halliday, Resnick e Walker

06 Um gás ideal monoatômico à temperatura inicial T0 (em Kelvins) se expande do


volume inicial V0 até o volume final 2V0 , por
3,0
cada um dos processos indicados na figura ao
lado. No processo AF a temperatura final é 2,5 B
Temperatura(T0)

de 0,63T0 .
2,0
Em que processo a expansão é: C
1,5 D
a) isotérmica A
1,0
b) Isobárica (pressão constante) E
c) adiabática 0,5 F
Explique as suas respostas.
0,0
0 1 2 3
d) Em quais dos processos a entropia do gás Volume(V0)
diminui?

Cap 21 www.fisica.ufpb.br/~romero 14
www.cliqueapostilas.com.br

Prof. Romero Tavares da Silva

a) Numa expansão isotérmica de um gás temos que, obviamente, a temperatura


permanece constante e, portanto isso acontece no processo AE .

b) Numa expansão isobárica de um gás ideal temos que:

Vi µR V f V 
= = ∴ Tf = Ti  f  = 2Ti
Ti p Tf  Vi 

e, portanto isso acontece na expansão AC .

c) Numa expansão adiabática de um gás ideal temos que:

pV γ = const ⇒ TV γ −1 = const

Para um gás ideal monoatômico:

5R
CP 5 2
γ = = 2 = ∴ γ −1=
CV 3R 3 3
2
e desse modo:
γ −1
γ −1 γ −1 V  Ti
T i Vi = T f Vf ∴ Tf = Ti  i  = 2
= 0,629Ti
 Vf  2 3

e, portanto isso acontece na expansão AF .

d) Numa expansão isotérmica desse tipo, como mostrado no problema 06, a en-
tropia varia da forma:
V 
Vf
dV
∆S = S f − S i = µR ∫ = µR ln f  = µR ln 2 > 0
Vi
V  Vi 

Numa expansão adiabática dQ = 0 e desse modo ∆S = 0

Numa variação genérica em um gás ideal temos que:

dQ = dE + p dV = µ CV dT + p dV
ou seja:
dQ p dV
dS = = µCV dT + dV = µCV dT + µR
T T V
e, portanto
T  V 
∆S = µCV ln f  + µR ln f 
 Ti   Vi 

Numa expansão isobárica desse tipo

∆S = (µCV + µR ) ln 2 > 0

Cap 21 www.fisica.ufpb.br/~romero 15
www.cliqueapostilas.com.br

Prof. Romero Tavares da Silva

Capítulo 21 - Halliday, Resnick e Walker

07 a) Qual a variação de entropia de um cubo de gelo de 12,0g que se derrete com-


pletamente em um balde de água cuja temperatura está logo acima do ponto de
congelamento da água?

m = 12,0g = 0,012kg
T = 00C = 273K
LF = 79,5cam/g = 333x103J/kg

Q mLF
∆S = = = 14,63J / K
T T

b) Qual a variação de entropia de uma colherada de 5,0g de água que evapora


completamente em cima de um prato quente cuja temperatura está ligeiramente
acima do ponto de ebulição da água?

m = 5,0g = 0,005kg
T = 1000C = 373K
LV = 539cam/g = 2.256x103J/kg

Q mLV
∆S = = = 30,24J / K
T T

Capítulo 21 - Halliday, Resnick e Walker

09 Em um experimento, 200g de alumínio (com calor específico de 900J/kg.K) a


1000C são misturados com 50,0g de água a 20,00C , com a mistura termicamente
isolada.

a) Qual a temperatura de equilíbrio?

ma = 200g = 0,2kg mA = 50g = 0,05kg


ca = 900J/kg.K cA = 1cal/g 0C = 4.190J/kg.K
Ta = 1000C = 373K TA = 200C = 293K

Como o sistema composto por alumínio e água está isolado, ele não troca calor
com a vizinhança, e desse modo:

∆Q = ∆Qa + ∆QA = 0

e desse modo alcançam uma temperatura de equilíbrio T :

ma ca (T - Ta) + mA cA (T - TA) = 0
ou seja:
m a c aTa + m A c AT A
T = = 56,97 0 C = 329,97K
ma c a + m A c A

Cap 21 www.fisica.ufpb.br/~romero 16
www.cliqueapostilas.com.br

Prof. Romero Tavares da Silva

b) Qual a variação de entropia do alumínio?

T 
Tf
dQ mcdT dT
dS = = ∴ ∆S = S f − S i = mc ∫ = mc ln f 
T T Ti
T  Ti 
ou seja:
T 
∆S a = m a c a ln  = −22,07J / K
 Ta 
c) Qual a variação de entropia da água?

T 
∆S A = m A c A ln  = +24,86J / K
 TA 

d) Qual a variação de entropia do sistema água - alumínio?

∆S = ∆Sa + ∆SA ⇒ ∆S = +2,79J/K

Capítulo 21 - Halliday, Resnick e Walker

11 A figura abaixo mostra dois blocos idênticos de massa m = 1,5kg . O bloco E da


esquerda está a uma temperatura TiE = 600C e o bloco D da direita está a uma
temperatura TiD = 200C . Os blocos estão em uma caixa isolada termicamente e es-
tão separados por uma divisória isolante. Quando levantamos a divisória, os blocos
acabam chegando a uma temperatura de equilíbrio Tf = 400C .

E D

a) Qual a variação de entropia resultante do sistema de dois blocos durante esse


processo irreversível?
Depois que a divisória isolante é retirada, os blocos trocam calor até atingirem o
equilíbrio térmico. Ou seja:
∆Q = ∆Qe + ∆Qd = 0

Te + T d
mc (T − Te ) + mc (T − Td ) = 0 ∴ T =
2

Para calcular a entropia neste processo irreversível, usamos o fato que a entropia
é uma função de estado e, portanto o seu valor depende apenas do estado em
que se encontra, não importando o processo através do qual alcançou este esta-
do. Podemos imaginar que cada um dos blocos alcançou o seu estado final atra-
vés de processos reversíveis.

Cap 21 www.fisica.ufpb.br/~romero 17
www.cliqueapostilas.com.br

Prof. Romero Tavares da Silva

Por exemplo, cada um bloco poderia ter a sua temperatura modificada lentamen-
te através da troca de calor com um banho térmico (reservatório) adequado até
que atingissem a temperatura de equilíbrio original T . Desse modo podemos
calcular a variação de entropia para cada um dos blocos:

T 
Tf
dQ dT dT
dS = = mc ∴ ∆S = mc ∫ = mc ln f 
T T Ti
T  Ti 
ou seja:
 T 
∆S e = mc ln 
  Te 

 T 
∆S e = mc ln 
  Td 

  T  T   T2   (Te + Td )2 
∆ST = ∆S e + ∆S d = mc ln  + ln  = mc ln  = mc ln 
  Te   Td  TeTd   2TeTd 

No entanto:
(Te + Td )2
(Te + Td ) 2
= T + T + 2TeTd
e
2
d
2
∴ (Te + Td ) 2
> 2TeTd ⇒ >1
2TeTd
ou seja:
 (T + Td )2 
ln e >0
 2TeTd 
e, portanto
∆ST > 0

b) Mostre que se o processo acontecesse no sentido inverso, a entropia do sistema


diminuiria, violando a segunda lei da termodinâmica.

Se o processo acontecer no sentido inverso

 I  Te 
 ∆S e = mc ln 
 T 

 I  Td 
∆S d = mc ln 
 T 

 2TeTd 
∆STI = ∆S eI + ∆S dI = mc ln 2 
<0
 (Te + Td ) 

Cap 21 www.fisica.ufpb.br/~romero 18
www.cliqueapostilas.com.br

Prof. Romero Tavares da Silva

Capítulo 21 - Halliday, Resnick e Walker

18 Um cilindro contém µ moles de um gás ideal monoatômico. Se o gás sofrer uma


expansão isotérmica reversível do volume
inicial Vi até o volume final Vf ao longo 1,2
da trajetória I da figura ao lado, a sua i
1
variação de entropia é:

Pressão
0,8
 Vf  I
∆S = µR ln  0,6 Isoterma
 Vi  0,4 II f T
0,2 Adiabática x
Agora considere a trajetória II da figura 0
ao lado, que leva o gás do mesmo estado 0 1 2 3 4
inicial i até o estado x por meio de uma Volume
expansão adiabática reversível, e depois
deste estado x até o mesmo estado final
f por meio de um processo reversível a volume constante

a) Descreva como você poderia realizar os dois processos reversíveis para a traje-
tória II

b) Mostre que a temperatura do gás no estado x é dada por:


2/3
V 
T x = Ti  i 
 Vf 

A transformação ix é adiabática, e numa transformação desse tipo para um gás


ideal temos que:

pV γ = const ⇒ TV γ −1 = const ∴ Ti Vi γ −1 = T xV xγ −1
mas
5R
CP 5 2
γ = = 2 = ∴ γ −1=
CV 3R 3 3
2
ou seja:
2/3
V 
T x = Ti  i 
 Vf 

c) Qual a energia QI transferida sob a forma de calor ao longo da trajetória I e a


energia QII transferida sob a forma de calor ao longo da trajetória II ? Elas são
iguais?
Ao longo da trajetória I temos um processo isotérmico. Considerando a primei-
ra lei da termodinâmica, para uma transformação isotérmica ( dE = 0) , temos
que:

dE = dQ - dW ⇒ dQ = dW = p dV
Ou seja:

Cap 21 www.fisica.ufpb.br/~romero 19
www.cliqueapostilas.com.br

Prof. Romero Tavares da Silva

V 
Vf
dV dV
dQ = µRT
V
∴ QI = µRTi
Vi

V
= µRT ln f
 Vi


Ao longo da trajetória II temos um processo adiabático ( ix ) e um outro isovo-


lumétrico ( xf ) . Ou seja:

QII = Qix + Qxf

Como no processo adiabático não existe troca de calor temos que Qix = 0 . Para
o processo isovolumétrico, temos que ∆Vxf = 0 .Usando a primeira lei da termo
dinâmica, temos que
dQxf = dExf + p dVxf
ou seja:
dQ xf = dE xf = µCV dT xf ∴ Q xf = µCV (Tf − T x )

d) Qual a variação da entropia ∆S para a trajetória II ?A variação de entropia para


a trajetória I é igual a ela?

dQI dW I pdV dV V 
dS I = = = = µR ⇒ ∆S i = µR ln f 
T T T V  Vi 

∆SII = ∆Six + ∆Sxf

Como o processo ix é adiabático, temos então que ∆Six = 0 e, portanto:

dQ xf dE xf dT xf T 
dS II = dS xf = = = µCV ⇒ ∆S II = µCV ln f 
T T T  Tx 
Por outro lado
2
Tf T V  3
= i =  f 
T x T x  Vi 
e, portanto
 2

 3R   Vf  3  V 
∆SII = µ   ln   = µR ln f 
 2   Vi    Vi 
 
onde encontramos que:
∆SI = ∆SII

e) Calcule Tx , QI , QII e ∆S para µ = 1 , T = 500K e Vf /Vi = 2 .

Cap 21 www.fisica.ufpb.br/~romero 20
www.cliqueapostilas.com.br

Prof. Romero Tavares da Silva

Capítulo 21 - Halliday, Resnick e Walker

19 Um mol de um gás ideal percorre o ciclo da figura à seguir.

a) Qual o trabalho realizado pelo gás para ir p


do estado a ao estado c ao longo da tra- c
jetória abc ? 2p0

Wabc = Wab + Wbc p0 a b

Como o processo bc é isovolumétrico, o V


trabalho para realizá-lo é nulo, e desse V0 4V0
modo:
Vb Vb

W abc = ∫ pdV = p ∫ dV = p (V
Va
a
Va
a b − Va ) = 3 p 0V0

b) Quais as variações de energia interna para ir de b para c e para percorrer um


ciclo completo?
dE = µCV dT ∴ ∆E bc = µCV (Tc − Tb ) = (µRTc − µRTb ) = (p cVc − p bVb )
3 3
2 2

∆E bc =
3
[(2p0 )(4V0 ) − (p0 )(4V0 )] = 3 (4 p0V0 ) = 6 p0V0 = 6µRT0
2 2

Como a energia interna é uma função de estado, a sua variação em um ciclo


completo é nula.

c) Quais as variações de entropia para ir de b para c e para percorrer um ciclo


completo?
Como o processo bc é isovolumétrico, o trabalho para realizá-lo é nulo, e desse
modo a primeira lei da termodinâmica toma a forma:

dQ bc = dE bc = µCV dTbc
Mas
dQ bc dTbc
dS bc = = µCV
T T
logo
T 
Tc
dT
∆S bc = µCV ∫
Tb
T
= µCV ln c
 Tb


No entanto
p c Vc
Tb µR p c 2 p 0 3
= = = =2 ⇒ ∆S bc = µR ln 2
Tc p bVb pb p0 2
µR

Como a entropia é uma função de estado, a sua variação em um ciclo completo é


nula.

Cap 21 www.fisica.ufpb.br/~romero 21
www.cliqueapostilas.com.br

Prof. Romero Tavares da Silva

Capítulo 21 - Halliday, Resnick e Walker

23 Um motor de Carnot opera entre 2350C e 1150C , absorvendo 6,3x104J por ciclo
na temperatura mais alta.

a) Calcule a eficiência do motor.

Ta = 2350C = 508K
Tb = 1150C = 388K
Qa = 6,3x104J

Tb
ε (%) = 23,62%
388
ε = 1− = 1− = 0,2362 ⇒
Ta 508

b) Quanto trabalho por ciclo este motor é capaz de realizar?

W
ε= ⇒ W = εQa = 1,48 x10 4 J
Qa

Capítulo 21 - Halliday, Resnick e Walker

27 Um mol de um gás ideal monoatômico percorre o ciclo reversível mostrado na figura


ao lado. O processo BC é uma ex-
3,0
pansão adiabática, com pB=10atm e
-3 3
VB = 10 m . p2,5
B B
2,0
Pre ssão

a) Determine a energia adicionada 1,5 Adiabático


ao gás sob a forma de calor. 1,0
pA A C
0,5
pB = 10atm = 1,013x105Pa
VB = 10-3m3 0,0
0 1 2 3 4 5 6 7 8 9
Volume(Vb)
Como a transformação BC é
adiabática:
γ 5
V  V  3
pB
γ
p BVB = pCVC γ
⇒ pC = p B  B  = p B  B  ∴ pC =
 VC   8VB  32

QT = QAB + QBC + QCA

Como o processo BC é adiabático, temos que QBC = 0 . Por outro lado, o pro-
cesso AB é isovolumétrico, de modo que o trabalho dWAB = 0 e, portanto a pri-
meira lei da termodinâmica toma a forma:

dQAB = dEAB + dWAB ⇒ dQAB = µCVdTAB ∴ QAB = µCV (TB - TA)

Q AB =
3
(µRTB − µRT A ) = 3 (pBVB − p AV A ) = 3  pBVB − pB VB 
2 2 2 32 

Cap 21 www.fisica.ufpb.br/~romero 22
www.cliqueapostilas.com.br

Prof. Romero Tavares da Silva

3 31
Q AB = . pBVB = 147,20J
2 32

dQCA = dECA + dWCA = µCVdT + pdV


ou seja:
QCA = µCV (TA - TC) + pA (VA - VC)

QCA =
3
(p AV A − pCVC ) + p A (V A − VC ) = 5 p A (V A − VC )
2 2

5 pB
QCA = . (VB − 8VB ) = − 5 . 7 pBVB = −55,39J
2 32 2 32

É fácil concluir que QAB é a energia adicionada ao gás sob a forma de calor.

b) Determine a energia que deixa o gás sob a forma de calor.

Por outro lado, também é fácil concluir que QCA é a energia que deixa o gás sob
a forma de calor.

c) Determine o trabalho resultante realizado pelo gás.

W = QAB + QCA = 147,20J - 55,39J = +91,81J

d) Determine a eficiência do ciclo.

W 91,81
ε= = = 0,6237
Q AB 147,20

Capítulo 21 - Halliday, Resnick e Walker

29 Um mol de um gás ideal monoatômico percorre o ciclo mostrado na figura ao lado.


Suponha que p = 2p0 ; V = 2V0 ; p0=1,01x105Pa
e V0 = 0,0225m3 .

a) Calcule o trabalho realizado durante o ciclo B C


V, p
Pressão

W abcda = ∫ pdV = (p − p )(V − V )


abcda
0 0

V0 , p0
W abcda = (2 p0 − p0 )(2V0 − Vo ) = p 0V0 A D

Wabcda = 2.272,50J
Volume

b) Calcule a energia adicionada sob a forma de calor durante o tempo ABC do mo-
tor.

Cap 21 www.fisica.ufpb.br/~romero 23
www.cliqueapostilas.com.br

Prof. Romero Tavares da Silva

Qabc = Qab + Qbc

∴ Qab = µCV (Tb − Ta ) = (µRTb − µRTa )


3
dQab = dE ab = µCV dT
2
= (p bVb − p aVa ) = [(2 p0 )(V0 ) − (p 0 )(V0 )] = p 0V0
3 3 3
Qab
2 2 2
Qab = 3.408,75J

dQ bc = dE bc + dW bc = µCV dT bc + pdVbc ∴ Q bc = µCV (Tc − Tb ) + p b (Vc − Vb )

Q bc = (pcVc − p bVb ) + p b (Vc − Vb ) = p b (Vc − Vb ) = (2 p 0 )[(2V0 ) − V0 ] = 5 p 0V0


3 5 5
2 2 2
Qbc = 11.362,50J
e, portanto
3 13
Qabc = p 0V0 + 5 p 0V0 = p 0V0
2 2
Qabc = 14,771,25J

c) Calcule a eficiência do ciclo.

W abcda p 0V0 2
ε = = = 0,1538
Qabc 13 13
p 0V0
2
ε(%) = 15,38%

d) Qual a eficiência de um motor de Carnot operando entre as temperaturas mais


alta e mais baixa que ocorrem no ciclo? Como essa eficiência se compara com a
calculada em (c) .
pV p V  pV p V 
Ta = a a =  0 0  Tb = b b = 2 0 0  = 2Ta
µR  µR  µR  µR 
p cVc p V  p d Vd p V 
Tc = = 4 0 0  = 4Ta Td = = 2 0 0  = 2Ta
µR  µR  µR  µR 

Ta T 3
εC = 1− = 1 − a = = 0,75
Tc 4Ta 4
εC(%) = 75%

Capítulo 21 - Halliday, Resnick e Walker

30 No primeiro estágio de um motor de Carnot de dois estágios, a energia Q1 é absor-


vida sob a forma de calor a uma temperatura T1 , o trabalho W1 é realizado e a
energia Q2 é expelida sob a forma de calor a uma temperatura T2 .O segundo está-
gio absorve essa energia Q2 , realiza o trabalho W2 e expele a energia Q3 a uma
temperatura ainda mais baixa T3 . Prove que a eficiência do motor de dois estágios é
(T1 - T3)/T1 .

Cap 21 www.fisica.ufpb.br/~romero 24
www.cliqueapostilas.com.br

Prof. Romero Tavares da Silva

absorve Q1 em T1 absorve Q 2 em T2
 
Estágio 1 produz W1 Estágio 2  produz W 2
 rejeita Q em T  rejeita Q em T
 2 2  3 3

Essa máquina interage com a vizinhança absorvendo Q1 numa temperatura T1 ,


rejeitando Q3 numa temperatura T3 , e produzindo um trabalho WT = W1 + W2 . Em
outras palavras:
WT = W1 + W2 = |Q1| - |Q3|

WT Q1 − Q3 Q3 T T − T3
ε= = = 1− = 1− 3 = 1
Q1 Q1 Q1 T1 T1

Capítulo 21 - Halliday, Resnick e Walker

32 Um mol de um gás ideal é usado como substância de trabalho de um motor que ope-
ra no ciclo mostrado na figura abaixo. BC e DA são processos adiabáticos reversí-
veis
p
a) O gás é monoatômico, diatômico ou
poliatômico? p0 A B

Como o processo BC é adiabático,


temos que:

p BVBγ = pCVCγ
γ p0/32 D C
p B  VC 
= 
pC  VB  V0 2V0 8V0 16V0 V
γ
p0  16V0 
=  ∴ 32 = 8 γ ⇒ ln 32 = γ ln 8
p0  2V0 
32
ln 32 5 ln 2 5
γ = = =
ln 8 3 ln 2 3
Para um gás monoatômico:
5R
CP 5
γ = = 2 =
CV 3R 3
2
e, portanto o gás utilizado é monoatômico.

b) Qual a eficiência do motor?


dQ = µ CV dT + pdV

Se a transformação entre os estados inicial e final acontece com a pressão cons-


tante, temos que:

Cap 21 www.fisica.ufpb.br/~romero 25
www.cliqueapostilas.com.br

Prof. Romero Tavares da Silva

dQ = µ CV dT + pdV
Se a transformação entre os estados inicial e final acontece com a pressão cons-
tante, temos que:
Qif = µCV (Tf - Ti) + pi(Vf - Vi)
Q if = (pf Vf − p i Vi ) + p i (Vf − Vi ) ⇒ Q if = p i (Vf − Vi )
3 5
2 2
Desse modo:
Q AB = p A (VB − V A ) = p 0 (2V0 − V0 ) = p 0V0
5 5 5
2 2 2
QBC = 0 , pois se trata de um processo adiabático
5 p
QCD = pC (VD − VC ) = . 0 (8V0 − 16V0 ) = − p 0V0
5 5
2 2 32 8
QDA = 0 , pois se trata de um processo adiabático
Usando a primeira lei da termodinâmica, temos que em um ciclo:
5   5  15
W = Q AB + QCD =  p 0V0  +  − p 0V0  = p 0V0
2   8  8
15
p0V0
Q AB + QCD W 8 3
ε= = = = = 0,75 ⇒ ε(%) = 75%
Q AB Q AB 5 4
p 0V0
2

Capítulo 21 - Halliday, Resnick e Walker

33 A operação de um motor a gasolina de combustão interna está representada pelo


ciclo na figura ao lado. Suponha que a p
mistura gasolina - ar de admissão é um 3p1 2
gás ideal e use a razão de compressão Centelha
4:1 (V4 = 4V1) . Suponha que p2 = 3p1 . Adiabático
p1 1 3
a) Determine a pressão e a temperatura Tomada de ar
em cada um dos pontos de vértice do Adiabático 4
diagrama p - V , em termos de p1 , T1,
e a razão γ entre os calores especí- V1 4V1 V
ficos molares do gás.
pV T 4 = 4 1−γ T1
T1 = 1 1
µR  V2 
γ
γ γ
p 2V2 = p 3V3 ∴ p 3 = p 2    
p 2V2 (3 p1 )(V1 ) p1V1  V3 
T2 = = =3 = 3T1 V 
γ
µR µR µR p 3 = (3 p1 ) 1  = 3.4 −γ p1
 4V1 
pV
T3 = 3 3 =
(
3.4 p1 (4V1 )
−γ
) pV 
= 3.4 1−γ  1 1  γ
µR µR  µR  γ γ  V1 
p1V1 = p 4V4 ∴ p 4 = p1  
T3 = 3.4 1−γ T1  V4 
γ
V 
p 4V 4 ( )
4 −γ p1 (4V1 ) pV  p 4 = (p1 ) 1  = 4 −γ p1
T4 = = = 4 1−γ  1 1   V1 
µR µR  µR 
Cap 21 www.fisica.ufpb.br/~romero 26
www.cliqueapostilas.com.br

Prof. Romero Tavares da Silva

b) Qual a eficiência do ciclo?

QT = Q12 + Q23 + Q34 + Q41

Como as transformações 23 e 41 são adiabáticas, temos que:

Q23 + Q41 = 0

As transformações 12 e 34 acontecem a volume constante, e quando usamos


a primeira lei da termodinâmica, temos que:

dQif = dEif = µCVdTif ⇒ Qif = µCV(Tf - Ti)


e desse modo:
Q12 = µCV (T2 − T1 ) = µCV [(3T1 ) − (T1 )] = 2µCV T1
e
[( ) ( )]
Q34 = µCV (T 4 − T3 ) = µCV 4 1−γ T1 − 3.4 1−γ T1 = 2.4 1−γ µCV T1
e finalmente
Q 34 2.4 1−γ µCV T1
ε = 1− = 1− = 1 − 4 1−γ
Q12 2µCV T1

Capítulo 21 - Halliday, Resnick e Walker

35 Um condicionador de ar de Carnot pega energia da energia térmica de uma sala a


700F e a transfere para um ambiente externo, que está a 960F . Para cada Joule de
energia elétrica necessária para operar o condicionador de ar, quantos Joules de ca-
lor serão removidos do quarto?

TF = 700F TF = 294,26K
TQ = 960F TQ = 308,70K

Para efetuar as transformações das escalas de temperatura, usamos que:

 9
 TF = 5 TC + 32

 ⇒ TK = 273,15 +
5
(TF − 32)
T = T − 273,15 9
 C K

O coeficiente de desempenho de um refrigerador de Carnot é definido como:

QF QF TF
κ= = =
W QQ − QF TQ − TF

κ = 20,37

E, portanto podemos dizer que para cada Joule de trabalho W fornecido pelo motor
elétrico serão retirados 20,37Joules de calor do quarto.

Cap 21 www.fisica.ufpb.br/~romero 27
www.cliqueapostilas.com.br

Prof. Romero Tavares da Silva

Capítulo 21 - Halliday, Resnick e Walker

37 Uma bomba térmica é usada para aquecer um edifício. A temperatura externa é de


-5,00C , e a temperatura dentro do edifício deve ser mantida a 220C . O coeficiente
de desempenho da bomba é de 3,8 , e a bomba térmica entrega 7,54MJ de calor
para o edifício a cada hora. Se a bomba térmica for um motor de Carnot trabalhando
no sentido inverso, a que taxa deve-se realizar trabalho para fazer funcionar a bomba
térmica?

TF = - 50C = 268,15K |QQ| / t = 7,5 x 106 Joules/hora


TQ = 220C = 295,15K κ = 3,8

QF QF κ
κ= = ⇒ QF = QQ
W Qq − QF 1+ κ
portanto
1 W 1 QQ
W = QQ − QF = QQ ∴ P= =
1+ κ t 1+ κ t

P = 1,56 x 106J/hora = 434,02 watts

Capítulo 21 - Halliday, Resnick e Walker

41 Um motor de Carnot trabalha entre as temperaturas T1 e T2 . Ele aciona um refrige-


rador de Carnot que trabalha entre as
temperaturas T3 e T4 . Determine a T1
razão Q3/Q1 em termos de T1 , T2 , T3 T3
e T4
A eficiência de uma máquina de Car- Q1
not é definida como: Q3
W QQ − QF W
ε= =
QQ QQ
Q2 Q4
O coeficiente de desempenho de um
refrigerador de Carnot é definido como:
Q QF TF T2 T4
κ= F = =
W QQ − QF TQ − TF
Considerando que: Q1 − Q2 T2
ε= = 1−
Q1 T1
1 – fonte quente
2 – fonte fria
3 – fonte quente T4 Q4
κ= =
4 – fonte fria Q3 − Q4 T3 − T4
Por outro lado, como a máquina e o refrigerador estão conectados, os trabalhos
envolvido em ambos os processos são iguais, ou seja:

W = |Q1| - |Q2| = |Q3| - |Q4|

Cap 21 www.fisica.ufpb.br/~romero 28
www.cliqueapostilas.com.br

Prof. Romero Tavares da Silva

Q4 κ
κ= ⇒ Q4 = Q3
Q3 − Q4 1+ κ
Logo
W Q4 Q κ Q3
εκ = = 4 ⇒ εκ =
Q1 W Q1 1 + κ Q1
ou seja:
Q3  1+ κ 
= (ε κ )  = ε (1 + κ )
Q1  κ 
ou ainda:
 T2 
1 − 
Q3  T2  T4   T2  T3   T1 
= 1 − 1 +  = 1 −  =
Q1  T1  T3 − T4   T1  T3 − T4   T4 
1 − 
 T3 

Capítulo 21 - Halliday, Resnick e Walker

44 Uma caixa contém N moléculas de gás igualmente divididas entre as suas duas me-
tades. Para N = 50 :
a) Qual a multiplicidade desta configuração central?

Se um sistema tem N componentes idênticos que podem ocupar duas situações


distintas, a multiplicidade de um estado com n componentes em uma situação e
os coponentes restantes N-n na outra situação é dada por

N!
CNn =
n! N!

Na situação específica do problema, temos que:

50!
25
C50 = ≅ 1,26 x1014
25! 25!

b) Qual o número total de microestados para o sistema?

O número de microestados de um sistema com N componentes que podem o-


cupar duas situações, é dado por 2N , nesse caso temos:

250 = 1,13x1015

c) Que percentagem de tempo o sistema gasta em sua configuração central?

Como todos os estados são igualmente prováveis, o sistema passará em tese o


mesmo tempo em cada um desses estados. No entanto os estados têm multipli-
cidade diferentes e desse modo o sistema passará um tempo τ(N,n) em um de-
terminado estado proporcional a probabilidade da ocorrência deste estado, ou
seja:

Cap 21 www.fisica.ufpb.br/~romero 29
www.cliqueapostilas.com.br

Prof. Romero Tavares da Silva

multiplicidade do estado CNn


τ (N , n ) = =
número total de estados 2N

25
C50
τ (50,25) = 50
= 0,111
2

Cap 21 www.fisica.ufpb.br/~romero 30
www.cliqueapostilas.com.br
www.cliqueapostilas.com.br

Exercícios
Recomendados
www.cliqueapostilas.com.br
www.cliqueapostilas.com.br

Física 2 - 2009
1a Lista de Exercícios

1. (Ex. 4 do Cap. 15 - Física 2 Resnick, Halliday e Krane - 5a Edição)


As arestas de um cubo maciço de cobre possuem 85,5 cm de comprimento. Qual é o valor da pressão que deve ser
aplicada ao cubo para que o comprimento das arestas seja reduzido para 85,0cm? O módulo de compressibilidade
do cobre é de 140 GPa.

2. (Ex. 7 do Cap. 15 - Física 2 Resnick, Halliday e Krane - 5a Edição)


Calcule a diferença de pressão hidrostática no sangue entre o cérebro e os pés de uma pessoa com 1,83 m de altura.

3. (Ex. 10 do Cap. 15 - Física 2 Resnick, Halliday e Krane - 5a Edição) 1,0 Corcovado


De acordo acordo com o modelo de temperatura constante da atmosfera
p (atm) Pico da Neblina
terrestre, (a) qual é a pressão (em atm) a uma altitude de 5,00 km; e (b) a
que altitude a pressão vale 0,500 atm? Compare suas respostas com o
0,5 Aconcágua, Argentina
preconizado na figura ao lado. Everest, Nepal
Atmosfera padrão
4. (Ex. 13 do Cap. 15 - Física 2 Resnick, Halliday e Krane - 5a Edição) p = p0e-h/a
Qual seria a altura da atmosfera se a massa específica do ar (a) fosse 0,0
constante; e (b) diminuísse linearmente até zero em função da altitude? 0 10 20 30
Altitude h (km)
Admita que a massa específica do nível do mar seja de 1,21 kg/m3.

5. (Ex. 17 do Cap. 15 - Física 2 Resnick, Halliday e Krane - 5a Edição)


A tração atuante no cabo que mantém um bloco maciço abaixo da superfície de um líquido (cuja
massa específica é maior que a do material do bloco) é To quando o recipiente está em repouso.
Mostre que a tração T, quando o recipiente está sujeito a uma aceleração vertical a para cima, pode
ser expressa por To(1+a/g).

6. (Ex. 23 do Cap. 15 - Física 2 Resnick, Halliday e Krane - 5a Edição)


Admita que a massa específica dos pesos de latão seja de 8,0 g/cm3. Qual é o erro percentual cometido ao se
desprezar a sustentação do ar quando se pesa um objeto com massa específica de 3,4 g/cm3 em uma balança de
pratos?

7. (Prob. 3 do Cap. 15 - Física 2 Resnick, Halliday e Krane - 5a Edição)


A água possui uma profundidade D atrás da face vertical a montante de
L
uma barragem, conforme mostrado na figura ao lado. Seja L a largura da
barragem. (a) Determine a força horizontal resultante exercidasobre a
barragem pela pressão manométrica da água; e (b) o momento
resultante devido à pressão manométrica exercida pela água, em relação D
a uma linha paralela à largura da barragem e que passa pelo ponto O.
(c) Onde se situa a linha de ação da força resultante equivalente? O

4,6cm2
8. (Prob. 4 do Cap. 15 - Física 2 Resnick, Halliday e Krane - 5a Edição)
1,8m Um barril cilíndrico possui um tubo esbelto fixado em sua superfície superior, conforme a figura ao
lado. O recipiente é cheio com água até o topo do tubo. Calcule a relação entre a força hidrostática
exercida sobre o fundo do barril e o peso da água nele contido. Por que esta relação não é igual a
um? (Despreze a ação da atmosfera)
1,8m
1,2m 

9. (Prob. 6 do Cap. 15 - Física 2 Resnick, Halliday e Krane - 5a Edição)


(a) Mostre que a massa específica ρ da água a uma profundidade y do oceano está relacionada com a massa
específica na superfície ρs por: ρ ≈ ρs [1+(ρsg/B)y] onde B = 2,2 GPa é o módulo de compressibilidade da água.
Despreze as variações na temperatura. (b) De quanto a massa específica a uma profundidade de 4200 m excede a
massa específica na superfície?
www.cliqueapostilas.com.br

10. (Prob. 9 do Cap. 15 - Física 2 Resnick, Halliday e Krane - 5a Edição)


(a) Considere a aceleração horizontal de uma massa de líquido em um reservatório aberto. Uma aceleração deste
tipo causa um abaixamento da superfície do líquido na parte frontal do reservatório e uma elevação na parte traseira.
Mostre que a superfície do líquido se inclina de um ângulo θ em relação à horizontal, onde tg θ = a/g, sendo a a
aceleraçãohorizontal. (b) Neste caso, como a pressão varia com h, a profundidade vertical abaixo da superfície?

ω
11. (Prob. 12 do Cap. 15 - Física 2 Resnick, Halliday e Krane - 5a Edição)
(a) Um fluido gira com velocidade angular constante ω em relação ao eixo vertical central de um
reservatório cilíndrico. Mostre que a variação da pressão na direção radial é expressa por
dp r
= ρω2r
dr y

(b) Faça p = pc no eixo de rotação (r = 0) e mostre que a pressão p em um ponto qualquer a uma
distância r vale
1
p = pc + ρω2r .
2

(c) Mostre que a superfície do líquido possui a forma parabolóide de revolução; isto é, uma seção transversal vertical
da superfície pode ser representada pela curva y = ω2r2/2g. (d) Mostre que a variação da pressão com a profundidade
é p = ρgh.

12. (Ex. 5 do Cap. 16 - Física 2 Resnick, Halliday e Krane - 5a Edição)


Um rio com 21m de largura e 4,3m de profundidade (média) drena uma região de 8500 km2 de área onde a
precipitação pluviométrica média é de 48 cm/ano. Um quarto desta água retorna à atmosfera por evaporação, mas o
restante permanece no rio. Qual é a velocidade média da água do rio?
Reservatório

Entrada
13. (Ex. 7 do Cap. 16 - Física 2 Resnick, Halliday e Krane - 5a Edição)
A entrada de água em uma represa possui uma área de seção reta de 7,60 ft2.
572 ft Prédio do A água escoa com velocidade de 1,33 ft/s. No prédio do gerador, que está 572
ft abaixo do ponto de entrada da água, esta flui a 31,0 ft/s. (a) Calcule a
diferença de pressão, em lb/in2, entre a entrada e a saída da água. (b) Qual é a
Saída
área da tubulação na saída? O peso específico da água é de 62,4 lb/ft3.

14. (Ex. 11 do Cap. 16 - Física 2 Resnick, Halliday e Krane - 5a Edição)


Em um furacão, o ar (massa específica de 1,2kg/m3) sopra sobre o telhado de uma casa a uma velocidade de 110
km/h. (a) Qual é a diferença de pressão entre o interior e o exterior da casa que tende a arrancar o telhado? (b) Qual
é o módulo da força de sustentação que seria aplicada a um telhado de 93m2?

15. (Ex. 14 do Cap. 16 - Física 2 Resnick, Halliday e Krane - 5a Edição)


A figura ao lado mostra a descarga de um líquido através de um orifício situado a uma distância
h abaixo da superfície do líquido contido em um tanque de grandes dimensões. O tanque é c
aberto na parte superior.
(a) Aplique a equação de Bernoulli à linha de corrente que liga os pontos 1, 2 e 3 e mostre que a
h
velocidade com que o líquido sai pelo orifício pode ser expressa por v = 2gh Este resultado é
v
conhecido como lei de Torricelli. d
(b) Se a saída do orifício apontasse diretamente para cima, qual seria a altura máxima atingida
pelo jato de líquido? e
(c) Como a viscosidade ou turbulência afetariam esta análise?


16. (Ex. 17 do Cap. 16 - Física 2 Resnick, Halliday e Krane - 5a Edição)
e  Considere um tubo em U, uniforme, com um diafragma em sua parte inferior, contendo um
líquido a diferentes alturas em cada um dos seus ramos. Imagine agora que o diafragma é
perfurado de modo que o líquido escoe da esquerda para a direita. (a) Mostre que a
d  aplicação da equação de Bernoulli aos pontos 1 e 3 leva a uma contradição. (b) Explique
porque a equação de Bernoulli não é aplicável a este problema. (Sugestão: O escoamento
neste caso é estacionário?)
Diafragma
www.cliqueapostilas.com.br

17. (Ex. 21 do Cap. 16 - Física 2 Resnick, Halliday e Krane - 5a Edição) vo


Um tubo oco possui um disco DD fixado a uma de suas extremidades. Quando o ar
com massa específica ρ é soprado através do tubo, o disco atrai o cartão CC. Seja A
a área do cartão e v a velocidade média do ar entre o cartão e o disco. Determine a
força resultante direcionada para cima que atua em CC. Despreze o peso do cartão e
admita que vo « v, onde vo é a velocidade do do ar no interior do tubo. (Nota: para isso
a distância entre o disco e o cartão tem de ser muito pequena.)

D D
v v
C C

18. (Ex. 23 do Cap. 16 - Física 2 Resnick, Halliday e Krane - 5a Edição)


O ar escoa sobre a parte superior da asa de um avião cuja área é A, com velocidade vs, e sob a parte inferior da asa
com velocidade vi. Mostre que a equação de Bernoulli prevê que a força de sustentação F orientada para cima sobre
a asa será
F = ½ρA(vs2 - vi2),
onde ρ é a massa específica do ar.

19. (Prob. 3 do Cap. 16 - Física 2 Resnick, Halliday e Krane - 5a Edição)


h Um tanque é cheio com água até uma altura H. À profundidade h abaixo do nível
da água, é feito um pequeno orifício em sua parede. (a) Mostre que a distância x
da base da parede até o local em que o jato atinge o solo pode ser expressa por
H x = 2[h(H-h)]1/2. (b) Poderia ser perfurado um orifício a uma outra profundidade de
modo que o segundo jato tivesse o mesmo alcance que o do item anterior? Em
caso afirmativo, qual seria esta profundidade? (c) A que profundidade deveria ser
x feito um orifício para que a água que por ele sai apresente o alcance máximo
relativamente à base? Qual deve ser este alcance máximo?

h1
20. (Prob. 4 do Cap. 16 - Física 2 Resnick, Halliday e Krane - 5a Edição)
Um sifão é um dispositivo utilizado para remover um líquido de um recipiente que não pode
ser tombado. Ele funciona conforme ilustra a figura a lado.Inicialmente, o tubo deve ser
d
cheio, porém tão logo isto tenha sido feito, o líquido passará a escoar até que seu nível
fique abaixo da abertura do tubo em A. O líquido possui uma massa específica ρ e A
viscosidade desprezível. (a) Com que velocidade o líquido sai do tubo em C? (b) Qual é a
pressão no líquido no ponto mais alto em B? (c) Qual é a maior altura h possível para a
qual um sifão pode fazer subir a água? h2

21. (Prob. 5 do Cap. 16 - Física 2 Resnick, Halliday e Krane - 5a Edição)


(a) Considere um fluido de massa específica ρ que escoa com
p1 velocidade v1 e passa abruptamente de uma tubulação cilíndrica com
p2
área de seção transversal a1, para outra tubulação cilíndrica mais larga,
v1 v 2 cuja área de seção transversal é a2. O jato de líquido que emerge da
a1 tubulação estreita mistura-se com o que se encontra na tubulação mais
larga, depois ele escoa quase uniformemente com velocidade média v2.
a2 Sem se preocupar com os detalhes de menor importânciarelacionados
à mistura, utilize o conceito de momento linear para mostrar que o
aumento de pressão devido à mistura é aproximadamente igual a
p2 - p1 = ρv2 (v1 - v2)
(b) Mostre, partindo da equação de Bernoulli, que em uma tubulação cuja seção transversal aumente gradativamente
esta diferença de pressão pode ser expressa por
p2 - p1 = ρ (v12 - v22)
(c) Determine a perda de pressão devido ao alargamento brusco da tubulação. Você seria capaz de fazer uma
analogia com os choques eleásticos e inelásticos entre partículas, estudado na mecânica?
www.cliqueapostilas.com.br

22. (Prob. 7 do Cap. 16 - Física 2 Resnick, Halliday e Krane - 5a Edição)


Considere o ar estagnado na borda frontal da asa de um avião e o ar fluindo na superfície superior desta asa a uma
velocidade v. Admita que a pressão na borda dianteira seja aproximadamente igual à pressão atmosférica e
determine o maior valor possível para v na linha de corrente do escoamento. Admita que o ar seja incompressível e
utilize a equação de Bernoulli. Considere que a massa específica do ar vale 1,2 kg/m3. Como esta velocidade pode
ser comparada com a velocidade do som (340 m/s) nessas condições? Você pode explicar esta diferença? Por que
deveria haver alguma relação entre essas grandezas?

p1 p2 23. (Prob. 9 do Cap. 16 - Física 2 Resnick, Halliday e Krane - 5a Edição)


A1 A2 Considere o medidor de Venturi, mostrado na figura ao lado, contendo água.
v1 v2 Seja A1 = 4,75.A2. Suponha que a pressão no ponto 1 seja 2,12 atm.

(a) Calcule os valores de v1 no ponto 1 e v2 no ponto 2 que fariam com que a pressão no ponto 2 se anulasse.
(b) Calcule a vazão correspondente considerando que o diâmetro no ponto 1 seja de 5,20 cm. O fenômeno que
ocorre no ponto 2 quando p2 cai aproximadamente a zero é conhecido como cavitação. A água vaporiza em
pequenas bolhas.

24. (Prob. 33E do Cap. 16 - Fundamentos de Física 2 Resnick, Halliday e Walker - 4a Edição)
Cerca de um terço do corpo de um estudante de Física, nadando no Mar Morto, ficará acima da superfície.
Considerando a densidade do corpo humano 0,98 g/cm3, encontre a densidade da água do Mar Morto. (Por que ela é
tão maior que 1,0 g/cm3?)

25. (Prob. 56P do Cap. 16 - Fundamentos de Física 2 Resnick, Halliday e Walker - 4a Edição)
A água é bombeada continuamente para fora de um porão inundado, a uma velocidade de 5,0 m/s, através de uma
mangueira uniforme de raio 1,0 cm. A mangueira passa por uma janela 3,0 m acima do nível da água. Qual é a
potência da bomba?

26. (Prob. 76P do Cap. 16 - Fundamentos de Física 2 Resnick, Halliday e Walker - 4a Edição)
Uma placa de 80 cm2 e 500 g de massa é presa por dobradiças em um de seus lados. Se houver ar soprando
apenas sobre a sua superfície superior, que velocidade deverá ter o ar para sustentar a placa na posição horizontal?
 
www.cliqueapostilas.com.br

Física 2 - 2009
2a Lista de Exercícios

1. (Ex. 5 do Cap. 17 - Física 2 Resnick, Halliday e Krane - 5a Edição)


Em um barbeador elétrico a lâmina se move para frente e para trás com um curso de 2,00 mm. O movimento é
harmônico simples, com frequência de 120 Hz.Determine (a) a amplitude, (b) a velocidade máxima da lâmina e (c) a
aceleração máxima da lâmina.

2. (Ex. 7 do Cap. 17 - Física 2 Resnick, Halliday e Krane - 5a Edição)


Um corpo oscila com movimento harmônico simples de acordo com a equação:
x = (6,12 m).cos[(8,38 rad/s)t + 1,92 rad]
Determine (a) o deslocamento, (b) a velocidade e (c) a aceleração no instante t = 1,90 s. Determine também (d) a
frequência e (e) o período do movimento.

3. (Ex. 16 do Cap. 17 - Física 2 Resnick, Halliday e Krane - 5a Edição)


Um tubo em U é preenchido com um líquido homogêneo. O líquido é temporariamente pressionado por um pistão em
um dos lados do yubo. O pistão é removido e o nível do líquido em cada um dos lados passa a oscilar. Mostre que o
período de oscilação deste movimento é π.[2L/g]1/2, onde L é o comprimento total do líquido no interior do tubo.

4. (Ex. 19 do Cap. 17 - Física 2 Resnick, Halliday e Krane - 5a Edição)


Um estilingue grande (hipotético) é distendido de 1,53 m a fim de lançar um projeto de 130 g com velocidade
suficiente para escapar da Terra (11,2 km/s - tremendamente hipotético). (a) Qual deve ser a constante elástica deste
dispositivo, supondo que toda a energia potencial seja convertida em energia cinética? (b) Admita que uma pessoa
normal possa exercer uma força de 220 N. Quantas pessoas seriam necessárias para distender este hipotético
estilingue?

5. (Ex. 25 do Cap. 17 - Física 2 Resnick, Halliday e Krane - 5a Edição)


Determine o comprimento de um pêndulo simples cujo período é de 1,00 s em um local onde g = 9,82 m/s2.

6. (Ex. 34 do Cap. 17 - Física 2 Resnick, Halliday e Krane - 5a Edição)


Um engenheiro deseja determinar o momento de inércia de um objeto de forma bizarra, com massa igual a 11,3 kg,
em torno de um eixo que passa pelo centro de massa. O objeto é pendurado por um fio que passa pelo centro de
massa e apóia-se sobre o eixo desejado. A constante elástica torcional do fio é κ = 0,513 N.m. O engenheiro observa
que o pêndulo executa 20,0 ciclos em 48,7 s. Qual é o valor do momento de inércia do objeto?

7. (Ex. 46 do Cap. 17 - Física 2 Resnick, Halliday e Krane - 5a Edição)


Um oscilador harmônico amortecido consiste em um bloco (m = 1,91 kg), uma certa mola (k = 12,6 N/m) e uma força
amortecedora F = -bvx. Inicialmente o bloco oscila com amplitude de 26,2 cm; por causa do amortecimento a
amplitude reduz-se para três quartos deste valor inicial, após quatro ciclos completos. (a) Qual é o valor de b? (b)
Qual é a quantidade de energia dissipada durante estes quatro ciclos?

8. (Ex. 47 do Cap. 17 - Física 2 Resnick, Halliday e Krane - 5a Edição)


Considere as oscilações forçadas de um sistema massa-mola amortecido. Mostre que na ressonância (a) a amplitude
das oscilações é xm = Fm/bω e (b) a velocidade máxima do bloco oscilante é vm = Fm/b.

9. (Ex. 52 do Cap. 17 - Física 2 Resnick, Halliday e Krane - 5a Edição)


F
(
Partindo da equação xm = m cos(ω' ' t − β ) , onde G = m 2 ω' ' 2 −ω 2 )
2 ⎛ bω' ' ⎞
+ b 2 ω' ' 2 e β = cos −1 ⎜ ⎟ , encontre a
G ⎝ G ⎠
velocidade vx (= dx/dt) do movimento oscilatório forçado. Mostre que a amplitude da velocidade é vm = Fm/[(mω'' -
k/ω'')2+b2]1/2.
www.cliqueapostilas.com.br

10. (Prob. 2 do Cap. 17 - Física 2 Resnick, Halliday e Krane - 5a


Edição)
A figura ao lado mostra uma astronauta em um dispositivo de
medida de massa corporal (BMMD "body mass measuring
device"). Projetado para uso em veículos espaciais em órbita, sua
finalidade é permitir que os astronautas possam medir suas
massas nas condições de ausência de "peso aparente" do fato de
estar em órbira. O BMMD consiste em uma cadeira dotada de
molas. O astronauta mede o período de suas oscilações na
cadeira; sua massa é obtida da expressão do período em um
sistema massa-mola oscilante. (a) Sendo M a massa do
astronauta e m a massa efetiva da parte do BMMD, que também
oscila, mostre que M = (k/4π2)T2 – m onde T é o período de
oscilação e k é a constante elástica. (b) A constante elástica do
BMMD é k = 605,6 N/m e o período de oscilação da cadeira vazia é 0,90149 s. Calcule a massa efetiva da cadeira.
(c) Com um astronauta na cadeira, o período de oscilação passa para 2,08832 s. Calcule a massa do astronauta.

11. (Prob. 3 do Cap. 17 - Física 2 Resnick, Halliday e Krane - 5a Edição)


Dois blocos (m = 1,22 kg e M = 8,73 kg) e uma determinada mola (k = 344 N/m) estão m
arranjados em uma superfície horizontal, sem atrito, conforme mostra a figura ao lado. k
O coeficiente de atrito estático entre os blocos é de 0,42. Determine a amplitude M
máxima possível do movimento harmônico simples para que não haja deslizamento
entre os blocos.

12. (Prob. 5 do Cap. 17 - Física 2 Resnick, Halliday e Krane - 5a Edição)


m Duas molas são fixadas a um bloco de massa m, que pode deslizar sem atrito
k1 k2 sobre uma superfície horizontal, conforme mostrado na figura ao lado. Mostre
1 k1 + k 2
que a frequência de oscilação do bloco é f == f12 + f 22 , onde f1 e
2π m
f2 são as frequências com as quais o bloco oscilaria se fosse conectado apenas à mola 1 ou à mola 2.

13. (Prob. 6 do Cap. 17 - Física 2 Resnick, Halliday e Krane - 5a Edição)


Duas molas são unidas e conectadas a um bloco de massa m, conforme a figura k11 k22
ao lado. As superfícies são lisas (sem atrito). se as molas separadamente m
possuem constantes elásticas k1 e k2, mostre que a frequência de oscilação do
bloco será
1 k1 ⋅ k 2 f1 ⋅ f 2
f = = .
2π (k 1 + k 2 )m f12 + f 22

(f1 e f2 são as frequências com as quais o bloco oscilaria se fosse conectado apenas à mola 1 ou à mola 2.)

14. (Prob. 7 do Cap. 17 - Física 2 Resnick, Halliday e Krane - 5a Edição)


Uma determinada mola de massa desprezível e constante elástica igual a 3,60 N/cm, é
partida em dois pedaços iguais. (a) Qual é a constante elástica de cada pedaço? (b) Os dois
pedaços, suspensos separadamente, suportam um bloco de massa M, como mostra a figura
ao lado. O sistema vibra com frequência de 2,87 Hz. Calcule o valor da massa M.

15. (Prob. 11 do Cap. 17 - Física 2 Resnick, Halliday e Krane - 5a Edição)


Um bloco de massa M, em repouso sobre uma mesa horizontal sem atrito, é fixado M
v k
a um suporte rígido através de uma mola cuja constante elástica é k. Um projétil de
massa m e velocidade v atinge o bloco, conforme mostra a figura ao lado; o projétil m
fica preso ao bloco. Determine a amplitude do movimento harmônico simples
resultante, em função de m, M, v e k.
www.cliqueapostilas.com.br

k 16. (Prob. 13 do Cap. 17 - Física 2 Resnick, Halliday e Krane - 5a Edição)


Um cilindro sólido está preso a uma mola horizontal sem massa, de tal modo que
M ele pode rolar sem deslizar sobre uma superfície horizontal, conforme mostrado
na figura ao lado. A constante elástica k da mola é de 2,94 N/cm. Sabendo-se
que o sistema foi abandonado do repouso na posição em que a mola está
distendida de 23,9 cm, calcule as energias cinética (a) de translação e (b) de rotação do cilindro, quando este passar
pela posição de equilíbrio. (c) Mostre que, nestas condições, o centro de massa do cilindro executa movimento
3M
harmônico simples com período de T = 2π onde M é a massa do cilindro.
2k

17. (Prob. 15 do Cap. 17 - Física 2 Resnick, Halliday e Krane - 5a Edição)


Um pêndulo físico consiste em um disco sólido uniforme de massa M = 563 g e raio R = 14,4 cm, Pivô
mantido no plano vertical por um eixo preso a uma distância d = 10,2 cm do centro do disco, d
conforme a figura ao lado. Desloca-se o disco de um pequeno ângulo e, em seguida, ele é
liberado. Encontre o período do movimento harmônico resultante. R

k 18. (Prob. 18 do Cap. 17 - Física 2 Resnick, Halliday e Krane - 5a Edição)


Uma roda pode girar em torno de seu eixo fixo. Uma determinada mola está
R
r presa a um de seus raios, a uma distância r do eixo. Supondo que a roda seja
um aro de raio R e massa M, obtenha a frequência angular para pequenas
M oscilações deste sistema em função de M, R, r e da constante elástica da
mola k. Analise os casos particulares em que r = R e r = 0.

19. (Prob. 21 do Cap. 17 - Física 2 Resnick, Halliday e Krane - 5a Edição)


Um disco de 2,50 kg e 42,0 cm de diâmetro está suspenso por uma barra fina de 76,0 cm
de comprimento, articulada no seu extremo, conforme mostrado na figura ao lado. (a)
Inicialmente a mola de torção leve está desconectada. Qual é o período de oscilação? (b) A
mola é então conectada de tal forma que, na condição de equilíbrio, a barra oscila em torno 76,0cm
da posição vertical. Qual deve ser a constante elástica de torção da mola, se o período
agora for 500 ms menor do que era inicialmente?

42,0cm
20. (Prob. 22 do Cap. 17 - Física 2 Resnick, Halliday e Krane - 5a Edição)
Um pêndulo simples de comprimento L e massa m está preso a um carro que se move com
velocidade constante v em uma trajetória circular de raio R. Qual será o período do
movimento sabendo-se que o pêndulo executa pequenas oscilações em torno da posição de equilíbrio?

21. (Prob. 25 do Cap. 17 - Física 2 Resnick, Halliday e Krane - 5a Edição)


Suponha que você esteja examinando as características do sistema de suspensão de um automóvel de 2000 kg. A
suspensão "cede" 10 cm quando todo o peso do automóvel é colocado sobre ela, e a amplitude da oscilação diminui
de 50% durante um ciclo completo. Determine os valores das constantes k e b da mola e do sistema absorvedor de
choque de cada roda. Suponha que cada roda suporte 500 kg.

R
L
m

v
R
www.cliqueapostilas.com.br

22. (Prob. 122 do Cap. 14 - Física 1 Tipler e Mosca - 5a Edição)


x
Túnel
Um túnel reto é escavado através da Terra, como mostra a figura ao lado. Suponha
m que as paredes do túnel sejam isentas de atrito. (a) A força gravitacional exercida
r pela Terra em uma partícula de massa m a uma distância r do centro da Terra,
quando r < RT, é Fr = -(GmMT/RT3)r, onde MT é a massa da Terra e RT o seu raio.
Mostre que a força resultante na partícula de massa m atuando a uma distância x do
meio do túnel é dada por Fx = = -(GmMT/RT3)x, e que o movimento da partícula é um
RT movimento harmônico simples. (b) Mostre que o período do movimento é dado por T
= 2π.[RT/g]1/2 e calcule seu valor em minutos. (Esse é o mesmo período de um
satélite orbitando próximo à superfície da Terra e é independente do comprimento do
túnel.)

Resposta:
(a) Sendo a força gravitacional igual Fr = -(GmMT/RT3)r, a sua componente segundo o eixo x será dada por:
Fx = Fr.senθ onde θ = arcsen(x/r)
∴ Fx = - GmMT/RT3)r .x/r = = -(GmMT/RT3)x ↔ Fx = -kx onde k = GmMT/RT3
(b) T = 2π/ω0
Como ω0 = [k/m]1/2 = [GMT/RT3]1/2 e g = GMT/RT2 → ω0 = [g/RT]1/2
Logo: T = 2π/ωo = 2π.[RT/g]1/2 = 2π. [6,4x106m/9,8m/s2]1/2 = 5078 s = 84 min

23. (Prob. 123 do Cap. 14 - Física 1 Tipler e Mosca - 5a Edição)


Um oscilador harmônico amortecido tem uma frequência ω' que é 10% menor do que uma frequência não
amortecida. (a) Qual é o fator de decréscimo da amplitude de oscilação em cada oscilação? (b) De que fator a
energia do sistema é amortecida durante cada oscilação?

Resposta:
b2 b2 b2 b
(a) ω’ = ω 02 − 2
= 0,90.ωo → ω 02 − 2
= 0,81 ω02 → 0,19 ω02 = 2
→ = 0,44 ω0
4m 4m 4m 2m
O período de uma oscilação é dado por:
2π 2π
T= =
ω' 0,90ω 0
Logo, como a amplitude cai na forma A(t) = A0.e-b/2m.t , temos que:

ΔA A (t ) − A (t + T )
b −0,44 ω0 ⋅
− T
0,90 ω0
= = 1 − e 2m = 1 − e = 0,95 = 95%
A A (t )

(b) A energia é proporcional à amplitude ao quadrado, ou seja, E(t) = E0.e-b/m.t . Logo:



ΔE E(t ) − E(t + T )
b−0,87 ω0 ⋅
− T
0,90 ω0
= = 1− e m = 1− e = 0,997 = 99,7%
E E(t )

24. (Prob. 125 do Cap. 14 - Física 1 Tipler e Mosca - 5a Edição)


Neste problema será determinada a expressão da potência média proporcionada por uma força de excitação a um
oscilador forçado.
(a) Mostre que a potência instantânea aplicada por uma força excitadora é dada por: P = Fv = -AωFo cosωt sen(ωt-δ)

(b) Use a identidade trigonométrica sen(θ1 - θ2) = senθ1 cosθ2 -cosθ1 senθ2 para mostrar que a equação obtida em (a)
pode ser escrita como P = AωFo senδ cos2ωt - AωFo cosδ cosωt senωt

(c) Mostre que o valor médio do segundo termo do resultado de (b), em um ou mais períodos, é zero e que, por
conseguinte, Pméd = ½. AωFo senδ

(d) A partir da equação tg δ = bω/[m(ωo2-ω2)] construa um triângulo retângulo no qual o lados oposto ao ângulo δ é bω
e o lado adjacente é [m(ωo2-ω2)], e use esse triângulo para mostrar que
bω bωA
senδ = =
( 2
)
m 2 ω o2 − ω 2 + b 2 ω 2
Fo
www.cliqueapostilas.com.br

(e) Use o resultado de (d) para eliminar ωA do resultado de (c), e então a potência média de entrada poderá ser
escrita como
2 ⎡ bω 2 Fo2 ⎤
1 Fo 1
Pméd = sen 2 δ = ⎢ ⎥
2 b
⎣ o (
2 ⎢ m 2 ω2 − ω2 2 + b 2 ω2 ⎥
⎦ )

25. Um oscilador harmônico tem fator de qualidade Q=10 (Q = ω/(b/m)). Partindo da posição de equilíbrio é-lhe
comunicada uma velocidade inicial de 5 m/s. Verifica-se que a energia total do oscilador diminui numa taxa, por
segundo, igual a duas vezes sua energia total instantânea. Calcule o deslocamento x(t) do oscilador (em metros) em
função do tempo (em segundos).

Resposta:
Q = 10, portanto se trata de um OHA. Então,
x(t) = A.e-b t/2m. sen(ωt +ϕ) onde
1/ 2
⎡ ⎛ bx 0 ⎞
2⎤
⎢ ⎜ + v0 ⎟ ⎥
x0ω
A = ⎢ x 02 + ⎜ 2m ⎟ ⎥ e tgϕ =
⎢ ⎜ ω ⎟ ⎥ bx 0
⎢ ⎜ ⎟ ⎥ + v0
⎣⎢ ⎝ ⎠ ⎦⎥ 2m

Do problema, temos que:


x0 = x(0) = 0 (parte da posição de equilíbrio)
v0 = v(0) = 5 m/s (parte da posição de equilíbrio)
dE b
Q = ω/(b/m) = 10 → ω = 20.b/m » b/m → ≈ − E T = -2.ET → b/m = 1s
-1
dt m
ω = Q.b/m =20 rad/s
A ≈ v0/ω = 5m/s / 20 s−1 = 0,25 m e tgϕ = 0 x(t) = (0,25 m).e-t sen(20s-1 t)

26. Um oscilador harmônico não amortecido de massa m e freqüência própria ω0 move-se sob ação de uma força
externa F=F0.sen(ωt), partindo da posição de equilíbrio com velocidade inicial nula. Determine o deslocamento x(t)
que a massa m realiza em função do tempo.

Resposta:
Temos um OH sob ação de uma força externa harmônica. Então,
x1(t) = A1.e-b/2m.t sen(ω1t +ϕ) onde ω1 = [ω02 – b2/4m2]1/2
F0 m ωb m
x2(t) = A2.sen(ωt -β) onde A2 = e tgβ = 2
2
(
ω 02 − ω 2 + ω 2 b 2 m ) ω0 − ω2

Do problema, temos que:


F0 m
b = 0 (não amortecido) → tgβ = 0, A2 = e ω1 = ω0
ω 02 − ω 2
x0 = x(0) = 0 m (parte da posição de equilíbrio) → x(0) = x1(0) + x2(0) = A1.sen(ϕ) = 0 → ϕ = 0
v0 = v(0) = 0 m/s (velocidade inicial nula)→ v(0) = ω1A1 cosϕ + ωA2 = 0 → A1 = -ω/ω1 A2
F m ⎛ ω ⎞
∴ x(t) = 2 0 2 . ⎜⎜ sen(ωt ) − sen(ω 0 t )⎟⎟
ω0 − ω ⎝ ω0 ⎠

 
www.cliqueapostilas.com.br

Física 2 – 2009
3a Lista de Exercícios

1. (Ex. 3 do Cap. 18 - Física 2 Resnick, Halliday e Krane - 5a Edição)


Uma onda senoidal viaja em uma corda. O tempo para que um ponto em particular movimente-se da posição de
deslocamento máximo para a deslocamento zero é de 178ms. O comprimento de onda é de 1,38m. Encontre (a) o
período , (b) a frequência e (c) a velocidade da onda.

2. (Ex. 12 do Cap. 18 - Física 2 Resnick, Halliday e Krane - 5a Edição)


Na figura à esquerda, a corda 1 tem massa específica linear de 3,31 g/m e a corda 2 tem massa específica linear de
4,87 g/m. Ambas estão submetidas ao peso da massa M = 551 g pendurado. (a) Qual é a velocidade de onda em
cada corda? (b) O bloco é subdividido, agora, em dois outros (com M1 + M2 = M) e a montagem é reorganizada como
indica a figura à direita. Quais são os valores de M1 e M2 para que as velocidades de onda nas duas cordas sejam
iguais?

corda 1 corda 2 corda 1 corda 2

M2

M1
M

3. (Ex. 14 do Cap. 18 - Física 2 Resnick, Halliday e Krane - 5a Edição)


Em um sistema esférico simétrico, a equação tridimensional da onda é dado por

 
(a) Demonstre que

 
é solução para essa equação de onda. (b) Qual é a dimensão da constante A?

4. (Ex. 18 do Cap. 18 - Física 2 Resnick, Halliday e Krane - 5a Edição)


(a) Mostre que a intensidade I é o produto da energia específica u (energia por unidade de volume) e a velocidade v
de propagação de uma onda; isto é, que I = uv. (b) Calcule a energia específica em uma onda sonora distante 4,82
km de uma sirene de 47,5W, admitindo que as ondas sejam esféricas, que a propagação seja isotrópica em absorção
atmosférica e que a velocidade do som seja 343 m/s.

6,0cmcm
5. (Ex. 22 do Cap. 18 - Física 2 Resnick, Halliday e Krane - 5a Edição)
Dois pulsos viajam em uma corda em sentidos opostos conforme a figura ao v
lado. (a) Se a velocidade de onda é de 2,0 m/s e os pulsos estão afastados de
6,0 cm, qual é o esboço dos padrões para eles depois de 5,0, 10, 15, 20 e
25ms? (b) O que aconteceu com a energia no instante t = 15ms?
-v

6. (Ex. 29 do Cap. 18 - Física 2 Resnick, Halliday e Krane - 5a Edição)


As vibrações de um diapasão de 622Hz determinam ondas estacionárias em uma corda presa em ambas as
extremidades. A velocidade de onda na corda é de 388 m/s. A onda estacionária tem quatro laços e amplitude de
1,90 mm. (a) Qual é o comprimento de onda? (b) Escreva uma equação para os deslocamento da corda como uma
função da posição e do tempo.
www.cliqueapostilas.com.br

7. (Ex. 34 do Cap. 18 - Física 2 Resnick, Halliday e Krane - 5a Edição)


Uma corda de 75,6 cm é tracionada entre dois suportes fixos. Observa-se que apresenta frequências ressonantes de
420 Hz e 315 Hz, e nenhuma outra entre elas. (a) Qual é a frequência ressonante mais baix para essa corda? (b)
Qual é a velocidade de onda para essa corda.

8. (Prob. 2 do Cap. 18 - Física 2 Resnick, Halliday e Krane - 5a Edição)


Que equação descreve uma onda que se desloca no sentido negativo ao longo do eixo x, com amplitude de 1,12 cm,
frequência de 548 Hz e de velocidade de 326 m/s?

9. (Prob. 11 do Cap. 18 - Física 2 Resnick, Halliday e Krane - 5a


S1 Edição)
r1 P Os violinos no tempo de Handel eram construídos para tocar um
lá em 422,5Hz. (Como se pode saber disso?) As orquestras
modernas, no entanto, estão afinadas para tocar o lá em 440Hz.
Admitindo que todas as outras coisas são iguais, de quanto um
músico precisa aumentar, percentualmente, a força de tração
r2 nas cordas para afinar um violino do tempo de Handel para tocar
nos dias de hoje?

S2

10. (Prob. 13 do Cap. 18 - Física 2 Resnick, Halliday e Krane - 5a Edição)


Considere duas fontes pontuais S1 e S2 na figura ao lado, as quais emitem ondas de mesma frequência f e amplitude
A. As ondas começam na mesma fase, e esta relação entre as fases das fontes é mantida longo do tempo.
Considere um ponto P no qual r1 é muito próximo de r2. (a) Mostre que a superposição dessas duas ondas gera uma
onda cuja amplitude ym varia com a posição P aproximadamente com

onde r = (r1 + r2)/2. (b) Em seguida, mostre que uma anulação total ocorre quando r1 - r2 = (n + ½)λ, com n sendo
qualquer número inteiro, e que haverá um reforço integral quando r1 - r2 = nλ. O lugar geométrico dos pontos cuja
distância a dois pontos fixos é constante é uma hipérbole, sendo os pontos fixos chamados focos. Assim, cada valor
de n corresponde a uma hipérbole de interferência construtiva e uma hipérbole de interferência destrutiva. Nos
pontos em que r1 e r2 não são aproximadamente iguais (como próximos às fontes), as amplitudes de S1 e S2 diferem
e a anulação é apenas parcial. (Isto é a base do sistema de navegação OMEGA.) 

11. (Prob. 20 do Cap. 18 - Física 2 Resnick, Halliday e Krane - 5a Edição)


A interferência pode ocorrer para ondas com frequências diferentes. (a) Mostre que a resultante de duas ondas
y1(x,t) = ym sen (k1x - ω1t) e y2(x,t) = ym sen (k2x - ω2t) pode ser escrita como

(b) O que é ω'/k´? (c) Como se pode descrever, qualitativamente o movimento dessa onda?

12. (Prob. 22 do Cap. 18 - Física 2 Resnick, Halliday e Krane - 5a Edição)


Um fio de alumínio de comprimento L1 = 60,0cm e área transversal 1,00x10-2cm2 é conectado a um fio de aço com
mesma área de seção transversal. O fio composto, carregado com um bloco de massa m = 10,0kg é disposto
conforme indicado na figura abaixo a fim de que a distância L2 da junção até a polia de suporte seja de 86,6cm.
Ondas transversais são induzidas no fio usando-se uma fonte externa de frequência variável. (a) Qual é a menor
frequência de excitação em que ocorrem ondas estacionárias e em que a junção dos fios é um nó na corda? (b) Qual
é o número total de nós observado nessa frequência, excluindo-se os nós nas duas extremidades do fio? A massa
específica do alumínio é de 2,60 g/cm3 e a do aço é de 7,80 g/cm3.
L1 L2

Alumínio Aço
m
www.cliqueapostilas.com.br

1) (Exercício 1-Capítulo 19)-Uma onda senoidal contínua é enviada através de uma


mola helicoidal por uma fonte vibrante atrelada a ela. A freqüência da fonte é de
25Hz, e a distância entre as rarefações sucessivas na mola é de 24cm. (a)
Determine a velocidade da onda. (b) Se o deslocamento longitudinal máximo de
uma partícula na mola é de 0,30cm e a onda se move no sentido negativo do
eixo x, escreva a equação da onda. Admita que a fonte está em x=0 e o
deslocamento s=0 na fonte quando t=0.
2) (Exercício 2-Capítulo 19)-A figura 19-19 mostra uma imagem notavelmente
detalhada de um transistor em um circuito microeletrônico, formada por um
microscópio acústico. As ondas sonoras têm freqüência de 4,2GHz. A
velocidade dessas ondas no hélio líquido em que está imerso o dispositivo, é de
240m/s. (a) Qual é o comprimento de onda dessas ondas acústicas de freqüência
ultra-elevadas? (b) Os condutores em forma de tira da figura têm largura de
2pm. A quantos comprimento de ondas corresponde essa dimensão?

3) (Exercício 10-Capítulo 19)-Mostre que a intensidade de onda sonora I pode ser


escrita em termos da freqüência f e da amplitude de deslocamento sm na forma
I = 2" 2 !vf 2 s 2m .
4) (Exercício 13-Capítulo 19)- Uma onda sonora de intensidade 1,60 µW / m 2 passa
através de uma superfície com área 4,7cm 2 . Quanta energia passa através de
uma hora?
5) (Exercício 17-Capítulo 19)-Determine a densidade de energia de uma onda
sonora a 4,82 km de distância de uma sirene de emergência de 5,2kW,
admitindo que as ondas são esféricas e que a propagação é isotrópica, sem
absorção pela atmosfera. Considere a velocidade do som igual a 343m/s.
6) (Exercício 25-Capítulo 19)-Uma fonte sonora esférica está localizada em P1
próxima a uma parede refletora AB e um microfone está localizado no ponto P2,
como mostrado na Fig.19-23. A freqüência da fonte sonora é variável.
Determine as duas menores freqüências para as quais a intensidade sonora ,
como observado no ponto 2, será um máximo. Não há mudança de fase na
reflexão; o ângulo de incidência é igual ao ângulo de reflexão. Dica: A onda
www.cliqueapostilas.com.br

sonora refletida é igual aquela emitida pela sua imagem formada pela parede que
funciona como um “espelho”.
7) (Exercício 33-Capítulo 19)- determine a velocidade das ondas em uma corda de
violino com 22,0cm , 820mg e a freqüência fundamental de 920Hz. (b) Calcule
a tração na corda.
8) (Problema 4-Capítulo 19)-(a) Se duas ondas sonoras, uma no ar e outra na água,
são iguais em intensidade, qual a razão entre a amplitude de pressão da segunda
em relação à primeira? (b) Se em vez disso, as amplitudes de pressão forem
iguais, qual será a razão entre as intensidades das ondas? Admita que a
temperatura da água é de 20 o C .
9) (Problema 11-Capítulo 19)-Na Fig 19-28, uma vara R é presa por seu centro;
um disco D em sua extremidade projeta-se para dentro de um tubo de vidro,
com o interior preenchido com raspas e cortiça. Um êmbolo P é colocado na
outra extremidade do tubo. Faz-se a vara vibrar longitudinalmente e o êmbolo é
deslocado até que sejam formados nós e antinós na cortiça (as raspas ficam
nitidamente enrugadas nos anitnós de pressão). Se a freqüência de vibração
longitudinal da var é conhecida, uma medição da distância média de entre dois
antinós sucessivos determinará a velocidade do som v no gás no interior do tubo.
Mostre que
v = 2fd.
Esse é o método de Kundt para a determinação da velocidade do som em vários
gases.

10) (Problema 16-Capítulo 19)- Dois diapasões idênticos oscilam a 442Hz. Uma
pessoa é colocada em algum lugar no caminho entre os dois. Calcule a
freqüência de batimento percebida por esse indivíduo se (a) a pessoa permanece
imóvel e os diapasões são deslocados para a direita a 31,3m/s, e (b) os diapasões
permanecem estacionários e o ouvinte move-se para a direita a 31,3 m/s.
11) (Problema 17-Capítulo 19)- Um avião desloca-se a 396m/s em uma latitude
constante. O choque sonoro alcança um observador no solo 12,0s após o
sobrevôo. Determine a altitude do avião. Admitia que a velocidade do som é de
330m/s. Resposta correta: 7164m/s.
12) (Problema 19-Capítulo 19)-Dois submarinos estão em curso de colisão proa-
proa durante uma manobra no Atlântico Norte. O primeiro submarino move-se a
20,2km/h e o segundo , a 94,6 km/h. O primeiro envia um sinal de sonar (ondas
sonoras na água) a 1030Hz. As ondas do sonar propagam-se a 5470km/h. (a) O
segundo submarino capta o sinal. Qual a freqüência captada pelo segundo
sonar?. (b) O primeiro submarino capta o sinal refletido. Qual freqüência será
detectada por ele? Veja a Fig. 19-30. O oceano está calmo. Admita que não
existem correntes.
www.cliqueapostilas.com.br

Física 2 - 2009
5a Lista de Exercícios

1. (Cap. 21 - Física 2 Resnick, Halliday e Krane - 5a Edição)


Usando a figura 21-11, determine o volume de um kg de água a 20 oC.

2. (Ex. 7, Cap. 21 - Física 2 Resnick, Halliday e Krane - 5a Edição)


Um termômetro de resistência é um termômetro no qual a resistência elétrica varia com a temperatura. É possível
definir as temperaturas medidas por este termômetro em kelvins (K) como sendo diretamente proporcionais à
resistência R, medida em ohms (Ω). Um certo termômetro de resistência apresenta uma resistência R de 90,35Ω
quando o seu bulbo é colocado em água à temperatura do ponto tríplice (273,16 K). Que temperatura o termômetro
indica se o bulbo for colocado em um ambiente no qual a sua resistência elétrica é de 96,28Ω?

3. (Ex. 12, Cap. 21 - Física 2 Resnick, Halliday e Krane - 5a Edição)


Um mastro de alumínio de uma bandeira possui 33 m de altura. De quanto aumenta o seu comprimento quando a
temperatura sobe 15oC? (α = 23x10-6 oC-1)

4. (Ex. 15, Cap. 21 - Física 2 Resnick, Halliday e Krane - 5a Edição)


Trilhos de trem são instalados quando a temperatura é igual a -5oC. Uma seção do trilho tem 12,0 m de comprimento.
Que folga deve ser deixada entre os trilhos de modo a não ocorrer compressão quando a temperatura subir até
42oC?

5. (Ex. 29, Cap. 21 - Física 2 Resnick, Halliday e Krane - 5a Edição)


Um frasco de vidro a 100oC está completamente cheio com 891 g de mercúrio. Qual é a massa de mercúrio
necessária para que o frasco fique cheio a -35oC? (O coeficiente de dilatação linear do vidro é 9,0x10-6/oC; o
coeficiente de dilatação volumétrica do mercúrio é 1,8x10-4/oC.)

6. (Ex. 38, Cap. 21 - Física 2 Resnick, Halliday e Krane - 5a Edição)


O melhor vácuo que pode ser obtido em laboratório corresponde a uma pressão de aproximadamente 10-18atm, ou
1,01x10-13Pa. Quantas moléculas existem por centímetro cúbico nesta pressão a 22oC?

7. (Ex. 43, Cap. 21 - Física 2 Resnick, Halliday e Krane - 5a Edição)


Uma bolha de ar de 19,4cm3 de volumeestá no fundo de um lago com 41,5 m de profundidade, onde a temperatura é
de 3,80oC. A bolha sobe até a superfície, que está à temperatura de 22,6oC. Considere que a temperatura do ar na
bolha é a mesma da água em sua volta e determine o seu volume do instante imediatamente anterior à chegada da
bolha à superfície.

8. (Prob. 1, Cap. 21 - Física 2 Resnick, Halliday e Krane - 5a Edição)


Observa-se que os objetos quentes e frios esfriam ou aquecem até a temperatura das suas vizinhanças. Se a
diferença de temperatura ΔT entre um objeto e sua vizinhança (ΔT = Tobj - Tviz) não for muito grande, a taxa de
aquecimento do objeto é aproximadamente proporcional a essa diferença de temperatura, isto é: dΔT/dt = -A(ΔT)
onde A é uma constante. O sinal negativo está presente porque ΔT diminui com o tempo se ΔT for positivo e aumenta
se ΔT for negativo. Isto é conhecido como a lei de resfriamento de Newton.
(a) A constante A depende de que fatores? Quais são as suas dimensões?
(b) Se em um determinado instante t = 0 a diferença de temperatura é ΔT0, mostre que, para um instante posterior t,
ela é: ΔT = ΔTo e-At

9. (Prob. 13, Cap. 21 - Física 2 Resnick, Halliday e Krane - 5a Edição)


Um cubo de alumínio de 20cm de aresta flutua em mercúrio. De quanto mais o cubo afunda quando a temperatura
aumenta de 270 para 320K? (O coeficiente de dilatação volumétrica do mercúrio é 1,8x10-4/oC.)

10. (Prob. 19, Cap. 21 - Física 2 Resnick, Halliday e Krane - 5a Edição)


A variação da pressão na atmosfera da Terra, supondo que esteja a uma temperatura constante, é dada por
p = po.e-Mgy/RT, onde M é a massa molar do ar. Mostre que o número de moléculas por unidade de volume é dado
por: nv = nv0 e-Mgy/RT.
www.cliqueapostilas.com.br

11. (Prob. 21, Cap. 21 - Física 2 Resnick, Halliday e Krane - 5a Edição)


Um manômetro de mercúrio com dois braços de mesma seção transversal é selado de modo a ter-se a mesma
pressão p nos dois braços, conforme é mostrado na figura ao lado. Mantendo a temperatura constante, adiciona-se
10,0cm3 de mercúrio através da torneira na parte de baixo. O nível da esquerda aumenta de 6,00 cm e o da direita
aumenta de 4,00 cm. Determine a pressão p.

 
www.cliqueapostilas.com.br

Física 2 - 2009
6a Lista de Exercícios

1. (Ex.5, Cap. 23 - Física 2 Resnick, Halliday e Krane - 5a Edição)


k1 k2 Quatro peças quadradas de isolantes de dois materiais diferentes, todas
k1 k1 com a mesma espessura e área A, estão disponíveis para cobrir uma área
aberta de 2A. Isto pode ser feito através de uma das duas formas
mostradas na figura ao lado. Se k2 ≠ k1, qual montagem, (1a) ou (1b),
fornece o menor fluxo de calor?
k2 k2 k2 k1

(1a) (1b)

2. (Ex. 8, Cap. 23 - Física 2 Resnick, Halliday e Krane - 5a Edição) 0oC 100oC


Duas hastes de metal idênticas são soldadas extremidade com
extremidade, conforme mostrado na figura (2a) ao lado, e 10J de (2a)
calor fluem através das hastes em 2,0min. Quanto tempo levaria
para 30J fluírem através das hastes se elas estivessem soldadas
0oC 100oC
conforme mostrado na Figura (2B)?
(2b)

p a f 3. (Ex. 11, Cap. 23 - Física 2 Resnick, Halliday e Krane - 5a Edição)


Quando um sistema é levado de um estado i para um estado f ao longo de um caminho iaf
indicado na figura (3), observa-se que Q = 50J e Wviz= -20J. Ao longo do caminho ibf, Q =
36J. (a) Qual é o valor de Wviz ao longo do caminho ibf? (b) Se Wviz=+13J para o caminho
i b curvo fi, qual é o valor de W para este caminho? (c) Considere Eint,i = 10J. Qual é o valor
de Eint,f? Se Eint,b = 22J, determine Q para os processos e if.
V

4. (Ex. 28, Cap. 23 - Física 2 Resnick, Halliday e Krane - 5a Edição)


(a) Um litro de gás com γ = 1,32 está a 273K e 1,00 atm de pressão. Ele é repentinamente (adiabaticamente)
comprimido até a metade do seu volume original. Determine a pressão e temperaturas finais. (b) Em seguida, o gás é
resfriado de volta até 273K a pressão constante. Determine o volume final. (c) Determine o trabalho total realizado
sobre o gás.

5. (Ex. 33, Cap. 23 - Física 2 Resnick, Halliday e Krane - 5a Edição)


Um gás ideal experimenta uma compressão adiabática de p = 122kPa, V = 10,7 m3, T = -23oC para p = 1450kPa, V =
1,36 m3. (a) Calcule o valor de γ. (b) Determine a temperatura final. (c) Quantos moles de gás estão presentes? (d)
Qual é a energia cinética de translação total por mol, antes e após a compressão? (e) Calcule a razão entre as
velocidades rms antes e depois da compressão.

6. (Ex. 39, Cap. 23 - Física 2 Resnick, Halliday e Krane - 5a Edição)


Um recipiente contém uma mistura de três gases inertes: n1 moles do primeiro gás com calor específico molar a
volume constante C1, e assim por diante. Determine o calor específico molar a volume constante da mistura, em
termos dos calores específicos molares e quantidades dos três gases separados.

7. (Ex. 47, Cap. 23 - Física 2 Resnick, Halliday e Krane - 5a Edição)


Suponha que 29,0 J de calor sejam adicionados a um determinado gás ideal. Como resultado, o seu volume varia de
63,0 para 113 cm3 enquanto a pressão permanece constante em 1,00 atm. (a) De quanto a energia interna do gás
varia? (b) Se a quantidade de gás presente é 2,00x10-3mol, determine a capacidade térmica molar a pressão
constante. (c) Determine a capacidade térmica molar a volume constante.
www.cliqueapostilas.com.br

8. (Prob. 3, Cap. 23 - Física 2 Resnick, Halliday e Krane - 5a Edição)


Supondo que k é constante, mostre que a taxa radial de fluxo de calor em uma substância entre duas esferas
concêntricas é dada por
(T − T )4πkr1r2
H= 1 2
r2 − r1

onde a esfera interna tem um raio r1 e temperatura T1, e a esfera externa tem um raio r2 e temperatura T2.

9. (Prob. 5, Cap. 23 - Física 2 Resnick, Halliday e Krane - 5a Edição)


A baixas temperaturas (abaixo de 50K), a condutividade térmica de um metal é proporcional à temperatura absoluta;
isto é, k = aT, onde a é uma constante com um valor numérico que depende do material. Mostre que a taxa de fluxo
de calor através de uma haste de comprimento L e seção transversal A, cujas extremidades estão às temperaturas T1
e T2, é dada por
H=
aA 2
2L
(
T1 − T22 )

10. (Prob. 6, Cap. 23 - Física 2 Resnick, Halliday e Krane - 5a Edição)


Um recipiente de água foi colocado no exterior exposto ao frio até formar uma camada de
5,0cm de espessura de gelo na sua superfície. O ar acima do gelo está a -10oC. Calcule a
taxa de formação do gelo (em centímetros por hora) sobre a superfície inferior da cama de
gelo. Considere a condutividade térmica e a massa específica do gelo como sendo,
respectivamente, 1,7 W/m.K e 0,92 g/cm3. Suponha que nenhum calor flua das paredes do
recipiente.

11. (Prob. 7, Cap. 23 - Física 2 Resnick, Halliday e Krane - 5a Edição)


Uma pessoa faz uma porção de chá gelado misturando 520 g de chá quente (essencialmente água) a uma massa
igual de gelo a 0oC. Quais são a temperatura final e a massa de gelo remanescente, se o chá estiver, inicialmente, a
uma temperatura de (a) 90,0oC e (b) 70,0oC?

cMP(J/mol.K)
  12. (Prob. 11, Cap. 23 - Física 2 Resnick, Halliday e Krane - 5a
Edição)
25 Da figura ao lado estime a quantidade de calor necessária para
elevar a temperatura de 0,45 mol de carbono de 200 para 500 K.
20 (Sugestão: Aproxime a curva real por um segmento de reta.)

15 Carbono

10

0
0 100 200 300 400 500
T(K)

13. (Prob. 18, Cap. 23 - Física 2 Resnick, Halliday e Krane - 5a Edição)

14. (Prob. 19, Cap. 23 - Física 2 Resnick, Halliday e Krane - 5a Edição)

 
www.cliqueapostilas.com.br

Física 2 - 2009
7a Lista de Exercícios

1. (Ex.2, Cap. 22 - Física 2 Resnick, Halliday e Krane - 5a Edição)


(a) Determine o número de moléculas em 1,00 m3 de ar a 20,0oC sob pressão de 1,00 atm. (b) Qual é a massa desse
volume de ar? Suponha que 75% das moléculas sejam de nitrogênio (N2) e 25% de oxigênio (O2).

2. (Ex. 5, Cap. 22 - Física 2 Resnick, Halliday e Krane - 5a Edição)


Considere uma amostra de gás argônio a 35,0oC e sob pressão de a,22 atm. Suponha que o raios de um átomo
(esférico) de argônio seja de 0,710x10-10m. Calcule a fração do volume do recipiente que é realmente ocupada pelos
átomos.

3. (Ex. 7, Cap. 22 - Física 2 Resnick, Halliday e Krane - 5a Edição)


A 44,0oC e 1,23x10-2atm a massa específica de um gás é de 1,32x10-5g/cm2. (a) Determine a velocidade vrms para as
moléculas do gás. (b) Determine a massa molar do gás e identifique-o.

4. (Ex. 11, Cap. 22 - Física 2 Resnick, Halliday e Krane - 5a Edição)


A que frequência o comprimento de onda do som é da mesma ordem de grandeza da trajetória livre média no
nitrogênio à pressão de 1,02 atm e temperatura de 18,0oC? Considere o diâmetro da molécula de nitrogênio como
sendo 315pM.

5. (Ex. 15, Cap. 22 - Física 2 Resnick, Halliday e Krane - 5a Edição)


(a) Dez partículas se movem com a seguinte distribuição de velocidades: quatro a 200m/s, duas a 500m/s e quatro a
600m/s. Calcule as velocidades médias e média quadrática. É verdade que vrms>vméd? (b) Construa uma outra
distribuição de velocidades para 10 partículas e mostre que vrms ≥ vméd para a distribuição que você imaginou. (c) Sob
que condições vrms será igual a vméd?

6. (Ex. 29, Cap. 22 - Física 2 Resnick, Halliday e Krane - 5a Edição)


Calcule a fração das partículas de um gás que se movem com energia de translação entre 0,01kT e 0,03kT.
Sugestão: Para E<<kT o termo e-E/kt pode ser substituído por 1-E/kT. Por quê?)

7. (Prob. 1, Cap. 22 - Física 2 Resnick, Halliday e Krane - 5a Edição)


Na temperatura de 0oC e pressão de 1000 atm as massas específicas do ar, do oxigênio e do nitrogênio valem,
respectivamente, 1,293kg/m3, 1,429kg/m3 e 1,250kg/m3. Calcule a percentagem, em massa, de nitrogênio no ar, a
partir desses dados, supondo que apenas esses dois gases estejam presentes.

8. (Prob. 7, Cap. 22 - Física 2 Resnick, Halliday e Krane - 5a Edição)


Dois recipientes estão à mesma temperatura. O primeiro contém gás à pressão p1, cujas moléculas têm massa m1
sendo vrms,1 a sua velocidade média quadrática. O segundo recipiente contém moléculas de massa m2 à pressão
igual a 2p1, sendo sua velocidade média vméd,2 = 2vrms,1. Calcule a razão m1/m2 entre as massas de suas moléculas.

9. (Prob. 12, Cap. 22 - Física 2 Resnick, Halliday e Krane - 5a Edição)


O Sol pode ser considerado como uma enorme bola de gás ideal aquecido. A incandescência
que o envolve, na imagem ultravioleta vista na figura ao lado, é a coroa - atmosfera do Sol.
Sua temperatura e sua pressão valem 2,0x106K e 0,030Pa. Calcule a velocidade média
quadrática dos elétrons na coroa.

10. (Prob. 13, Cap. 22 - Física 2 Resnick, Halliday e Krane - 5a Edição)


Um determinado gás, à temperatura T e ocupando um volume V, é constituído de uma mistura de átomos, a saber,
Na átomos de massa ma, cada um tendo uma velocidade média quadrática va, e Nb átomos de massa mb cada um
tendo uma velocidade média quadrática vb. (a) Obtenha uma expressão para a pressão total exercida pelo gás. (b)
Suponha agora que Na = Nb e que os átomos diferentes se combinem a volume constante para formar moléculas de
massa ma+mb. Assim que a temperatura voltar ao seu valor original, qual será a razão entre as pressões depois e
antes da combinação?
www.cliqueapostilas.com.br

Provas
Anteriores
www.cliqueapostilas.com.br
www.cliqueapostilas.com.br

INSTITUTO DE FÍSICA
Universidade Federal do Rio de Janeiro
Física II-A - manhã
1° período de 2009
1a Prova – 29/04/2009
1a Questão (2,5 pontos)
Um cubo de aresta a flutua dentro de um recipiente contendo mercúrio (densidade ρm),
tendo um quarto de seu volume submerso.
(a) (1,0) Se o cubo for homogêneo, qual seria sua densidade em termos da densidade
do mercúrio?
(b) (0,5) Acrescentamos água no recipiente (densidade ρa) até o cubo inteiro estar
submerso com sua face superior abaixo do nível da água. Qual é o empuxo sofrido
pelo cubo em termos da variáveis a, ρa, ρm e a aceleração da gravidade g?
(c) (1,0) Após o acréscimo da água, qual fração do volume do cubo que estará imersa
(dentro) no mercúrio?
Resposta:
(a) (1,0) ρmgV/4 = ρcubogV , donde
ρcubo = ρm/4
(b) (0,5) Como o cubo inteiro está submerso, o empuxo sofrido
pelo cubo é igual a seu peso :
E = ρcubogV = ρmga3/4
(c) (1,0) Seja α a fração dentro do mercúrio, temos αρm + (1 − α)ρa = ρm/4, donde

α=
2a Questão (2,5 pontos)
Uma mola de comprimento relaxado de L = 10,0 cm tem
constante elástica k = 240N/m. Ela é cortada em dois
pedaços: o primeiro de comprimento L1 = 6,0cm e o
segundo de L2 = 4,0cm. As duas molas assim obtidas são
amarradas sem deformação entre duas paredes e nos
lados opostos de um bloco de massa M que pode deslizar, sem atrito, em cima de
uma mesa horizontal ao longo do eixo Ox. Veja a figura ao lado.
(a) (1,0) As constantes elásticas, de cada uma das molas obtidas, são k1 = 5/3 k e k2 =
5/2 k. Justifique estes valores.
(b) (1,0) Para uma massa M = 100 g qual é a frequência angular de oscilação do
bloco?
(c) (0,5) Uma força externa periódica é aplicada na direção do eixo Ox com a
frequência fext = 50,0Hz. Qual deve ser o valor da massa M para que o bloco oscile
com amplitude máxima?
Resposta:
(a) (1,0) Usando o fato da constante elástica efetiva de duas molas em série ser
dada por:

verificamos que k1 = k×5/3 = 400,0 N/m e k2=k×5/2 = 600,0 N/m satisfazem esta
relação.
Um argumento mais completo consiste na consideração seguinte:
Ao dividir a mola em N = 5 partes iguais, cada parte terá uma constante N k = 5 k.
Juntando n pedaços em série, obtemos uma mola com constante elástica N k/n. No
caso, N = 5 e n = 2, 3.
www.cliqueapostilas.com.br

(b) (1,0) A constante da mola efetiva é:


Kef = k1 + k2 = 1000 N/m
A frequência angular de oscilação do bloco é

ω= = 100rad/s
(c) (0,5) A condição de ressonância fornece ωext = 2πfext = 100πrad/s
M = Kef/ωext2 = 1000N/m/(100π rad/s)2 = 10,1 g
3a Questão (2,5 ponto)
Uma das extremidades de uma corda de 20 cm é presa a uma
parede. A outra extremidade está ligada a um anel sem massa
que pode deslizar livremente ao longo de uma haste vertical
sem atrito, conforme a figura apresentada ao lado.
(a) (1,5) Quais são os três maiores comprimentos de ondas estacionárias possíveis
nesta corda?
(b) (1,0) Esboce as ondas estacionárias correspondentes?
Resposta:
(a) (1,5) λ1 = 4L = 80,0 cm
λ2 = 4L/3 = 26, 6 cm
λ3 = 4L/5 = 16, 0 cm
(b) (1,0)
www.cliqueapostilas.com.br

4a Questão (2,5 ponto)


Uma fonte sonora de freqüência de 100 Hz se desloca a uma velocidade de v = 36 km/
h em uma via retilínea.
(a) (1,0) Qual é a freqüência que um observador, parado na via, percebe enquanto a
fonte sonora se afasta?
(b) (1,0) A via termina em um grande muro perpendicular a ela. O som da fonte é
refletido neste muro e retorna ao observador. Qual é a frequência do som refletido no
muro que retorna ao observador?

(c) (0,5) O observador percebe, então, na superposição do som vindo diretamente da


fonte com o refletido pelo muro um batimento. Qual é a frequência do batimento?
Resposta:
Conforme a figura, a fonte se afasta do observador parado na direção do muro.
Sendo vsom = 340m/s a velocidade do som no ar parado.
A velocidade da fonte é vfonte = 36,0 km/h = 10,0m/s

(a) (1,0) fo =

= = 97Hz
(b) (1,0) A frequência do som refletido pelo muro é :

fmuro =

= = 103Hz
(c) (0,5) fbat = (103 − 97)Hz = 6 Hz
__________________________________
Formulário
vsom = 340 m/s, g=10m/s2, ρágua= 1,0x103kg/m3, sen(a) + sen(b) = 2.sen [(a+b)/2].cos[(a-b)/2]
P+½ρv2+ρgy = const., v = [F/µ]1/2, v = [B/ρ0]1/2, f' = (v±vO)/(vvS).f, ∆pm = v.ρ.ω.sm, I = ½ρvω2sm2, Pméd=
A(∆pm)2/(2ρv), ∆p(t) = [2∆pmcos[(ω1-ω2)t/2].sen[(ω1+ω2)t/2], NIS = 10 log(I/Io),
www.cliqueapostilas.com.br

IN S T I T U T O DE FÍSI C A

Universidade Federal do Rio de Janeiro


Física II-A - tarde
1° período de 2009
1a Prova – 29/04/2009

1a Questão (2,0 pontos)

Uma casca esférica oca, feita de ferro, flutua quase completamente submersa na
água, conforme mostrado na figura ao lado. O diâmetro externo da esfera é de
60,0 cm e a massa específica do ferro é de 7,9 g/cm3. Determine o volume da parte
oca da esfera.

Resposta:

Em termos do diâmetro D da esfera, o seu volume é dado por:

3
4π  D  π 3
  D
V= 3  2  =6

O empuxo, i.e. o peso do volume de água deslocado, que é o volume da esfera, é igual
ao peso da esfera;

ρag(π/6)D3 = ρFeg(π/6) (D3 − d3)

onde d é o diâmetro da parte oca da esfera. Logo:

ρ Fe − ρ a π 3
× D
ρ Fe 6
Voca = (π/6) d3 = ≈ 99x103cm3

2a Questão (3,0 pontos)

A figura mostra um bloco de massa M, em cima de uma mesa horizontal m


sem atrito, preso a uma mola cuja outra extremidade é fixada a uma
k
parede. Ao ser puxado e posteriormente, no instante t = 0 s, ser solto o
M
bloco oscila harmonicamente entre as posições x1 = 0,8m e x2 = 1,2m.
Algum tempo depois, um segundo bloco de massa m = 1,5 kg é colocado x
sobre o primeiro, quando da passagem deste por um de seus pontos de x x
retorno. Sabendo que o coeficiente de atrito estático entre os dois blocos é μ= 0,40,
que o sistema com apenas um bloco oscila 50 vezes em 1,0 min e o sistema composto
pelos dois blocos oscila 40 vezes neste mesmo intervalo de tempo, determine:

a) (0,5+0,5) a massa M do primeiro bloco, e a constante elástica k da mola;


www.cliqueapostilas.com.br

b) (0,2+0,2+0,2+0,2) a função x(t), com os valores todas as constante determinadas


(posição de equilíbrio xeq, amplitude A, frequência angular ω e fase φ), que dá a
posição do primeiro bloco com o passar do tempo, antes da colocação do segundo
bloco;

c) (0,4+0,4) as velocidades máximas de oscilação v(1)max e v(2)max do sistema com o


primeiro bloco e com os dois blocos;

d) (0,4) a amplitude máxima Amax que poderia ter o movimento harmônico descrito pelo
sistema com os dois blocos sem que o bloco de cima viesse a escorregar.

Resposta:

(a) (0,5+0,5) Temos duas relações 2πf = [k/M]1/2 e 2πf' = [k/(m +M)]1/2,

onde f = 5/6 s−1 , f' = 4/6 s−1 , m = 1,5 kg.

Achamos k = Mω2 = (M+m)ω'2 → M(ω2-ω'2) = mω'2 ↔ M(f2-f'2) = mf'2

f2 − f'2 16
2
m × 1,5kg
∴ M = f' = 9 = 8/3 kg ≈ 2,7 kg

k = Mω2 = M.4π2f2 = 8/3kgx4π2x(5/6Hz)2 = 73,1 N/m

(b) (0,2+0,2+0,2+0,2)

x(t) = xeq + A cos(ωt) = xeq + A sin(ωt + π/2)

xeq = (x1 + x2)/2 = 1,0 m; A = (x2 − x1)/2 = 0,2 m

ω = 2πf = 5,2 s−1; ϕ = π/2

(c) (0,4+0,4)

v1(max) = Aω = 1,0 m/s; v2(max) = Aω' = v1(1)×4/5 = 0,8 m/s


www.cliqueapostilas.com.br

(d) (0,4) As equações de Newton para os dois blocos são :

ma = −Fat, M d2x/dt2 = −k(x − xeq) + Fat

Os dois blocos se movem juntos se d2x/dt2 = a.

Eliminando Fat, temos

(M + m) d2x/dt2 = −k(x − xeq),

com solução x(t) = xeq + A cos(ω't).

A força de atrito é :

kA cos( ω 't )
m
Fat = m + M

A condição |Fat| ≤ μmg resulta em:

m+ M µg
µg 2
(0,6) A ≤ k = ω ' ≈ 0, 22 m

3a Questão (2,5 pontos)

Uma corda de violino de 30,0 cm de comprimento com densidade linear de massa de


0,650 g/cm é colocada próxima de um auto-falante que está conectado a um oscilador
de áudio de frequência variável. Descobre-se que a corda oscila somente nas
frequências de 880 Hz e 1320 Hz, quando a frequência do oscilador varia entre 500 Hz
e 1500 Hz. Determine:

(a) (1,0) a velocidade das ondas na corda;

(b) (0,5) a frequência do fundamental

(c) (1,0) a tensão na corda.


www.cliqueapostilas.com.br

Resposta:

Os comprimentos de onda da corda são dados por λn = 2L/n, com n = 1, 2, 3,...

e as frequências por:

v
n
fn = v/λn = 2L

onde v é a velocidade de propagação da onda.

A corda está em ressonância com as frequências sequenciais de:

880Hz = nv/(2L) e

1320Hz = (n+1)v/(2L)

Logo:

v/(2L) = 440Hz e

(a) (1,0) v = 264m/s

(b) (0,5) A frequência do modo fundamental corresponde a n = 1:

f1 = v/(2L) = 440 Hz

(c) (1,0) Finalmente, a tensão é obtida como:

T = μv2 = 65×10−3kg/m × (264m/s)2 = 4530N


www.cliqueapostilas.com.br

4a Questão (2,5 pontos)

Dois alto-falantes estão localizados a 20,0m e a 22,0m, respectivamente, de um


ouvinte em um auditório. Um gerador de áudio coloca os dois alto-falantes em fase
com as mesmas freqüências e com as amplitudes iguais na posição do ouvinte. As
freqüências podem ser ajustadas dentro do intervalo audível de 20 Hz a 20 kHz.

(a) (1,5) Quais são as três mais baixas freqüências para as quais o ouvinte irá ouvir
um sinal de mínimo, devido à interferência destrutiva?

(b) (1,0) Quais são as três mais baixas freqüências para as quais o ouvite ouvirá um
sinal máximo?

Resposta:

y(x,t) = y1(x,t) + y2(x,t)

= A sin(kx1 − ωt) + A sin(kx2 − ωt)

= 2A sin(k(x1 + x2)/2 − ωt) × cos(k∆x/2)

com ∆x = x2 − x1 = 2,0 m.

(a) (1,5)

Teremos um sinal mínimo se k∆x/2 = (n+1/2)π, ou

vsom
( n + 1 2)
fn = ∆ x

∴ f0 = 85 Hz, f1 = 255 Hz, f2 = 425 Hz

(b) (1,0)

Teremos um sinal máximo se k∆x/2 = nπ, ou

vsom
n
f'n = ∆ x

f'1 = 170Hz, f'2 = 340Hz, f'3 = 510Hz

Formulário
vsom = 340 m/s, g=10m/s2, ρágua= 1,0x103kg/m3, sen(a) + sen(b) = 2.sen [(a+b)/2].cos[(a-b)/2]

P+½ρv2+ρgy = const., v = [F/µ]1/2, v = [B/ρ0]1/2, f' = (v±vO)/(vmvS).f, ∆pm = v.ρ.ω.sm, I = ½ρvω2sm2, Pméd=
A(∆pm)2/(2ρv), ∆p(t) = [2∆pmcos[(ω1-ω2)t/2].sen[(ω1+ω2)t/2], NIS = 10 log(I/Io),
www.cliqueapostilas.com.br

IN S T I T U T O DE FÍSI C A

Universidade Federal do Rio de Janeiro


Física II – Turmas do horário de 10 h às 12 h
1° período de 2009
2a Prova
1a Questão (2,5 pontos)

Nu m a máqui n a té r m i c a , o age n te exte r n o é um P ressã


gás idea l di a t ô m i c o que e xe c u t a o cic l o da fi g u r a B
abai x o , onde BC é uma adi a b á t i c a e CA é uma
isote r m a . Assu m a que du r a n t e to d o o cic l o ,
ape n a s gr a u s de li be r d a d e de tr a n s l a ç ã o e de
rot aç ã o são ex c i t a d o s no gás. p0 A

(a) (0,6) Ex p r i m a os vol u m e s , te m p e r a t u r a s e


pre s s õ e s no s po n t o s B e C em ter m o s do s C
res p e c t i v o s val o r e s V 0, T0, P 0 no po n t o A e da raz ã o de V0 rV 0
Volum
co m p r e s s ã o r.

(b) (1,4) D et e r m i n e e m ca d a eta p a do cicl o (A → B), (B → C) e (C → A) a vari a ç ã o da


en e r g i a inter n a , o cal o r ab s o r v i d o e o tra b a l h o feito pel o sist e m a . Fa ç a u m a tab e l a de
se u s res u l t a d o s .

(c) (0,5) C al c u l e a efi ci ê n c i a da m á q u i n a . M o s t r e qu e el a só de p e n d e da raz ã o de


co m p r e s s ã o r.

Resposta:

Como o gás é diatômico: γ = CP/CV = 7/5 = 1,4

(a) Ponto A:

pA = p0, TA = T0, VA = V0

Ponto C:

TC = TA ∴ TC = T0 (isoterma)

pCVC = pAVA = p0V0

VC = rV0 →pC = p0V0/rV0 ∴ pC = p0/r

Ponto B:

VB = VA ∴ VB = V0
www.cliqueapostilas.com.br

pBVBγ = pCVCγ = p0/r.(rV0)γ

pB =p0 rγ-1 → pB = r2/5.p0

e TB/TA = pBVB/pAVA → TB = r(γ-1).T0 ∴ TB = r2/5.T0


Ponto Volume Temperatura Pressão

A V0 T0 p0

B V0 r2/5.T0 r2/5.p0

C rV0 T0 p0/r

(b) A→B (volume constante)

WAB = 0

∆EintAB = nCV∆T = p0V0/RT0.5/2R.(r2/5.T0 - T0) = 5/2.p0V0.(r2/5-1)

QAB = WAB + ∆EintAB = 5/2.p0V0.(r2/5-1)

B→C (adiabática)

QBC = 0

∆EintBC=nCV∆T=n.5/2.R.(TC-TB)=p0V0/T0.5/2.(T0–r2/5T0) =5/2.p0V0.(1- r2/5) [Obs : ∆EintBC= -


∆EintAB]

WBC = QBC - ∆EintBC = 0 → WBC = - ∆EintBC = 5/2 p0V0.(r2/5-1)

C→A (isoterma)

∴ ∆EintCA = 0

WCA = ∫CA p.dV = ∫CA nRT0.dV/V = p0V0 ∫CAdV/V = p0V0.ln(VA/VC) ∴ WCA = -p0V0.lnr
QCA = WCA + ∆EintCA ∴ QCA = -p0V0.lnr
Etapa Q Wsist ∆Eint

A→B 5/2.p0V0.(r2/5-1) 0 5/2.p0V0.(r2/5-1)

B→C 0 5/2 p0V0.(r2/5-1) 5/2.p0V0.(1-r2/5)

C→A -p0V0.lnr -p0V0.lnr 0

(c) ε = W/QQ = [0 + 5/2.p0V0.(r2/5-1) - p0V0.lnr]/[5/2.p0V0.(r2/5-1)] =

= [5/2.(r2/5-1) - lnr]/[5/2.(r2/5-1)] = 1 - [lnr / (5/2.(r2/5-1))]

2a Questão (2,5 pontos)

U m cor p o de ca p a c i d a d e cal o r í fi c a C = 50 J/K na te m p e r a t u r a T1 = 450 K est á po s t o


e m co n t a t o co m u m res e r v a t ó r i o de te m p e r a t u r a a T2 = 300 K. Ju n t o s ele s for m a m u m
universo ter m o d i n â m i c o . A o ati n g i r o eq ui l í b r i o ;
www.cliqueapostilas.com.br

(a) (0,5) cal c u l e Q 1 , o cal o r ab s o r v i d o pel o cor p o , e Q 2 , o cal o r ab s o r v i d o pel o


res e r v a t ó r i o .

(b) (1,0) c alc u l e a vari a ç ã o da entr o p i a ∆S 1 do cor p o e ∆S 2 do res e r v a t ó r i o .

(c) (1,0) q ual é a vari a ç ã o da en e r g i a inter n a ∆E int, e da entr o p i a ∆S , de s s e uni v e r s o


ter m o d i n â m i c o ?

Resposta:

(a) Q1 = C.(T2 – T1) = 50J/K.(300K-450K) = -7,5 kJ

Q2 = -Q1 = +7,5 kJ

(b) ∆S1 = ∫T1T2dQ/T = ∫T1T2C.dT/T = C.ln(T2/T1) = C.ln(300/450) = C.ln(2/3) = C.[ln2-ln3] =

≈ 50J/K.[0,69 - 1,1] = -20,5 J/K

∆S2 = ∆Q/T2 = Q2/T2 = 7,5kJ/300K = 25 J/K

(c) Como não há trabalho, ∆Eint = Q, logo ∆EintUNIV = Q1 + Q2 = 0

∆SUNIV = ∆S1 + ∆S2 ≈ -20,5 J/K + 25 J/K = 4,5 J/K

3a Questão (2,5 pontos)

U m reci pi e n t e A co n t é m u m gá s ide al a u m a pr e s s ã o de 5,0 × 1 0 5 Pa e a u m a


te m p e r a t u r a de 300 K. El e est á co n e c t a d o atr a v é s de u m tub o fin o ao reci pi e n t e B qu e
te m qu a t r o ve z e s o vol u m e de A. B co n t é m o m e s m o gá s ide a l a u m a pr e s s ã o de
1,0 × 1 0 5 Pa e a u m a te m p e r a t u r a de 400 K. A vál v u l a de co n e x ã o , feit a de u m m a t e r i a l
de co n d u t i v i d a d e tér m i c a de s p r e z í v e l , é ab e r t a e o eq ui l í b r i o é ati n g i d o a u m a pr e s s ã o
co m u m en q u a n t o a te m p e r a t u r a de ca d a res e r v a t ó r i o é m a n t i d a co n s t a n t e no se u val o r
inici al.

(a) (1,0) C al c u l e a raz ã o entr e os nú m e r o s de m ol e s no s reci p i e n t e s A e B ant e s (n A /


n B ) e de p o i s (nA '/n B ') da ab e r t u r a da vál v u l a .

(b) (1,5) D e t e r m i n e a pr e s s ã o fin al do sist e m a .

Resposta:

(a) pA = 5,0x105Pa, pA’ = pF, TA = TA’ = 300K, VA =VA’ = V0

pB = 1,0x105Pa, pB’ = pF, TB = TB’ = 400K, VB =VB’ = 4V0

nA = pAVA/RTA e nB = pBVB/RTB → nA/ nB = pAVATB/(pBVBTA) = 5x 1/4 x 4/3 =


5/3

nA’ = pA’VA’/RTA’ e nB’ = pB’VB’/RTB’ → nA’/nB’ = pFVATB/(pFVBTA) = 1/4 x 4/3 = 1/3

(b) nA - nA’ = pAVA/RTA - pA’VA’/RTA’ = (pA – pF) x V0/RTA

nB - nB’ = pBVB/RTB - pB’VB’/RTB’ = (pB – pF) x 4V0/RTB


www.cliqueapostilas.com.br

Como ∆nA + ∆nB = 0 → (pA – pF) x V0/RTA + (pB – pF) x 4V0/RTB = 0

→ (pA/TA + 4pB/TB).V0/R = pF.(1/TA + 4/TB).V0/R


∴ pF = (pA/TA + 4pB/TB)/(1/TA + 4/TB) = (5/300 + 4x1/400)x105Pa/K / (1/300K +
4x1/400K) =

= 2,0x105Pa

4a Questão (2,5 pontos)

U m a m á q u i n a tér m i c a M op e r a entr e res e r v a t ó r i o s de te m p e r a t u r a s TA = 400 K e TB =


300 K, co m u m ren d i m e n t o r de 20 % . Po r cicl o é utiliz a d a u m a qu a n t i d a d e de cal o r Q A
= 1 0 0 J da font e qu e n t e .

(a) (0,6) C al c u l e W, o tra b a l h o feit o pel a m á q u i n a por cicl o, e Q B , o cal o r ce d i d o ao


res e r v a t ó r i o frio por cicl o.

(b) (0,6) Pa r a u m a m á q u i n a de C a r n o t M(Carnot), op e r a n d o co m os m e s m o s


res e r v a t ó r i o s , e us a n d o a m e s m a qu a n t i d a d e de cal o r por cicl o Q A = 1 0 0 J da font e
qu e n t e , cal c u l e W (Carnot), o tra b a l h o feit o por cicl o, e Q B(Carnot), o cal o r ce di d o , por cicl o, ao
res e r v a t ó r i o frio.

(c) (0,8) C al c u l e a vari a ç ã o de entr o p i a , por cicl o, ∆S , da m á q u i n a M e ta m b é m


∆S (Carnot) da m á q u i n a M(Carnot) de C a r n o t .

(d) (0,5) C al c u l e a raz ã o R entr e o trabalho perdido (W (Carnot) −W) e a vari a ç ã o de


entr o p i a , por cicl o, ∆S , e ou sej a, R = (W (Carnot) −W)/∆S .

Resposta:

(a) Máquina M:

r = W/QA → W = r.QA = 0,20x100J = 20J

QB = QA – W = 80J

(b) Máquina M(Carnot):

rCarnot = 1 – TB/TA = 1 – ¾ = 25%

→ WCarnot = rCarnot.QA = 0,25x100J = 25J

QB = QA – W = 75J

(c) Máquina M:
∆S = -QA/TA + QB/TB = = -100J/400K + 80J/300K = 0,0167J/K

∆SCarnot = -QA/TA + QB/TB = -100J/400K + 75J/300K = 0 (reversível)

(d) R = (25J-20J)/0,0167J/K = 300K = TB


www.cliqueapostilas.com.br

Formulário
ln2 = 0,69, ln3 = 1,1, ln5 = 1,6, 1atm = 1,0x105Pa, 1l = 10-3m3, R = 8,3 J/mol.K

pV = nRT, pVγ = const, CV = ½.f R, CP = CV + R, r = W/QQ, K = QF/W

IN S T I T U T O DE FÍSI C A

Universidade Federal do Rio de Janeiro


Física II - Turmas no horário de 15h às 17h
1° período de 2009 P ressã
2a Prova
p0 A B
1a Questão (2,5 pontos)

Nu m a máqui n a té r m i c a , o age n te exte r n o é um


gás idea l di a t ô m i c o que e xe c u t a o cic l o da fi g u r a
abai x o , onde BC é uma adi a b á t i c a e CA é uma
isote r m a . Assu m a que du r a n t e to d o o cic l o ,
ape n a s gr a u s de l i b e r d a d e de t r a n s l a ç ã o e de C
(a) (0,6) Ex p r i m a os vol u m e s , te m p e r a t u r a s e
V0 rV 0
pre s s õ e s no s po n t o s B e C em ter m o s do s Volum
res p e c t i v o s val o r e s V 0, T0, P 0 no po n t o A e da raz ã o de
(b) (1,4) D et e r m i n e e m ca d a eta p a do cicl o (A → B), (B → C) e (C → A) a vari a ç ã o da
en e r g i a inter n a , o cal o r ab s o r v i d o e o tra b a l h o feito pel o sist e m a . Fa ç a u m a tab e l a de
se u s res u l t a d o s .
(c) (0,5) C al c u l e a efi ci ê n c i a da m á q u i n a . M o s t r e qu e el a só de p e n d e da raz ã o de
co m p r e s s ã o r.
Resposta:

Como o gás é diatômico: γ = CP/CV = 7/5 = 1,4

(a) Ponto A:

pA = p0, TA = T0, VA = V0

Ponto C:

TC = TA ∴ TC = T0 (isoterma)

pCVC = pAVA = p0V0

VC = rV0 →pC = p0V0/rV0 ∴ pC = p0/r


www.cliqueapostilas.com.br

Ponto B:

pB = pA ∴ pB = p0

pBVBγ = pCVCγ = p0/r.(rV0)γ

VBγ = rγ-1.V0γ = r(γ-1)/γ.V0 → VB = r2/7.V0

e TB/TA = pBVB/pAVA → TB = r(γ-1)/γ.T0 ∴ TB = r2/7.T0


Ponto Volume Temperatura Pressão

A V0 T0 p0

B r2/7.V0 r2/7.T0 p0

C rV0 T0 p0/r

(b) A→B (pressão constante)

QAB = nCP∆T = p0V0/RT0.7/2R.(r(γ-1)/γ.T0 - T0) = 7/2.p0V0.(r(γ-1)/γ-1) ∴ QAB = 7/2.p0V0.(r2/7-1)

WAB = pA.∆V = p0.(r(γ-1)/γ.V0 - V0) = p0V0.(r(γ-1)/γ-1) ∴ WAB = p0V0.(r2/7-1)

∆EintAB = QAB – WAB = 7/2.p0V0.(r2/7-1) - p0V0.(r2/7-1) ∴ ∆EintAB = 5/2.p0V0.(r2/7-1)

B→C (adiabática)

QBC = 0

∆EintBC = nCV∆T = n.5/2.R.(TC-TB) = p0V0/T0.5/2.(T0- r(γ-1)/γ.T0) = 5/2.p0V0.(1- r(γ-1)/γ)

∴ ∆EintBC = 5/2 p0V0.(1 -r2/7) [Obs : ∆EintBC = - ∆EintAB]

QBC = WBC + ∆EintBC = 0 → WBC = - ∆EintBC ∴ WBC = 5/2 p0V0.(r2/7-1)

C→A (isoterma)

∴ ∆EintCA = 0

WCA = ∫CA p.dV = ∫CA nRT0.dV/V = p0V0 ∫CAdV/V = p0V0.ln(VA/VC) ∴ WCA = -p0V0.lnr
QCA = WCA + ∆EintCA ∴ QCA = -p0V0.lnr
Etapa Q Wsist ∆Eint

A→B 7/2.p0V0.(r2/7-1) p0V0.(r2/7-1) 5/2.p0V0.(r2/7-1)

B→C 0 5/2 p0V0.(r2/7-1) 5/2 p0V0.(1-r2/7)

C→A -p0V0.lnr -p0V0.lnr 0

(c) ε = W/QQ = [p0V0.(r2/7-1) + 5/2.p0V0.(r(γ-1)/γ-1) - p0V0.lnr]/[7/2.p0V0.(r(γ-1)/γ-1)] =

= [7/2.(r2/7-1) - lnr]/[ 7/2.(r2/7-1)] = 1 - [lnr / (7/2.(r2/7-1))]

2a Questão (2,5 pontos)


www.cliqueapostilas.com.br

U m cor p o de ca p a c i d a d e cal o r í fi c a C = 50 J/K na te m p e r a t u r a T1 = 300 K est á po s t o


e m co n t a t o co m u m res e r v a t ó r i o de te m p e r a t u r a a T2 = 450 K. Ju n t o s ele s for m a m u m
universo ter m o d i n â m i c o . A o ati n g i r o eq ui l í b r i o ;

(a) (0,5) cal c u l e Q 1 , o cal o r ab s o r v i d o pel o cor p o , e Q 2 , o cal o r ab s o r v i d o pel o


res e r v a t ó r i o .

(b) (1,0) c alc u l e a vari a ç ã o da entr o p i a ∆S 1 do cor p o e ∆S 2 do res e r v a t ó r i o .

(c) (1,0) q ual é a vari a ç ã o da en e r g i a inter n a ∆E int, e da entr o p i a ∆S , de s s e uni v e r s o


ter m o d i n â m i c o ?

Resposta:

(a) Q1 = C.(T2 – T1) = 50J/K.(450K-300K) = +7,5 kJ

Q2 = -Q1 = -7,5 kJ

(b) ∆S1 = ∫T1T2dQ/T = ∫T1T2C.dT/T = C.ln(T2/T1) = C.ln(450/300) = C.ln(3/2) = C.[ln3-ln2] =

≈ 50J/K.[1,1 - 0,69] = +20,5 J/K

∆S2 = ∆Q/T2 = Q2/T2 = -7,5kJ/450K = -16,7 J/K

(c) Como não há trabalho, ∆Eint = Q, logo ∆EintUNIV = Q1 + Q2 = 0

∆SUNIV = ∆S1 + ∆S2 ≈ +20,5 J/K – 16,7 J/K = 3,8 J/K

3a Questão (2,5 pontos)

U m reci p i e n t e de vol u m e igu a l a 30 l co n t é m u m gá s perf e i t o à te m p e r a t u r a de 0,0 O C .


D ei x a- se u m a part e do gá s es c a p a r par a o ext e ri o r do reci pi e n t e , m a n t e n d o- se a
te m p e r a t u r a co n s t a n t e en q u a n t o qu e a pr e s s ã o no reci pi e n t e di m i n u i de ∆p = 0,78
atm. A de n s i d a d e do gá s so b co n d i ç õ e s nor m a i s de te m p e r a t u r a e pre s s ã o , isto é, T =
0,0 o C e p = 1,0 atm, é igu a l a ρ = 1,3 g/l.

(a) (1,0) C al c u l e a vari a ç ã o do nú m e r o de m o l e s do gá s de n t r o do reci pi e n t e .

(b) (1,5) D e t e r m i n e a m a s s a do gá s qu e es c a p o u par a o ext e r i o r.

Resposta:

(a) Vi = V0 = 30l = 3,0x10-2m3, Ti = T0 = 273K, pi = p0, ni = n0

Vf = V0, Tf = T0, pf = p0 - ∆p, nf = ni-∆n (∆p = 0,78atm = 7,8x104Pa)

n0 = p0V0/RT0 nf = (p0-∆p)V0/RT0

∆n = nf - n0 = -∆p.V0/RT0 = -7,8x104Pa x 3,0x10-2m3/8,3J/mol.K/273K = -1,03 mol

(b) Na CNTP, o volume ∆V ocupado pelo gás que escapou é determinado por:

∆V = ∆n.R.T0/patm = (∆p.V0/RT0)x(R.T0/patm) = ∆p/patm.V0 = 0,78atm/1atm x 30l = 23,4l


www.cliqueapostilas.com.br

Logo: ∆m = ρ.∆V = 1,3 g/l x 23,4l = 30,4 g


www.cliqueapostilas.com.br

4a Questão (2,5 pontos)

U m refri g e r a d o r R op e r a entr e res e r v a t ó r i o s de te m p e r a t u r a s TA = 400 K e TB = 300 K,


co m u m co e fi ci e n t e de perf o r m a n c e ou ren d i m e n t o K = 2. Q u e r e m o s , e m ca d a cicl o,
tirar Q B = 600 J do res e r v a t ó r i o frio.

(a) (0,6) C al c u l e o tra b a l h o qu e, por cicl o, te m qu e ser feito pel o refri g e r a d o r , as si m


co m o Q A, o cal o r ce d i d o por cicl o ao res e r v a t ó r i o qu e n t e .

(b) (0,6) P ar a u m refri g e r a d o r de C a r n o t R(Carnot), op e r a n d o co m os m e s m o s


res e r v a t ó r i o s , e tira n d o a m e s m a qu a n t i d a d e de cal o r Q B por cicl o, cal c u l e W (Carnot) e
Q A(Carnot).

(c) (0,8) C al c u l e a vari a ç ã o de entr o p i a , por cicl o, ∆S , do refri g e r a d o r R e ta m b é m


∆S (Carnot) do refri g e r a d o r R(Carnot) de C a r n o t .

(d) (0,5) C al c u l e a raz ã o R entr e o trabalho extra (W-W (Carnot), qu e te m o s qu e for n e c e r


por nã o dis p o r da q u e l e refri g e r a d o r de C a r n o t , e a vari a ç ã o de entr o p i a , por cicl o, ∆S ,
ou sej a, R = (W-W (Carnot))/∆S .

Resposta:

(a) Refrigerador R:

K = QB/W → W = QB/K = 600J/2 = 300J

QA = QB + W = 600J + 300J = 900J

(b) Refrigerador R(Carnot):

KCarnot = TB/(TA-TB) = 300 – 100 = 3

→ WCarnot = QB/KCarnot = 600J/3 = 200J

QA = QB + W = 600J + 200J = 800J

(d) Refrigerador R:
∆S = QA/TA - QB/TB = = 900J/400K - 600J/300K = 0,25J/K

∆SCarnot = QA/TA - QB/TB = 800J/400K - 600J/300K = 0 (reversível)

(d) R = (300J-200J)/0,25J/K = 400K = TA

Formulário
ln2 = 0,69, ln3 = 1,1, ln5 = 1,6, 1atm = 1,0x105Pa, 1l = 10-3m3, R = 8,3 J/mol.K

pV = nRT, pVγ = const, CV = ½.f R, CP = CV + R, r = W/QQ, K = QF/W


www.cliqueapostilas.com.br

IN S T I T U T O DE FÍSI C A

Universidade Federal do Rio de Janeiro


Física II – Turmas do horário de 10 h às 12 h 1° período de 2009
Prova Final
1a Questão (2,5 pontos): A águ a esc o a po r um ca n o hor i z o n t a l par a a
at m o s f e r a a uma vel oc i d a d e v1 = 15m / s co m o mostr a a fi g u r a ao
lado . Os di â m e t r o s das seç õ e s es q u e r d a e di re i t a do tub o são
(a) (0,5) Q u e vol u m e de
d1 v2
d2 ág u a esc o a par a a
v1 =15m/s
at m o s f e r a du r a n t e um
(b) (1,0) Qual é a
ve l o c i d a d e de esc o a m e n t o
da águ a no lad o es q u e r d o
(c) (1,0) Q u a l é a pre s s ã o mano mét r ica (e m at m o s f e r a s ) no lado

Resposta:

∆V A1.∆ x π d12 4 × ∆ x π d12 ∆ x π d12


∆t ∆t ∆t ∆t v 1∆ t
(a) ∆V = ∆t = ∆t = ∆t = 4 ∆t = 4 =
(
π 3,0 × 10 − 2 m ) 2
15m / s.600s
4 =
= 6,36m3

∆V A d2 9
v1 1 = v1 12 = 15m / s.
∆ t = A1v1 = A2v2 → v2 = A2 d2 25
(b) = 5,4 m/s
(c) P1 = 1 atm
P1+½.ρv12+ρgy1 = P2+½.ρv22+ρgy2, onde y1=y2

Logo

P2 = P1 + ½.ρ[v12-v22] = 1 atm + ½.1,0x103kg/m3.[(15m/s)2-(5,4m/s)2]

= 1 atm + 97920 Pa = (1+0,98) atm

∴ P2(manométrico) = 0,98 atm

2a Questão (2,5 pontos): U m a co r d a de viol i n o de 30, 0 cm de


co m p r i m e n t o co m de n s i d a d e li ne a r de massa de 0, 6 5 0 g/c m é
co l o c a d a pró x i m a de um auto f a l a n t e que es t á co n e c t a d o a um
www.cliqueapostilas.com.br

os cil a d o r de áu d i o de fre q u ê n c i a vari á v e l . D e s c o b r e- se qu e a cor d a


os cil a so m e n t e na s fre q u ê n c i a s de 880 Hz e 1 3 2 0 Hz, qu a n d o a
fre q u ê n c i a do os cil a d o r vari a entr e 500 Hz e 1 50 0 Hz. D e t e r m i n e :

(a) (1,0) a vel o c i d a d e da s on d a s na cor d a ;

(b) (1,0) a fre q u ê n c i a do fun d a m e n t a l ;

(c) (0,5) a ten s ã o na cor d a .

Resposta:

(a) λ = 2L/n onde n = 1,2,3...

v v
f = v/λ = 2L n → ∆f = L
2

→ v = 2L.∆f = 2x0,30mx(1320-880)Hz = 264 m/s

v
(b) f1 = L .1 = ∆f = 440Hz
2

F
µ
(c) v = → F = µ.v2 = 0,650x10-3kg/(10-2m)x(264m/s)2 = 4,53 kN/m

3a Questão (2,5 pontos): U m cili n d r o , co m po s i ç ã o do eix o hori z o n t a l ,


te m par e d e s adi a b á t i c a s e fun d o di at é r m i c o (o lad o es q u e r d o per m i t e
a pa s s a g e m de cal o r). O cilin d r o est á fec h a d o por u m ê m b o l o m ó v e l
de áre a A, qu e ta m b é m é adi a b á t i c o . O ê m b o l o est á lig a d o a uma m o l a de
co n s t a n t e elá s ti c a k, pre s a à par e d e direit a de m o d o qu e o est a d o rel a x a d o da m ol a
corr e s p o n d e à po si ç ã o do ê m b o l o no fun d o do cilin d r o (figur a à es q u e r d a). U m a
qu a n t i d a d e de n m ol e s de u m gá s ide al, m o n o a t ô m i c o , é inj et a d a no cili n d r o e, no
eq ui l í b r i o , o ê m b o l o fic a a u m a dist â n c i a x do fun d o do cilin d r o (ver fig u r a).

(a) (0,5) Em função desses dados (A, x, k, n) e de constantes uni ve rsais, deter m i n e a pressão e a
tem peratu ra do gás.

Em um segund o processo, certa quanti da de de calor Q é fornec i d a ao gás lenta m e n te de for m a


que o êmbo l o fica agora até uma distânci a 3x/2. Neste processo, deter m i n e:

(b) (0,5) o trabal h o realizad o pel o gás;

(c) (0,5) a variação de sua energia interna ∆E int;

(d) (0,5) o calor absor v i d o Q;

(e) (0,5) a variação de entro p i a no processo.


www.cliqueapostilas.com.br

. . . .
. . . .
Q . .. .. .. .
. gás
. . .
. . . .
vácuo . . . . vácuo
x
Resposta:

(a) A força que o gás exerce sobre o êmbolo é igual a que a mola faz do outro
lado. Logo:

kx
p.A = kx → p = A

pV kx ⋅ Ax kx 2
V = A.x → T(x) = nR = A ⋅ nR = nR

Vf xf
kx
∫ p ⋅ dV ∫ A
.Adx xf 5 2
kx
2 xi
(b) Wgás = Vi
= xi
= ½.kx = ½k.[(3x/2) -x ] = 8
2 2

3  k ( 3 x 2) kx 2  15
2

n R (T ( 3 x 2 ) − T ( x ) )
3 n R −
2  nR nR  kx 2
(c) ∆Eint = nCV∆T = 2 =  = 8

5 2 15 2 5 2
kx kx kx
(d) Q = Wgás + ∆Eint = 8 + 8 =2
f f f f f f
dW dEint pdV nCV dT nRT dV 3 dT
f
∫i
dQ
∫ T ∫ T ∫ T ∫ T ∫ V T ∫ n 2R T
(e) ∆S = T = i + i = i +i = i +i =

 3 k ( 3x / 2) 
2
Vf
 V  nR  ln 2 xA + 3 ln nR 
f T
dV 3 dT 3 T
nR ∫ nR ∫ nR  ln f + ln f  kx 2
Vi
V 2 Ti
T  Vi 2 Ti =  xA 2 nR 
= + = =

 3  9  2
1

nR  ln + 3 ln    3 3 3
 2  4   nR  ln + 3 ln  4nR ln
=   =  2 2 = 2
www.cliqueapostilas.com.br

4a Questão (2,5 pontos): U m gá s ide a l, diat ô m i c o , oc u p a u m vol u m e V1


= 2,5 l, a pr e s s ã o p1 = 1,0 bar = 1,0 x 1 0 5 Pa e à te m p e r a t u r a T1 = 300 K. El e é su b m e t i d o
ao s se g u i n t e s pro c e s s o s rev e r s í v e i s : (1 ⇒ 2) u m aq u e c i m e n t o iso v o l u m é t r i c o até a
su a pr e s s ã o qui n t u p l i c a r , de p o i s (2 ⇒ 3) u m a ex p a n s ã o isot é r m i c a até a pre s s ã o
ori gi n a l e, final m e n t e , (3 ⇒ 1) u m a tra n s f o r m a ç ã o iso b á r i c a volt a n d o ao est a d o inici al.
(a) (0,5) D e s e n h e o cicl o do gá s no dia g r a m a de Bo y l e- Cl a p e y r o n (p ver s u s V),
indi c a n d o a es c a l a da s uni d a d e s .

(b) (1,5) C al c u l e , e m joul e s, o cal o r Q qu e el e ab s o r v e , o tra b a l h o W feito pel o gá s e a


su a vari a ç ã o de en e r g i a inter n a ∆E int e m ca d a eta p a do cicl o. C o n s t r u a u m a tab e l a
co m os val o r e s de Q, W e ∆E int par a os trê s pr o c e s s o s .

(c) (0,5) O ren d i m e n t o da m á q u i n a op e r a n d o se g u n d o est e cicl o.

Resposta:

p (a)
5p1 2

p1
1 3
V1 V

(b)

1 ⇒ 2:

W12 = 0,

p1V1 p2V2 5 p1V1


= =
T1 T2 T2 ∴ T = 5T
2 1

p1V1 5
⋅ R ⋅ 4T1
→ ∆Eint,12 = n.CV.∆T = RT1 2 = 10.p1V1 = 10x1,0x105Pax2,5x10-3m3 = 2,5
kJ
Q12 = W12 + ∆Eint,12 = 2,5 kJ

2 ⇒ 3:

T3 = T2 = 5T1

→ ∆Eint,23 = 0

Vf Vf
nRT
∫ p.dV ∫ V
.dV ln
Vf
Vi Vi Vi .
W23 = = = nRT .

Como T = T2 = const. piVi = pfVf → Vf = pi/pf.Vi = 5p1/p1.V1 → Vf = 5.V1 e Vi = V1


www.cliqueapostilas.com.br

p1V1 5V
⋅ T2R ⋅ ln 1
RT V1 = p1V1 ⋅ 5 ln 5 = 1,0 × 10 5 Pa × 2,5 × 10 − 3 m 3 5 ln 5 = 2,0 kJ
∴ W23 = 1

Q23 = W23 + ∆Eint,23 = 2,0 kJ

3 ⇒ 1:
−3 3
W31 = p.∆V = p1.(V1-V3) = p1.(V1-5V1) = - 4.p1V1 = - 4 × 1,0 × 10 Pa × 2,5 × 10 m
5

= -1,0 kJ

p1V1 7
⋅ R ⋅ ( − 4T1 ) −3 3
= - 14.p1V1 = − 14 × 1,0 × 10 Pa × 2,5 × 10 m =
RT1 2 5
Q31 = n.CP.∆T =
-3,5 kJ

p1V1 5
⋅ R ⋅ ( − 4T1 )
∆Eint,31 RT 1 2
= n.CV.∆T = = -10.p1V1 = -10x1,0x105Pax2,5x10-3m3 =
-2,5 kJ

ou ∆Eint,31 = Q31 - W31 = -3,5 kJ + 1,0 kJ

ou Q31 = W31 + ∆Eint,31 = -1,0 kJ - 2,5 kJ = - 3,5 kJ

Q (kJ) W (kJ) ∆Eint (kJ)

1⇒2 2,5 0 2,5

2⇒3 2,0 2,0 0

3⇒1 -3,5 -1,0 -2,5

W W12 + W23 + W31 0 + 2,0kJ − 1,0kJ


Q12 + Q23 = 2,5kJ + 2,0kJ = 22%
QQ
(c) ε = =

Formulário
vsom = 340 m/s, g=10m/s2, ρágua= 1,0x103kg/m3, 1atm = 1,0x105Pa, 1l = 10-3m3, R = 8,3 J/mol.K

P+½ρv2+ρgy = const., v = [F/µ]1/2, f' = (v±vO)/(vmvS).f, ∆p(t) = [2∆pmcos[(ω1-ω2)t/2].sen[(ω1+ω2)t/2],


NIS = 10 log(I/Io),pV = nRT, pVγ = const, CV = ½.f R, CP = CV + R, ε = W/QQ, K = QF/W
www.cliqueapostilas.com.br

IN S T I T U T O DE FÍSI C A

Universidade Federal do Rio de Janeiro


Física II - Turmas no horário de 15h às 17h
1° período de 2009
Prova Final
a
1 Questão (2,5 pontos)

D oi s tub o s cilí n d r i c o s A e B, de se ç ã o ret a tra n s v e r s a l de


áre a igu a l a 40,0 c m 2 est ã o co n e c t a d o s por m e i o de u m
ter c e i r o tub o cili n d ri c o C, de se ç ã o ret a de áre a igu a l a 10,0 C
c m 2 . O s trê s tub o s tê m o m e s m o eix o de si m e t r i a , est ã o na A B
hori z o n t a l e ág u a flui atr a v é s del e s a u m a va z ã o vol u m é t r i c a
de 6,00 x 1 0 3 c m 3 /s.

(a) D e t e r m i n e a vel o c i d a d e de es c o a m e n t o na part e m a i s larg a e na co n s t ri ç ã o .

(b) D e t e r m i n e as dife r e n ç a s de pr e s s ã o entr e os tub o s A, B e C.

Resposta:

∆V 1 ∆V
(a) ∆ t = v.A → v = A ∆ t

1 ∆V 6,00 × 10 3 cm 3 / s
AA ∆ t 40,0cm 2
∴ vA = = = 150 cm/s

vAAA = vBAB = vCAC → vB = vA = 150 cm/s

AA 40cm 2
AB 2
→ vC = vA. = 150 cm/s. 10cm = 600 cm/s

(b) PA +½ρvA2+ρgyA = PB +½ρvB2+ρgyB = PC +½ρvC2+ρgyC

Como y = constante:

PA +½ρvA2 = PB +½ρvB2 = PC +½ρvC2

De (a) temos que vA = vB → PA = PB ou seja,

∆PAB =0
www.cliqueapostilas.com.br

e PA - PC = ½ρvC2 - ½ρvA2 = ½ρ (vC2-vA2) = ½.1,0x103kg/m3x[(6,00m/s)2-(1,50m/


s)2]

∴ ∆PAC = 1,69 x104Pa

2a Questão (2,5 pontos) L


U m a da s extr e m i d a d e s de u m a cor d a de co m p r i m e n t o L é pr e s a a
u m a par e d e . A outr a extr e m i d a d e est á liga d a a u m an e l se m m a s s a qu e po d e
de s li z a r livr e m e n t e ao lon g o de u m a ha s t e verti c a l se m atrit o, co n f o r m e a figu r a
apr e s e n t a d a ao lad o.
(a) (1,0) Es b o c e as on d a s est a c i o n á r i a s corr e s p o n d e n t e s ao s trê s maior e s
co m p r i m e n t o s de on d a s ?

(b) (1,5) Q u a i s sã o est e s trê s m a i o r e s co m p r i m e n t o s de on d a s est a c i o n á r i a s po s s í v e i s


ne s t a cor d a ?

Resposta:

(a)
L L L

(b) λ1 = 4L

4
L
λ2 = 3

4
L
λ3 = 5

3a Questão (2,5 pontos)

U m m ol de u m gá s ide al, m o n o a t ô m i c o , à te m p e r a t u r a T1 = 300 K é su b m e t i d o ao s


se g u i n t e s pr o c e s s o s rev e r s í v e i s : (1 ⇒ 2) u m aq u e c i m e n t o iso v o l u m é t r i c o até a su a
te m p e r a t u r a do b r a r T2 = 600 K, de p o i s (2 ⇒ 3) u m a ex p a n s ã o adi a b á t i c a até o est a d o
3, on d e a pr e s s ã o é igu a l à pre s s ã o inici al: p3 = p1 e, final m e n t e , u m a tra n s f o r m a ç ã o
iso b á r i c a (3 ⇒ 1).
(a) (0,5) R e p r e s e n t e o cicl o e m u m dia g r a m a {p, V}.
www.cliqueapostilas.com.br

(b) (1,5) C al c u l e a te m p e r a t u r a T3 e, e m joul e s , o tra b a l h o feit o W pel o gá s, o cal o r


tro c a d o Q e a su a vari a ç ã o de en e r g i a inte r n a ∆E int e m ca d a eta p a do cicl o. C o n s t r u a
u m a tab e l a co m os val o r e s de Q, W e ∆E int no s trê s pr o c e s s o s .

(c) (1,0) S e p1 = 1,0 x 1 0 5 Pa, cal c u l e {V 1, V 3}.

N u m e r o l o g i a : 2 0 ,4 = 1,32 ; 2 0 ,5 = 1,4 1 ; 2 0 ,6 = 1,52 ; 2 0 ,7 = 1,62.

Resposta:

(a)
p
2

p1
1 3

V1 V

(b) 1 ⇒ 2:

p1 p 2 p2
= =
T1 T 2 2 × T1
→ p2 = 2p1

W12 = 0 (volume constante)

R × (T2 − T1 ) 8,3J /( mol .K ) × ( 600K − 300K )


3 3

∆Eint,12 = n.CV.∆T = 2 = 2 =
3,735 kJ

Q12 = ∆Eint,12 + W12 = 3,735 kJ

2 ⇒ 3:

2 p1V1γ
p1
p2V2γ = p3V3γ onde γ = 5/3, V2 = V1, p2 = 2p1 e p3 = p1 → V3γ = → V3 =
3/5
2 V1

p 3V3 pV
= 2 0 ,6 1 1 = 2 0,6 T1
T3 = nR nR = 455K

∴ ∆Eint,23 = nCV∆T23 = 1x3/2x8,3J/(mol.K)x(455K-600K) = -1,805 kJ


www.cliqueapostilas.com.br

e Q23 = 0 = ∆Eint,23 + W23

→ W23 = -∆Eint,23 = +1,805 kJ

3 ⇒ 1:

Q31 = nCP (T1-T3) = 1x5/2x8,3J/(mol.K)x(300K-455K) = -3,216 kJ

∆Eint,31 = nCV.(T1-T3) = 1x3/2x8,3J/(mol.K)x(300K-455K) = -1,930 kJ

W31 = Q31 -∆Eint,31 = -3,216 kJ + 1,930 kJ = - 1,286 kJ

Q (kJ) W (kJ) ∆Eint(kJ)

1⇒2 3,7 0 3,7

2⇒3 0 1,8 -1,8

3⇒1 -3,2 -1,3 -1,9

nRT1 1 × 8,3J /( mol .K ) × 300K


V1 = =
(c) p1 1,0 × 10 5 Pa = 0,0249m3 ≈ 25 l

p1V1 pV T3
= 3 3 V3 = V1 455K 25 
nRT1 nRT3 T1
→ = 300K ≈ 38 l

4a Questão (2,5 pontos)

U m m o t o r tér m i c o fun ci o n a entr e doi s res e r v a t ó r i o s tér m i c o s , u m na te m p e r a t u r a de


600 K e o outr o a 350 K. A ca d a cicl o o m o t o r retir a 900 J de en e r g i a da font e qu e n t e e
pro d u z u m tra b a l h o útil de 1 50 J.

(a) (1,0) Q u a l a vari a ç ã o de entr o p i a do uni v e r s o nu m cicl o de fun c i o n a m e n t o de s s e


motor?

(b) (0,5) Q u a l o co e fi c i e n t e de ren d i m e n t o de u m refri g e r a d o r de C a r n o t qu e op e r e


entr e es s e s m e s m o s res e r v a t ó r i o s tér m i c o s ?

(c) (1,0) Q u a l seri a o tra b a l h o re ali z a d o a ca d a cicl o por u m m o t o r ide a l de C a r n o t , qu e


fun c i o n a s s e entr e es s e s m e s m o s res e r v a t ó r i o s tér m i c o s e retir a s s e a m e s m a en e r g i a
(900 J) da font e qu e n t e a ca d a cicl o?

Resposta:
TQ
(a)TQ = 600K, QQ= 900J
QQ
TF = 350K, QF= QQ- W = 900J – 150J = 750J

QF ∆SU = ∆SQ + ∆SM + ∆SF

∆SM = 0 (Motor opera em ciclo fechado)


TF
www.cliqueapostilas.com.br

∆SQ = -QQ/TQ = -900J/600K = -1,50 J/K

(negativo pois o calor sai do reservatório quente)

∆SF = +QF/TF = 750J/350K = 2,14 J/K

(positivo pois o calor entra no reservatório frio)

∆SU = -1,50 J/K + 0 + 2,14 J/K = 0,64 J/K 1,0 pto

TF 350K
TQ − TF
(b) Rendimento de Carnot: KCar= = 600K − 350K = 1,4

TQ − TF QQ − Q F W
TQ QQ QQ
(c) Eficiência de Carnot: εCar= = =

TQ − TF 250J
QQ 900J .
TQ
∴ W= = 600J = 375J

Formulário
vsom = 340 m/s, g=10m/s2, ρágua= 1,0x103kg/m3, 1atm = 1,0x105Pa, 1l = 10-3m3, R = 8,3 J/mol.K

P+½ρv2+ρgy = const., v = [F/µ]1/2, f' = (v±vO)/(vmvS).f, ∆p(t) = [2∆pmcos[(ω1-ω2)t/2].sen[(ω1+ω2)t/2],


NIS = 10 log(I/Io),pV = nRT, pVγ = const, CV = ½.f R, CP = CV + R, ε = W/QQ, K = QF/W
www.cliqueapostilas.com.br

IN S T I T U T O DE FÍSI C A

Universidade Federal do Rio de Janeiro


Física II – Turmas do horário de 10 h às 12 h 1° período de 2009

Prova de 2a Chamada

1a Questão (2,5 pontos):

U m a m o l a de m a s s a de s p r e z í v e l e co n s t a n t e elá s ti c a k = 400 N/m est á


su s p e n s a verti c a l m e n t e e u m pra t o de m a s s a m = 0 ,200 kg est á su s p e n s o e m re p o u s o
na su a extr e m i d a d e infe ri o r. O aç o u g e i r o dei x a cair so b r e o pr at o de u m a altur a de h =
0 ,40 m u m a po s t a de car n e de M = 2 ,2 kg. A po s t a de car n e pr o d u z u m a coli s ã o
co m p l e t a m e n t e inel á s t i c a co m o prat o e faz o sist e m a ex e c u t a r u m M H S . C al c u l e :

(a) (1,0) a vel o c i d a d e do prat o e da car n e log o ap ó s a coli s ã o ;

(b) (1,0) a a m p l i t u d e da os cil a ç ã o su b s e q u e n t e ;

(c) (0,5) o perí o d o da os cil a ç ã o .

Resposta:

(a) (1,0) No momento do impacto a velocidade da carne é v = 2gh e a conservação do


momento implica na velocidade do conjunto, logo após a colisão:

M
2gh
V0 = M + m = 2,59 m/s.
www.cliqueapostilas.com.br

(b) (1,0) A posição do prato antes do choque, a partir da posição relaxada da mola, é
dada por

mg
x0 = k = 0,5cm.

A equação de Newton :

d 2x
(M + m)
dt 2 = -kx+(M+m) g = −k (x−x1),

g
com x1 = (M+m) k = 5,5 cm, resulta em

k
x − x1 = a senωt + b cosωt, com ω = M + m = 12,9 s−1.

As condições iniciais fornecem

V0
b = x0 − x1 = −5,0 cm e a = ω = 20, 1 cm.

A amplitude é:

A= a 2 + b 2 = 20,7 cm.

O mesmo resultado pode ser obtido usando a conservação da energia após o choque
inelástico.


(c) (0,5) T = ω = 0,49 s.

2a Questão (2,5 pontos):


3
Uma onda tr a n s v e r s a l har m ô n i c a
2
si m p l e s p r o p a g a - se ao lo n g o de 1
uma co r d a no se n t i d o do ei x o x
y(cm)

ne g a t i v o (- x). A fig u r a ao lad o -1

-2
mostr a um grá f i c o do de s l o c a m e n t o
-3
0 20 40 60 80 100
x(cm)
www.cliqueapostilas.com.br

inst a n t e t = 0. A inte n s i d a d e da for ç a de tra ç ã o na cor d a é de


3,6 N e a m a s s a es p e c í f i c a line a r é de 25 g/m. D e t e r m i n e :

(a) (0,5) a a m p l i t u d e ,

(b) (0,5) o co m p r i m e n t o de on d a ,

(c) (0,5) a vel o c i d a d e da on d a ,

(d) (0,5) a fre q u ê n c i a ,

(e) (0,5) e u m a eq u a ç ã o y(x, t) qu e de s c r e v a a pr o p a g a ç ã o da on d a .

Resposta:

(a) (0,5) A = 3,0 cm

(b) (0,5) λ = 50 cm

F 3,6N
µ 25 × 10 − 3 kg / m
(c) (0,5) v = = = 12 m/s

v 12m / s
(d) (0,5) f = λ = 0,5m = 24 Hz

(e) (0,5) y(x,t) = A.sen(kx+ωt+δ) (onda se propagando no sentido -x), onde:


k = λ = 4π rad/m

ω = 2π.f = 48π rad/s

e, para t = 0, y(0,0) = A.sen(δ) ≈ 2,5 cm, sendo que:


www.cliqueapostilas.com.br

∂ y( x,t )
∂t
x ,t = 0 = ωA.cos(kx+ωt+δ)
v(0,0) = x,t=0 = ωA.cos(δ) e, pelo gráfico v(0,0) <0

→ 3π/2 >δ > π/2

Logo:

 2,5  56,5 o
arcsen   ≈ o
δ=  3,0  123,5 → δ = 123,5o ≈ 0,686π rad

3a Questão (2,5 pontos):

Dentro de u m cal o r í m e t r o , isol a d o adi a b a t i c a m e n t e , de ca p a c i d a d e


cal o r í fi c a de s p r e z í v e l , col o c a- se M g elo = 0 ,5 kg de gel o a te m p e r a t u r a de Tg elo = -
1 0 ,0 o C e M á g u a = 1 ,0 kg de ág u a a Tá g u a = 30 ,0 o C .

(a) (0,5) Q u a l é a te m p e r a t u r a de eq u il í b r i o ? (Verifi q u e se é po s s í v e l der r e t e r tod o o


gel o.)

(b) (1,0) Q u a l é a qu a n t i d a d e de ág u a líq ui d a no est a d o de eq ui l í b r i o ?

(c) (1,0) C al c u l e ne s t e pro c e s s o a vari a ç ã o da entr o p i a do uni v e r s o ter m o d i n â m i c o


for m a d o pel o cal o r í m e t r o e se u co n t e ú d o .

Resposta:

(a) (0,5) ∆Qgelo + ∆Qágua=0:

Para levar o gelo até 0oC são necessários:

∆Qgelo(Tg→0oC) = Mgelo.c(gelo). (0oC-Tgelo) = 0,5kg x 2220J x 10oC = 1,110x104J

Já para conseguir derreter todo o gelo:

∆Qgelo→água = ∆Qgelo(Tg→0oC) + Mgelo.Lfusão(gelo) = 1,110x104J + 0,5kg x 333x103J =


5
1,776x10 J
www.cliqueapostilas.com.br

Se toda água esfriasse até 0oC o calor liberado seria de:

∆Qágua = Mágua.c(água).(Tf - Tágua) = 1,0kg x 4190J/(kg.oC) x 30,0oC = 1,257x105J

Ou seja,

∆Qgelo(Tg→0oC) < ∆Qágua < ∆Qgelo→água ⇔ não dá para derreter todo o gelo mas
ele chega até 00C!

∴ Tf = 0oC = 273 K

(b)(1,0) O calor cedido pela água será utilizado para aquecer o gelo até 0oC e derreter
parte dele. Portanto, o calor a ser utilizado para o derretimento do gelo será de:

∆Qderretimento = mgelo.Lf,gelo = ∆Qágua - ∆Qgelo(Tg→0oC) = 1,257x105J - 1,110x104J =


1,146x105J

Logo a massa de gelo derretida é de:

(
∆ Qa − ∆ Qg Tg → 0o C ) 1,1146 × 105 J
= 333 × 10 J = 0,344kg
Lf 3
mgelo =

∴ Mágua(final)= Mágua + mgelo = 1,344 kg, com Tf = 273K.

Tf Tf
dQ gelo Mg cg dT mg Lf Tf mg Lf
∫ T ∫ T
+
Tf
M g cf ln +
Tg Tg Tg Tf
(c)(1,0) ∆Sgelo = = = =

273K 0,344kg × 333 x10 3 J / kg


0,5kg × 2220J / ( kg .K ) × ln +
= 263K 273K = 461 J/K
www.cliqueapostilas.com.br

Tf Tf
dQágua MacadT
∫ T ∫ T Maca ln
Tf
1,0kg × 4190 J / ( kg .K ) × ln
273K
Ta 303K = -
∆Ságua = Ta
= Ta
= =
437 J/K

∴ ∆Suniverso = 24 J/K.

4a Questão (2,5 pontos):

N e s s e pr o b l e m a dis p o m o s de trê s res e r v a t ó r i o s tér m i c o s R A , R B , R C co m te m p e r a t u r a s


TA > TB > TC .

(a) (0,5) N u m a pri m e i r a op e r a ç ã o tra n s f e r i m o s , por co n d u ç ã o atr a v é s de u m co n d u t o r


de cal o r, cert a qu a n t i d a d e de cal o r Q do res e r v a t ó r i o R A até o res e r v a t ó r i o R B . C al c u l e ,
ne s s a op e r a ç ã o , a vari a ç ã o de entr o p i a ∆S a do sist e m a co n s t i t u i d o pel o s doi s
res e r v a t ó r i o s e o co n d u t o r e cal o r.

(b) (1,0) Ac o p l a m o s o sist e m a aci m a a u m a m á q u i n a de C a r n o t C BC op e r a n d o entr e R B


e R C . C al c u l e o tra b a l h o pr o d u z i d o W 1 us a n d o a qu a n t i d a d e de cal o r Q dis p o n í v e l no
res e r v a t ó r i o R B .

(c) (1,0) Sej a ag o r a u m a m a q u i n a de C a r n o t C AC op e r a n d o dire t a m e n t e entr e R A e R C .


C al c u l e o tra b a l h o W 2 pro d u z i d o por es s a m á q u i n a de C a r n o t us a n d o a mes m a
qu a n t i d a d e de cal o r Q extr a í d a do R A .

Resposta:

−Q Q  1 1 
+ Q ⋅  − 
(a) (0,5) ∆Sa = ∆SA + ∆SB =
T A TB
=  TB T A 

TC W1
1−
TB
(b) (1,0) εCarnot = = Q

 T 
Q 1 − C 
= 
TB 
∴ W1
www.cliqueapostilas.com.br

TC W2
1−
TA
(c) (1,0) εCarnot = = Q

 T 
Q 1 − C 
= 
TA 
∴ W2

Formulário
vsom=340 m/s, g=10m/s2, ρágua=1,0x103kg/m3, R=8,3J/mol.K, c(gelo)=2220J/(kg.oC), c(água)=4190J/(kg.oC),
Lfusao(gelo)=333kJ/kg, 1l=10-3m3, 1atm=1,0x105Pa.

P+½ρv2+ρgy = const., v = [F/µ]1/2, f' = (v±vO)/(vmvS).f, ∆p(t) = [2∆pmcos[(ω1-ω2)t/2].sen[(ω1+ω2)t/2],


NIS = 10 log(I/Io),pV = nRT, pVγ = const, CV = ½.f R, CP = CV + R, ε = W/QQ, K = QF/W

IN S T I T U T O DE FÍSI C A

Universidade Federal do Rio de Janeiro


Física II – Turmas do horário de 15 h às 17 h 1° período de 2009
Prova de 2a Chamada
1a Questão (2,5 pontos):

O volante de um relóg i o mecâni c o oscila com ampl i t u d e angular máx i m a de π rad e com
perío d o T = 0,50 s. Deter m i n e:

(a) (0,5) sua veloci da de angular máxi m a;

(b) (1,0) a veloc i d a d e angular quand o o desloca m e n t o angul ar for de π/2 rad.

(c) (1,0) a aceleração angular, quand o seu desloca m e n t o for de π/4 rad.
www.cliqueapostilas.com.br

Resposta:

 2π  dθ ( t ) 2π  2π 
θ ( t ) = π ⋅ cos  t+ δ = −π ⋅ sen t+ δ
 T  , dt T  T ,

d 2θ (t )
2 2
 2π   2π   2π 
= −π  ⋅ cos t + δ  = −  θ (t )
dt 2  T   T   T 

dθ ( t ) 2π 2
=
dt máx T
(a) (0,5) = 79 rad/s

 2π t  3
 + δ
(b) (1,0) Quando θ = π/2, o cosseno de  T  vale 0,5 e o seno ± 2 , logo:

dθ ( t )
= 4π 2 3rad / s
dt ≈ m 34,2 rad/s

d 2θ (t )
2
 2π  π
= −  ⋅
(c) (1,0)
dt 2  T  4
= - 2π3 rad/s2 ≈ -124 rad/s2

2a Questão (2,5 pontos): L1 L2

Um fi o de al u m í n i o de co m p r i m e n t o L 1 = 60,0 c m e áre a de
se ç ã o tra n s v e r s a l A = 1,0 0 x 1 0 -2 c m 2 é co n e c t a d o a u m fio de aç o co m
m e s m a áre a de se ç ã o tra n s v e r s a l . O fio co m p o s t o , carr e g a d o co m u m bl o c o m de
m a s s a 10,0 kg, é dis p o s t o co n f o r m e indi c a d o na fig u r a ao lad o a fi m de qu e a dist â n c i a
L 2 da jun ç ã o até a poli a de su p o r t e sej a de 86,6 cm. O n d a s tra n s v e r s a i s sã o ind u z i d a s
no fio us a n d o- se u m a font e ext e r n a de fre q u ê n c i a vari á v e l .

(a) (1,0) Q u a i s sã o as vel o c i d a d e s de pro p a g a ç ã o de u m a on d a na s du a s cor d a s e m


fun ç ã o da s vari á v e i s m, ρ1, ρ2, A e da ac e l e r a ç ã o gra v i t a c i o n a l g?

(b) (1,0) Q u a l é a m e n o r fre q u ê n c i a de ex ci t a ç ã o e m qu e oc o r r e m on d a s est a c i o n á r i a s


e e m qu e a jun ç ã o do s doi s fios é u m nó da on d a ?

(c) 0,5) Q u a l é o nú m e r o total de nó s ob s e r v a d o ne s s a fre q u ê n c i a , ex cl u i n d o- se os nó s


na s du a s extr e m i d a d e s do fio?
www.cliqueapostilas.com.br

O b s: A m a s s a es p e c í f i c a ρ1 do al u m í n i o é 2,60 g/cm3 e a do aç o ρ2 é de 7,80 g/cm3.

Resposta:

mg mg mg
F m1 ρ 1 ⋅ V1 ρ 1 ⋅ A1 ⋅ L1 mg
µ L1 L1 L1 ρ 1A
(a) (1,0) v1 = = = = = e v2 =
mg
ρ 2A

2L1 v1 v1 v2
n1 n2
n1 λ1 2L1 2L 2
(b) (1,0) λ1 = →f= = =

n2 L v L2 ρ2 86,6cm 7,80g / cm 3
= 2 1
n1 L1v 2 L1 ρ1 60,0cm 2,60g / cm 3
∴ = = = 2,5

A menor frequência implica em menor valor de n, logo

n2 = 5 e n1 = 2

v1 mg 2
n1
2L1 ρ 1A 2L1
→f = = =

10,0kg × 10m / s 2 1
−6
2,60 x10 kg / m × 1,00 × 10
3 3
m 2 0,60m
= 323 Hz

(c) (0,5) No fio 1 temos 2 nós (excluída a extremidade esquerda) e no fio 2


temos 5 nós (excluída a extremidade direita), sendo um nó comum aos dois fios
(junção). Logo o número total de nós é de:

n = n1 + n2 - 1 = 6 nós

3a Questão (2,5 pontos):


www.cliqueapostilas.com.br

Dentro de um cal o r í m e t r o , isol a d o adi a b a t i c a m e n t e , de ca p a c i d a d e cal o r í fi c a


de s p r e z í v e l , col o c a- se M g elo = 2 ,0 kg de gel o a te m p e r a t u r a de Tg elo = -20 ,0 o C e M ag u a =
0 ,500 kg de ág u a a Tag u a = 30 ,0 o C .

(a) (0,5) Q u a l é a te m p e r a t u r a de eq ui l í b r i o ? (Verifi q u e se o gel o ch e g a a der r e t e r .)

(b) (1,0) Q u a l é a qu a n t i d a d e de ág u a líq ui d a ne s t e est a d o de eq u il í b r i o ?

(c) (1,0) C al c u l e ne s t e pro c e s s o a vari a ç ã o da entr o p i a do uni v e r s o ter m o d i n â m i c o


for m a d o pel o cal o r í m e t r o e se u co n t e ú d o .

Resposta:

(a) (0,5) ∆Qgelo + ∆Qágua=0

Para levar o gelo até 0oC são necessários:

∆Qgelo(Tg→0oC) = Mgelo.c(gelo). (0oC-Tgelo) = 2,0kg x 2220J/(kg.oC)x20oC =


8,880x104J

Se toda água esfriasse até 0oC o calor liberado seria de:

- ∆Qágua = - Mágua.c(água).(Tf - Tágua) = 0,500kg x 4190J/(kg.oC) x 30,0oC =


6,285x104J

O que é insuficiente para levar o gelo até 0oC. Logo, parte da água vai congelar e o calor
liberado será utilizado para aquecer o restante do gelo.

∆Qágua→gelo + ∆Qágua + ∆Qgelo(Tg→0oC) = 0

→ -∆Qágua→gelo = máguaLfusão = ∆Qgelo(Tg→0oC)+∆Qágua = 8,880x104J - 6,285x104J =


2,595x104J

Ou seja, a massa de água que irá congelar será:


www.cliqueapostilas.com.br

∴ magua= [∆Qgelo(Tg→0oC) - ∆Qágua]/Lfusão = 2,595x104J/(333x103J/kg) = 0,078 kg <


0,500kg

Portanto, nem toda água congela e a temperatura final fica em

Tf = 0oC

(b)(1,0) A água líquida que sobra é

∆Mágua = Mágua - magua = 0,500 kg - 0,078 kg = 0,422 kg

Tf Tf
dQ gelo M g c g dT Tf
∫ T ∫ T M g c f ln
Tg Tg Tg
(c)(1,0) ∆Sgelo = = = =

J 273K
2,0kg × 2220 × ln
= kg ⋅ K 253K = 338 J/K

Tf Tf
dQágua M a c a dT ∆ Qf − ma Lf
∫ T ∫ T
+
Tf Maca ln
Tf
Ta Tf
∆Ságua = Ta
= Ta = + =

273K 0,078kg × 333 × 10 3 J / kg


0,500kg × 4190J / ( kg .K ) × ln
= 303K - 273K = -314
J/K

∴ ∆Suniverso = +24 J/K.

4a Questão (2,5 pontos):

Um cor p o de ca p a c i d a d e tér m i c a C e te m p e r a t u r a Ti é col o c a d o de n t r o do


co m p a r t i m e n t o frio de u m refri g e r a d o r de C a r n o t, op e r a n d o co m res e r v a t ó r i o s tér m i c o s
www.cliqueapostilas.com.br

de te m p e r a t u r a s TA e TB res p e c t i v a m e n t e . S u p õ e- se qu e TA > Ti > TB . N o pro c e s s o de


resfri a m e n t o até a te m p e r a t u r a TB , o cor p o ce d e cert a qu a n t i d a d e de cal o r Q B à font e
fria, qu e por su a ve z ce d e es s a m e s m a qu a n t i d a d e de cal o r ao sist e m a refri g e r a n t e
qu e op e r a e m cicl o, ga s t a n d o u m a en e r g i a W e tra n s f e r i n d o o cal o r Q A à font e qu e n t e .
E m fun ç ã o do s da d o s do cor p o {Ti,C } e do refri g e r a d o r de C a r n o t {TA , TB }, det e r m i n e :

(a) (0,5) o cal o r Q B , a en e r g i a W ga s t a no pr o c e s s o e a qu a n t i d a d e de cal o r Q A ce di d a


à font e qu e n t e ;

(b) (1,5) a vari a ç ã o de entr o p i a ∆S C do cor p o , ∆S S do sist e m a refri g e r a n t e op e r a n d o


e m cicl o e, final m e n t e , ∆S A e ∆S B do s res e r v a t ó r i o s tér m i c o s .

(c) (0,5) C al c u l e ∆S UNIVERS O e ex p li q u e , fisic a m e n t e , por qu e nã o po d e se an u l a r ?

Resposta:

(a) (0,5) QB = C (T i − TB )

QB TB
T − TB
RCarnot = W = A

TA − TB T A − TB
QB C (T i − TB )
TB TB
→ W= =

T A − TB  T − TB 
C (T i − TB ) C (T i − TB )  A + 1
+ C (T i − T B ) =   =
TB T B
QA = W + QB =
TA
C (T i − T B )
TB

TB TB
dQC dTC TB Ti
∫ TC ∫C TC C ln − C ln
Ti TB
(b) (1,5) ∆SC = Ti = Ti = =

∆SS = SS(f) - SS(i) = 0 (estado final igual ao estado inicial no ciclo)


www.cliqueapostilas.com.br

∆ QA C (T i − T B ) T A C (T i − T B )
TA TA TB TB
∆SA = = =

∆ QB
TB
∆SB = . Como a fonte fria recebe QB do corpo e cede a mesma quantidade
de calor QB para a fonte quente, ∆QB = 0, logo:

∆SB = 0

Ti C (T i − T B )
− C ln
TB TB
(c) (0,5) ∆SUNIVERSO = ∆SC + ∆SS + ∆SA +∆SB = +0+ +0=
 (T − TB ) T 
C i − ln i 
 TB TB 

Para ∆SU se anular o processo tem de ser reversível. Como parte do processo é
irreversível (a do corpo C entrar em equilíbrio térmico com o reservatório B), então ∆SU >
0.

Matematicamente:

Ti
Se fizermos x =
TB
>1, então ∆SU = C [ ( x − 1) − ln x ] . Mas [ ( x − 1) − ln x ] > 0 para x > 1, →
∆SU > 0

Formulário
vsom=340 m/s, g=10m/s2, ρágua=1,0x103kg/m3, R=8,3J/mol.K, c(gelo)=2220J/(kg.oC), c(água)=4190J/(kg.oC),
Lfusao(gelo)=333kJ/kg, 1l=10-3m3, 1atm=1,0x105Pa.

P+½ρv2+ρgy = const., v = [F/µ]1/2, f' = (v±vO)/(vmvS).f, ∆p(t) = [2∆pmcos[(ω1-ω2)t/2].sen[(ω1+ω2)t/2],


NIS = 10 log(I/Io),pV = nRT, pVγ = const, CV = ½.f R, CP = CV + R, ε = W/QQ, K = QF/W

S-ar putea să vă placă și